• Shuffle
    Toggle On
    Toggle Off
  • Alphabetize
    Toggle On
    Toggle Off
  • Front First
    Toggle On
    Toggle Off
  • Both Sides
    Toggle On
    Toggle Off
  • Read
    Toggle On
    Toggle Off
Reading...
Front

Card Range To Study

through

image

Play button

image

Play button

image

Progress

1/780

Click to flip

Use LEFT and RIGHT arrow keys to navigate between flashcards;

Use UP and DOWN arrow keys to flip the card;

H to show hint;

A reads text to speech;

780 Cards in this Set

  • Front
  • Back
At which of the following ages does fetal movement first occur?
B. 2 months
C. 4 months
D. 6 months
E. 7 months




The correct answer is B. Neuromuscular development is sufficient to allow fetal movement in the eighth week of life. Other features of Week 8 include the first appearance of a thin skin, a head as large as the rest of the body, forward-looking eyes, appearance of digits on the hands and feet, appearance of testes and ovaries (but not distinguishable external genitalia), and a crown-rump length of approximately 30 mm. By the end of the eighth week, nearly all adult structures have at least begun to develop, and the fetus "looks like a baby."
Most of the oocytes in the ovary of a prepubescent girl are in which meiotic stage?
A. Anaphase of the second meiotic division
B. Metaphase of the first meiotic division
C. Metaphase of the second meiotic division
D. Prophase of the first meiotic division
E. Telophase of the first meiotic division




The correct answer is D. The first meiotic division is the "reduction" meiotic division, in which the diploid complement of DNA is reduced to a haploid complement. The bulk of oocytes in premenopausal women, girls, and babies are arrested at prophase of the first meiotic division. Postmenopausal women have very few viable oocytes. It is important to note that ovulation occurs before the oocyte is completely mature. The secondary oocyte leaving the follicle is in metaphase of the second meiotic division (choice C). The cell's metabolic operations have been discontinued and the oocyte drifts in a state of "suspended animation," awaiting the necessary stimulus for further development. If fertilization does not occur, the oocyte disintegrates without completing meiosis.
An abrasion results in the total loss of epidermis over a large area of an arm, but one month later, the abrasion has healed, with regrowth of the epidermis. Which of the following mechanisms accounts for the restoration of the epidermis over the abraded area?
A. Growth of epidermis from hair follicles and sweat glands in the dermis
B. Migration of endothelial cells from newly grown capillaries
C. Transformation of dermal fibroblasts into epidermal cells
D. Transformation of macrophages into epidermal cells
E. Transformation of melanocytes into epidermal cells




The correct answer is A. The dermis contains skin appendages (e.g., hair follicles), which contain epithelial stem cells. In the process of healing a large area where the epidermis has been lost but the dermis is intact, re-epithelialization occurs by growth of epidermal cells from the underlying skin appendages, as well as from the intact epidermis along the wound edges. Physiologically, the dermis lies beneath the epidermis. It has two major components, a superficial papillary layer and a deeper reticular layer. The papillary layer contains the capillaries and the sensory neurons, which supply the surface of the skin. The reticular layer consists of an interwoven meshwork of dense irregular connective tissue.

None of the other cell types are known to directly contribute to the regeneration of epidermis over abraded skin.
Hirschsprung's disease indicates a developmental abnormality in which of the following embryonic tissues?
A. Ectoderm
B. Endoderm
C. Neural crest
D. Neural ectoderm
E. Splanchnic mesoderm




The correct answer is C. The baby has Hirschsprung's disease, which is due to an absence of ganglion cells in the wall of the colon. Neural crest cells contribute to the formation of many adult structures. Among these are all of the postganglionic neurons of the autonomic nervous system and the sensory neurons of the peripheral nervous system.

Ectoderm (choice A) forms the epidermis of the skin and the parenchymal cells of glands associated with the skin such as the sweat glands, sebaceous glands, and mammary glands.

Endoderm (choice B) forms the epithelial lining of the gut tube and the parenchymal cells of glands associated with the gut tube, such as the liver and pancreas.

Neural ectoderm (choice D) forms the central nervous system, the somatic motor neurons of the peripheral nervous system, and the preganglionic neurons of the autonomic nervous system.

Splanchnic mesoderm (choice E) forms the visceral peritoneum, visceral pleura, visceral pericardium, and the stroma and muscle of the wall of the gut, among other structures.
Injury at the lower border of a rib will most likely damage which structure?
A. intercostal artery
B. intercostal nerve
C. intercostal vein
D. internal intercostal muscle




The correct answer is B. The three structures in the intercostal space are, from superior to inferior, the intercostal vein, artery, and nerve. The proper site for insertion of an intercostal drain is superior to a rib, not directly at the level of the superior border but slightly higher to avoid the collateral branches of the nerve, artery, and vein. The nerve is the most inferior structure and thus most likely to be damaged by the drain (producing an anesthetic dermatome). In addition, the nerve is the least protected by the costal groove.

The intercostal artery (choice A) is the middle structure and thus is not the most likely to be damaged.

The intercostal vein (choice C) is the most superior structure and is thus the least likely to be damaged. In addition, the vein is the structure best protected by the costal groove (the further superior the structure, the more protected by the costal groove).

The internal intercostal fibers (choice D) will tend to be separated by the drain. In addition, the internal intercostal muscle is membranous (internal intercostal membrane) posteriorly, from the neck of the rib to its angle; therefore, many drains, when inserted, will not even pass through a layer of internal intercostal muscle.
Following the healing of a tibial fracture, a patient forms a small, new piece of viable bone near the joint (heterotopic ossification), which is interfering with joint mobility. Which of the following is the source of the osteoblasts that formed this bone?
A. Cancellous bone near the fracture
B. Circulating stem cells
C. Compact bone near the fracture
D. Marrow
E. Periosteum




The correct answer is E. The source of bone-forming cells following a fracture is the damaged periosteum. Occasionally, some of these cells become "lost" and can form small pieces of inappropriately located bone that impair joint movement. Except in joint cavities, where they are covered by a layer of hyaline cartilage, bone surfaces are covered by the periosteum. The periosteum is composed of an outer fibrous and inner cellular layer. The periosteum assists in the attachment of a bone to surrounding tissues and associated tendons and ligaments.

Surprisingly, neither the cancellous (choice A) nor the compact (choice C) bone near the fracture is the source of the bone-forming cells.

Circulating stem cells (choice B) can repopulate damaged bone marrow (choice D), but neither the bone marrow nor the circulating stem cells is the source of the bone-forming cells.
A patient has a severe headache, and is unable to move his right leg. There is no higher cortical function loss, but deep deep tendon reflexes and Babinski's sign are absent. The medial aspect of the cerebral hemispheres is most affected. Which artery is most likely involved?
A. left anterior cerebral artery
B. left middle cerebral artery
C. left posterior cerebral artery
D. right anterior cerebral artery
E. right posterior cerebral artery




The correct answer is A. The medial aspect of the cerebral hemispheres is supplied by the anterior cerebral arteries. The small portion of primary motor cortex located here sends fibers to spinal-cord segments innervating the lower limbs. Because the corticospinal tracts cross over to the opposite side in the medulla oblongata, the left cerebral hemisphere controls the right side of the body. Hence, paralysis of the right leg results from loss of blood flow to the portion of the left hemisphere supplied by the left anterior cerebral artery.

The middle cerebral artery (choice B) supplies the lateral convexity of the brain. Primary motor cortices on the lateral aspects of the hemispheres send fibers to the brainstem (innervating the face) and to spinal-cord segments innervating the upper limbs, the trunk, and the proximal part of the lower extremities. As with the anterior cerebral artery territories, the right hemisphere innervates the left body, and the left hemisphere innervates the right body.

The posterior cerebral artery (choices C and E) supplies the occipital cortex; interruption of blood flow through this artery would probably produce visual deficits rather than paralysis.

The right anterior cerebral artery (choice D) supplies blood to the medial aspect of the right cerebral hemisphere, which controls the left leg. This patient has paralysis of the right leg.
An infant is born with an abnormally developed falciform ligament. The hepatogastric and hepatoduodenal ligaments are also malformed. These developmental anomalies are most likely due to abnormal development of the
A. dorsal mesoduodenum
B. dorsal mesogastrium
C. pericardioperitoneal canal
D. pleuropericardial membranes
E. ventral mesentery




The correct answer is E. The ventral mesentery forms the falciform ligament, ligamentum teres, and lesser omentum, which can be divided into the hepatogastric and hepatoduodenal ligament.

The dorsal mesoduodenum (choice A) is the mesentery of the developing duodenum, which later disappears so that the duodenum and pancreas lie retroperitoneally.

Both omental bursa and the greater omentum are derived from the dorsal mesogastrium (choice B), which is the mesentery of the stomach region.

The pericardioperitoneal canal (choice C) embryologically connects the thoracic and peritoneal canals.

The pleuropericardial membranes (choice D) become the pericardium and contribute to the diaphragm.
Which of the following structures is lined with epithelium derived from mesoderm of the ureteric bud?
A. Bowman's capsule
B. Distal convoluted tubule
C. Loop of Henle
D. Proximal convoluted tubule
E. Ureter




The correct answer is E. The transitional epithelium that lines the ureter, the renal pelvis, and the major and minor calyces is derived from mesoderm of the ureteric bud, as is the cuboidal epithelium of the collecting tubules.

The simple squamous epithelium lining Bowman's capsule (choice A) is derived from mesoderm of the metanephric vesicle.

The simple cuboidal epithelium lining the distal convoluted tubule (choice B) is derived from mesoderm of the metanephric vesicle.

The simple squamous epithelium lining the loop of Henle (choice C) is derived from mesoderm of the metanephric vesicle.

The proximal convoluted tubule (choice D) is lined with simple columnar epithelium derived from mesoderm of the metanephric vesicle.
A patient's left hypoglossal nerve (CN XII) is injured during a carotid endarterectomy. Which of the following would most likely result from this injury?
A. Decreased gag reflex on the left
B. Decreased salivation from the left submandibular and sublingual salivary glands
C. Deviation of the tongue to the left on protrusion
D. Inability to elevate the pharynx on the left during swallowing
E. Inability to perceive sweet and salt taste sensation on the anterior part of the left side of the tongue




The correct answer is C. The hypoglossal nerve is a pure motor nerve (general somatic efferent) to the intrinsic and most extrinsic muscles of the tongue. If the nerve is damaged, denervation atrophy of the affected side will permit the intact musculature of the opposite side to operate unopposed, thereby protruding the tongue to the side of the injury.

The gag reflex (choice A) is mediated by the glossopharyngeal nerve (CN IX; afferent limb) and the vagus nerve (CN X; efferent limb).

Choice B is incorrect because the preganglionic parasympathetic fibers that regulate these two salivary glands are carried by the chorda tympani (which joins with the lingual nerve) to the submandibular ganglion. Postganglionic fibers are then distributed to these glands.

The muscles responsible for elevation of the pharynx (choice D) are innervated primarily by the vagus nerve (CN X).

Choice E is incorrect because the taste fibers for the anterior two thirds of the tongue are carried via the chorda tympani to the facial nerve (CN VII) and hence to the brainstem.
A patient with paresthesia of the medial side of the hand and forearm undergoes surgery to correct it. If the anterior scalene muscle is cut during this surgery, what nearby nerve must be avoided?
A. Phrenic nerve
B. Recurrent laryngeal nerve
C. Superior laryngeal nerve
D. Suprascapular nerve
E. Vagus nerve




The correct answer is A. The patient is suffering from scalene triangle syndrome (thoracic outlet syndrome). The lower trunk of the brachial plexus and the subclavian artery are being compressed between the anterior scalene muscle and the middle scalene muscle. Incision of the anterior scalene muscle will relieve this compression. The phrenic nerve lies on the anterior surface of the anterior scalene muscle deep to the prevertebral fascia. If this nerve is cut, the diaphragm on that side of the body will be paralyzed. Anatomically, the phrenic nerve is one of the nerves of the cervical plexus. This nerve distributes to the diaphragm other nerves of the cervical plexusm including the ansa cervicalis, lesser occiptital, transverse cervical, supraclavical, and greater auricular nerves, as well as the cervical nerves.

The recurrent laryngeal nerve (choice B) is a branch of the vagus nerve. On the left side it recurs around the aortic arch, and on the right side it recurs around the right subclavian artery. It ascends to the larynx in the tracheoesophageal groove. The nerve innervates several laryngeal muscles and the laryngeal mucosa inferior to the vocal folds. It is not in contact with the anterior scalene muscle.

The superior laryngeal nerve (choice C) is a branch of the vagus nerve that arises just after the vagus nerve passes through the jugular foramen. It innervates the cricothyroid muscle of the larynx and the laryngeal mucosa above the vocal folds. The superior laryngeal nerve is not in contact with the anterior scalene muscle.

The suprascapular nerve (choice D) is a branch of the upper trunk of the brachial plexus. It arises from the upper trunk after the upper trunk has passed between the anterior and middle scalene muscles. The suprascapular nerve innervates the supraspinatus and infraspinatus muscles.

The vagus nerve (choice E) lies within the carotid sheath within the neck. The carotid sheath is anterior to the prevertebral fascia. The vagus nerve is not in contact with the anterior scalene muscle.
Compression of a cranial nerve by a large aneurysm of the right superior cerebellar artery immediately distal to its origin from the basilar artery would cause which of the following clinical findings?
A. Loss of abduction of the right eye
B. Loss of adduction of the right eye
C. Loss of depression of the right eye from the adducted position
D. Loss of sensation on the right side of the face
E. Loss of visual field of the right eye




The correct answer is B. The oculomotor nerve emerges from the interpeduncular fossa of the midbrain and then passes between the superior cerebellar artery and the posterior cerebral artery immediately lateral to the basilar artery. Aneurysm of any of these three arteries may compress the nerve. The oculomotor nerve innervates a number of extraocular muscles in the orbit, including the medial rectus muscle, which is responsible for adduction of the eye.

Abduction of the eye (choice A) is accomplished by the lateral rectus muscle. This muscle is innervated by the abducens nerve, which arises more caudally from the brainstem and is not in contact with this artery.

Depression of the eye from the adducted position (choice C) is accomplished by the superior oblique muscle. This muscle is innervated by the trochlear nerve, which emerges from the dorsal surface of the midbrain and passes lateral to the cerebral peduncle before entering the cavernous sinus. A superior cerebellar artery aneurysm would not compress this nerve.

Sensation on the face (choice D) is mediated by the trigeminal nerve. This nerve arises from the anterolateral surface of the pons and is not in contact with this artery.

Loss of visual field (choice E) results from lesions of the optic nerve or other elements of the visual pathway. These are not in contact with this artery.
The two most important hormones controlling Calcium metabolism are secreted from the:
A. Parathyroid and thyroid glands
B. Thyroid and thymus glands
C. Adenohypophysis and parathyroid glands
D. Adrenal cortex and thyroid glands
E. Parathyroid and adrenal glands




The correct answer is A.PTH, or parathyroid hormone, is considered to be the most important hormone controlling calcium metabolism. It increases serum calcium by increasing bone mineral resorption by osteoclasts, increasing digestive absorption of calcium, and decreasing calcium excretion by the kidney. PTH is secreted by principal cells of the parathyroid. The antagonist hormone, calcitonin, is secreted by parafollicular cells of the thyroid (follicular cells secrete thyroxin). It reduces serum calcium by promoting calcium depositon in bones, decreasing digestive absorption of calcium, and promoting calcium excretion. Note that the term adenohypophysis in answer choice C refers to the anterior pituitary. This term and neurohypophysis (posterior pituitary) are often used on NBDE.
During a cranial nerve test, the patient cannot elevate her right eye from the abducted position. Which of the following muscles is paralyzed?
A. Right inferior oblique
B. Right inferior rectus
C. Right lateral rectus
D. Right superior oblique
E. Right superior rectus




The correct answer is E. The superior rectus muscle can elevate and adduct the eye from the neutral position. From the abducted position, it is the only muscle that can elevate the eye.

The inferior oblique muscle (choice A) can elevate and abduct the eye from the neutral position. From the adducted position, it is the only muscle that can elevate the eye.

The inferior rectus muscle (choice B) can depress and adduct the eye from the neutral position. From the abducted position, it is the only muscle that can depress the eye.

The lateral rectus muscle (choice C) can abduct the eye.

The superior oblique muscle (choice D) can depress and abduct the eye from the neutral position. From the adducted position, it is the only muscle that can depress the eye.
Several arterial branches penetrate into the distal end of the lesser curvature of the stomach. Which of the following arteries usually supplies these branches?
A. Left gastric
B. Left gastroepiploic
C. Right gastric
D. Right gastroepiploic
E. Short gastric




The correct answer is C. The arterial supply of the stomach is complex; it therefore has a good potential to appear on the NBDE. The right gastric artery supplies the distal lesser curvature.

The left gastric artery (choice A) supplies the proximal lesser curvature.

The left gastroepiploic artery (choice B) supplies the proximal greater curvature below the splenic artery.

The right gastroepiploic artery (choice D) supplies the distal greater curvature.

The short gastric artery (choice E) supplies the proximal greater curvature above the splenic artery.
Contraction of which of the following muscles contributes most to the backward movement of the lower jaw during the process of mastication?
A. Digastric
B. Lateral pterygoid
C. Medial pterygoid
D. Mylohyoid
E. Temporalis




The correct answer is E. Mastication is a complex process involving alternating elevation, depression, forward movement, and backward movement of the lower jaw. The backward-movement step is accomplished by the posterior fibers of the temporalis muscle. The temporalis originates along the temporal lines of the temporal bone and inserts into the coronoid process of the mandible. It is innervated by the manibular branch of the trigeminal nerve.

The digastric (choice A) helps to depress (open) the lower jaw during chewing.

The lateral pterygoid (choice B) helps to move the lower jaw forward, laterally, or downward during chewing.

The medial pterygoid (choice C) helps to elevate (close) the lower jaw during chewing.

The mylohyoid (choice D) helps to depress (open) the lower jaw during chewing.
Most fascia of the body that attach to bones attach by which of the following mechanisms?
A. Blending with the covering periosteum
B. Inserting deeply into the cancellous bone
C. Inserting deeply into the cartilage
D. Inserting deeply into the diaphysis
E. Inserting deeply into the marrow




The correct answer is A. Fascial straps (retinacula) and fascial coverings of muscles or muscle groups characteristically attach to nearby bones by blending with the covering periosteum. No deep attachments are usually made by fascia.

Cancellous bone (choice B) is spongy bone, which is usually found in marrow and is not the site for fascial attachment.

Fascia do not usually attach to cartilage (choice C).

Fascia attaches to bony shafts, or diaphyses (choice D), superficially via the periosteum.

Fascia do not penetrate the bone to reach the marrow (choice E).
A newborn baby is noted to have a left unilateral cleft lip. There are no abnormalities of the baby's palate. Which of the following developmental defects accounts for this occurrence?
A. Failure of the left lateral palatine process to fuse with the median palatine process
B. Failure of the left maxillary prominence to unite with the left medial nasal prominence
C. Failure of the primary palate to fuse with the secondary palate
D. Failure of the right and left medial nasal prominences to merge
E. Failure of the right palatine process to fuse with the left palatine process




The correct answer is B. In the formation of the upper lip, the right and left medial nasal prominences merge to form the philtrum of the upper lip. The lateral maxillary prominence then merges with the merged medial nasal prominences. Failure of this merger to occur results in a unilateral cleft lip.

Failure of a lateral palatine process to fuse with the median palatine process (choice A), which is synonymous with a failure of the primary palate to fuse with the secondary palate (choice C), results in a primary cleft palate. Primary clefts of the palate are found anterior to the incisive foramen of the palate. Primary cleft palates may occur along with cleft lips but are the result of a separate developmental defect.

Failure of the right and left medial nasal prominences to merge (choice D) results in a median cleft lip. This is a rare anomaly. Normally, the right and left medial nasal prominences merge into a single prominence that forms the philtrum of the lip.

Failure of the right and left palatine processes to fuse (choice E) results in a secondary cleft palate. Secondary cleft palates are found posterior to the incisive foramen. Normally, the right and left palatine processes fuse together and fuse to the primary palate.
A patient experiences visual difficulties. When a light is shined in her right eye, there is no pupillary response in either eye. However, upon shining a light in her left eye, both ipsilateral and contralateral pupillary responses are apparent. Her extraocular movements are intact. What is the most likely location of her lesion?
A. Oculomotor nerve, left side
B. Oculomotor nerve, right side
C. Optic nerve, left side
D. Optic nerve, right side
E. Trochlear nerve, left side




The correct answer is D. Know your cranial nerves! This woman has a "Marcus-Gunn pupil" with a defect in the afferent pathway of the optic nerve (in this case, on the right side). Recall that the afferent limb of the pupillary light reflex is the optic nerve (CN II); the efferent limb is the oculomotor nerve (CN III; parasympathetic fibers). When light is shined into her right eye, because her right optic nerve is not functioning properly, the light signal is not transmitted to the central nervous system (CNS), resulting in no pupillary response. As light is shined into her left eye, the left optic nerve transmits the signal to the CNS, which then sends an outbound signal through both the right and left oculomotor nerves to cause pupillary constriction in both eyes. Anatomically, the optic nerve is a special sensory nerve that originates in the retina of the eye, passes through the optic foramen of the sphenoid bone, and has a destination in the diencephalon via the optic chiasm.

The oculomotor nerve (choices A and B) innervates all extraocular muscles except the lateral rectus (innervated by the abducens nerve) and the superior oblique (innervated by the trochlear nerve choice E). The oculomotor nerve also mediates pupillary constriction (parasympathetic fibers), eyelid opening (levator palpebrae), and innervates the ciliary muscle (allowing accommodation).

A patient with a lesion of the left optic nerve (choice C) would have no pupillary responses in either eye when shining a light in the left eye; pupillary responses would be present in both eyes when shining a light in the right eye.
If the tongue deviates to the right side when protruded, the most likely cause is paralysis of which of the following muscles?
A. Left genioglossus
B. Left hyoglossus
C. Left palatoglossus
D. Right genioglossus
E. Right hyoglossus




The correct answer is D. The genioglossus muscle is innervated by the hypoglossal nerve. The function of the genioglossus muscle is to pull the tongue forward (protrude) and toward the opposite side. When the right genioglossus muscle is paralyzed, the left genioglossus muscle pulls the tongue forward and to the right.

If the left genioglossus muscle were paralyzed (choice A), the tongue would deviate toward the left on protrusion because of the unopposed action of the right genioglossus muscle. The left genioglossus muscle is innervated by the left hypoglossal nerve.

The hyoglossus muscles (choices B and E) are innervated by the hypoglossal nerves. The function of these muscles is to retract the tongue. These muscles are not active during protrusion of the tongue.

The palatoglossus muscle (choice C) are innervated by the vagus nerves rather than the hypoglossal nerves. Their function is to pull the tongue back (retract) and upward toward the palate.
Which of the following is a derivative of the second pharyngeal arch?
A. Eustachian tube
B. External auditory meatus
C. Palatine tonsil
D. Stylohyoid muscle
E. Tensor tympani




The correct answer is D. The stylohyoid is derived from the second pharyngeal arch, which also gives rise to the muscles of facial expression, the stapedius, the posterior belly of the digastric muscle, Reichert's cartilage, and the facial nerve.

The Eustachian tubes (choice A) are derived from the first pharyngeal pouch, which also gives rise to the middle ear cavity and the inner epithelial lining of the tympanic membrane.

The external auditory meatus (choice B is derived from the first pharyngeal cleft, which also gives rise to the outer epithelial lining of the tympanic membrane.

The palatine tonsil (choice C) is derived from the epithelial lining of the second pharyngeal pouch.

The tensor tympani (choice E) is derived from the first pharyngeal arch, which also gives rise to the muscles of mastication, the anterior belly of the digastric muscle, the mylohyoideus, the tensor veli palantini, the maxillary and mandibular bones, and the maxillary and mandibular divisions of the trigeminal nerve.

Note that knowing the embryology of these structures helps you remember the innervation of the muscles of the face.

First pharyngeal arch → muscles of mastication → trigeminal nerve

Second pharyngeal arch → muscles of facial expression → facial nerve
Damage to the parasagittal region and falx cerebri will most likely result in which of the following neurologic deficits?
A. Altered taste
B. Leg paralysis
C. Loss of facial sensation
D. Ptosis
E. Unilateral deafness




The correct answer is B. A meningioma of the parasagittal region and the falx cerebri would be located at the top of the brain, near the midline. In this position, it could compress the sensory or motor cortex supplying the lower extremities. The falx cerebri is a fold of dura mater that projects between the cerebral hemispheres in the longitudinal tissues. Its interior portions attach anteriorly to the crista galli and posteriorly to the internal occipital crest.

Taste (choice A) is supplied by cranial nerves VII, IX, and X. These nerves arise from the brainstem.

Facial sensation (choice C) is supplied by cranial nerve V, the nuclei of which are in the brainstem. Furthermore, the area of the sensory cortex that subserves the face is on the lateral aspect of the cortex and would not be affected by a tumor in the parasagittal region.

Ptosis (choice D) can be caused by a deficit in cranial nerve III, which arises from the brainstem.

Unilateral deafness (choice E) suggests damage to cranial nerve VIII, which arises from the brainstem.
In which of the following structures are the opacifications located with cataract formation?
A. Aqueous humor
B. Cornea
C. Lens
D. Optic nerve
E. Retina




The correct answer is C. Cataracts are lens opacifications. It is not known whether senile cataracts represent disease or normal opacification with age. Cataracts may occur as a consequence of diabetes mellitus, long-term steroids, or congenital infections. They are successfully treated at present with lens extraction and implantation of prosthetic lenses. The diagnostic characteristics for cataracts are as follows: 1) blurred vision that is progressive over months to years, 2) no pain or redness is seen, and 3) lens opacities may be invisible or grossly visible.

Aqueous humor (choice A) is continually replaced due to active secretion by the ciliary body. As such, it does not undergo opacification; it is in constant flux.

Corneal opacification (choice B) is generally a consequence of squamous metaplasia, in which the transparent, nonkeratinized, squamous cells are replaced by opaque, keratinized, squamous cells. Squamous metaplasia is a reparative process, usually due to friction injury to the cornea or a vitamin A deficiency.

The optic nerve (choice D) is not transparent, and it does not undergo opacification injury. It may, however, atrophy due to ischemic, traumatic, infective, or metabolic insults.

The retina (choice E) consists of multiple layers of neural cells. The retina is transparent but is not the site of cataract formation.
A sharp instrument passing through the superior orbital fissure would most likely sever which of the following structures?
A. Abducens nerve
B. Facial nerve
C. Mandibular nerve
D. Maxillary nerve
E. Middle meningeal artery




The correct answer is A. A good way to remember what passes through the superior oribital fissure is that everything that innervates the eye, other than the optic nerve, passes through this fissure. This incudes the oculomotor nerve(CN III), the trochlear nerve (CN IV), the ophthalmic nerve (V1), and the abducens nerve (CN VI).

The facial nerve (CN VII; choice B) passes through the internal auditory meatus.

The mandibular nerve (V3; choice C) passes through the foramen ovale.

The maxillary nerve (V2; choice D) passes through the foramen rotundum.

The middle meningeal artery (choice E) passes through the foramen spinosum.
A knife wound to the neck damages the posterior cord of the brachial plexus. Which of the following muscles would be most likely be paralyzed?
A. Deltoid
B. Flexor carpi ulnaris
C. Flexor digitorum superficialis
D. Flexor pollicis brevis
E. Palmaris longus




The correct answer is A. The posterior cord supplies the axillary and radial nerves. Of the muscles listed, only the deltoid is supplied by one of these two nerves, specifically the axillary nerve. The deltoid originates from the clavicle and scapula. It inserts into the deltoid tuberosity of the humerus. This muscle is responsible for abduction of the arm.

The flexor carpi ulnaris (choice B) is supplied by the ulnar nerve.

The flexor digitorum superficialis (choice C), the flexor pollicis brevis (choice D), and the palmaris longus (choice E) are supplied by the median nerve.
During the process of meiosis, a single homologous chromosome pair fails to separate during the first meiotic division. This failure would be most likely to produce which of the following conditions if fertilization occurs and an embryo later develops?
A. Balanced translocation
B. Triploidy
C. Trisomy
D. Unbalanced translocation
E. Uniploidy




The correct answer is C. Meiosis is cell division that produces gametes with half of the normal somatic chromosome complement. The process described is nondisjunction, which will cause one daughter cell to have 24 chromosomes, while the other will have 22 chromosomes. When a gamete with the normal 23 chromosomes combines at fertilization with a gamete with 22 or 24 chromosomes, the embryo will have 47 chromosomes (trisomy) or 45 chromosomes (monosomy). Nondisjunction can occur in either the first or second meiotic division.

Balanced translocation (choice A) occurs when non-homologous chromosomes exchange genetic material in such a way that no critical genetic material is lost.

Triploidy (choice B) is the term used when a cell has 69 chromosomes (3N or 3 sets), and can occur in tumors or when an egg is fertilized by two sperm.

An unbalanced translocation (choice D) occurs when non-homologous chromosomes exchange genetic material with a net loss or gain of critical genetic material.

Uniploidy (choice E) is the state of having 23 chromosomes, seen normally in sperm and eggs.
The superior ophthalmic vein directly communicates with which of the following dural venous sinuses?
A. Cavernous sinus
B. Occipital sinus
C. Sigmoid sinus
D. Superior petrosal sinus
E. Straight sinus




The correct answer is A. The anterior continuation of the cavernous sinus, the superior ophthalmic vein, passes through the superior orbital fissure to enter the orbit. Veins of the face communicate with the superior ophthalmic vein. Because of the absence of valves in emissary veins, venous flow may occur in either direction. Cutaneous infections may be carried into the cavernous sinus and result in a cavernous sinus infection, which may lead to an infected cavernous sinus thrombosis. The cavernous sinus is lateral to the pituitary gland and contains portions of cranial nerves III, IV, V1, V2, and VI, and the internal carotid artery.

The occipital sinus (choice B) is at the base of the falx cerebelli in the posterior cranial fossa. It drains into the confluence of sinuses.

The sigmoid sinus (choice C) is the anterior continuation of the transverse sinus in the middle cranial fossa. The sigmoid sinus passes through the jugular foramen and drains into the internal jugular vein.

The superior petrosal sinus (choice D) is at the apex of the petrous portion of the temporal bone and is a posterior continuation of the cavernous sinus. The superior petrosal sinus connects the cavernous sinus with the sigmoid sinus.

The straight sinus (choice E) is at the intersection of the falx cerebri and the falx cerebelli in the posterior cranial fossa. The straight sinus connects the inferior sagittal sinus with the confluence of sinuses.
In preparation for a procedure to remove the fingernail on an index finger, the physician would most likely anesthetize a branch of the
A. anterior interosseus nerve
B. median nerve
C. musculocutaneous nerve
D. radial nerve
E. ulnar nerve




The correct answer is B. The median nerve supplies the surface of the lateral palm, the palmar surface of the first three digits, and the distal dorsal surface of the index and middle fingers (including the nail beds). Therefore, prior to performing surgery in this area, it is essential to anesthetize a branch of this nerve (possibly a proper digital branch) to eliminate pain sensation around the nail bed of the index finger. The median nerve of the branchial plexus distributes to the flexor muscles on the forearm (flexor carpi radialis and palmaris longus), the pronators (p. quadratus and p. teres), digital flexors, and skin over the lateral surface of the hand.

Neither the anterior interosseus (choice A) nor the musculocutaneous (choice C) nerves supplies the hand. The anterior interosseous nerve supplies the flexor pollicis longus, the lateral half of flexor digitorum profundus, and pronator quadratus. The musculocutaneous nerve supplies the coracobrachialis, biceps, and most of the brachialis muscle, then becomes the lateral cutaneous nerve of the forearm.

The radial nerve (choice D) supplies skin on the radial side of the dorsal surface of the hand, but not the fingertips.

The ulnar nerve (choice E) supplies the palmar and dorsal surfaces of the medial hand, including the palmar and dorsal surfaces of the fourth and fifth digits.
Which glandular area secretes hormones which are products of tyrosine metabolism?
A. Alpha cells of pancreas
B. Beta cells of pancreas
C. Adrenal cortex
D. Adrenal medulla
E. Testes




The correct answer is D.The products of the adrenal medulla are epinephrine (adrenalin) and norepinephrine (noradrenalin). The pathway of production of these compunds is a s follows: tyrosine to DOPA to dopamine to norepinephrine to epinephrine. The hormone secreted by alpha cells of the pancreas is glucagon, while beta cells secrete insulin. Both are peptide hormones. The adrenal cortex secretes a variety of hormones including cortisol and aldosterone, both steroid hormones. The testes secrete testosterone, also a steroid hormone.
A newborn infant has some of its abdominal viscera protruding through a defect in the abdominal wall. Which of the following is the likely cause of this defect?
A. Failure of the intestinal loop to retract from the umbilical cord
B. Failure of the yolk stalk to degenerate
C. Failure of peritoneal fusion
D. Incomplete fusion of the lateral body folds
E. Umbilical herniation




The correct answer is D. During the fourth week of development, the lateral body folds move ventrally and fuse in the midline to form the anterior body wall. Incomplete fusion results in a defect that allows abdominal viscera to protrude from the abdominal cavity, a condition known as gastroschisis.

During development, the midgut normally herniates into the umbilical cord and then subsequently retracts into the abdominal cavity. Failure of the intestinal loop to retract from the umbilical cord (choice A) results in omphalocele.

Failure of the yolk stalk to degenerate (choice B) results in an ileal (Meckel's) diverticulum or a vitelline fistula or cyst. In the early embryo, the gut tube is connected to the yolk sac by a narrow connection known as the yolk stalk. Normally, this connection degenerates.

During development, certain peritoneal organs fuse with the posterior abdominal wall to become secondarily retroperitoneal. Failure of this peritoneal fusion (choice C) will result in certain organs that are normally immobile being mobile (e.g., mobile cecum).

Umbilical herniation (choice E) results from abdominal viscera protruding through a weakness in the abdominal wall after development. Such protrusions are covered by subcutaneous fascia and skin, distinguishing them from gastroschisis.
If a patient has a drooping right eyelid and a dilated right pupil, which of the following neural structures is most likely affected?
A. Cervical sympathetic chain
B. Facial nerve
C. Oculomotor nerve
D. Superior cervical ganglion
E. Trigeminal nerve




The correct answer is C. The oculomotor nerve innervates the levator palpebrae superioris, which elevates the eyelid. This nerve also innervates the inferior oblique muscles, as well as the superior, inferior, and medial rectus muscles. The oculomotor nerve also contains preganglionic parasympathetic fibers that synapse, in the ciliary ganglion, on postganglionic parasympathetic nerve fibers that innervate the sphincter pupillae muscle, which constricts the pupil. A lesion of the oculomotor nerve may therefore result in both drooping of the eyelid (ptosis) and dilation of the pupil (mydriasis).

The cervical sympathetic chain (choice A) contains preganglionic sympathetic nerve fibers, arising from the upper thoracic spinal cord, which ascend to the cervical sympathetic ganglia. A lesion of these nerves may result in Horner's syndrome, which includes a ptosis and miosis (pupillary constriction) and, often, anhidrosis (lack of sweating).

The facial nerve (choice B) innervates the muscles of facial expression, including the orbicularis oculi muscle. A lesion of this nerve may therefore result in the inability to close the eye.

The superior cervical ganglion (choice D) contains the cell bodies of postganglionic sympathetic nerves that innervate structures in the head. A lesion of this structure will cause Horner's syndrome.

The trigeminal nerve (choice E) provides sensory innervation to much of the head. A lesion of this nerve may interfere with the corneal blink reflex.
An injection to anesthetize pain from a fracture of the seventh rib should be gien in what area?
A. Seventh intercostal space immediately below the seventh rib in the midclavicular line
B. Seventh intercostal space immediately below the seventh rib just lateral to the angle of the rib
C. Seventh intercostal space immediately below the seventh rib just medial to the angle of the rib
D. Sixth intercostal space immediately above the seventh rib in the midclavicular line
E. Sixth intercostal space immediately above the seventh rib just lateral to the angle of the rib




The correct answer is B. The seventh intercostal nerve (the anterior ramus of the seventh thoracic spinal nerve) innervates the seventh rib. After passing through the intervertebral foramen between the seventh and eighth thoracic vertebrae, the nerve lies in the seventh intercostal space. After passing the angle of the rib, it occupies a position along the lower border of the rib, in the costal groove. Use of a local anesthetic at this point will anesthetize the rib.

By the time the intercostal nerve has reached the midclavicular line (choice A), it has already innervated most of the rib. Use of an anesthetic at this point would not be effective.

While the intercostal nerve is in the intercostal space medial to the angle of the rib (choice C), it is not along the lower border of the rib.

The intercostal nerve does not lie along the upper border of the rib (choices D and E). Thus, injection at these sites would not be effective.
The nucleus that lies immediately medial (and deep) to the uncus is the
A. amygdala
B. caudate nucleus
C. claustrum
D. hippocampus
E. putamen




The correct answer is A. The uncus, which is the medial protrusion of the parahippocampal gyrus, is an external structure seen on the ventral surface of the temporal lobe. The amygdala is a collection of nuclei that lies directly beneath the uncus.

The caudate nucleus (choice B) is a deep nuclear structure that lies lateral to the lateral ventricles.

The claustrum (choice C) is a thin and elongated nucleus that lies just medial to the insular cortex.

The hippocampus (choice D) is a nuclear structure that lies in the interior of the parahippocampal gyrus.

The putamen (choice E) is a nuclear structure that resides lateral to the caudate and medial to the claustrum.
A nursing home patient who aspirates while lying on his back would be most likely to develop pneumonia involving which of the following sites?
A. Anterior segment of the right upper lobe
B. Apical segment of the right lower lobe
C. Inferior lingular segment of the left upper lobe
D. Lateral segment of the right middle lobe
E. Superior lingular segment of the left upper lobe




The correct answer is B. Aspiration pneumonia is a common complication observed in nursing home patients. The most probable site of the pneumonia can be anticipated by knowing the anatomy of the bronchial tree because the aspirated fluid usually flows downhill. In a supine or nearly supine patient, the fluid flows into the trachea and then into either of the (typically the right) main bronchi. The first posteriorly located branch is the one leading to the apical aspect of (either) lower lobe. The lateral and posterior segments of the lower lobes are also supplied by posteriorly branching segmental bronchi. In contrast, the posterior aspects of the upper lobes are somewhat protected by an initial anteriorly directed bifurcation before their segmental bronchi arise. All other segments of the bronchial tree and their corresponding portions of lung are more anterior.
The hormone most responsible for regulating sodium balance is secreted from:
A. Zona glomerulosa of the adrenal cortex
B. Zona reticularis of the adrenal medulla
C. Zona fasiculata of the adrenal medulla
D. Zona fasiculata of the adrenal cortex
E. Zona glomerulosa of the adrenal medulla




The correct answer is choice A.Firstly, the hormone involved is aldosterone, which acts to increase sodium resorption in the kidney. Note that aldosterone (a mineralcorticoid) and the glucocorticoids (cortisol, cortisone) are produced by the adrenal cortex, not medulla. The medulla, with a different developmental origin and cell type, produces catecholamines such as epinephrine and norepinephrine. The cortex, which is outside the medulla, is in three regions. On the outside is the Zona Glomerulosa, source of the mineralcorticoids. Inside of that is the Zona Fasiculata, which together with the innermost layer of the cortex, the Zona Reticularis, produce glucocortoids. Remember that interior to the Zona Reticularis, you will find the adrenal medulla. Also note that as a memory aid, the cortex layers from the outside in are G-F-R, like the GFR of the kidney.
During embryological development, hematopoiesis occurs in different organs at different times. Which of the following are the correct organs, in the correct sequence, at which hematopoiesis occurs embryologically?
A. Amnion, yolk sac, placenta, bone marrow
B. Placenta, liver and spleen, yolk sac, bone marrow
C. Placenta, spleen and lymphatic organs, bone marrow
D. Yolk sac, bone marrow, liver and spleen
E. Yolk sac, liver, spleen and lymphatic organs, bone marrow




The correct answer is E. By the third week of development, hematopoiesis begins in the blood islands of the yolk sac. Beginning at 1 month of age and continuing until 7 months of age, blood elements are also formed in the liver. Hematopoiesis occurs in the spleen and lymphatic organs between 2 and 4 months, and in the bone marrow after 4 months.
A woman suffers a fracture of the left tenth and eleventh ribs. Which of the following organs is most likely to have been injured by these fractured ribs?
A. Descending colon
B. Jejunum
C. Left adrenal gland
D. Left kidney
E. Spleen




The correct answer is E. The spleen is a soft, friable organ with a thin capsule and is subject to injury upon trauma to the left side of the abdomen. It is located in the upper left quadrant of the abdomen, deep to the left ninth, tenth, and eleventh ribs. It is the most commonly injured organ in the abdomen. The adult spleen contains the largest number of lymphoid tissues in the human body.

The descending colon (choice A) lies in a retroperitoneal position on the left side of the posterior abdominal wall. The descending colon begins at the splenic flexure immediately inferior to the spleen.

The jejunum (choice B) is a peritoneal structure suspended by a long mesentery. It is located primarily in the upper left quadrant of the abdomen. Its long mesentery allows the jejunum to be highly mobile and thus is not likely to be injured by trauma to the body wall.

The left adrenal gland (choice C) is a retroperitoneal structure that lies near the upper pole of the left kidney. It is embedded within fat and is thus well protected from injury.

The left kidney (choice D) is a retroperitoneal structure that is well protected by fat.
The tongue will move in which direction when protuded with surgical damage to the right hypoglossal nerve?
A. Downward
B. Upward
C. Directly forward
D. To the right
E. To the left




The correct answer is D. There are two ways to answer this question. The formula method is that a tongue with muscle or nerve injury will protrude toward the side of injury, in this case the right side. This is similar to the case of the mandible protruding toward the side of injury when a lateral pterygoid is injured.The logical method is to imagine intrinsic muscles and extrinsic protruders of the tongue on the right side not receiving stimulation from innervation. In this case, only the left side protruders will operate. The tongue will protrude only on the left side, with the immobile right side acting as a stationary pivot while the tongue moves from left to right (toward the injured side).
You are asked to hold your upper arm against your lateral chest wall, with the palm upward. You then rotate the hand so that the palm faces downward, without bending the wrist. This motion is known as:
A. abduction of the forearm
B. adduction of the forearm
C. flexion of the forearm
D. pronation of the forearm
E. supination of the forearm




The correct answer is D. When the forearm is rotated from anatomic position (palms facing forward, thumbs out) so that the palm faces posteriorly, the forearm is said to be pronated.

Abduction (choice A) raises the arm to a horizontal position away from the body; adduction (choice B) is the reverse.

Flexion (choice C) brings the arm or forearm forward, in front of the plane of the body.

Rotation of the forearm so that the palm faces forward (i.e., into anatomic position) is referred to as supination (choice E).
An otherwise healthy student taking no medications is concerned because he has noticed several painless uniform "large bumps" at the back of his tongue. These are most likely
A. aphthous ulcers
B. candidal colonies
C. circumvallate papillae
D. filiform papillae
E. fungiform papillae




The correct answer is C. The large bumps at the back of his tongue are circumvallate papillae. These are large circular structures surrounded by moat-like depressions. The lateral surfaces of these papillae contain taste buds. There are also small serous-only salivary glands in these papillae.

Aphthous ulcers (choice A) are small, white, or red mouth lesions.

Candidal colonies (choice B) appear in thrush, which occurs more commonly in the immunocompromised host or in those taking antibacterial drugs. You are told that the patient is healthy and not taking medications, making this condition unlikely.

Filiform papillae (choice D) are the most numerous papillae of the tongue. They are small, elongated cones that create the tongue's rough texture. They do not contain taste buds.

Fungiform papillae (choice E) are mushroom-shaped structures scattered among the filiform papillae. They frequently contain taste buds. They are intermediate in size between filiform and circumvallate papillae.
Attempts to straighten out a flexed thigh cause great pain in a patient with appendicitis. This is due to the position of the appendix near which muscle?
A. Adductor magnus
B. Biceps femoris
C. Gluteus maximus
D. Gracilis
E. Psoas major




The correct answer is E. The path of the psoas major lies in the retroperitoneum and comes close to the appendix. Acute appendicitis can cause either infection or a sympathetic inflammation of the psoas. This produces clinically a "positive psoas sign," in which attempts to straighten the patient's flexed (to relieve pain) hip produce sometimes marked exacerbation of the pain. None of the other muscles listed pass near the appendix.
Which of the following structures does the fetal allantoic duct become in the adult?
A. Cloaca
B. Medial umbilical ligament
C. Urachus
D. Ureter
E. Urethra




The correct answer is C. The urachus is a fibrous remnant that extends from the umbilicus to the urinary bladder. It is also known as the median umbilical ligament of the anterior abdominal wall.

The cloaca (choice A) is the primitive, endoderm-lined region that receives the terminal portion of the hindgut. It is later subdivided into urogenital and anal areas.

The medial umbilical ligament (choice B) is a paired structure located deep to the peritoneum of the anterior abdominal wall. It is formed by the obliterated umbilical artery.

The ureter (choice D) is the muscular tube that conveys urine from the kidney to the urinary bladder.

The urethra (choice E) is the passageway that carries urine from the bladder to the perineum.
The smooth part of the right atrium derives from which of the following embryonic structures?
A. Bulbus cordis
B. Primitive atrium
C. Primitive ventricle
D. Sinus venosus
E. Truncus arteriosus




The correct answer is D. The smooth part of the right atrium (the sinus venarum) is derived from the sinus venosus. The coronary sinus and the oblique vein of the left atrium also derive from the sinus venosus.

The bulbus cordis (choice A) gives rise to the smooth part of the right ventricle (conus arteriosus) and the smooth part of the left ventricle (aortic vestibule).

The primitive atrium (choice B) gives rise to the trabeculated part of the right and left atria.

The primitive ventricle (choice C) gives rise to the trabeculated part of the right and left ventricles.

The truncus arteriosus (choice E) gives rise to the proximal part of the aorta and the proximal part of the pulmonary artery.
Microscopic examination of a PAS-stained histological section of a Graafian follicle demonstrates a bright reddish-pink, acellular ring around the ovum. Which of the following terms most accurately describes this ring?
A. Corona radiata
B. Cumulus oophorus
C. Theca externa
D. Theca interna
E. Zona pellucida




The correct answer is E. The ring described is the zona pellucida, which surrounds the ovum. The zona pellucida is rich in polysaccharides and glycoproteins and consequently stains brightly pink or red with PAS stain. Binding of the sperm cell membrane to the zona pellucida triggers the acrosome reaction, during which acrosomal enzymes are released that digest the zona pellucida, allowing the spermatozoon to contact and fuse with the ovum cell membrane. Anatomically, as layers of granulosa cells develop around the primary oocyte, microvilli from the surrounding granulosa cells intermingle with cells of the primary oocyte. The microvilli are surrounded by a layer of glycoproteins, and the entire region is called the zona pellucida.

The follicular cells immediately outside the zona pellucida form the corona radiata (choice A). The larger cumulus oophorus (choice B) is the hill of follicular cells that surrounds the ovum.

The theca interna (choice D) and externa (choice C) are formed from the connective tissue surrounding the follicle.
Following a surgical procedure on the right side of the neck, a patient can no longer raise his right arm above the horizontal position. The patient also cannot shrug his right shoulder. Which of the following nerves was injured?
A. Axillary nerve
B. Great auricular nerve
C. Greater occipital nerve
D. Spinal accessory nerve
E. Transverse cervical nerve




The correct answer is D. The spinal accessory nerve crosses the posterior triangle of the neck immediately deep to the investing fascia of the neck. This nerve innervates the trapezius muscle, which is responsible for upward rotation and elevation of the scapula. A lesion of this nerve in the posterior triangle leads to paralysis of the trapezius. Without the ability to upwardly rotate the scapula, abduction and flexion of the arm above the horizontal plane is not possible. Also, shrugging of the shoulder is impaired with paralysis of the trapezius muscle.

The axillary nerve (choice A) does not pass through the neck. It is a branch of the brachial plexus, and it leaves the axilla to innervate the deltoid and teres minor muscles.

The great auricular nerve (choice B) and transverse cervical nerve (choice E) are branches of the cervical plexus, which provide cutaneous innervation to the skin of the neck. No muscles are innervated by these nerves.

The greater occipital nerve (choice C) is the dorsal ramus of the second cervical spinal nerve. It provides cutaneous innervation to the skin of the back of the head. No muscles are innervated by this nerve.
Zygomycosis, a destructive fungal infection of the sinuses, is likely to reach the brain by which of the following routes?
A. Cavernous sinus
B. External carotid artery
C. Internal carotid artery
D. Superior sagittal sinus
E. Superior vena cava




The correct answer is A. This question requires knowledge of pathophysiology with a basic understanding of anatomy. The cavernous sinuses are located on either side of the body of the sphenoid bone and become a potential route of infection because they receive blood both from the face (via the ophthalmic veins and sphenoparietal sinus) and from some of the cerebral veins. The spread of infection into the cavernous sinus can produce either central nervous system (CNS) infection or cavernous sinus thrombosis, both of which are potentially fatal.

The route from the face to the brain is not arterial (choices B and C).

The superior sagittal sinus (choice D) is located in the falx cerebri and drains venous blood from the brain to other dural sinuses, from which it eventually drains into the jugular vein. Zygomycosis does not reach the brain by way of the superior sagittal sinus.

The superior vena cava (choice E) drains blood from the upper part of the body into the heart.
A surgeon inadvertently sections the recurrent laryngeal nerve during a procedure. Which of the following muscles would retain its innervation subsequent to this injury?
A. Cricothyroid
B. Lateral cricoarytenoid
C. Posterior cricoarytenoid
D. Thyroarytenoid
E. Vocalis




The correct answer is A. The recurrent laryngeal nerve is a branch of the vagus nerve, which innervates all of the intrinsic laryngeal muscles, except for the cricothyroid muscle. The cricothyroid is attached to the cricoid cartilage and the thyroid cartilage; contraction of this muscle tends to stretch and adduct the vocal ligament. The cricothyroid is innervated by the external laryngeal nerve.

The lateral cricoarytenoid muscle (choice B) is innervated by the recurrent laryngeal nerve and is attached to the cricoid cartilage and the arytenoid cartilage. Its contraction causes adduction of the vocal ligament.

The posterior cricoarytenoid muscle (choice C) is innervated by the recurrent laryngeal nerve and is attached to the cricoid cartilage and the arytenoid cartilage. Its contraction causes abduction of the vocal ligament.

The thyroarytenoid muscle (choice D) is innervated by the recurrent laryngeal nerve and is attached to the thyroid cartilage and the arytenoid cartilage. Its contraction causes slackening of the vocal ligament.

The vocalis muscle (choice E) is the most medial part of the thyroarytenoid muscle. It attaches to either the thyroid cartilage and the vocal ligament, or the arytenoid cartilage and the vocal ligament. It is innervated by the recurrent laryngeal nerve. Its contraction causes tension on segments of the vocal ligament.
As a result of a viral infection, a patient has swelling of the left facial nerve within the facial canal. The patient's face appears asymmetrical, and he complains that saliva drips from his mouth while he is chewing. Paralysis of which of the following muscles accounts for these symptoms?
A. Buccinator
B. Masseter
C. Palatoglossus
D. Palatopharyngeus
E. Temporalis




The correct answer is A. Compression of the facial nerve within the facial canal may result in facial palsy (Bell's palsy). Because the muscles on one side of the face are paralyzed, the face appears asymmetrical. The buccinator muscle, which is located within the cheek and is innervated by the facial nerve, functions to hold food against the teeth while it is being chewed. Paralysis of this muscle can result in food and saliva accumulating between the teeth and the cheek. The buccinator originates from the alveolar processes of the maxilla and mandible. It inserts into the fibers of the obicularis oris.

The masseter and temporalis muscles (choices B and E) are innervated by the mandibular division of the trigeminal nerve. These muscles of mastication function to elevate the mandible.

The palatoglossus (choice C) and palatopharyngeus (choice D) muscles are innervated by the vagus nerve. The palatoglossus, with its mucosal covering, forms the palatoglossal fold (anterior pillar of the fauces), immediately anterior to the palatine tonsil. This muscle functions to draw the tongue and soft palate closer together, as occurs during swallowing. The palatopharyngeus, with its mucosal covering, forms the palatopharyngeal fold (posterior pillar of the fauces), which is immediately posterior to the palatine tonsil. This muscle causes elevation of the pharynx, as occurs during swallowing.
Biopsy demonstrates epithelial metaplasia. Which of the following cell types was most likely observed in the involved areas?
A. Ciliated columnar epithelium
B. Cuboidal epithelium
C. Keratinizing squamous epithelium
D. Nonciliated columnar epithelium
E. Nonkeratinizing squamous epithelium




The correct answer is D. The medical condition is Barrett's esophagus, in which the normally nonkeratinizing squamous epithelium (choice E) of the esophagus undergoes metaplasia to gastric or intestinal-like epithelium composed of nonciliated columnar epithelial cells. Barrett's esophagus typically develops in the setting of chronic gastroesophageal reflux and significantly increases the risk of later development of adenocarcinoma of the distal esophagus.

Ciliated columnar epithelium (choice A) is found in the respiratory tract.

Cuboidal epithelium (choice B) is found in the kidney, peritoneal lining, and pleural lining.

Keratinizing squamous epithelium (choice C) is found in skin.

Nonkeratinizing squamous epithelium (choice E), in addition to being the normal epithelium of the esophagus, is found in mouth, nose, and vagina.
Despite blockage of the celiac trunk, the organs receiving their blood supply from the trunk continue to operate normally. This is due to anastomoses between which vessels?
A. Left gastroepiploic artery and right gastroepiploic artery
B. Left gastroepiploic artery and right gastroepiploic artery
C. Proper hepatic artery and gastroduodenal artery
D. Right colic artery and middle colic artery
E. Superior pancreaticoduodenal artery and inferior pancreaticoduodenal artery




The correct answer is E. The superior pancreaticoduodenal artery is a branch of the gastroduodenal artery, which is a branch of the common hepatic artery, itself a branch of the celiac trunk. The inferior pancreaticoduodenal artery is a branch of the superior mesenteric artery. Occlusion of the celiac trunk would allow blood from the superior mesenteric artery to reach the branches of the celiac trunk via the connections between the superior and inferior pancreaticoduodenal arteries.

The left gastric and right gastric arteries (choice A) both receive their blood from the celiac trunk. The left gastric artery is a direct branch of the celiac trunk. The right gastric artery is usually a branch of the proper hepatic artery, which is a branch of the common hepatic artery (a branch of the celiac trunk).

The left and right gastroepiploic arteries (choice B) both receive their blood supply from the celiac trunk. The left gastroepiploic artery is a branch of the splenic artery, which is a branch of the celiac trunk. The right gastroepiploic artery is a branch of the gastroduodenal artery, which is a branch of the common hepatic artery (a branch of the celiac trunk).

The proper hepatic and gastroduodenal arteries (choice C) are branches of the common hepatic artery, which is a branch of the celiac trunk.

The right colic and middle colic arteries (choice D) are both branches of the superior mesenteric artery.
Despite blockage of the celiac trunk, the organs receiving their blood supply from the trunk continue to operate normally. This is due to anastomoses between which vessels?
A. Left gastroepiploic artery and right gastroepiploic artery
B. Left gastroepiploic artery and right gastroepiploic artery
C. Proper hepatic artery and gastroduodenal artery
D. Right colic artery and middle colic artery
E. Superior pancreaticoduodenal artery and inferior pancreaticoduodenal artery




The correct answer is E. The superior pancreaticoduodenal artery is a branch of the gastroduodenal artery, which is a branch of the common hepatic artery, itself a branch of the celiac trunk. The inferior pancreaticoduodenal artery is a branch of the superior mesenteric artery. Occlusion of the celiac trunk would allow blood from the superior mesenteric artery to reach the branches of the celiac trunk via the connections between the superior and inferior pancreaticoduodenal arteries.

The left gastric and right gastric arteries (choice A) both receive their blood from the celiac trunk. The left gastric artery is a direct branch of the celiac trunk. The right gastric artery is usually a branch of the proper hepatic artery, which is a branch of the common hepatic artery (a branch of the celiac trunk).

The left and right gastroepiploic arteries (choice B) both receive their blood supply from the celiac trunk. The left gastroepiploic artery is a branch of the splenic artery, which is a branch of the celiac trunk. The right gastroepiploic artery is a branch of the gastroduodenal artery, which is a branch of the common hepatic artery (a branch of the celiac trunk).

The proper hepatic and gastroduodenal arteries (choice C) are branches of the common hepatic artery, which is a branch of the celiac trunk.

The right colic and middle colic arteries (choice D) are both branches of the superior mesenteric artery.
Damage to which of the following structures might produce hair cell loss?
A. Basilar membrane
B. Organ of Corti
C. Reissner's membrane
D. Scala tympani
E. Scala vestibuli




The correct answer is B. Hearing is the detection of sound, which consists of pressure waves conducted through air or water. The receptors of the cochlear duct provide us with a sense of hearing that allows us to detect the quietest whisper and yet remain functional in a crowded, noisy environment. The organ of Corti contains hair cells from the cochlear branch of the vestibulocochlear nerve (CN VIII). These cells rest on the basilar membrane (choice A), which separates the scala tympani (choice D) from the scala media. The hair cells are embedded in the tectorial membrane, and movement of the basilar membrane below the cells causes the hairs to bend, which generates action potentials. The tectorial membrane that lies on the hair cells does not form a boundary between the different scala; the membrane separating the scala media from the scala vestibuli (choice E) is Reissner's (vestibular) membrane (choice C).
Which of the following locations in the embryo later forms the dorsal horn of the spinal cord?
A. Alar plate
B. Basal plate
C. Neural crest
D. Rostral end of neural tube
E. Sulcus limitans




The correct answer is A. The spinal cord arises from the caudal end of the neural tube. During development, an alar and a basal plate is formed, separated by a longitudinal groove called the sulcus limitans (choice E). The alar plate forms the dorsal (posterior) part of the spinal cord and becomes the sensory or afferent portion of the cord. The basal plate (choiceB) is the ventral (anterior) part of the cord and becomes the motor, or efferent, portion of the spinal cord and therefore would contain anterior horn cells.

The neural crest (choice C) develops into multipolar ganglion cells of autonomic ganglia, pseudounipolar cells of spinal and cranial nerve ganglia, leptomeningeal cells, Schwann cells, melanocytes, chromaffin cells of the adrenal medulla, and odontoblasts.

The brain forms from the rostral end of the neural tube (choice D).
From which of the following fetal vessels do the umbilical arteries arise?
A. Aorta
B. Carotid arteries
C. Ductus arteriosus
D. Iliac arteries
E. Pulmonary arteries




The correct answer is D. The paired umbilical arteries arise from the iliac arteries. They supply unoxygenated fetal blood to the placenta. The single umbilical vein takes the newly oxygenated fetal blood from the placenta to the liver and then to the inferior vena cava via the ductus venosus. Near the level of vertebra L4, the terminal segment of the abdominal aorta divides to form the right and left common iliac arteries. These arteries carry blood to the pelvis and lower limbs. As these arteries travel along the inner surface, they descend behind the cecum and sigmoid colon, where each divides to form the internal iliac artery and external iliac artery.
A mass in the anterior midline of the neck, slightly above the larynx is mobile and elevates upon protrusion of the tongue. This mass is most likely a cyst that developed from which of the following embryonic structures?
A. First pharyngeal cleft
B. First pharyngeal pouch
C. Second pharyngeal cleft
D. Second pharyngeal pouch
E. Thyroglossal duct




The correct answer is E. The thyroglossal duct develops as an evagination of the floor of the pharynx in the region where the tongue develops. The adult foramen cecum of the tongue marks the site of this evagination. The distal end of this duct normally forms the thyroid gland; the proximal part of the duct normally degenerates. Failure of a part of the duct to degenerate may lead to a thyroglossal duct cyst or a median cervical cyst, as seen in this patient.

The first pharyngeal cleft (choice A) forms the external ear canal. This cleft normally remains patent.

The first pharyngeal pouch (choice B) forms the middle ear cavity and the auditory tube. This pouch normally remains patent.

The second pharyngeal cleft (choice C) normally does not remain patent. It is typically covered over by the overgrowth of the second pharyngeal arch. If part of this pouch does remain patent, it may form a lateral cervical cyst, which is seen on the lateral side of the neck along the anterior border of the sternocleidomastoid muscle.

The second pharyngeal pouch (choice D) forms the tonsillar fossa of the pharynx. The pharyngeal mucosa in this area arises from the endoderm of the pouch. Ingrowth of mesoderm cells results in the formation of the palatine tonsil.
Which of the following sites contains striated muscle that is not under voluntary control?
A. Bladder
B. Colon
C. Esophagus
D. Gallbladder
E. Stomach




The correct answer is C. Striated (skeletal) muscle not under voluntary control is an unusual feature of the upper and middle thirds of the esophagus. The middle third of the esophagus contains roughly half striated and half smooth muscle; the lower third contains only smooth muscle. All the other structures listed in the answer choices contain smooth muscle.

As a side note, cardiac muscle cells do not rely on nerve activity to start a contraction. Instead, specialized pacemaker cells establish a regular rate of contraction. Because these pacemaker cells regulate the heart and the central nervous system does not, cardiac muscle is also considered striated, involuntary muscle.
An ulcer damages an artery supplying the area of the greater curvature of the stomach. Which artery is involved?
A. Left gastric
B. Left gastroepiploic
C. Right gastric
D. Right gastroepiploic
E. Short gastric




The correct answer is D. The right gastroepiploic artery, off the gastroduodenal artery, supplies the right half of the greater curvature of the stomach and could be directly affected by ulceration of the greater curvature of the stomach at a site this close (4 cm) to the pyloric sphincter.

The left gastric artery (choice A), off the celiac trunk, supplies the left half of the lesser curvature of the stomach.

The left gastroepiploic artery (choice B), off the splenic artery, supplies the left half of the greater curvature of the stomach. Although it anastomoses with the right gastroepiploic artery, it is unlikely that this artery would be directly damaged by ulceration of the stomach near the pyloric sphincter.

The right gastric artery (choice C), off the proper hepatic artery, supplies the right half of the lesser curvature of the stomach.

The short gastric artery (choice E), actually one of several (4 to 5) short gastric arteries, off the splenic artery (occasionally the left gastroepiploic), supplies the fundus of the stomach, which is the most distant from the pylorus.
An elderly patient has had multiple small strokes lending to an absence of the gag reflex. These findings suggest involvement of the nucleus of which of the following cranial nerves?
A. Facial (VII)
B. Glossopharyngeal (IX)
C. Hypoglossal (XII)
D. Spinal accessory (XI)
E. Vestibulocochlear (VIII)




The correct answer is B. Cranial nerve IX is the glossopharyngeal nerve, which has a nucleus in the medulla and is necessary for the gag reflex. The gag reflex is elicited by touching either side of the posterior pharynx with a tongue blade, producing bilateral elevation of the palate and bilateral contraction of the pharyngeal muscles. The afferent of this reflex arc consists of the ipsilateral glossopharyngeal nerve, while the vagus nerve, bilaterally, supplies the efferent limb. Although the glossopharyngeal nerve may seem to be one of the less important cranial nerves, you should remember to test for its function, as a loss of gag reflex can lead to the patient's death secondary to an aspiration pneumonia. The glossopharyngeal nerve is a mixed sensory and motor nerve to the head and neck. It originates from the posterior 1/3 of the tongue, pharynx, palate, and carotid arteries of the neck. Its destination includes sensory nuclei of the medulla oblongata, as well as the pharyngeal muscles involved in swallowing.

Cranial nerve VII (choice A) is the facial nerve, which supplies motor function to the face, but it does not supply the oropharynx.

Cranial nerve XII (choice C) is the hypoglossal nerve, which supplies the intrinsic and most extrinsic muscles of the tongue. It is not involved in the gag reflex.

Cranial nerve XI (choice D) is the spinal accessory nerve, which supplies the trapezius and sternocleidomastoid.

Cranial nerve VIII (choice E) is the vestibulocochlear nerve, responsible for hearing and equilibrium.
Which of the following respiratory system components is derived from neural crest?
A. Endothelial cells
B. Epithelium of primary bronchi
C. Laryngeal cartilage
D. Tracheal glands
E. Type I pneumocytes




The correct answer is C. Laryngeal cartilages are derived from neural crest. The larynx is composed of three cartilages that form the "body of the larynx": the thyroid cartilage, the cricoid cartilage, and the epiglottis. The larynx also contains three pairs of smaller hyaline cartilages: the arytenoid, corniculate, and cuneiform cartilages.

The endothelial cells (choice A), in the simple squamous epithelium that lines the pulmonary capillaries, are derived from visceral mesoderm.

The epithelial lining of primary bronchi (choice B) is derived from endoderm.

Tracheal glands (choice D) and epithelium both derive from endoderm.

Type I pneumocytes (choice E) are derived from endoderm.
The extraocular muscles are derived from which of the following structures?
A. Branchial arches
B. Optic cup ectoderm
C. Somites
D. Somitomeres
E. Splanchnic mesoderm




The correct answer is D. The somitomeres are specialized masses of mesoderm found in the head region that give rise to the muscles of the head. The extraocular muscles are derived from somitomeres 1, 2, 3, and 5.

The branchial arches (choice A) give rise to muscles of mastication (arch 1), muscles of facial expression (arch 2), and muscles of the pharynx and larynx (arches 3-6), as well as additional small muscles.

The optic cup ectoderm (choice B) gives rise to the muscles of the iris (sphincter and dilator pupillae). These are the only muscles not formed from mesoderm.

Somites (choice C) give rise to the inferior muscles of the neck.

Splanchnic mesoderm (choice E) gives rise to smooth muscle of the viscera and the heart muscle.
A CT scan reveals a small tumor at the cerebellopontine angle of the brain. Which of the following nerves is most likely to be affected by this tumor?
A. Facial nerve
B. Glossopharyngeal nerve
C. Optic nerve
D. Trigeminal nerve
E. Vagus nerve




The correct answer is A. The facial nerve and the vestibulocochlear nerves emerge from the brain stem at the cerebellopontine angle. These are the two nerves that will be initially affected by a tumor in this region. The entire anatomy of the facial nerve is as follows: the facial nerve originates from the taste receptors on the anterior 2/3 of the tongue (sensory) and from nuclei of the pons (motor). It passes through the internal acoustic canal of the temporal bone to reach the stylomastoid foramen.

The glossopharyngeal and vagus nerves (choices B and E) emerge from the brain stem at the postolivary sulcus. This is caudal to the cerebellopontine angle.

The optic nerve (choice C) exits from the optic chiasm on the ventral surface of the diencephalon. This is rostral to the cerebellopontine angle.

The trigeminal nerve (choice D) emerges from the brain stem at the anterolateral surface of the pons. This is rostral and ventral to the cerebellopontine angle.
A patient is unable to close her right eye. Physical examination reveals weakness of the right orbicularis oculi. Which of the following additional symptoms would likely also be present?
A. Blurred vision
B. Hyperacusis
C. Inability to chew
D. Inability to feel the face
E. Inability to shrug the shoulder




The correct answer B. The facial nerve innervates the muscles of facial expression. The visceral motor portion innervates the lacrimal gland and nasal mucous glands via the sphenopalatine ganglion. The submandibular and sublingual salivary glands are innervated via the submandiublar ganglion. This patient has a lesion of the facial nerve (VII), which leads to an inability to close the ipsilateral eye because of damaged motor fibers to the orbicularis oculi. This patient would also lose her corneal reflex on that side because of an inability to blink and would have ipsilateral paralysis of the muscles of facial expression distal to the lesion. If the lesion affected the facial nerve more proximally, additional findings would be hyperacusis (increased sensitivity to sound because of stapedius muscle paralysis), lack of taste sensation in the anterior two-thirds of the tongue, and disturbed lacrimation and salivation.

Blurred vision (choice A) could occur with lesions of the oculomotor (CN III), abducens (CN VI), or trochlear (CN IV) nerves, which innervate the extraocular muscles. CN III innervates the medial rectus, inferior rectus, superior rectus, and inferior oblique muscles. CN VI innervates the lateral rectus and CN IV innervates the superior oblique.

An inability to chew (choice C) would probably be the result of a lesion of the trigeminal nerve (CN V). Motor fibers of CN V innervate the muscles of mastication (temporalis, masseter, and medial and lateral pterygoid muscles), and a lesion of these fibers may cause the jaw to deviate to the side of the weak muscles.

An inability to feel the face (choice D) would also be the result of a CN V lesion. This lesion could result in the ipsilateral loss of general sensation of the face and also of the mucous membranes of the oral and nasal cavities.

A lesion of the accessory nerve (CN XI) would cause paralysis of the trapezius muscle, which results in a sagging of the shoulder and a weakness in attempting to shrug the shoulder (choice E).
A patient received a severe blow to the lateral side of the head, resulting in an epidural hematoma. Which of the following blood vessels was most likely torn?
A. Anterior cerebral artery
B. Middle cerebral artery
C. Middle meningeal artery
D. Superficial temporal artery
E. Superior cerebral vein




The correct answer is C. The middle meningeal artery is in the interior of the lateral portion of the cranial cavity, embedded in the periosteal (outer) layer of the dura. A tear of this artery results in blood entering the potential space between the outer dural layer and the skull (epidural space), causing an epidural hematoma.

The anterior cerebral artery and middle cerebral artery (choices A and B) lie on the surface of the brain. The anterior cerebral arteries supply the medial surface of the cerebral hemispheres, and the middle cerebral arteries supply the lateral surface of the cerebral hemispheres. A tear of either of these arteries would result in blood entering the subarachnoid space (subarachnoid hemorrhage).

The superficial temporal artery (choice D) is a branch of the external carotid artery and is external to the skull. This artery supplies the skin and other tissue of the temple region.

The superior cerebral veins (choice E) drain the cerebral hemisphere and enter the superior sagittal sinus. A tear of these veins results in blood entering the potential space between the dura and arachnoid (subdural space), causing a subdural hematoma.
The x-ray of a child's arm after a fall appears to show a fracture near, but not at, the distal end of the ulna. Before diagnosing a fracture, you should also consider the possibility that this is actually which of the following?
A. Articular cartilage
B. Epiphyseal plate
C. Perichondrium
D. Primary ossification center
E. Secondary ossification center




The correct answer is B. The epiphyseal plate of the bone contains cartilage that is radiolucent. The plate in a bone that is not yet fully ossified can produce a "line" crossing the bone near the end. This may be easily mistaken for a fracture by the inexperienced. Anatomically, the epiphyseal plate separates the epiphysis from the diaphysis.

Articular cartilage (choice A) is radiolucent, but occurs at the very tip of the long bones.

Perichondrium (choice C) is usually difficult to see on x-ray.

Primary (choice D) and secondary (choice E) ossification centers are radiopaque.
Which of the following embryonic structures gives rise to the adrenal cortex?
A. Ectoderm
B. Endoderm
C. Mesoderm
D. Mesonephros
E. Neural crest cells




The correct answer is C. The mesoderm gives rise to the adrenal cortex. In addition, it also gives rise to connective tissue, cartilage, bone, muscle, blood and lymph vessels, kidneys, gonads, serous membranes lining body cavities, and the spleen.

The ectoderm (choice A) gives rise to the central nervous system, peripheral nervous system, epidermis and its appendages, mammary glands, pituitary gland, tooth enamel, and the neural crest.

The endoderm (choice B) gives rise to the parenchyma of the tonsils, thyroid and parathyroid glands, thymus, liver, pancreas, the epithelial lining of the gastrointestinal and respiratory tracts, urinary bladder, urethra, and auditory tube.

The mesonephros (choice D) functions as an interim kidney in the embryo.

The neural crest cells (choice E) give rise to cells of the spinal and cranial nerves, autonomic ganglia, melanocytes, leptomeninges, connective tissue and bone of branchial arch origin, and the adrenal medulla.
The major structural component of the sperm flagellum is the:
A. microtubule
B. microfilament
C. actin filament
D. mysosin filament
E. ciliary body




The correct answer is choice A. This is a basic histological question of a type found commonly on NBDE, which asks about basic cellular organelles and structures. The sperm cell flagellum is notable for a “9+2” arrangement of fibers formed from microtubules. The 9 single fibers form a circular ring in cross section around the 2 doubled fibers in the center. One of the significant facts about the fibers is that they are composed of microtubules, made up of tubulin. The other significant fact is that all eucaryotic flagella and cilia have this same structure (for example, cilia form the tracheal lining). Note that procaryotic (bacterial) flagellea do NOT share this structure.
In cases of dysphagia where the esophagus is compressed, which structure would be most likely to cause the compression?
A. Left atrium
B. Left ventricle
C. Pulmonary trunk
D. Right atrium
E. Right ventricle




The correct answer is A. The left atrium forms most of the posterior wall of the heart. The esophagus passes immediately posterior to the heart. Enlargement of the left atrium may compress the esophagus and cause dysphagia. Anatomically, the esophagus begins posterior to the cricoid cartilage, at the level of vertebrum C6. From this point, it descends toward the thoracic cavity posterior to the trachea, passes inferiorly along the dorsal wall of the mediastinum, and enters the abdominopelvic cavity through an opening in the diaphragm, the diaphragmatic hiatus.

The left ventricle (choice B) forms most of the left border of the heart and most of the diaphragmatic surface of the heart. The left ventricle is not related to the esophagus.

The pulmonary trunk (choice C) emerges from the right ventricle on the anterior surface of the heart. The pulmonary trunk is not related to the esophagus.

The right atrium (choice D) forms the right border of the heart. It is not related to the esophagus.

The right ventricle (choice E) forms most of the anterior wall of the heart and a small portion of the diaphragmatic surface of the heart. It is not related to the esophagus.
Which of the following tissues normally has the highest percentage of mucus-secreting cells?
A. Esophageal mucosa
B. Oral mucosa
C. Parotid gland
D. Sublingual gland
E. Submandibular gland




The correct answer is D. The sublingual salivary glands are located beneath the mucous membrane of the floor of the mouth. Numerous sublingual ducts open along either side of the lingual frenulum. Salivary glands can contain predominantly serous cells, predominantly mucous cells, or both in their acini. As you proceed from the midline laterally, acini in the sublingual gland are almost pure mucous cells, whereas acini in the submandibular gland (choice E) contain a mixture of serous and mucous cells. Acini in the parotid gland (choice C) are mostly pure serous cells.

The esophageal mucosa (choice A) and the oral mucosa (choice B) are squamous epithelia that do not form acini.
The primary arterial supply to the nasal mucosa is a direct branch of which of the following arteries?
A. Facial artery
B. Maxillary artery
C. Superficial temporal artery
D. Superior labial artery
E. Transverse facial artery




The correct answer is B. The major source of blood supply to the nasal mucosa is the sphenopalatine artery, which is the terminal branch of the maxillary artery. The sphenopalatine artery enters the nasal cavity from the pterygopalatine fossa by passing through the sphenopalatine foramen in the lateral wall of the nasal cavity.

The facial artery (choice A) is a direct branch of the external carotid artery. It provides most of the blood supply to the superficial face.

The superficial temporal artery (choice C) is a terminal branch of the external carotid artery. The other terminal branch is the maxillary artery. The superficial temporal artery provides blood supply to the temporal region and the lateral portion of the scalp.

The superior labial artery (choice D) is a branch of the facial artery. The superior labial artery provides blood supply to the upper lip. It has a septal branch that provides some of the blood supply to the anterior portion of the septal mucosa of the nasal cavity. However, this is not the major blood supply to the nasal cavity.

The transverse facial artery (choice E) is a branch of the superficial temporal artery. It provides blood supply to the parotid gland, the parotid duct, and the skin of the lateral face.
Which of the following cranial nerves carries the pain sensation from the tip of tongue?
A. V2
B. V3
C. VII
D. IX
E. X




The correct answer is B. This question is essentially asking "which of the following nerves innervates the tip of the tongue?" The innervation of the tongue is complex. The mandibular division of the trigeminal nerve (V3) carries general somatic sensation from the anterior two-thirds of the tongue. The mandibular branch is also a sensory nerve for the lower gingiva, teeth, and lips, as well as the palate.

The maxillary division (V2, choice A) carries somatic sensation from the palate, upper gingiva, and upper lip, as well as the lower eyelid, cheek, nose, and a portion of the pharynx.

The facial nerve (VII, choice C) carries taste from the anterior two-thirds of the tongue, as well as innervating the lacrimal gland and the submandibular and sublingual salivary glands.

The glossopharyngeal nerve (IX, choice D) carries sensation and taste from the posterior one-third of the tongue.

The vagus nerve (X, choice E) carries sensation from the lower pharynx.
A mass is noted at the back of a young man's tongue. A biopsy's pathology report comes back with a diagnosis of normal thyroid tissue. This finding is related to the embryonic origin of the thyroid near which of the following structures?
A. First pharyngeal pouch
B. Foramen cecum
C. Nasolacrimal duct
D. Second pharyngeal arch
E. Third pharyngeal pouch




The correct answer is B. The thyroid gland originates as a mass of endodermal tissue near the foramen cecum, which is near the tuberculum impar (which becomes the central part of the tongue). During development, the thyroid descends in front of the pharynx, maintaining a connection to the tongue via the thyroglossal duct. Usually, the thyroglossal duct disappears. Uncommonly, residual ectopic thyroid tissue can be left anywhere along the path, including at the back of the tongue. (In rare cases, all of the thyroid tissue remains at this site, forming a mass that should not be excised, for obvious reasons!) Anatomically, the thyroid gland curves across the anterior surface of the trachea just below the thyroid cartilage that forms the majority of the anterior surface of the larynx. The two lobes of the thyroid gland are united by a slender connection, the isthmus.

The first pharyngeal pouch (choice A) develops into the middle ear and eustachian tube.

The nasolacrimal ducts (choice C) connect the eyes to the oropharynx.

The second pharyngeal arch (choice D) develops into many muscles of the face and the styloid process of the temporal bone.

The third pharyngeal pouch (choice E) develops into the thymus and inferior parathyroid glands.
If the nerve that accompanies the superior laryngeal artery is damaged, which of the following functional losses will ensue?
A. Loss of sensation in the laryngeal mucosa above the vocal folds
B. Loss of sensation in the laryngeal mucosa below the vocal folds
C. Loss of sensation in the pharyngeal mucosa
D. Paralysis of the cricothyroid muscle
E. Paralysis of the lateral cricoarytenoid muscle




The correct answer is A. The superior laryngeal artery is a branch of the superior thyroid artery. It enters the larynx by passing through the cricothyroid membrane. In this region, it is accompanied by the internal branch of the superior laryngeal nerve. This nerve provides sensory innervation to the laryngeal mucosa above the vocal folds.

The laryngeal mucosa below the vocal folds (choice B) receives its sensory innervation from the recurrent laryngeal nerve.

The pharyngeal mucosa (choice C) receives its sensory innervation from the glossopharyngeal nerve.

The cricothyroid muscle (choice D) receives its motor innervation from the external branch of the superior laryngeal nerve.

The lateral cricoarytenoid muscle (choice E) receives its motor innervation from the recurrent laryngeal nerve.
An atrial septal defect results from failure of the
A. ostium primum to form within the septum primum
B. ostium secundum to form within the septum primum
C. septum primum to fuse with the endocardial cushions
D. septum primum to fuse with the septum secundum
E. septum secundum to fuse with the endocardial cushions




The correct answer is C. The septum primum (first interatrial septum) develops by growing from the cranial end of the embryonic atrium toward the endocardial cushions. The gap that exists between the two atria during this period is the ostium primum. As the septum primum continues its growth, the ostium primum gets smaller until it is closed when the septum primum completes its growth and completely fuses with the endocardial cushions. Failure of the septum primum to fuse completely with the endocardial cushions leaves a persistent ostium primum, known as a primum-type atrial septal defect.

The ostium primum does not form within the septum primum (choice A). The ostium primum is the communication between the two atria that exists during the formation of the septum primum. That is, the ostium primum is the space within the developing atrium not yet occupied by the septum primum.

The ostium secundum normally forms within the septum primum (choice B) before the ostium primum closes by fusion of the septum primum with the endocardial cushions. Failure of the ostium secundum to form would result in embryonic death because there would be no pathway for blood to pass from the right atrium to the left atrium when the ostium primum closes, thus depriving the embryo of oxygenated blood.

Most of the septum primum normally disappears. The part that remains forms the valve of the foramen ovale. This part of the septum primum normally does not fuse with the septum secundum (choice D) during prenatal life. After birth, the valve of the foramen ovale is pushed against the septum secundum as a result of the increased pressure in the left atrium. This achieves functional closure of the foramen ovale. Fusion does not normally occur at this time; it usually occurs later in life in most people. In some people, however, complete fusion never occurs (probe patency).

The septum secundum normally does not fuse with the endocardial cushions (choice E).
A CT scan of the head demonstrates a mass in the olfactory groove area. The axons likely compressed by this mass project to which of the following structures?
A. Insula
B. Nucleus ambiguus
C. Postcentral gyrus
D. Precentral gyrus
E. Pyriform cortex




The correct answer is E. The olfactory nerves can be damaged by head injury, severe infection, and tumors (such as meningiomas) that may compress the olfactory bulbs. Unilateral damage is often not noticed by the patient, but bilateral damage (which surprisingly can also be missed if it develops slowly) may lead to occasional bizarre errors in odor identification. The olfactory nerves are technically the neurons on the mucosal side of the cribriform plate that send processes through the plate to synapse in the olfactory bulb, which in turn sends axons to the pyriform cortex (the primary olfactory cortex).

The insula (choice A) is thought to contain the primary gustatory (taste) cortex.

The nucleus ambiguus (choice B) provides the special visceral efferent fibers carried by the glossopharyngeal and vagus nerves.

The postcentral gyrus (choice C) contains the primary sensory cortex.

The precentral gyrus (choice D) contains the primary motor cortex.
A biopsy specimen demonstrates a ciliated columnar epithelium. From which of the following locations in the female genital tract was the biopsy obtained?
A. Cervix
B. Endometrium
C. Fallopian tube
D. Ovary
E. Vagina




The correct answer is C.The fallopian tube is the only structure in the female genital tract with a ciliated columnar epithelium; the beating of the cilia helps move the egg into the uterus. This fact is also sometimes clinically helpful because dilated and deformed fallopian tubes can be microscopically distinguished from cystic ovarian tumors by the presence of the cilia. Each uterine tube is a hollow, muscular tube measuring approximately 13 cm in length. Each uterine tube is divided into three regions: the infundibulum, the ampulla, and the isthmus.

The cervix (choice A) and vagina (choice E) are lined by squamous epithelium.

The endometrium (choice B) is lined by columnar epithelium (although a few ciliated cells may be present).

The covering of the ovary (choice D) is cuboidal epithelium, and cysts within the ovary can be lined by cuboidal or nonciliated columnar epithelium.
A person sits with his legs together and then opens his legs against lateral resistance. Which nerve controls this type of movement?
A. deep peroneal nerve
B. femoral nerve
C. obturator nerve
D. sciatic nerve
E. superior gluteal nerve




The correct answer is E. The motion described is hip abduction. The superior gluteal nerve from roots L4-S1 is responsible for hip abduction.

The deep peroneal nerve (choice A) controls dorsiflexion of the foot. If you see a patient on the NBDE with "foot drop," think of damage to the deep peroneal nerve.

The femoral nerve (choice B) is responsible for knee extension.

The obturator nerve (choice C) is responsible for hip adduction. The hip adduction machine would provide resistance to the medial aspects of the thighs, and the individual would push his/her legs together against the machine's resistance.

The sciatic nerve (choice D) controls knee flexion.
During anatomy lab, a dental student notes a fibrous band that runs on the visceral surface of the liver. It is attached on one end to the inferior vena cava and on the other end to the left branch of the portal vein. In the embryo, this structure corresponds to the
A. ductus venosus
B. ligamentum teres
C. ligamentum venosum
D. umbilical arteries
E. umbilical vein




The correct answer is A. This question could have tricked you if you didn't catch the key words, "in the embryo." If you read the question too quickly and thought you were going to be asked to identify the structure described, you probably chose choice C (ligamentum venosum) because that is indeed the structure in question. In the embryo, however, this fibrous band is actually the ductus venosus. The ductus venosus is an embryonic vessel that allows blood to bypass the fetal liver; this prevents the depletion of oxygen and nutrient-rich blood in the hepatic sinusoids.

The embryonic umbilical arteries (choice D) become the medial umbilical ligaments.

The embryonic umbilical vein (choice E) actually becomes the fibrous ligamentum teres (choice B). The ligamentum teres is located in the free margin of the falciform ligament.
Damage to the upper fibers of the trapezius muscle would most likely impair which of the following movements?
A. Depression of the medial end of the scapula
B. Elevation of the acromion
C. Flexion of the neck
D. Raising of the medial border of the scapula
E. Turning of the face sideways




The correct answer is B. The trapezius is a large muscle arising in the posterior midline from the superior nuchal line of the skull, ligamentum nuchae, and spinous processes of all of the thoracic vertebrae. The upper fibers, which are potentially damaged by wounds to the posterior and lateral neck, insert on the crest of the spine of the scapula, the medial border of the acromion, and the lateral third of the clavicle. The upper fibers serve to elevate the acromion.

The lower fibers of the trapezius depress the medial end of the scapula (choice A).

The sternocleidomastoids are important in extension of the head at the atlanto-occipital joint and flexion of the cervical vertebral column (choice C).

The rhomboids and levator scapulae are important in raising the medial border of the scapula (choice D).

Contraction of a single sternocleidomastoid muscle is important in turning of the face sideways (choice E) in the contralateral direction.
A man's chest is compressed during a car accident, causing a posterior displacement of the clavicle at the sternoclavicular joint. Which of the following structures would be most at risk?
A. Aorta
B. Esophagus
C. Heart
D. Superior vena cava
E. Trachea




The correct answer is E. The sternoclavicular joint is quite strong and dislocates only with difficulty. Dislocations, particularly posterior dislocations, are occasionally seen and may be dangerous because of impingement onto the trachea, causing respiratory difficulties. The trachea is a tough flexible tube with a diameter of approximately 2.5 cm and a length of approximately 11 cm. The trachea begins anterior to the vertebra C6 in a ligamentous attachment to the cricoid cartilage and ends in the mediastinum at the level of vertebra T5. At this point, it branches to form the right and left primary bronchi.

Both the aorta (choice A) and the superior vena cava (choice D) are more posterior than the trachea and are therefore less vulnerable.

The esophagus (choice B) is located behind the trachea and is less vulnerable.

The heart (choice C) lies below the clavicle and would not be at particular risk.
Which of the following cell types is derived from neuroepithelial cells?
A. Astrocytes
B. Enterochromaffin cells
C. Melanocytes
D. Odontoblasts
E. Schwann cells




The correct answer is A. Astrocytes and oligodendrocytes are both derived from glioblasts, which, in turn, are derived from neuroepithelial cells. Other neuroepithelial cell derivatives include neuroblasts and ependymal cells. The astrocytes are the largest and most numerous glial cells. These cells are responsible for maintaining the blood-brain barrier, creating a three-dimensional framework for the central nervous system, performing repairs in damaged neural tissues, and controlling the interstitial environment.

All the other choices are derived from neural crest cells. Other neural crest derivatives include the neurons of the parasympathetic and sympathetic ganglia (including the adrenal medulla), the dorsal root ganglia of the peripheral nervous system, the sensory ganglia of cranial nerves V, VII, IX, and X, and the leptomeninges (pia and arachnoid).
Following a fracture of the humerus, which of the following is responsible for producing the majority of the new bone that will reunite the two fragments?
A. Cancellous bone
B. Cartilage
C. Compact bone
D. Marrow
E. Periosteum




The correct answer is E. When the periosteum is torn during a fracture, it supplies cells that develop into osteoblasts and are the major producers of the new bone that reunites the two ends. Heterotopic ossification (bone formed outside the regular bone) can occur as a complication of fracture if some of the osteoblastic cells are misdirected into adjacent tissues. The periosteum assists in the attachment of the osteoblasts to surrounding tissues and to associated tendons and ligaments. This cellular layer functions in bone growth and participates in repair after an injury.

Pre-existing cancellous bone (choice A) and compact bone (choice C) are not the major source of osteoblasts that form the new bone.

Cartilage (choice B) and marrow (choice D) do not contribute to new bone formation after fracture.
Which membrane is in the way is you try to reach the lesser peritoneal sac and head of the pancreas after penetrating the greater peritoneal sac?
A. Falciform ligament
B. Gastrohepatic ligament
C. Gastrosplenic ligament
D. Hepatoduodenal ligament
E. Splenorenal ligament




The correct answer is B. The gastrohepatic ligament is the part of the lesser omentum that separates the greater peritoneal sac from the right portion of the lesser peritoneal sac. This portion of the lesser omentum has no significant blood vessels within it and may be incised for surgical access.

The falciform ligament (choice A) is a mesenteric membrane between the liver and the anterior abdominal wall. This ligament is within the greater peritoneal sac and does not separate it from the lesser peritoneal sac.

The gastrosplenic ligament (choice C) passes from the greater curvature of the stomach to the spleen. It separates the greater peritoneal sac from the left portion of the lesser peritoneal sac. Incision of this structure would be the appropriate surgical approach to gain access to the left side of the lesser peritoneal sac.

The hepatoduodenal ligament (choice D) is part of the lesser omentum and separates the greater peritoneal sac from the right portion of the lesser peritoneal sac. It forms the anterior border of the epiploic foramen. However, the hepatoduodenal ligament contains the common bile duct, the proper hepatic artery, and the portal vein, and therefore may not be incised for surgical access.

The splenorenal ligament (choice E) passes from the spleen to the parietal peritoneum on the anterior surface of the left kidney. It separates the greater peritoneal sac from the left portion of the lesser peritoneal sac. This ligament contains the splenic artery, splenic vein, and the tail of the pancreas, and therefore may not be incised to gain access to the lesser peritoneal sac.
Which of the following fibers provide the only output from the cerebellar cortex?
A. Climbing
B. Golgi cell
C. Granule cell
D. Mossy
E. Purkinje




The correct answer is E. Two basic things about cerebellar circuitry that are well-worth knowing are that Purkinje cells of the cerebellar cortex project to the deep cerebellar nuclei and that these nuclei project out of the cerebellum. Purkinje cells are located in the second (Purkinje) cell layer of the cerebellar cortex and form inhibitory synapses on the deep cerebellar nuclei.

Climbing fibers (choice A) are afferents to the cerebellum. Specifically, they originate from the medullary olivary nuclear complex, enter the cerebellum through the inferior cerebellar peduncle, and project to the deep cerebellar nuclei and the lateral cerebellar hemispheres.

Golgi cell bodies (choice B) reside in the granule cell layer of the cerebellar cortex and project predominantly to the granule cells, where they form inhibitory synapses. Therefore, they reside in and project to the cerebellar cortex.

Granule cells (choice C) reside in the granule cell layer of the cerebellar cortex and send their axons to the moleculary layer, where they bifurcate into parallel fibers. Granule cells, therefore, both reside in and project to the cerebellar cortex.

Mossy fibers (choice D) consist of all of the afferents (except the climbing fibers) to the cerebellum. These fibers project to deep cerebellar nuclei and glomeruli in the granular layer of the cerebellar cortex.
A child who has had abnormal development of the membranous bones has a broad skull with associated facial and dental anomalies. Which other bones are most likely to also be affected?
A. Clavicles
B. Femurs
C. Metatarsals
D. Phalanges
E. Tibias




The correct answer is A. In a syndrome called cleidocranial dysostosis, absence of part of the clavicles accompanies a broad skull and facial and dental anomalies. This syndrome affects bones formed by intramembranous ossification.

The femurs (choice B), metatarsals (choice C), phalanges (choice D), and tibias (choice E) are cartilaginous (formed by endochondral ossification) rather than membranous bones.
If cirrhosis causes obstruction of the portal circulation within the liver, portal blood could still be conveyed to the caval system via which of the following?
A. Azygos and hemiazygos veins
B. Gonadal veins
C. Internal iliac veins
D. Splenic vein
E. Vesical venous plexus




The correct answer is A. The esophageal venous plexus, which drains into the azygos and hemiazygos veins within the thorax, has anastomoses with branches of the left gastric vein. Thus, following blockage of the portal vein, portal blood may enter the superior vena cava via the azygos system. Other important portacaval connections include the superior rectal vein with the middle and inferior rectal veins; paraumbilical veins and epigastric veins (engorgement of these vessels results in caput medusae); and the colic and splenic veins with renal veins and veins of the posterior body wall.

The gonadal veins (choice B) exclusively drain the gonads (although in the female, the ovarian vein communicates with the uterovaginal plexus). These vessels have no anastomoses with portal veins.

The internal iliac veins (choice C), which drain most of the pelvis and much of the inferior extremities, have no demonstrated portal anastomoses.

The splenic vein (choice D) is incorrect because it's in itself a component of the portal venous system.

The vesical venous plexus (choice E), which is situated well within the pelvis and drains the bladder and the prostate (or uterus and vagina) has no association with portal vessels.
Secretory basket cells are located
A. inside secretory acini
B. surrounding the intercalated duct
C. between secretory cells and the basal lamina
D. surrounding the striated duct




The correct answer is C. Secretory basket cells are found in salivary gland acini and are also called myoepithelial cells. The name implies that they are epithelial cells with muscle-like contractile properties. The are found between secretory cells and the basal lamina and exert pressure on the secretory cells to release their secretions. Secretory acini themselves are composed of mucous or serous secreting cells. Intercalated duct cells are nonspecialized cuboidal epithelial cells. Striated duct cells are columnar epithelial cells with striations (stripes) of rows of mitochondria used for ATP generation for active transport.
Which muscle is the chief mover of the mandible TOWARD the left?
A. Left medial pterygoid
B. Left lateral pterygoid
C. Right medial pterygoid
D. Right lateral pterygoid




The correct answer is D. There are two methods for answering this question. The formula method states that the right lateral pterygoid moves the mandible left, whereas the left lateral pterygoid moves the mandible right. The understanding method begins with the idea that the medial pterygoid is a closer (elevator). The lateral pterygoid connects to the condyle and is a protruder. If the right lateral pterygoid pulls the right condyle out while the left lateral pterygoid and condyle remain stable, the left side will act as a stationary pivot point while the right side protrudes outward and also toward the left (medially).
Which of the following cells is the germ cell closest to the basal lamina in the seminiferous tubule?
A. Primary spermatocyte
B. Secondary spermatocyte
C. Spermatid
D. Spermatogonia
E. Spermatozoa




The correct answer is D. Maturation of germ cells (spermatogenesis) within the seminiferous tubules occurs in a concentric pattern, with the less mature spermatogonia near the basal lamina and the mature forms near the tubule center. Along this route the developing sperm are nurtured by sertoli cells. Spermatogonia are 2N cells and mature into larger primary spermatocytes (4N) (choice A). These mature into secondary spermatocytes (2N) (choice B), and finally into spermatids (1N) (choice C). Spermatids undergo spermiogenesis to become mature spermatozoa (choice E). Acrosomes form from the Golgi apparatus, and a flagellum forms from microtubules. Unneeded organelles are shed. The seminiferous tubules of a reproductive-age male should exhibit all stages of maturation, with mature flagellated sperm in their centers, no longer associated with sertoli cells.
If a person has normal musculature, but has difficulty swallowing, which nerves should be tested for function?
A. Hypoglossal and phrenic
B. Hypoglossal and splanchnic
C. Glossopharyngeal and vagus
D. Phrenic and vagus
E. Splanchnic and vagus




The correct answer is E. The upper 2/3 of the esophagus contains striated muscle. It is derived from the pharyngeal arches and innervated by the vagus nerve (CN X). The lower 1/3 contains smooth muscle from splanchnic mesoderm and is innervated by the splanchnic plexus. The vagus nerve has mixed sensory and motor functions. The sensory fibers innervate the sensory nuclei and autonomic centers of the medulla oblongata. The motor fibers innervate muscles of the palate, pharynx, and respiratory and cardiovascular systems.

The hypoglossal nerve (choices A and B), or CN XII, moves the tongue.

The phrenic nerve (choices A and D), derived from C3, C4, and C5, innervates the muscle of the diaphragm.

The glossopharyngeal nerve (choice C), or CN IX, functions in taste, swallowing, and salivation, as well as monitoring the activity of the carotid body.
The left adrenal vein drains directly into which of the following veins?
A. Hemiazygos vein
B. Inferior vena cava
C. Left renal vein
D. Splenic vein
E. Superior mesenteric vein




The correct answer is C. The left adrenal vein and the left gonadal vein (either testicular or ovarian) drain into the left renal vein. The left renal vein then drains into the inferior vena cava. In contrast, the right adrenal vein and right gonadal vein drain directly into the inferior vena cava.

The hemiazygos vein (choice A) receives the venous drainage from the body wall on the left side of the thorax and abdomen. No visceral organs drain directly to the azygos or hemiazygos veins.

The inferior vena cava (choice B) receives the direct venous drainage from the right adrenal vein, but not the left adrenal vein. Remember, the inferior vena cava is on the right side of the abdomen.

The splenic vein (choice D) receives the venous drainage from the spleen and part of the pancreas and stomach. The splenic vein is part of the portal venous system.

The superior mesenteric vein (choice E) receives venous drainage from much of the intestinal tract. It is part of the portal venous system and joins with the splenic vein to form the portal vein.
Which of the following is present in males but not in females?
A. Bulbospongiosus muscle
B. Bulbourethral gland
C. Corpus cavernosum
D. Membranous urethra
E. Perineal body




The correct answer is B. The bulbourethral glands are paired structures located within the deep perineal pouch, embedded within the sphincter urethrae. Their ducts pass to the spongy urethra. The homologous female structures are the greater vestibular (Bartholin's) glands, which are located in the superficial perineal pouch.

The bulbospongiosus muscles (choice A) lie superficial to the bulb of the penis in males and to the bulbs of the vestibule in females.

The corpora cavernosa (choice C) are paired structures, consisting of cavernous erectile tissue that form a large portion of the penile shaft in males and of the body of the clitoris in females.

The membranous urethra (choice D) is the portion of the urethra that passes through the urogenital diaphragm in both males and females.

The perineal body (choice E) is the centrally located tendinous structure that provides attachment for perineal musculature in both males and females. It separates the urogenital area from the anal area and is an important obstetric landmark.
Luminal narrowing of which of the following vessels would compromise blood flow through the renal arteries?
A. Abdominal aorta
B. Celiac trunk
C. Common iliac artery
D. Inferior mesenteric artery
E. Superior mesenteric artery




The correct answer is A. The renal arteries emerge from the abdominal aorta at about the level of the L1/L2 intervertebral disk and travel at nearly right angles to it (on the right, passing posterior to the inferior vena cava) to enter the hilum of the kidney.

The celiac trunk (choice B) gives off the common hepatic, splenic, and left gastric arteries.

The common iliac artery (choice C) gives off the internal and external iliac arteries. In addition, an unascended pelvic kidney may be supplied by the common iliac artery.

The inferior mesenteric artery (choice D) gives off the superior rectal, sigmoid, and left colic arteries.

The superior mesenteric artery (choice E) gives off the inferior pancreaticoduodenal, intestinal (ileal and jejunal), right colic, middle colic, and ileocolic arteries.
Which muscle has fibers that enter the articular disc and capsule?
A. Medial pterygoid
B. Buccinator
C. Temporalis
D. Lateral pterygoid
E. Masseter




The correct answer is D. Only the lateral pterygoid enters the temperomandibular joint (TMJ). Its fibers join the articular disc and capsule. These fibers are from the superior head of the muscle. Fibers from the inferior head enter the bony head of the mandibular condyle. The lateral pterygoid protrudes the mandible, depresses (opens) it, and moves it left and right. None of the other muscles listed enter the TMJ. Medial pterygoid and masseter primarily elevate (close) the mandible, whereas temporalis elevates (anterior fibers) and retrudes (posterior fibers). Buccinator forms the substance of the cheek and is also a muscle of facial expression. It is not a muscle of mastication.
While lying supine in bed eating, a child aspirates a peanut. Which of the following bronchopulmonary segments would this foreign object most likely enter?
A. Apical segment of the left upper lobe
B. Apical segment of the right upper lobe
C. Medial segment of the right middle lobe
D. Posterior basal segment of the left lower lobe
E. Superior segment of the right lower lobe




The correct answer is E. Because the right main bronchus is wider and more vertical than the left, foreign objects are more likely to be aspirated into the right main bronchus. The superior segmental bronchus of the lower lobar bronchus is the only segmental bronchus that exits from the posterior wall of the lobar bronchi. Therefore, if a patient is supine at the time of aspiration, the object is most likely to enter the superior segmental bronchus of the lower lobe.

None of the segmental bronchi of the left lung (choices A and D) are likely to receive the object because the object is less likely to enter the left main bronchus.

The apical segment of the right upper lobe (choice B) is not likely to receive the foreign object because of the sharp angle that the upper lobar bronchus makes with the right main bronchus and the sharp angle that the apical segmental bronchus makes with the lobar bronchus.

The medial segmental bronchus of the right middle lobe (choice C) arises from the anterior wall of the right middle lobar bronchus. Therefore, when the patient is supine, the effect of gravity will tend to prevent the object from entering this segmental bronchus.
A physician notes weakness of a patient's thumb in extension, although rotation, flexion, abduction, adduction, and opposition are normal. Which of the following nerves is most likely involved?
A. Median and radial
B. Median and ulnar
C. Median only
D. Radial only
E. Ulnar only




The correct answer is D. All three of the nerves listed innervate muscles that supply the thumb. Extension is provided by the extensors pollicis longus and brevis, which are innervated by the radial nerve.

The median nerve (choices A, B, and C) supplies the thenar group, which allows the thumb to oppose, flex, abduct, and rotate.

The ulnar nerve (choices E and B) supplies the adductor pollicis, which adducts the thumb.
From which of the following arteries does the sphenopalatine artery arise?
A. External carotid
B. Facial
C. Maxillary
D. Ophthalmic
E. Transverse facial




The correct answer is C. The sphenopalatine artery is the terminal branch of the maxillary artery. The maxillary artery arises from the external carotid artery and then passes through the infratemporal fossa, giving off branches to structures in this region. The artery then passes through the pterygomaxillary fissure to enter the pterygopalatine fossa. The terminal branch then passes through the sphenopalatine foramen to enter the nasal cavity and supply much of the nasal mucosa, particularly in the posterior region of the nasal cavity.

The external carotid artery (choice A) arises from the common carotid artery at the carotid bifurcation. The external carotid artery has eight branches that supply the head and neck region: the superior thyroid, ascending pharyngeal, occipital, lingual, facial, posterior auricular, maxillary, and superficial temporal arteries.

The facial artery (choice B) arises from the external carotid artery. It provides blood supply to much of the facial region. The nasal cavity (particularly the anterior portion) receives some of its blood supply from branches of the facial artery.

The ophthalmic artery (choice D) arises from the internal carotid artery immediately after the internal carotid artery emerges from the cavernous sinus. The ophthalmic artery passes through the optic canal to supply structures in the orbit.

The transverse facial artery (choice E) is a branch of the superficial temporal artery. It supplies blood to structures in the upper portion of the lateral face.
Which of the following helps to anchor an epithelial cell to the basement membrane?
A. Adherent junction
B. Connexon
C. Gap junction
D. Hemidesmosome
E. Tight junction




The correct answer is D. Desmosomes are specializations of the lateral surface of the cell formed from the juxtaposition of two disc-shaped structures in adjacent cells, acting much like spot welds between the two cells. Hemidesmosomes are basically desmosomes between a single cell and an extracellular matrix structure, such as a basement membrane.

Adherent junctions (choice A), or zonula adherens are bandlike junctions that help attach adjacent epithelial cells to each other.

Gap junctions consist of a hexagonal lattice of tubular proteins called connexons (choice B), which form channels allowing communication between cells.

Tight junctions (choice E), or zonula occludens, are formed by the fusion of the outer leaflets of apposed cell membranes on the lateral cell surfaces, just beneath the apical poles. They form a barrier to permeability, or a seal around the cell.
A patient is involved in an accident which tears the left lateral pterygoid muscle completely. On attempting to open, the patients mandible will move:
A. Left
B. Right
C. In an elevating direction
D. In a direct protruding direction




The correct answer is A.There are two ways to answer this question. The formulaic method says that damage to a lateral pterygoid or the innervation to it will make the mandible deviate toward the side of the damage. A deeper understanding of the question would show that the lateral pterygoid is the primary protruder of the mandible. Contraction of both lateral pterygoids produces straight protrusion. If the left lateral pterygoid is torn, the right muscle will begin to protrude the right side, with the left side remaining stable, and acting as a stationary pivot point. As a result, the mandible will pivot out and toward the left (the side of injury). Note that damage to muscles or innervation to one side of a tongue will also produce deviation toward the injured side.
Which of the tongue papillae are NOT vascular?
A. Fungiform
B. Filiform
C. Vallate
D. Foliate




The correct answer is B. The four major lingual papillae types are listed in the answer. Fungiform papillae are fairly numerous, vascular, have taste buds, and are found primarily on the anterior tongue. Filiform are avasacular, without taste buds, are the most numerous, and are found in rows. Vallate are largest, usually 7-9 in number, contain taste buds, and serous salivary glands of Von Ebner. Foliate are on the lateral surface of the tongue in ridges but are usually considered rudimentary and nonfunctional in man.
Mucopurulent exudate from maxillary sinusitis would be most likely to drain through an ostium in the
A. bulla ethmoidalis
B. hiatus semilunaris
C. inferior nasal meatus
D. sphenoethmoidal recess
E. superior nasal meatus




The correct answer is B. This patient has two risk factors for sinusitis: chronic rhinitis and allergy. She probably also has aspirin allergy, which is associated with the triad of nasal polyps, asthma, and sinusitis. In maxillary sinusitis, exudate may drain into the middle meatus through an ostium in the hiatus semilunaris, which contains openings to the frontal and maxillary sinuses and anterior ethmoidal cells.

The bulla ethmoidalis (choice A), also part of the middle meatus, contains an opening to the middle ethmoidal air cells.

The inferior nasal meatus (choice C) receives fluid from the nasolacrimal duct, which drains tears from the medial aspect of the orbit to the nasal cavity.

The sphenoethmoidal recess (choice D) is located above the superior concha and contains an opening for the sphenoid sinus.

The superior nasal meatus (choice E) is located above the superior concha and contains an opening for the sphenoid sinus.
Mucopurulent exudate from maxillary sinusitis would be most likely to drain through an ostium in the
A. bulla ethmoidalis
B. hiatus semilunaris
C. inferior nasal meatus
D. sphenoethmoidal recess
E. superior nasal meatus




The correct answer is B. This patient has two risk factors for sinusitis: chronic rhinitis and allergy. She probably also has aspirin allergy, which is associated with the triad of nasal polyps, asthma, and sinusitis. In maxillary sinusitis, exudate may drain into the middle meatus through an ostium in the hiatus semilunaris, which contains openings to the frontal and maxillary sinuses and anterior ethmoidal cells.

The bulla ethmoidalis (choice A), also part of the middle meatus, contains an opening to the middle ethmoidal air cells.

The inferior nasal meatus (choice C) receives fluid from the nasolacrimal duct, which drains tears from the medial aspect of the orbit to the nasal cavity.

The sphenoethmoidal recess (choice D) is located above the superior concha and contains an opening for the sphenoid sinus.

The superior nasal meatus (choice E) is located above the superior concha and contains an opening for the sphenoid sinus.
A wound to the posterior left axillary line, between the ninth and tenth rib, and extending approximately 5 cm deep, will most likely damage which organ?
A. Ascending colon
B. Duodenum
C. Left kidney
D. Left lobe of the liver
E. Spleen




The correct answer is E. The spleen follows the long axes of ribs 9 to 11 and lies mostly posterior to the stomach, above the colon, and partly anterior to the kidney. It is attached to the stomach by a broad mesenterial band, the gastrosplemic ligament. Therefore, it is the most likely organ of the group to be pierced by a sharp object penetrating just above rib 10 at the posterior axillary line. Note that the pleural cavity, and possibly the lower part of the inferior lobe of the lung, would be pierced before the spleen.

The ascending colon (choice A) is on the wrong side (the right) to be penetrated by a sharp instrument piercing the left side.

Most of the duodenum (choice B) is positioned too far to the right to be affected by this injury. Even the third part of the duodenum, which runs from right to left, would still be out of harm's way. In addition, the duodenum lies at about levels L1 to L3, placing it too low to be injured in this case.

The superior pole of the left kidney (choice C) is bordered by the lower part of the spleen. However, it is crossed by rib 12 and usually does not extend above rib 11. It would probably be too low and medial to be injured in this case because this penetration is at the posterior axillary line.

The left lobe of the liver (choice D) is positioned just beneath the diaphragm, just over and anterior to the stomach. The anterior positioning of this structure makes it an unlikely candidate for injury in this case. Even with deep penetration at the correct angle, it would not be penetrated before the spleen.
If a patient has a drooping right eyelid and a dilated right pupil, which of the following neural structures is most likely affected?
A. Cervical sympathetic chain
B. Facial nerve
C. Oculomotor nerve
D. Superior cervical ganglion
E. Trigeminal nerve




The correct answer is C. The oculomotor nerve innervates the levator palpebrae superioris, which elevates the eyelid. This nerve also innervates the inferior oblique muscles, as well as the superior, inferior, and medial rectus muscles. The oculomotor nerve also contains preganglionic parasympathetic fibers that synapse, in the ciliary ganglion, on postganglionic parasympathetic nerve fibers that innervate the sphincter pupillae muscle, which constricts the pupil. A lesion of the oculomotor nerve may therefore result in both drooping of the eyelid (ptosis) and dilation of the pupil (mydriasis).

The cervical sympathetic chain (choice A) contains preganglionic sympathetic nerve fibers, arising from the upper thoracic spinal cord, which ascend to the cervical sympathetic ganglia. A lesion of these nerves may result in Horner's syndrome, which includes a ptosis and miosis (pupillary constriction) and, often, anhidrosis (lack of sweating).

The facial nerve (choice B) innervates the muscles of facial expression, including the orbicularis oculi muscle. A lesion of this nerve may therefore result in the inability to close the eye.

The superior cervical ganglion (choice D) contains the cell bodies of postganglionic sympathetic nerves that innervate structures in the head. A lesion of this structure will cause Horner's syndrome.

The trigeminal nerve (choice E) provides sensory innervation to much of the head. A lesion of this nerve may interfere with the corneal blink reflex.
Several arterial branches penetrate into the distal end of the lesser curvature of the stomach. Which of the following arteries usually supplies these branches?
A. Left gastric
B. Left gastroepiploic
C. Right gastric
D. Right gastroepiploic
E. Short gastric




The correct answer is C. The arterial supply of the stomach is complex; it therefore has a good potential to appear on the NBDE. The right gastric artery supplies the distal lesser curvature.

The left gastric artery (choice A) supplies the proximal lesser curvature.

The left gastroepiploic artery (choice B) supplies the proximal greater curvature below the splenic artery.

The right gastroepiploic artery (choice D) supplies the distal greater curvature.

The short gastric artery (choice E) supplies the proximal greater curvature above the splenic artery.
A woman who recently gave birth has elevated prolactin levels. The gland responsible for secretion of this hormone is derived from which of the following structures?
A. Cerebral vesicle
B. Infundibulum
C. Neurohypophysis
D. Proctodeum
E. Rathke's pouch




The correct answer is E. The anterior pituitary produces prolactin. The structure originates from Rathke's pouch, which is itself a diverticulum of the roof of the stomodeum.

The cerebral vesicle (choice A) lies close to Rathke's pouch.

The infundibulum (choice B) comes in contact with Rathke's pouch at the fifth week of development.

The neurohypophysis (choice C) gives rise to the posterior pituitary.

The proctodeum (choice D) is also known as the anal pit.
During a mastectomy, the surgeon notes that the breast tumor has spread to involve the muscle layer immediately deep to the breast. Which muscle is involved?
A. External oblique
B. Pectoralis major
C. Platysma
D. Rectus abdominis
E. Sternocleidomastoid




The correct answer is B. The muscle deep to the breast is the pectoralis major. Breast cancer can invade this muscle but usually does not because the relatively thick, deep fascial layer serves as a barrier. Formerly, the pectoralis major was removed during radical mastectomy, leaving the patient with a major loss of function of the adjacent arm. The now more frequently used modified radical mastectomy spares the pectoralis major.

The external obliques (choice A) and rectus abdominis (choice D) muscles are in the abdomen.

The platysma (choice C) and sternocleidomastoid (choice E) muscles are in the neck.
Which of the following cell types is derived from neuroepithelial cells?
A. Astrocytes
B. Enterochromaffin cells
C. Melanocytes
D. Odontoblasts
E. Schwann cells




The correct answer is A. Astrocytes and oligodendrocytes are both derived from glioblasts, which, in turn, are derived from neuroepithelial cells. Other neuroepithelial cell derivatives include neuroblasts and ependymal cells. The astrocytes are the largest and most numerous glial cells. These cells are responsible for maintaining the blood-brain barrier, creating a three-dimensional framework for the central nervous system, performing repairs in damaged neural tissues, and controlling the interstitial environment.

All the other choices are derived from neural crest cells. Other neural crest derivatives include the neurons of the parasympathetic and sympathetic ganglia (including the adrenal medulla), the dorsal root ganglia of the peripheral nervous system, the sensory ganglia of cranial nerves V, VII, IX, and X, and the leptomeninges (pia and arachnoid).
Sensory innervation of the tongue, in order, from anterior to posterior, shows the effect of development of which branchial arches?
A. First, second, third
B. Third, second, first
C. Second, third, fourth
D. Fourth, third, second




The correct answer is A. Although many students memorize the innervation of the tongue, it can also be explained in a developmental way. In particular, the tongue is derived from the first three pharyngeal (branchial) arches, and the cranial nerves associated with each arch appear from anterior to posterior in the tongue innervation. Note that some texts also state that the most posterior sections of the tongue are developed form the fourth arch as well. The cranial nerves of each arch are as follows: Arch 1 (CN V), Arch 2 (CN VII), Arch 3 (CN IX), and Arch 4 (CN X). Note that general sensation from the anterior two thirds of the tongue is supplied by CN V, taste by CN VII, taste and general sensation from the posterior one third are supplied by CN IX, and some innervation of the most posterior tongue by CN X.
A mutation affecting the development of the diencephalon could interfere with the secretion of which of the following hormones?
A. Adrenocorticotrophic hormone (ACTH)
B. Epinephrine
C. Oxytocin
D. Prolactin
E. Thyroid-stimulating hormone (TSH)




The correct answer is C. The neurohypophysis (posterior pituitary) is derived from an evagination of the diencephalic neurectoderm. This structure is responsible for releasing oxytocin and vasopressin to the general circulation. Both hormones are synthesized in cell bodies contained within the hypothalamus. Oxytocin is a hormone produced by the hypothalamic cells and secreted into capillaries at the posterior pituitary. It stimulates smooth muscle contractions of the uterus or mammary glands in the female, but has no known function in the male.

ACTH (choice A), prolactin (choice D), and TSH (choice E) are all synthesized and released by the anterior pituitary, or adenohypophysis, which is derived from an evagination of the ectoderm of Rathke's pouch, a diverticulum of the primitive mouth. Remnants of this pouch may give rise to a craniopharyngioma in later life.

Epinephrine (choice B) is synthesized and released into the circulation by the adrenal medulla, a neural crest derivative.
Which of the following neurologic deficits would a large meningioma involving the brain's parasagittal region and the falx cerebri be expected to produce?
A. Altered taste
B. Leg paralysis
C. Loss of facial sensation
D. Ptosis
E. Unilateral deafness




The correct answer is B. A meningioma (a tumor of the meninges) of the parasagittal region and the falx cerebri would be located superiorly, between the two hemispheres. In this position, it could compress the sensory (postcentral gyrus) or motor cortex (precentral gyrus) supplying the lower extremities.

Taste (choice A) is supplied by cranial nerves VII, IX, and X. These nerves arise in the brainstem.

Facial sensation (choice C) is supplied by cranial nerve V, the nuclei of which are in the brainstem.

Ptosis (choice D) can be caused by a deficit in cranial nerve III, which arises from the brainstem.

Unilateral deafness (choice E) suggests damage to cranial nerve VIII, which arises from the brainstem.
Nissl bodies correspond to which of the following cytoplasmic organelles?
A. Golgi apparatus
B. Mitochondria
C. Nucleoli
D. Rough endoplasmic reticulum
E. Smooth endoplasmic reticulum




The correct answer is D. Rough endoplasmic reticulum present in neurons is called Nissl substance, or Nissl bodies. Nissl bodies stain intensely with basic dyes and are found in the cell body and proximal dendrites, but not in the axon hillock or axon.
Which of the following anatomic landmarks is the site of the anterior fontanelle in an infant?
A. Bregma
B. Coronal suture
C. Lambda
D. Pterion
E. Sagittal suture




The correct answer is A. Bregma represents the point where the coronal and sagittal sutures intersect; it is the site of the anterior fontanelle.

The coronal suture (choice B) lies between the frontal and parietal bones.

Lambda (choice C) represents the point where the sagittal and lambdoid sutures intersect; it is the site of the posterior fontanelle in infants.

The pterion (choice D) is the point on the lateral aspect of the skull where the greater wing of the sphenoid, parietal, frontal, and temporal bones converge. Recall that the pterion is the landmark for the middle meningeal artery and that a blow to the temple (e.g., as could occur in boxing) can lead to a middle meningeal arterial bleed and an epidural hemorhage.

The sagittal suture (choice E) is located between the two parietal bones.
A patient suffers severe head trauma, including a fracture in the region of the foramen ovale. Which of the following functional losses would most likely be related to this injury?
A. Loss of abduction of the eye
B. Loss of sensation over the forehead
C. Loss of sensation over the zygoma
D. Loss of taste sensation on the anterior 2/3 of the tongue
E. Paralysis of muscles of mastication




The correct answer is E. The mandibular nerve (V3) passes through the foramen ovale and may be injured by this fracture. The mandibular nerve is responsible for the innervation of all of the muscles of mastication: the masseter, the temporalis, the medial pterygoid, and the lateral pterygoid muscles, as well as the tensor palatini, tensor tympani, and mylohyoid.

Abduction of the eye (choice A) is produced by the lateral rectus muscle, which is innervated by the abducens nerve. The abducens nerve leaves the cranial cavity and enters the orbit by passing through the superior orbital fissure.

Sensation in the skin over the forehead (choice B) is provided by the ophthalmic division of the trigeminal nerve. The ophthalmic division leaves the cranial cavity and enters the orbit by passing through the superior orbital fissure.

Sensation in the skin over the zygoma (choice C) is provided by the maxillary division of the trigeminal nerve. The maxillary division leaves the cranial cavity by passing through the foramen rotundum and enters the pterygopalatine fossa.

Taste sensation on the anterior 2/3 of the tongue (choice D) is provided by the chordae tympani, a branch of the facial nerve. The facial nerve leaves the cranial cavity by passing into the internal auditory meatus and enters the temporal bone. The chordae tympani leaves the temporal bone by passing through the petrotympanic fissure and enters the infratemporal fossae.
A neonate is observed to have a cleft lip. Which of the following is the most likely etiology of this malformation?
A. Abnormal development of the third and fourth pharyngeal pouches
B. Bony defects of the malar bone and mandible
C. Failure of the maxillary processes and medial nasal swellings to fuse
D. Incomplete joining of the palatine shelves
E. Insufficient migration of neural crest cells




The correct answer is C. Cleft lip is caused by the failure of the maxillary processes to fuse with the medial nasal swellings.

Abnormal development of the third and fourth pharyngeal pouches (choice A) can give rise to DiGeorge syndrome, which is characterized by the underdevelopment or absence of several structures, including the thymus and parathyroids.

Bony defects of the malar bone and mandible (choice B) are associated with mandibulofacial dysostosis, which is mainly due to abnormal development of derivatives of the first arch. This condition is characterized by downward sloping palpebral fissures; hypoplasia of the malar and mandibular bones; macrostomia; high or cleft palate; abnormally shaped, low-set ears; and unusual hair growth patterns. Insufficient migration of neural crest cells (choice E) is an important factor as well.

When the palatine shelves fail to join together (choice D), cleft palate results. Cleft lip and cleft palate commonly co-occur.
Which of the following is the embryologic precursor of the fibrous remnant that runs in a fissure on the visceral surface of the liver?
A. Ductus venosus
B. Hepatic portal vein
C. Lateral splanchnic artery
D. Ligamentum venosum
E. Vitelline vein




The correct answer is A. The structure described is the ligamentum venosum (choice D), which is derived from the ductus venosus.

The hepatic portal vein (choice B) is derived from the vitelline vein (choice E).

The lateral splanchnic arteries (choice C) arise from each side of the dorsal aorta. They supply intermediate mesoderm and derivatives and give rise to renal, suprarenal, phrenic, and testicular or ovarian arteries.
Which of the following veins empties into the left renal vein?
A. Hepatic
B. Left suprarenal
C. Right gonadal
D. Right renal




The correct answer is B. The left suprarenal vein empties into the left renal vein, which crosses the vertebral column to reach the inferior vena cava. The left renal vein also receives the left gonadal vein.

Hepatic veins (choice A) convey blood from the liver to the inferior vena cava as it approaches the caval orifice of the diaphragm.

The right gonadal vein (testicular in male, ovarian in female; choice C) drains directly to the inferior vena cava.

The left and right (choice D) renal veins enter the inferior vena cava at vertebral level L2.
The postganglionic signals carrying the impulses to constrict arterioles are transmitted along which of the following fiber types?
A. A-Δ fibers
B. B fibers
C. C fibers
D. Ia fibers
E. Ib fibers




The correct answer is C. There are two systems currently used for classifying nerve fibers. The first system groups both sensory and motor fibers together, describing A-α±, A-β, A-Γ, A-Δ, B, and C fibers. Another system relates only to sensory fibers, describing Ia, Ib, II, III, and IV categories. Both classification schemes begin with large, myelinated fibers, progressing to finer, unmyelinated fibers.

The C fiber (or IV fibers) is the only type of fiber that is unmyelinated. Remember that preganglionic neurons are myelinated, but postganglionic neurons are unmyelinated. Neurons that carry slow pain and temperature information are also classified as C fibers. See the table below for more information.

A-α± or Ia (choice D) Alpha motor neurons, primary afferents of muscle spindles

A-α± or Ib (choice E) Golgi tendon organ afferents, touch and pressure

A-β or II Secondary afferents of muscle spindles, touch and pressure

A-Γ Gamma motor neurons

A-Δ (choice A) Touch, pressure, pain and temperature (fast)

B (choice B) Preganglionic autonomic, visceral afferents

C (choice C) IV Postganglionic autonomic, pain and temperature (slow)
Injury to which nerve would result in wrist drop, and inability to make a tight fist, even though all finders can be flexed?
A. Axillary nerve
B. Long thoracic nerve
C. Median nerve
D. Musculocutaneous nerve
E. Radial nerve




The correct answer is E. Physiologically, the distribution of the radial nerve is as follows: extensor muscles on the arm and forearm (triceps bracii, brachioradialis, extensor carpi radials, and extersor carpi ulnaris), ditigal extensors and abductor pollicis, and skin over the posterolateral surface of the arm. The radial nerve lies in the musculospiral groove of the humerus and is subject to injury in association with a fracture of the midshaft of the humerus. The radial nerve innervates the extensor muscles of the forearm, including the muscles that extend the wrist; paralysis of these muscles results in a wrist drop. Although the muscles responsible for flexing the digits are not innervated by the radial nerve, the making of a tight fist requires that the wrist be stabilized with the wrist extensors.

The axillary nerve (choice A) leaves the axilla through its posterior wall and is not located in the region of the midshaft of the humerus. The axillary nerve innervates the deltoid and the teres minor muscles, neither of which have any function in the hand.

The long thoracic nerve (choice B) lies against the chest wall and is not subject to injury as a result of a humeral fracture. The long thoracic nerve innervates the serratus anterior muscle.

The median nerve (choice C) passes though the arm, but is not in close contact with the humerus in the midshaft region. The nerve is separated from the bone by the brachialis muscle. The median nerve innervates many muscles of the anterior compartment of the forearm responsible for flexion of the wrist and digits.

The musculocutaneous nerve (choice D) is not in contact with the humerus. It innervates the muscles in the anterior compartment of the arm. These muscles have no function in the hand.
After falling on his laterally outstretched arm, a patient suffered a dislocation of the glenohumeral joint. Which of the following nerves is most likely to have been injured from this dislocation?
A. Axillary nerve
B. Dorsal scapular nerve
C. Lateral pectoral nerve
D. Medial pectoral nerve
E. Suprascapular nerve




The correct answer is A. When the head of the humerus dislocates from the glenohumeral joint, it exits inferiorly, where the joint capsule is the weakest. Immediately inferior to the glenohumeral joint, the axillary nerve exits from the axilla by passing through the quadrangular space. At this location, the downward movement of the head of the humerus can stretch the axillary nerve. The axillary nerve innervates the deltoid muscle after leaving the axilla.

The dorsal scapular nerve (choice B) passes along the medial border of the scapula to innervate the rhomboid muscles. The nerve does not pass in the region of the glenohumeral joint.

The lateral and medial pectoral nerves (choices C and D) branch from the lateral and medial cords of the brachial plexus, respectively, and exit through the anterior wall of the axilla to innervate the pectoralis major and minor. These nerves do not pass in the region of the glenohumeral joint.

The suprascapular nerve (choice E) is a branch of the upper trunk of the brachial plexus and passes over the superior border of the scapula to innervate the supraspinatus and infraspinatus muscles. This nerve does not pass in the region of the glenohumeral joint.
Which of the following areas of the heart have most likely suffered ischemic necrosis following a myocardial infarct due to blockage of the left circumflex artery in a patient that has left dominant coronary circulation?
A. Apex of left ventricle and anterior portion of septum
B. Lateral left ventricular wall and posterior portion of the septum
C. Lateral wall of the left ventricle only
D. Posterior portion of the septum only
E. Right ventricular wall




The correct answer is B. A right dominant coronary circulation is present when the posterior descending branch originates from the right coronary artery (80% of individuals). On the contrary, the posterior descending artery originates from the left circumflex artery in a left dominant circulation (20% of individuals). The posterior descending branch gives blood to the posterior half of the interventricular septum. Occlusion of the left circumflex artery in a left dominant circulation will therefore lead to ischemic necrosis in the left ventricular wall and the posterior interventricular septum.

The apex of the left ventricle (choice A) is dependent on the anterior descending branch; thus, occlusion of the left circumflex does not affect this portion of the left ventricle.

Infarction of the lateral (free) wall alone (choice C) will result from occlusion of the circumflex in a right dominant circulation.

An isolated infarct of the posterior interventricular septum (choice D) arises from occlusion of the posterior descending branch.

Isolated infarcts of the right ventricular wall (choice E) are very rare and would be caused by occlusion of branches of the right coronary artery.
A man is injured when a bony fragment penetrates the lateral portion of the dorsal columns in his neck. Which of the following functions would most likely be affected by a lesion at this site?
A. Fine motor control of the ipsilateral fingers
B. Motor control of the contralateral foot
C. Sweating of the ipsilateral face
D. Proprioception from the ipsilateral leg
E. Vibratory sense from the ipsilateral arm




The correct answer is E. At this level, the lateral portion of the dorsal columns (funiculus) is composed of the fasciculus cuneatus. Axons carrying tactile, proprioceptive, and vibratory information from the ipsilateral arm enter the spinal cord via the dorsal root, ascend the cord in the fasciculus cuneatus, and synapse in the nucleus cuneatus of the caudal medulla. Secondary neurons from this nucleus give rise to internal arcuate fibers, which decussate and ascend to the thalamus (ventral posterolateral nucleus, VPL) as the medial lemniscus. Tertiary neurons from the VPL project to the ipsilateral somatosensory cortex. Because the fibers that carry this information do not cross until they reach the medulla, damage to the fasciculus cuneatus would result in a deficit in tactile, proprioceptive, and vibratory sense in the ipsilateral arm.

Fine motor control of the fingers (choice A) would be carried principally by the ipsilateral lateral corticospinal tract in the lateral funiculus of the cord.

Motor control of the contralateral foot (choice B) is carried by the ipsilateral corticospinal tract in the lateral funiculus of the cord.

Hemianhidrosis (lack of sweating) over half of the face (choice C) could be produced by interruption of sympathetic innervation to the face, which projects from the hypothalamus to the intermediolateral cell column at levels T1 and T2. It descends in the lateral funiculus of the cord. Interruption of this tract results in Horner's syndrome (miosis, ptosis, and hemianhidrosis).

Proprioception from the ipsilateral leg (choice D) is carried by the fasciculus gracilis in the medial part of the dorsal columns.
Damage from extension of a tumor in the body of the right lateral ventricle would most likely affect which other brain structures?
A. Caudate nucleus
B. Cerebellum
C. Hippocampus
D. Hypothalamus
E. Pons




The correct answer is A. Tumors of the ventricular system of the brain can affect the brain tissue either directly, via pressure on or invasion into a physically close structure, or indirectly, by obstructing cerebral spinal fluid (CSF) flow and causing hydrocephalus. The caudate nucleus is a C-shaped structure that composes part of the wall of the lateral ventricle throughout its extent. The only structure listed that is adjacent to the body of the lateral ventricle and would therefore be directly affected by the large tumor described in the question is the caudate nucleus. Anatomically, the caudate nucleus has a massive head and a slender curving tail that follows the curve of the lateral ventricle. The head of the caudate nucleus lies superior to the lentiform nucleus.

The cerebellum (choice B) overlies the fourth ventricle.

The hippocampus (choice C) is adjacent to the inferior (temporal) horn of the lateral ventricle.

The hypothalamus (choice D) abuts the third ventricle.

The pons (choice E) forms part of the floor of the fourth ventricle.
Examination of a karyotype taken from a metaphase preparation reveals the presence of an extra chromosome. Which of the following is the most common mechanism of producing this phenomenon?
A. Balanced translocation
B. Chromosomal breakage
C. Fertilization by two sperm
D. Nondisjunction
E. Unbalanced translocation




The correct answer is D. Nondisjunction can occur in both meiosis and mitosis and refers to a failure of paired chromosomes to separate and go to different daughter cells. When this happens, one daughter cell gets an extra chromosome, and the other daughter cell is "short" one chromosome.

A balanced translocation (choice A) is an exchange of genetic material between nonhomologous chromosomes that preserves all critical genetic material.

Chromosomal breakage (choice B) produces fragmented chromosomes and can contribute to tumorigenesis.

Fertilization by two sperm (choice C) produces triploidy and is seen in many spontaneously aborted fetuses.

An unbalanced translocation (choice E) occurs when nonhomologous chromosomes exchange genetic material with a net loss or gain of critical genetic material.
To anesthetize an area under the nailbed of the index finger, which nerve should be injected?
A. Axillary
B. Median
C. Musculocutaneous
D. Radial
E. Ulnar




The correct answer is B. The tumor in question is probably a benign glomus tumor, which is notorious for producing pain far out of proportion to its small size. The question is a little tricky because the most distal aspect of the dorsal skin of the fingers, including the nail beds, is innervated by the palmar digital nerves rather than the dorsal digital nerves. Specifically, the median nerve, through its palmar digital nerves, supplies the nail beds of the thumb, index finger, middle finger, and half the ring finger.

The axillary nerve (choice A), musculocutaneous nerve (choice C), and radial nerve (choice D) do not supply the nail beds. The radial nerve does supply the more proximal skin of the back of the index finger.

The ulnar nerve (choice E) supplies the nail beds of the small finger and half of the ring finger.
An individual has an eye that is persistently directed toward his nose. A lesion of which of the following nerves could produce this finding?
A. CN III
B. CN IV
C. CN V
D. CN VI
E. CN VII




The correct answer is D. Cranial nerve VI is the abducens nerve, which supplies the abductor of the eye, the lateral rectus. A paralysis of the lateral rectus leads to unopposed adduction, causing the eye to point toward the nose.

Cranial nerve III (choice A) is the oculomotor nerve, which supplies all of the muscles of the eye, except the superior oblique and lateral rectus. Paralysis of III would impair adduction, not abduction, of the eye. The eye would tend to rotate downward and outward.

Cranial nerve IV (choice B) is the trochlear nerve, which supplies the superior oblique muscle. This muscle serves to depress and abduct (down and out) the eyeball. Paralysis of IV tends to produce double vision but does not cause an obvious deficit in conjugate gaze without careful testing.

Cranial nerve V (choice C) is the trigeminal nerve, which is a mixed sensory and motor nerve that supplies the face. It provides sensory innervation to the face and innervates the muscles of mastication. It does not innervate the eye muscles.

Cranial nerve VII (choice E) is the facial nerve, which innervates the muscles of facial expression but not the muscles of the orbit. CN VII is additionally involved in salivation, lacrimation, taste, and general sensation from the external ear.
The patient closes both of his eyelids in response to the left eye being touched with a thin wisp of cotton as he looks to the right. Which of the following cranial nerves is responsible for the motor limb of this reflex?
A. Abducens
B. Facial
C. Optic
D. Trigeminal
E. Trochlear




The correct answer is B. The corneal reflex is tested by touching the cornea of one eye with a cotton wisp; this causes both eyes to close. The afferent, or sensory, component of the corneal reflex is mediated by the ophthalmic division of the ipsilateral trigeminal nerve (V-1). The efferent, or motor, component is mediated by the facial nerve (CN VII), bilaterally.

The abducens nerve (CN VI; choice A) innervates the lateral rectus muscles, which abduct the eyes.

The optic nerve (CN II; choice C) is responsible for vision, providing the afferent limb of the pupillary light reflex. The Edinger Westphal nucleus mediates part of this reflex.

The trigeminal nerve (CN V; choice D) is responsible for the afferent limb of the corneal reflex. It also innervates the muscles of mastication and provides sensory innervation to the face.

The trochlear nerve (CN IV; choice E) innervates the superior oblique muscles, which depress, intort, and abduct the eyes.
Which of the following veins may be anastomosed to accomplish a porto-caval shunt?
A. Left renal vein and left testicular vein
B. Right renal vein and right suprarenal vein
C. Splenic vein and left renal vein
D. Superior mesenteric vein and inferior mesenteric vein
E. Superior mesenteric vein and splenic vein




The correct answer is C. The splenic vein drains directly into the portal vein. The left renal vein drains directly into the inferior vena cava. Anastomosis of these veins would allow blood from the portal vein to drain retrograde though the splenic vein into the renal vein and then into the inferior vena cava.

The left renal vein (choice A) drains directly into the inferior vena cava. The left testicular vein drains directly into the left renal vein. Thus, these veins are already in communication, and neither vein is part of the portal venous system.

The right renal vein (choice B) drains directly into the inferior vena cava. The right suprarenal vein also drains directly into the inferior vena cava. Thus, neither vein is part of the portal venous system.

The superior mesenteric vein (choice D) drains directly into the portal vein. The inferior mesenteric vein drains into the splenic vein, which then drains into the portal vein. Thus, neither vein is part of the caval venous system.

The superior mesenteric vein (choice E) drains directly into the portal vein. The splenic vein also drains directly into the portal vein. Thus, neither vein is part of the caval venous system.
In a histological section of a normal ovary, an oocyte is surrounded by several layers of follicular cells. A small antrum is present. Which of the following is the correct term for the entire structure, composed of the oocyte, follicular cells, and antrum?
A. Corpus luteum
B. Graafian follicle
C. Primary follicle
D. Primordial follicle
E. Secondary follicle




The correct answer is E. Follicles in different stages of maturation have different appearances. The most primitive follicles, primordial follicles (choice D), are inactive reserve follicles that contain primary oocytes (arrested in prophase of the first meiotic division) surrounded by a single layer of flattened follicular cells. Primary follicles (choice C), the next stage, are slightly larger and contain a central oocyte surrounded by one or several cuboidal follicular cells. When several small spaces in the follicular mass fuse to form the antrum (follicular cavity), the follicle is termed a secondary follicle (choice E). The secondary follicles continue to enlarge and develop a more complex structure that includes cumulus oophorus, corona radiata, theca interna, theca externa, and zona pellucida. The Graafian follicle (choice B) is the mature form of the follicle, which extends through the entire cortex and bulges out at the ovarian surface. After it ruptures and releases the ovum, the corpus luteum (choice A)develops and secretes progesterone as the cells of the follicle and the theca interna cells enlarge, become epithelioid, and secrete estrogen. If pregnancy occurs, it is maintained throughout the pregnancy; if pregnancy does not occur, the corpus luteum eventually degenerates.
Within which of the following peritoneal structures is the cystic artery located?
A. Falciform ligament
B. Gastrocolic ligament
C. Gastrohepatic ligament
D. Hepatoduodenal ligament
E. Splenorenal ligament




The correct answer is D. The hepatoduodenal ligament is the portion of the lesser omentum that connects the liver to the first part of the duodenum. Within the hepatoduodenal ligament are found the proper hepatic artery and its branches, the common bile duct and its branches, and the portal vein. The cystic artery is usually a branch of the right hepatic artery, which is a branch of the proper hepatic artery.

The falciform ligament (choice A) is a mesentery that connects the liver to the anterior abdominal wall. Within the free edge of the falciform ligament is found the round ligament of the liver (ligamentum teres hepatis), the adult remnant of the umbilical vein.

The gastrocolic ligament (choice B) is the portion of the greater omentum between the greater curvature of the stomach and the transverse colon. The gastroepiploic arteries lie within the gastrocolic ligament along the greater curvature of the stomach.

The gastrohepatic ligament (choice C) is the portion of the lesser omentum between the liver and the lesser curvature of the stomach. The right and left gastric arteries are within the gastrohepatic ligament along the lesser curvature of the stomach.

The splenorenal ligament (choice E) is the mesentery that connects the spleen to the posterior abdominal wall. The splenic artery and splenic vein are within the splenorenal ligament.
While performing a tonsillectomy, the surgeon accidentally damages the glossopharyngeal nerve. Which of the following functional losses is likely to result from this injury?
A. Loss of sensation on the posterior 1/3 of the tongue
B. Loss of taste on the anterior 2/3 of the tongue
C. Paralysis of the constrictor muscles of the palate
D. Paralysis of the muscles of the soft palate
E. Paralysis of the muscles of the tongue




The correct answer is A.The glossopharyngeal nerve, or cranial nerve IX, lies in the tonsillar fossa. This nerve provides general sensory innervation to the mucosa of the pharynx, and general sensory and taste sensation to the mucosa of the posterior 1/3 of the tongue. The nerve also innervates the stylopharyngeus muscle, but this innervation occurs proximal to the point at which the nerve crosses the tonsillar fossa. The full anatomical picture of the glossopharyngeal nerve is as follows: It originates from the posterior 1/3 of the tongue, part of the pharynx and palate, as well as the carotid arteries of the neck (sensory). It passes through the jugular foramen of the temporal bone to foramen lacerum between the occipital and temporal bones. The destination is to the sensory nuclei of the medulla oblongata, pharyngeal muscles involved in swallowing, as well as the parotid.

Taste sensation to the anterior 2/3 of the tongue (choice B) is provided by the facial nerve. The chordae tympani branch of the facial nerve travels with the lingual nerve to reach the tongue.

The constrictor muscles of the pharynx (choice C) are innervated by the vagus nerve. The vagus nerve fibers contribute to the pharyngeal plexus.

The muscles of the soft palate (choice D) are mostly innervated by the vagus nerve. The tensor palati is innervated by the trigeminal nerve (V3).

The muscles of the tongue (choice E) are innervated by the hypoglossal nerve.
If the lateral wall of the left ventricle receives insufficient blood supply, which artery is most likely involved
A. Left anterior descending
B. Left circumflex
C. Left main coronary
D. Right coronary




The correct answer is B. In some patients with coronary artery disease, thallium stress tests may be performed instead of cardiac catheterization to determine the vessels involved and the extent of occlusion. The left circumflex (LCx) branch supplies the lateral wall of the left ventricle; in 10% of the population, it also supplies the posterior wall and AV node.

The left anterior descending (LAD) branch of the left coronary artery (choice A) supplies the anterior wall of the left ventricle and the anterior portion of the interventricular septum.

The left main coronary artery (choice C) gives rise to both LCx and the LAD.

The right coronary artery (RCA; choice D) supplies the right ventricle; in 90% of the population, it supplies the AV node and posterior and inferior walls of the left ventricle.
Which of the following products would mast cells be most likely to secrete?
A. Bradykinin
B. Complement factor 3a
C. Histamine
D. Interleukin-2
E. Nitric oxide




The correct answer is C. The cells in question are mast cells, which play an important role in IgE-mediated allergic responses. It is also important to note that histamine is found in basophils. They are a normal (minor) constituent of dermal skin and are most definitely identified with stains, such as the Giemsa stain, that highlight the granularity of their cytoplasm (mast cells are closely related to blood basophils). Mast cells secrete histamine, serotonin, many leukotrienes, and platelet-aggregating factor (PAF). Their histamine secretion after IgE stimulation helps to trigger the acute inflammatory part of the allergic response.

Bradykinin (choice A) is a nonapeptide found in the plasma. It is a potent vasodilator and stimulates pain receptors.

Complement factor 3a (choice B) is a plasma protein that induces vascular leakage as part of the acute inflammatory response.

Interleukin-2 (choice D) is secreted by lymphocytes and augments the immune response.

Nitric oxide (choice E) is released by macrophages and endothelium. It causes vasodilation and cytotoxicity.
Which of the following gastrointestinal hormones is structurally related to secretin?
A. Cholecystokinin
B. Gastrin
C. Glucagon
D. Somatostatin
E. Substance P




The correct answer is C. Glucagon, secretin, and vasoactive intestinal polypeptide (VIP) are all structurally related.

Cholecystokinin (choice A) and gastrin (choice B) form another family of related hormones.

Neither somatostatin (choice D) nor substance P (choice E) are structurally related to secretin. In addition to their role in the gastrointestinal system, both hormones are also present in the brain.
In which of the following regions of the liver is the oxidase P450 system located?
A. Bile ducts
B. Intermediate zone
C. "Ito" cells
D. Pericentral vein zone
E. Periportal zone




The correct answer is D. The liver can be divided into three zones. Zone 1 is periportal, zone 2 is intermediate (between 1 and 3), and zone 3 is associated with the central vein. Zone 3 contains the P450 oxidase enzyme system and is most sensitive to injury. The CP450 system is very important to be familiar with because it is the major site of drug interaction. For example, the macrolides, which are commonly used in dentistry in penicillin-allergic patients are potent hepatic enzyme inhibitors, as are antifungals like ketoconazole and fluconazole.

The P450 system is not located in the bile ducts (choice A).

The intermediate zone (choice B), or zone 2, is the second area most sensitive to ischemic injury.

"Ito" cells (choice C) are the fat-containing mesenchymal cells located in the space of Disse. This is the site of vitamin A storage.

The periportal zone (choice E), or zone 1, is the area most sensitive to toxic injury. This is the area that shows infiltration with hepatitis.
Which part of the hypothalamus controls satiety, and would lead to obesity if destroyed?
A. Lateral nucleus
B. Septal nucleus
C. Suprachiasmatic nucleus
D. Supraoptic nucleus
E. Ventromedial nucleus




The correct answer is E. The ventromedial nucleus is thought to be the satiety center of the brain. Bilateral destruction leads to hyperphagia, obesity, and savage behavior. Stimulation inhibits the urge to eat.

Destruction of the lateral nucleus (choice A) results in starvation, whereas stimulation of this nucleus induces eating.

Destruction of the septal nucleus (choice B) produces aggressive behavior.

The suprachiasmatic nucleus (choice C) receives direct input from the retina and plays a role in controlling circadian rhythms.

The supraoptic nucleus (choice D), along with the periventricular nucleus, regulates water balance and produces antidiuretic hormone (ADH) and oxytocin.
A premature infant develops a progressive difficulty with breathing over the first few days of life. Deficient surfactant synthesis by which of the following cell types may have contributed to the infant's respiratory problems?
A. Alveolar capillary endothelial cells
B. Bronchial mucous cells
C. Bronchial respiratory epithelium
D. Type I pneumocytes
E. Type II pneumocytes




The correct answer is E. This child has neonatal respiratory distress syndrome (hyaline membrane disease), which is caused by the inability of the immature lungs to synthesize adequate amounts of surfactant. Surfactant, which reduces surface tension, helps keep alveoli dry, and aids in expansion of the lungs, is synthesized by type II pneumocytes.

Alveolar capillary endothelial cells (choice A) are important in maintaining the capillary structure and permitting flow of gases into and out of the blood stream.

Bronchial mucous cells (choice B) produce the usually thin (in healthy individuals) coat of mucus that lines the bronchi.

The ciliated bronchial respiratory epithelium (choice C) is responsible for moving the dust-coated mucus layer out of the bronchi.

Type I pneumocytes (choice D) are the squamous cells that line alveoli and permit easy gas exchange. These cells tend to be immature (and thick) in premature infants but do not produce surfactant.
Inability to extend the hand at the wrist is often accompanied by loss of sensation to which area?
A. Lower part of the back of the little finger
B. Lower part of the back of the thumb
C. Palmar aspect of the forefinger
D. Palmar aspect of the little finger
E. Thumbnail bed




The correct answer is B. Associate "wrist drop" with a radial nerve lesion. The radial nerve also provides sensation to the dorsal aspect of the radial side of the hand, including the lower part of the dorsal aspect of the thumb and all the fingers, except for the little finger and half of the ring finger.

The lower part of the back of the little finger (choice A) is supplied by the ulnar nerve.

The palmar aspects of all of the fingers (choices C and D) are supplied by the median nerve, as are the nailbeds, including the thumbnail bed(choice E).
At which of the following locations might a penetrating wound to the heart damage the AV node?
A. Apex of the heart
B. Interatrial septum
C. Interventricular septum
D. Wall of the right atrium
E. Wall of the left atrium




The correct answer is B. This question is essentially asking "where is the AV node located on the heart?" The atrioventricular (AV) node is in the subendocardium of the interatrial septum. The AV node can also be described as being within the floor of the right atrium near the opening of the coronary sinus. From the AV node, the Purkinje fibers of the atrioventricular bundle enter the interventricular septum to carry impulses to the ventricle. The function of the AV node is to retard the conduction of the cardiac impulses so that ventricular systole occurs after atrial systole.

The apex of the heart (choice A) is composed of ventricular wall musculature of the left ventricle. There is no nodal tissue in this region.

The interventricular septum (choice C) contains the common bundle (AV bundle) and the right and left bundle branches of the cardiac conduction system. These bundles are composed of Purkinje fibers, which are specialized cardiac muscle cells.

The sinoatrial (SA) node is located in the right atrial wall (choice D), near the entrance of the superior vena cava. The SA node contains specialized cardiac muscle cells that depolarize more rapidly than do typical cardiac muscle cells, thereby serving as the pacemaker of the heart.

The wall of the left atrium (choice E) does not contain any nodal cells.
Following thyroid surgery, hoarseness and difficulty speaking can be attributed to damage to a branch of which cranial nerve?
A. Facial
B. Glossopharyngeal
C. Hypoglossal
D. Trigeminal
E. Vagus




The correct answer is E. The recurrent laryngeal nerves are branches of the vagus (CN X) and supply all intrinsic muscles of the larynx, except the cricothyroid. The right recurrent laryngeal nerve recurs around the right subclavian artery. The left recurrent laryngeal nerve recurs in the thorax around the arch of the aorta and ligamentum arteriosum. Both nerves ascend to the larynx by passing between the trachea and esophagus, in close proximity to the thyroid gland. The recurrent laryngeal nerves are therefore particularly vulnerable during thyroid surgery, and damage may cause extreme hoarseness.

The facial nerve (choice A) innervates the muscles of facial expression, the stapedius muscle, and the lacrimal, submandibular, and sublingual glands. It also mediates taste sensation from the anterior two-thirds of the tongue.

The glossopharyngeal nerve (choice B) innervates the stylopharyngeus muscle and the parotid gland. Visceral afferents supply the carotid sinus baroreceptors and carotid body chemoreceptors and mediate taste from the posterior one-third of the tongue. Somatosensory fibers supply pain, temperature, and touch information from the posterior one-third of the tongue, upper pharynx, middle ear, and eustachian tube.

The hypoglossal nerve (choice C) innervates the intrinsic muscles of the tongue, the genioglossus, hypoglossus, and styloglossus muscles.

The trigeminal nerve (choice D) receives sensory information from the face and also innervates the muscles of mastication.
A knife wound to the face selectively damages the facial nerve. Salivation from which of the following would be impaired?
A. Parotid and sublingual glands
B. Parotid and submandibular glands
C. Parotid gland only
D. Sublingual gland only
E. Submandibular and sublingual glands




The correct answer is E. The facial nerve (CN VII) is intimately related anatomically to the parotid gland but controls salivation from both the submandibular and sublingual glands. The sublingual salivary glands are located beneath the mucous membrane on the floor of the mouth. The submandibular salivary glands are found in the floor of the mouth along the inner surface of the mandible within the manibular groove.

The parotid gland (choices A, B, and C) is innervated by the glossopharyngeal nerve (CN IX). The parotid glands are located inferiorly to the zygomatic arch beneath the skin covering the lateral and posterior surfaces of the mandible. Although the facial nerve most often runs through the parotid gland, it does not supply any innervation there.
Which area of the stomach has numerous, normal cuboidal-to-columnar cells with apical, membrane-bound secretion granules in the gastric glands?
A. Cardiac region
B. Columns of Morgagni
C. Fundic region
D. Greater omentum
E. Pyloric region




The correct answer is C. The pathologist saw normal chief cells, which are abundant in the body and fundus of the stomach. Chief cells secrete pepsinogen, which is stored in apical membrane-bound granules. The body and fundus of the stomach contain high concentrations of four other types of cells in the epithelium. The parietal (oxyntic) cells are large, pyramidal, and acidophilic with central nuclei (like a "fried egg"). They make and secrete hydrochloric acid. The mucous neck cells secrete mucus and appear clear. The enteroendocrine cells have affinity for silver stains and exhibit a positive chromaffin reaction; these cells synthesize amines, polypeptides, or proteins.

The cardiac region (choice A) is a narrow, circular band at the transition between the esophagus and stomach, consisting of shallow gastric pits and mucous glands. It does not normally contain an abundance of chief cells.

The columns of Morgagni (choice B) are found in the rectum, not in the stomach. These are mucous membrane infoldings in the submucosa of the proximal anal canal. They would not contain chief cells.

The greater omentum (choice D) is a four-layered fold of peritoneum that hangs from the greater curvature of the stomach and attaches to the transverse colon. It would not contain chief cells.

The pyloric region (choice E) has deep gastric pits into which tubular glands open. The predominant secretion is mucus. It does not normally contain an abundance of chief cells.

Note that in this question you could have automatically eliminated choices B and D because they are not gastric structures. If nothing else, you have improved your guessing odds to 33%.
A man suffers a direct blow to the right eye, producing eye pain and severe bleeding from the nose. X-rays reveal a blowout fracture of the floor of the orbit. Involvement of which of the following structures is suggested by the patient's symptoms?
A. Ethmoid air cells
B. Maxillary sinus
C. Oral cavity
D. Pterygopalatine fossa
E. Sphenoid sinus




The correct answer is B. The floor of the orbit is also the roof of the maxillary sinus. The bone separating the orbit from the maxillary sinus is frequently quite thin and may fracture from increased intraorbital pressure caused by a direct blow to the eye. The subsequent bleeding into the maxillary sinus will result in blood draining from the sinus into the nasal cavity. The maxillary sinuses are the largest sinuses in the skull. These sinuses lighten the portion of the maxillae above the teeth and produce mucous secretions that help to "flush" the interior surfaces of the nasal cavities.

The ethmoid air cells (choice A) are medial to the orbit. This bone is also frequently quite thin.

The oral cavity (choice C) is separated from the orbit by the maxillary sinus.

The pterygopalatine fossa (choice D) is posteromedial to the orbit. The medial portion of the inferior orbital fissure communicates between the orbit and the pterygopalatine fossa.

The sphenoid sinus (choice E) is posteromedial to the orbit. There is no communication between the sphenoid sinus and the orbit.
A parent is startled to find that squeezing her infant's shoulders with her hand can bring them nearly together in front of the body. Congenital absence of which of the following bones should be suspected?
A. Clavicles
B. First ribs
C. Humeri
D. Scapulae
E. Sternum




The correct answer is A. Congenital absence of the clavicles is a fairly common and not particularly troublesome anomaly, although affected children have some trouble with sports. It can be confirmed radiologically. In rare (particularly, obese) patients, the defect is not diagnosed until a routine chest x-ray performed for other reasons demonstrates the anomaly.

Congenital defects in ribs (choice B) can occur but would not allow the shoulders to be brought together.

Congenital absence of the humeri (choice C) would produce absent or markedly shortened upper arms.

Congenital anomalies of the scapulae (choice D) are rare and would affect posterior shoulder movement.

Congenital absence of part of the sternum (choice E) produces a hole in the lower part of the body of the sternum; complete absence is rare.
Which of the following is the 3-carbon end-product of fatty acid metabolism?
A. Acetoacetate
B. Acetone
C. β-Hydroxybutyrate
D. Diacylglycerol
E. Monoacylglycerol




The correct answer is B. The theme of this question is lipid metabolism. Ketone bodies [acetoacetate (choice A), acetone (choice B), and β-hydroxybutyrate (choice C)] are lipid breakdown products. The levels of these molecules can increase in poorly controlled diabetes mellitus or starvation. Acetone is the 3-carbon fragment of this group.

Monoacylglycerols (choice E) and diacylglycerols (choice D) are neutral fats made from glycerol with one or two fatty acids esterified to the glycerol, respectively.
Folate plays a role in single-carbon unit transfer in the synthesis of nucleotides. Which of the following nucleotides require folate for synthesis?
A. Adenosine, cytosine, and uracil
B. Adenosine, guanine, and thymidine
C. Adenosine, guanine, and uracil
D. Cytosine, thymidine, and uracil
E. Guanine, thymidine, and uracil




The correct answer is B. Folate is involved in the transfer of carbons 2 and 8 of the purine nucleus (affecting adenosine and guanosine) and the 5-methyl group of thymidine. This means that folate is required for synthesis of 3 of the 4 nucleic acid bases of DNA and 2 of the 4 nucleic acid bases in RNA. It is thus no wonder that folate deficiency affects so many tissues with high mitotic rate. Megaloblastic changes analogous to those seen in erythrocytes and their precursors can also be seen in other cells produced by bone marrow (neutrophil, eosinophil, basophil, and macrophage and megakaryocyte lines) and in epithelia throughout the body, including skin, mucous membranes such as the mouth and vagina (where the changes can be seen on pap smear), stomach, intestinal linings, and cells from lung or liver. Similar megaloblastic changes are observed throughout the body when cobalamin (vitamin B12 ) deficiency is present, since cobalamin plays a role in methionine synthesis, which is the source of the one-carbon unit "active-formate." Cobalamin is also invovled in the conversion of methylmalonic acid to succinic acid and is required to maintain the integrity of nerve cells via an unknown biochemical pathway.
A patient is taking hormone supplements. The hormone binds to a receptor in the cell membrane, which activates tyrosine kinase activity. Which of the following hormones is this patient taking?
A. Calcitrol
B. Thyroxine
C. Retinoic acid
D. Insulin
E. Protein synthesis




The correct answer is D.Insulin, a water-soluble hormone, binds to receptors in the cell membrane. Receptor tyrosine kinase is activated, leading to protein phosphorylation. Tyrosine kinase receptors are also involved in signaling by growth factors like PDGF (platelet derived growth factors) and EGF (epidermal growth factor). Choice A, B and C - are all lipid soluble harmones, which diffuse through the cell membrane, and bind to their respective receptors inside the cell. The hormone-receptor complex then binds to hormone receptor elements in DNA, which then results in the observed hormone effect.
A 5-year-old child has blue-tinged sclera, hearing loss, and small, slightly blue, misshapen teeth. Radiologic studies confirm the presence of numerous fractures of various ages. No significant degree of bruising is seen over sites of recent fracture. The disease this child most likely has is related to abnormal metabolism involving which of the following substances?
A. Collagen
B. Glycogen
C. Mucopolysaccharides
D. Purines
E. Tyrosine




The correct answer is A. The suspected disease is osteogenesis imperfecta, which is a rare genetic disorder that occurs in both recessive and dominant forms. The clinical presentation, depending on the specific form, varies from death in utero, to that described in the question stem, to very mild disease with only a modest increase in bone fragility. Spontaneous fractures occur in utero or during childhood. The different types all have defects in the synthesis of type I collagen, often with insufficient or abnormal pro-1(1) or pro-2(1) chains. These deficits produce an unstable collagen triple helix that is not as strong as normal collagen. Less severe mutations on type I collagen genes are common, resulting in collagen disarray and predisposing to hypogonadal or idiopathic osteoporosis.

Defective glycogen (choice B) metabolism is associated with the various glycogen storage diseases, such as von Gierke disease and Pompe disease. These diseases tend to present with profound hypoglycemia, hepatomegaly, or muscle weakness.

Defective mucopolysaccharide (choice C) metabolism is associated with the mucopolysaccharidoses, such as Hurler and Hunter syndromes. These diseases tend to present with abnormal facies ("gargoylism"), deformed ("gibbus") back, claw hand, and stiff joints.

Abnormalities of purine metabolism (choice D) are present in gout, which presents with joint inflammation and often involves the great toe. The net result is due to chronic hyperuricemia.

Abnormalities of tyrosine metabolism (choice E) are associated with phenylketonuria (pale hair and skin, mental retardation, musty smelling urine), albinism (pale hair, skin, increased skin cancer), cretinism (decreased T3 and T4), tyrosinosis (liver and kidney disease), and alkaptonuria (chronic arthritis and urine that turns black upon standing).
An IgG2 molecule is composed of which of the following?
A. One alpha, one gamma2, and two kappa chains
B. One gamma1 chain and two kappa chains
C. Two gamma1 chains and one kappa and one lambda chain
D. Two gamma1 chains and two kappa chains
E. Two gamma2 chains and two kappa chains




The correct answer is E. IgG molecules contain two gamma heavy chains of a given subtype and two light chains (either kappa or lambda). The 2 in IgG2 indicates the subclass to which the molecule belongs. IgG2 contains two gamma2 chains (since a given B cell can only form one type of heavy chain). The IgG molecule will contain either two kappa chains or two lambda chains, but never one of each (choice C). The standard serum concentration is 12 mg/mL.

A given cell produces immunoglobulin molecules with a single type of heavy chain (compare with choice A).

IgG molecules with gamma1 chains (choices B, C, and D) would be of the IgG1 subclass.
What is the lipase enzyme that degrades stored triacylglycerols in adipocytes?
A. Gastric lipase
B. Pancreatic lipase
C. Lipoprotein lipase
D. Hormone-sensitive lipase




The correct answer is D. Hormone-sensitive lipase is found in and degrades stored triacylglcerols in adipocytes.

Gastric lipase (choice A) originates in the stomach, where it degrades dietary triacylglycerols.

Pancreatic lipase (choice B) is found in pancreas and degrades dietary triacylglycerols in the small intestine.

Lipoprotein lipase (choice C) is found in extrahepatic tissues and acts on surface endothelial-cell lining. The capillaries degrade triacylglycerols circulating in chylomicrons or VLDL.
A 50-year-old man presents with headache, nausea, memory loss, abdominal pain, dark colored lines in gums, neuropathy, and anemia. Which of the following enzyme/s is/are inhibited in this patient?
A. ALA dehydrase and ferrochelatase
B. ALA synthase
C. Uroporphyrinogen I synthase
D. UDP-glucuronyl transferase




The correct answer is A. This is a case of lead poisoning. Lead inhibits ALA dehydrase and ferrochelatase, the enzymes of the heme synthesis pathway. Other signs and symptoms include coarse basophilic stippling of erythrocytes, lead deposits in abdomen, a gingiva, and epiphysis of bone.

Choice B - ALA synthase is the rate-limiting enzyme of heme synthesis and is repressed by heme.

Choice C - is the enzyme of heme synthesis; its absence leads to acute intermittent porphyria.

Choice D - UDP-glucuronyl transferase is the enzyme of heme degradation, and is involved in the formation of the conjugated form of bilirubin.
Which of the following metabolic alterations would most likely be present in a chronic alcoholic compared to a non-drinker?
A. Fatty acid oxidation is stimulated
B. Gluconeogenesis is stimulated
C. Glycerophosphate dehydrogenase is stimulated
D. The ratio of lactate to pyruvate is decreased
E. The ratio of NADH to NAD+ is increased




The correct answer is E. The principal route of metabolism of ethanol is via alcohol dehydrogenase, which uses hydrogen from ethanol to form NADH from NAD+, markedly increasing the ratio of NADH to NAD+. The relative excess of NADH has a number of effects, including inhibiting, rather than stimulating fatty acid oxidation (choice A); inhibiting gluconeogenesis rather than stimulating it (choice B); inhibiting, rather than stimulating (choice C) glycerophosphate dehydrogenase; and favoring the formation of lactate rather than pyruvate from glycolysis (thereby increasing, rather than decreasing the lactate/pyruvate ratio; choice D).

Alcoholism is a syndrome consisting of two phases: problem drinking and alcohol addiction. Alcohol addiction is defined as a physiologic dependence as manifested by evidence of withdrawal when intake is interrupted. As a side note, thiamine is commonly deficient in chronic alcoholics. Remember, thiamine is a necessary ketolase enzyme cofactor.
A young adult with albinism is found to be at increased risk for skin cancer. Which of the following is the precursor of melanin?
A. Tryptophan
B. Tyrosine
C. Arginine
D. Glycine




The correct answer is B. Albinism is caused by congenital deficiency of tyrosinase. It results in an inability to synthesize melanin from tyrosine. Other derivatives of tyrosine are thyroxine, dopamine, norepinephrine, and epinephrine.

Choice A - derivatives of tryptophan are niacin, serotonin, and melatonin.

Choice C - derivatives of arginine are creatine and urea.

Choice D - porphyrin is derived from glycine and eventually from heme.
A chronic alcoholic develops severe memory loss with marked confabulation. Deficiency of which of the following vitamins would be most likely to contribute to the neurologic damage underlying these symptoms?
A. Folic acid
B. Niacin
C. Riboflavin
D. Thiamine
E. Vitamin B12




The correct answer is D. Wernicke-Korsakoff syndrome refers to the constellation of neurologic symptoms caused by thiamine deficiency. Among these, a severe memory deficit, which the patient may attempt to cover by making up bizarre explanations (confabulation), is prominent. Wernicke's encephalopathy consists of the of confusion, ataxia and ophthalmoplegia. Anatomical damage to the mamillary bodies and periventricular structures has been postulated as the cause. In the U.S., severe thiamine deficiency is seen most commonly in chronic alcoholics. Thiamine deficiency can also damage peripheral nerves ("dry" beriberi) and the heart ("wet" beriberi).

Folic acid deficiency (choice A) produces megaloblastic anemia without neurologic symptoms.

Niacin deficiency (choice B) produces pellagra, characterized by depigmenting dermatitis, chronic diarrhea, and anemia.

Riboflavin deficiency (choice C) produces ariboflavinosis, characterized by glossitis, corneal opacities, dermatitis, and erythroid hyperplasia.

Vitamin B12 deficiency (choice E) produces megaloblastic anemia accompanied by degeneration of the posterolateral spinal cord. This megaloblastic anemia is commonly known as pernicious anemia.
One of the enzymes of the citric acid cycle, which is on the inner mitochondria membrane, also functions as complex II of the electron transport chain. This enzyme is also responsible for converting succinate to fumarate, producing FADH2. The enzyme is
A. isocitrate dehydrogenase
B. a ketoglutarate dehydrogenase
C. succinate dehydrogenase
D. succinyl CoA synthase




The correct answer is choice C. All Krebs cycle components are in the matrix of the mitochondria except succinate dehydrogenase, which is in the inner mitochondrial membrane. Succinate dehydrogenase reoxidizes FADH2 and passes electrons directly to Coenzyme Q.

Choice A - Isocitrate dehydrogenase, the rate limiting enzyme of citric acid cycle, produces NADH. It is activated by ADP and inhibited by NADH.

Choice B - a ketoglutarate dehydrogenase, like pyruvate dehydrogenase, requires thiamine, lipoic acid, CoA, FAD, NAD, and produces NADH.

Choice D - Succinyl CoA synthase catalyses a substrate level phosphorylation of GDP to GTP.
The extremely potent vasodilator nitric oxide (NO) is produced naturally by the body from which of the following amino acids?
A. Arginine
B. Aspartate
C. Glutamate
D. Methionine
E. Proline




The correct answer is A. In the body, the very short acting (5-second half-life) signaling molecule nitric oxide (NO) is synthesized by nitric oxide synthase (NOS). This enzyme is found in many tissues and converts arginine to citrulline and NO. The vasodilation produced by NO normally occurs in response to appropriate local biologic triggers, such as infection. However, this process may also produce pathology if uncontrolled, since it is thought that the endotoxic shock that can complicate bacterial septicemia may be related to continued activity of NOS. NO also plays a role in macrophage activity, where its metabolites are toxic to ingested microorganisms.

Aspartate (choice B) is associated with donation of one of the nitrogens in urea, with the liver enzyme aspartate aminotransferase (AST), and with nucleotide synthesis.

Glutamate (choice C) is associated with amino acid transport into the cell via the gamma-glutamyl cycle, and with the liver enzymes aspartate aminotransferase (AST) and alanine aminotransferase (ALT).

Methionine (choice D) is associated with 1-carbon transfer reactions.

Proline (choice E) is associated with the collagen helix.
Which of the following amino acids would most likely be found on the surface of a protein molecule?
A. Alanine
B. Arginine
C. Isoleucine
D. Leucine
E. Phenylalanine




The correct answer is B. This question requires two logical steps. First, you need to appreciate that hydrophilic amino acids are more likely to appear on the surface of a protein molecule, whereas hydrophobic amino acids are most likely be found in the interior. Next, you need to figure out which of the amino acids listed is hydrophilic. If you recall that arginine is a basic amino acid that is positively charged at physiologic pH, you should be able to answer this question right away.

All the other choices have neutral side chains and are uncharged at physiologic pH. They would most likely be found in the hydrophobic core of the protein structure. Alanine (choice A), isoleucine (choice C), and leucine (choice D) all have aliphatic side chains; phenylalanine (choice E) and tryptophan have aromatic side chains.
Dietary intake of which of the following amino acids can substitute for a portion of the daily requirement of niacin?
A. Alanine
B. Asparagine
C. Methionine
D. Proline
E. Tryptophan




The correct answer is E. Tryptophan is an aromatic amino acid that contains an indole group. By a complex series of minor enzymatic reactions, a small amount (~2%) of the tryptophan can be converted to quinolinate, which can then be used in place of niacin (nicotinic acid) in NAD (nicotinamide adenine dinucleotide) synthesis. Very high tryptophan levels can replace a portion of the dietary requirements for niacin. The nutritional disease pellegra (characterized by swollen tongue, dermatitis, neurologic dysfunction, and gastrointestinal dysfunction) usually occurs in the setting of combined tryptophan and niacin deficiency.

Alanine (choice A), the amino acid with a methyl R group, is a substrate of the liver enzyme alanine amino transferase (ALT, formerly called SGPT).

Asparagine (choice B) is one of the sources of ammonia for the urea cycle.

Methionine (choice C) is a sulfur-containing amino acid that is associated with methyl group transfer.

Proline (choice D) is technically an imino acid, rather than an amino acid, with a ring structure. You should remember that collagen has a high proline concentration.
A child is noted to be severely retarded. Physical examination reveals a pot-bellied, pale child with a puffy face. The child's tongue is enlarged. Dietary deficiency of which of the following substances can produce this pattern?
A. Calcium
B. Iodine
C. Iron
D. Magnesium
E. Selenium




The correct answer is B. The disease is cretinism, characterized by a profound lack of thyroid hormone in a developing child, leading to mental retardation and the physical findings described in the question stem. Cretinism can be due to dietary deficiency of iodine (now rare in this country because of iodized salt), to developmental failure of thyroid formation, or to a defect in thyroxine synthesis. Iodine in the diet is absorbed at the digestive tract as iodide (I-). The follicle cells in the thyroid gland absorb 120-150 μg of iodide ions per day. Iodide ions are actively transported into the thyroid follicle cells. The active transport mechanism for iodide is stimulated by TSH.

Calcium deficiency (choice A) in children can cause osteoporosis or osteopenia.

Iron deficiency (choice C) can cause a hypochromic, microcytic anemia.

Magnesium deficiency (choice D) is uncommon, but can cause decreased reflexes, and blunts the parathyroid response to hypocalcemia.

Selenium deficiency (choice E) is rare, but may cause a reversible form of cardiomyopathy.
A chronically malnourished patient notices her "hair falling out." She is on a strict fat-free diet. She probably has a deficiency in which vitamins?
A. Vitamin A
B. Vitamin C
C. Vitamin D
D. Vitamin E
E. Vitamin K




The correct answer is A. While it is hard to develop a deficiency in oil-soluble vitamins (A, D, E, K) because the liver stores these substances, deficiency states can be seen in chronic malnutrition (specifically chronic fat deprivation) and chronic malabsorption. Vitamin A deficiency is one of the most common deficiencies in developing countries. Vitamin A is necessary for formation of retinal pigments (deficiency can cause night blindness) and for appropriate differentiation of epithelial tissues (including hair follicles, mucous membranes, skin, bone, and adrenal cortex).

Vitamin C (choice B), which is water soluble rather than oil soluble, is necessary for collagen synthesis. Deficiency is associated with development of scurvy.

Vitamin D (choice C) is important in calcium absorption and metabolism. Calcium and phosphate irregularities can occur in deficiencies.

Vitamin E (choice D) is a lipid antioxidant that is important in the stabilization of cell membranes.

Vitamin K (choice E) is necessary for normal blood coagulation. When vitamin K is deficient, clotting factors 2, 7, 9, and 10 will be decreased leading to an increased pT/INR.
Which of the following amino acids is most responsible for the buffering capacity of hemoglobin and other proteins?
A. Arginine
B. Aspartic acid
C. Glutamic acid
D. Histidine
E. Lysine




The correct answer is D. Histidine, lysine and arginine are amino acids with basic side chains. Remember that a buffer is most effective when its pKa is within the pH range of the surrounding medium. Histidine is the only amino acid with good buffering capacity at physiologic pH. The imidazole side chain of histidine has a pKa around 6.5 and can reversibly donate and accept protons at physiologic pH.

Arginine (choice A) and lysine (choice E) are basic amino acids with pKa's of 12.5 and 10.5, respectively; at physiologic pH both will behave as bases and accept protons.

Aspartic acid (choice B) and glutamic acid (choice C) are acidic amino acids with pKa's of approximately 4; at physiologic pH they will behave as acids and donate protons. Their side chains are almost always negatively charged.
Which of the following cofactors is required for decarboxylation of alpha-ketoacids?
A. Vitamin B1
B. Vitamin B2
C. Vitamin B3
D. Vitamin B5
E. Vitamin B6




The correct answer is A. Vitamin B1, or thiamine, is the coenzyme required (as the pyrophosphate) for the decarboxylation of alpha-ketoacids. An example of this reaction is pyruvate decarboxylase reaction in alcoholic fermentation. Other reactions such as that catalyzed by pyruvate dehydrogenase also rely on thiamine pyrophosphate for decarboxylation, but require other cofactors as well. Thiamine is also required for the generation of pentose phosphates for nucleotide synthesis in the pentose phosphate pathway (hexose monophosphate shunt), serving as a cofactor for transketolase. The primary cause of thiamine deficiency in the US is due to alcoholism. Thiamine deficiency is known as beri beri. As this condition progresses, Wernicke-Korsakoff syndrome, which is seen in chronic alcoholics, can develop.

Vitamin B2 (choice B), or riboflavin, is a constituent of FMN (flavin mononucleotide) and FAD (flavin adenine dinucleotide). It functions in hydrogen and electron transport. Deficiency is associated with the development of cheilosis, angular stomatitis and glossitis.

Vitamin B3 (choice C), or niacin (nicotinic acid), is a coenzyme that is also involved in hydrogen and electron transport. Nicotinic acid functions in the form of NAD and NADP. Niacin is used clinically for the treatment of hypercholesteremia and hypertriglyceredemia.

Vitamin B5 (choice D), or pantothenic acid, is conjugated with coenzyme A to act as a carboxylic acid carrier.

Vitamin B6 (choice E), or pyridoxine, is required as a cofactor for pyridoxal phosphate and pyridoxamine phosphate. Both of these cofactors are essential to protein metabolism and energy production. Deficiencies can lead to the development of mouth soreness, glossitis, cheilosis, weakness and irritability. Severe deficiency can cause peripheral neuropathy and anemia.
A stretch of 25 hydrophilic amino acids in a protein could be found in a
A. signal sequence
B. start transfer sequence
C. stop transfer sequence
D. transmembrane domain
E. triple helix




The correct answer is E. A triple helix, such as the one found in collagen, is composed of three polypeptide chains wound together to form one structure. The chains are highly enriched with the hydrophilic amino acids glycine and proline.

A signal sequence (choice A) contains a stretch of 30 hydrophobic amino acids.

The start transfer sequence (choice B) and stop transfer sequence (choice C) are each composed of 25-30 hydrophobic amino acids.

Transmembrane domains (choice D) contain 25-30 hydrophobic amino acids. This is enough to form an alpha helix that will span a lipid bilayer.
The process involved in formation of protein from RNA is
A. Replication
B. Translation
C. Transcription
D. Transformation




The correct answer is choice B.In translation, protein synthesis occurs by peptide bond formation between successive amino acids, whose order is specified by a gene and thus by an mRNA

Choice A - Replicationis a process in which two daughter DNA molecule are produced, that are each identical to the parental DNA molecule

Choice C - Transcription is a process in which mRNA is synthesized using DNA as template.

Choice D - Transformation is not involved in protein synthesis either from DNA or RNA

Putting the processes together:
Folate plays a role in single-carbon unit transfer in the synthesis of nucleotides. Which of the following nucleotides require folate for synthesis?
A. Adenosine, cytosine, and uracil
B. Adenosine, guanine, and thymidine
C. Adenosine, guanine, and uracil
D. Cytosine, thymidine, and uracil
E. Guanine, thymidine, and uracil




The correct answer is B. In summary, a nucleoside consists of a purine or pyrimidine base linked to a pentose, and a nucleotide is a phosphate ester of a nucleoside. For example, adenine is the base of the ribonucleoside adenosine and the ribonucleotide adenylate (AMP). Folate is involved in the transfer of carbons 2 and 8 of the purine nucleus (affecting adenosine and guanosine) and the 5-methyl group of thymidine. This means that folate is required for synthesis of 3 of the 4 nucleic acid bases of DNA and 2 of the 4 nucleic acid bases in RNA. It is thus no wonder that folate deficiency has effects on so many tissues with high mitotic rate. Megaloblastic changes analogous to those seen in erythrocytes and their precursors can also be seen in other cells produced by bone marrow (neutrophil, eosinophil, basophil, and macrophage as well as megakaryocyte lines) and in epithelia throughout the body, including skin, mucous membranes such as the mouth and vagina (where the changes can be seen on Pap smear), stomach and intestinal linings, and cells from lung or liver. Similar megaloblastic changes are observed throughout the body when cobalamin (vitamin B12) deficiency is present, since cobalamin plays a role in methionine synthesis, which is the source of the one-carbon unit "active-formate." Cobalamin is also involved in the conversion of methylmalonic acid to succinic acid and is required to maintain the integrity of nerve cells via an unknown biochemical pathway.
A 45-year-old mother has a 5-year-old daughter with a history of repeated respiratory infections. The child has mental retardation, short stature, hypotonia with depressed nasal bridge, upslanting palpebral fissure, and epicanthal fold. She is also at an increased risk for leukemia. What is the probable chromosome abnormality?
A. Trisomy 13
B. Trisomy 18
C. Trisomy 21
D. Turner syndrome (45,X)




The correct answer is C. Trisomy 21 is the most common autosomal trisomy, and it causes Down syndrome. The increased risk of Down syndrome with increased maternal age is well documented.

Trisomy 13 (choice A) features include oral facial clefts, microophthalmia, polydactyly, and renal defects. They usually do not survive to 1 year.

Trisomy 18 (choice B) features include small mouth and ears, congenital heart defects, and overlapping fingers. Ninety percent die during the first year of life.

Turner syndrome (choice D) features include reduced stature, webbed neck, and lymphedema of the wrists and ankles.
The biochemical structure of all the hormones secreted by the anterior pituitary, posterior pituitary, and pancreas can best be described as which of the following?
A. Amino acid derivatives
B. Catecholamines
C. Glucocorticoids
D. Peptides
E. Steroid hormones




The correct answer is D. In the human body there are three basic types of hormones: peptides (protein derivatives), amino acid derivatives, and steroid hormones. Some examples of peptide hormones are those produced in the anterior pituitary (growth hormone, adrenocorticotropin, thyroid stimulating hormone, luteinizing hormone, and prolactin), the posterior pituitary (vasopressin and oxytocin), and the pancreas (insulin and glucagon).

Amino acid derivatives (choice A) include tyrosine and triiodothyronine, as well as the catecholamines (choice B) dopamine, epinephrine, and norepinephrine.

Glucocorticoid hormones (choice C; e.g., cortisol) are a specific type of steroid hormone produced by the adrenal cortex.

Steroid hormones (choice E) include the hormones produced in the adrenal cortex (cortisol and aldosterone), the ovaries (estrogen and progesterone), the testes (testosterone), and the placenta (estrogen and progesterone).
Which of the following enzymes is stimulated by glucagon?
A. Acetyl-CoA carboxylase
B. Glycogen phosphorylase
C. Glycogen synthase
D. HMG-CoA reductase
E. Pyruvate kinase




The correct answer is B. Before you started analyzing all of the answer choices you should have reminded yourself that glucagon increases serum glucose. So an enzyme stimulated by glucagon might be involved in either the breakdown of glycogen to glucose (glycogenolysis) or in the creation of glucose from noncarbohydrate precursors (gluconeogenesis). Glycogen phosphorylase catalyzes the first step in glycogenolysis; it makes sense that it would be stimulated by glucagon.

Clinical correlate: patients with Type I diabetes lose their response to hypoglycemia (but not to amino acids in protein containing meals) within a year or so after developing diabetes. Type I diabetics rely on the sympathetic nervous system to counter regulate hypoglycemia.

Acetyl-CoA carboxylase (choice A) catalyzes the first step in fatty acid synthesis, an anabolic process that would be stimulated by insulin, not glucagon.

As its name implies, glycogen synthase (choice C) is involved in the synthesis of glycogen. Glucagon (and epinephrine) stimulate the phosphorylation and inactivation of glycogen synthase.

HMG-CoA reductase (choice D) is the key enzyme involved in the synthesis of cholesterol. Since this is an anabolic process that occurs in the well-fed state, you would expect it to be stimulated by insulin and inhibited by glucagon (which it is).

Pyruvate kinase (choice E) catalyzes the last reaction of glycolysis. You would expect it to be inhibited by glucagon (thus decreasing the amount of glucose consumption). Glucagon promotes the phosphorylation of pyruvate kinase, which renders it inactive.
The majority of ATP generated during a 100 meter race is derived from which of the following?
A. ATP stores
B. Creatine phosphate
C. Gluconeogenesis
D. Glycolysis
E. Lipolysis




The correct answer is D. The key to this question is understanding how and when the body utilizes fuel stores. The stores of ATP (choice A) will be used up in less than 1 second once the race has started. Creatine phosphate (choice B) will be the primary source of energy for the next 3 or 4 seconds. After the creatine phosphate stores are depleted, the majority of ATP needed to complete the race will be derived from glycolysis (anaerobic respiration). If the race were to last for an extended period of time, then the processes of gluconeogenesis (choice C) and lipolysis (choice E) might be utilized. Gluconeogenesis is the process of synthesizing glucose in the liver from non-carbohydrate sources, such as amino and fatty acids. Lipolysis is the splitting up or decomposition of fat in the body.
As cells in the erythrocytic lineage mature and lose their nuclei, mitochondria, and ribosomes, which of the following pathways can still be used to produce ATP?
A. Citric acid cycle
B. Electron transport chain
C. Glycolysis
D. Malate shuttle
E. Urea cycle




The correct answer is C. Circulating erythrocytes have a life span of about 60 days and are dependent on a functioning Na+/K+ ATPase in the plasma membrane. This pump provides the electrochemical force across the plasma membrane that helps to maintain the volume of the red cell at a constant level by regulating salt, and consequently water, flow into and out of the cell. When the pump stops, the erythrocytes tend to lyse. It is therefore of extreme importance to the erythrocyte to supply the ATP necessary to keep the pump running. This is accomplished through the use of glycolysis, which converts glucose to pyruvate, with a net production of 2 ATP per glucose molecule.

The Krebs, or citric acid, cycle (choice A) is the central metabolic degradative pathway of aerobic cells. However, it is located in mitochondria and requires the electron transport chain, making it unavailable for mature red cells to use.

The electron transport chain (choice B) is located on the mitochondrial cristae and helps to convert the energy of NADH and FADH2 to ATP.

The malate shuttle (choice D) is used to transport cytoplasmic NADH electrons into mitochondria and is not available to erythrocytes.

The urea cycle (choice E) is used by liver cells for processing nitrogenous wastes, not generating energy.
Megaloblastic anemia with folate deficiency is linked to an inability to perform which type of enzymatic process?
A. Acyl transfer
B. Carboxylation
C. Decarboxylation
D. Hydroxylation
E. Methylation




The correct answer is E. Folic acid is a pteridine vitamin that exists as tetrahydrofolate (TH4) in its most reduced form. TH4 can accept methyl, methylene, or formyl carbons and transfer them as methyl groups. This function is vital in nucleotide and amino acid synthesis.

Pantothenic acid is a key vitamin in acyl transfer reactions (choice A). It forms part of coenzyme A, which transfers acyl groups in thiol esters as acetyl-CoA, succinyl-CoA, and other acyl-CoA forms.

Important vitamins in carboxylation reactions (choice B) include biotin and vitamin K. Biotin carries the carboxyl group in the pyruvate carboxylase and acetyl-CoA carboxylase reactions. Vitamin K is used in post-translational carboxylation of amino acid residues in blood clotting factors.

Oxidative decarboxylation reactions (choice C) require thiamine (vitamin B1). Examples include the pyruvate dehydrogenase and alpha-ketoglutarate dehydrogenase complexes.

Ascorbic acid (vitamin C) is a coenzyme in the hydroxylation (choice D) of lysyl and prolyl residues for collagen synthesis.
Long term broad spectrum antibiotics with reduced clotting time in localized areas is an indication of a deficiency in what?
A. Vitamin C
B. Vitamin K
C. Cyanocobalamine (B12)
D. Thiamine (B1)




The correct answer is B. Prolonged treatment with broad-spectrum antibiotics eliminates intestinal bacteria that supply vitamin K. Vitamin K catalyses gamma carboxylation of glutamic acid residues on various proteins concerned with blood clotting. Deficiency is characterized by prolonged PT but normal bleeding time.

Choice A - Vitamin C deficiency leads to scurvy with increased bleeding time but normal PT. A diet deficient in vitamin C results in scurvy.

Choice C - Vitamin B12 deficiency is usually caused by malabsorption and results in megaloblastic anemia and progressive peripheral neuropathy.

Choice D - The most common cause of thiamine deficiency is alcoholism. Because alcohol interferes with absorption, deficiency results in dry and wet beri-beri.
The finding that almost all of the melanoma cells have very large, visible nucleoli suggests that these cells are making large amounts of which of the following?
A. Cell surface markers
B. Golgi apparatus
C. Immunoglobulins
D. New DNA
E. Ribosomes




The correct answer is E. The nucleolus is the site of manufacture of ribosomal RNA with its subsequent packaging into ribosomes. Consequently, very large nucleoli indicate an increased rate of ribsome production, which, in turn, suggests an increased rate of production of proteins. Large nucleoli are seen in many active cancers but can also be prominent in benign conditions characterized by high metabolic rate, such as tissue repair after trauma.

Depending on the cells involved, the increased protein production associated with more ribosomes might produce more cell surface markers (choice A), but it is impossible to predict this simply from the presence of a large nucleolus.

You should associate a large Golgi apparatus (choice B) with transport of material to the cell surface.

Plasma cells that produce large amounts of immunoglobulins (choice C) may have large nucleoli, but more specifically have a very large endoplasmic reticulum and Golgi apparatus.

The nucleolus is not involved with new DNA (choice D) synthesis; this is done elsewhere in the nucleus.
Which type of enzyme reaction is effected by a folic acid deficiency?
A. Acyl transfer
B. Carboxylation
C. Decarboxylation
D. Hydroxylation
E. Methylation




The correct answer is E. Folic acid deficiency results in the development of macrocytic anemia that yields macro-ovalocytes and hypersegmented neutrophils on the peripheral blood smear. Folic acid is a pteridine vitamin that exists as tetrahydrofolate (TH4) in its most reduced form. TH4 can accept methyl, methylene, or formyl carbons and transfer them as methyl groups. This function is vital in nucleotide and amino acid synthesis. By far the most common cause of folate deficiency is inadequate dietary intake. Alcoholics and those with poor diets are at the highest risk for developing this type of anemia.

Pantothenic acid is a key vitamin in acyl transfer reactions (choice A). It forms part of coenzyme A, which transfers acyl groups in thiol esters as acetyl CoA, succinyl CoA, and other acyl CoA forms.

Important vitamins in carboxylation reactions (choice B) include biotin and vitamin K. Biotin carries the carboxyl group in the pyruvate carboxylase and acetyl CoA carboxylase reactions, and vitamin K is utilized in post-translational carboxylation of amino acid residues in blood clotting factors.

Oxidative decarboxylation reactions (choice C) require thiamine (vitamin B1). Examples include the pyruvate dehydrogenase and alpha-ketoglutarate dehydrogenase complexes.

Ascorbic acid (vitamin C) is a coenzyme in the hydroxylation (choice D) of lysyl and prolyl residues of collagen.
Which of the following inhibits the activity of acetyl-CoA carboxylase?
A. Citrate
B. Glucagon
C. High-carbohydrate, low-fat diet
D. Insulin




The correct answer is B. The key thing to remember here is that acetyl-CoA carboxylase catalyzes the first and rate-limiting step of fatty acid synthesis. If you got that far, you could have figured out which of the choices would inhibit the synthesis of fatty acids. Certainly glucagon, a catabolic hormone released in response to low blood glucose, would be a likely candidate to inhibit the synthesis of fatty acids. In fact, glucagon inhibits fatty acid synthesis by a cAMP-dependent phosphorylation of acetyl-CoA carboxylase. Conversely, glucagon stimulates fatty acid oxidation.

Clinical correlate: Glycogen metabolism is profoundly affected by specific hormones. Insulin, a polypeptide hormone, increases the capacity of the liver to synthesize glycogen. When insulin levels are high, the production of glycogen is high. The action of insulin is opposed by both glucagon and epinephrine which will increase blood glucose levels.

Citrate (choice A) is a key player in fatty acid synthesis (citrate shuttle). Therefore, the presence of citrate would stimulate, not inhibit, acetyl-CoA carboxylase.

A high-carbohydrate, low-fat diet (choice C) would stimulate, not inhibit, the synthesis of fatty acids.

In contrast to glucagon, insulin (choice D) is an anabolic hormone that promotes fatty acid synthesis and therefore would stimulate acetyl-CoA carboxylase. It does so by dephosphorylating the enzyme.
From which intermediate in the glycolytic pathway does the pentose phosphate pathway (also known as the hexose monophosphate, or pentose, shunt) "shunt"?
A. Fructose-1,6-bisphosphate
B. Fructose-6-phosphate
C. Glucose-6-phosphate
D. Phosphoenolpyruvate
E. Pyruvate




The correct answer is C. The hexose monophosphate shunt is an alternative route for the oxidation of glucose; it supplies the cell with NADPH and pentose sugars. The NADPH is used in many biosynthetic processes (e.g., fatty acid and cholesterol synthesis), whereas the pentoses are involved in the synthesis of nucleotides and some coenzymes. The pathway "shunts" from glucose-6-phosphate, which is oxidized in a series of NADPH-generating reactions, to ribulose-5-phosphate. The nonoxidative phase, which involves the transfer of C2 and C3 units from one sugar to another, follows. One resulting intermediate is fructose-6-phosphate (choice B), which can serve as a re-entry point to glycolysis, thereby closing the"shunt" loop.

Although all the other choices are glycolytic intermediates, they are not involved in the hexose monophosphate shunt.
Loss of which of the following classes of molecules on the surface of a tumor cell target would result in loss of susceptibility to killing by host immune cells?
A. CD3
B. CD4
C. CD8
D. MHC class I
E. MHC class II




The correct answer is D. Major histocompatibility complex (MHC) class I proteins are found in the membranes of all nucleotide cells. If the cell is healthy and the peptides are normal, T cells will ignore them. If the cytoplasm contains abnormal peptides, they will appear in the cell membrane and the T cells will be activated which will lead to the destruction of the cell. After the MHC class I molecule has moved to the surface of the tumor cell, peptide fragments from the tumor are presented in a groove of the class I molecule. The peptide fragments are presented to cytotoxic CD8 T cells, which recognize the MHC class I molecules on the cell surface and kill the tumor cell. Loss of this molecule would therefore prevent the tumor cell from being killed.

The CD3 molecule (choice A) is a marker on all T cells. It is involved in signal transduction, but not antigen recognition. This molecule would not be on the surface of tumor cells.

The CD4 molecule (choice B) is not on the surface of a tumor cell, but it is on the surface of a CD4+ T helper lymphocyte.

The CD8 molecule (choice C) is not on the surface of a tumor cell, but it is on the surface of a CD8+ cytotoxic T lymphocyte.

MHC class II antigens (choice E) are not involved in killing of tumor cell targets. They present peptide fragments (derived from intracellular killing of extracellular organisms by macrophages) to CD4 T lymphocytes.
Which of the following citric acid cycle intermediates is required for heme synthesis?
A. α±-Ketoglutarate
B. Fumarate
C. Isocitrate
D. Oxaloacetate
E. Succinyl-CoA




The correct answer is E. The porphyrin ring of heme is derived from the citric acid cycle intermediate succinyl-CoA and the amino acid glycine. The initial synthetic step, which is rate-limiting, is catalyzed by aminolevulinic acid synthase (ALA synthase).
Which of the following metabolic processes occurs exclusively in the mitochondria?
A. Cholesterol synthesis
B. Fatty acid synthesis
C. Gluconeogenesis
D. Ketone body synthesis
E. Urea cycle




The correct answer is D. Of the processes listed, only ketone body synthesis occurs exclusively in the mitochondria. Other mitochondrial processes include the production of acetyl-CoA, the TCA cycle, the electron transport chain, and fatty acid oxidation. The most common cause of ketone body formation is ketoacidosis secondary to diabetes. The essential diagnostic characteristics include: hyperglycemia, acidosis (with blood pH < 7.3), serum bicarbonate < 15 and serum positive for ketones.

Processes that occur exclusively in the cytoplasm include cholesterol synthesis (choice A; in cytosol or in ER) and fatty acid synthesis (choice B).

Note that gluconeogenesis (choice C) and the urea cycle (choice E) occur in both the mitochondria and the cytoplasm.
An infant diagnosed with phenylketonuria would be expected to be deficient in which of the following nonessential amino acids, assuming that it is not obtained from dietary sources?
A. Asparagine
B. Cysteine
C. Glutamine
D. Proline
E. Tyrosine




The correct answer is E. The human body is able to synthesize roughly half the amino acids necessary to build protein. These amino acids, termed nonessential, include alanine, arginine, asparagine, aspartate, cysteine, glutamate, glutamine, glycine, proline, serine, and tyrosine. The amino acids that must be supplied in the diet are termed essential; these include histidine, isoleucine, leucine, lysine, methionine, phenylalanine, threonine, tryptophan, and valine. Phenylalanine undergoes hydroxylation to tyrosine, catalyzed by the enzyme phenylalanine hydroxylase. It is noteworthy that tyrosine becomes an essential amino acid in individuals lacking this enzyme. The hyperphenylalaninemias, which include phenylketonuria, result from impaired conversion of phenylalanine to tyrosine, and are also asociated with mental retardation. This condition is associated with increased phenylalanine in blood and increased phenylalanine and its by-products (e.g., phenylpyruvate, phenylacetate, and phenyllactate) in urine.

Asparagine (choice A) is a member of the "oxaloacetate family"; its immediate precursor is aspartate.

The immediate precursor of cysteine (choice B) is serine. Serine is also the precursor of the nonessential amino acid glycine.

Glutamine (choice C), proline (choice D), and arginine are produced from glutamate. The synthesis of glutamate occurs by the reductive amination of alpha-ketoglutarate.
Which of the following cofactors is required for decarboxylation of alpha-ketoacids?
A. Vitamin B1
B. Vitamin B2
C. Vitamin B3
D. Vitamin B5
E. Vitamin B6




The correct answer is A. Vitamin B1, or thiamine, is the coenzyme required for the decarboxylation of alpha-ketoacids. An example of this reaction is pyruvate → acetyl CoA, catalyzed by the pyruvate dehydrogenase complex (although this particular reaction requires other cofactors, thiamine is required in all alpha-ketoacid decarboxylations). Thiamine is also required for the generation of pentose phosphates for nucleotide synthesis in the pentose phosphate pathway (hexose monophosphate shunt), serving as a cofactor for transketolase.

Vitamin B2(choice B), or riboflavin, is a constituent of FMN (flavin mononucleotide) and FAD (flavin adenine dinucleotide). It functions in hydrogen and electron transport.

Vitamin B3(choice C), or niacin (nicotinic acid), is a coenzyme that is also involved in hydrogen and electron transport. Nicotinic acid functions in the form of NAD and NADP.

Vitamin B5(choice D), or pantothenic acid, is conjugated with coenzyme A to act as a carboxylic acid carrier.

Vitamin B6(choice E), or pyridoxine, is used as pyridoxal phosphate and pyridoxamine phosphate. Both of these cofactors are essential to protein metabolism and energy production.
Albinism is a disorder involving what substance?
A. Aromatic amino acids
B. Branched chain amino acids
C. Glycolipids
D. Glycoproteins
E. Sulfur-containing amino acids




The correct answer is A. The disease is albinism. The most common form of albinism is caused by a deficiency of copper-dependent tyrosinase (tyrosine hydroxylase), blocking the production of melanin from the aromatic amino acid tyrosine. Affected individuals lack melanin pigment in skin, hair, and eyes, and are prone to develop sun-induced skin cancers, including both squamous cell carcinomas and melanomas.

Maple syrup urine disease is an example of a disorder of branched chain amino acids (choice B) causing motor abnormalities and seizures.

Tay-Sachs disease is an example of a disorder of glycolipids (choice C). In this disorder, a deficiency of hexosaminidase A leads to accumulation of ganglioside GM2.

Hunter's disease is an example of a disorder of glycoproteins (choice D). This mucopolysaccharidosis is inherited as an X-linked recessive trait.

Homocystinuria disease is an example of a disorder of sulfur-containing amino acids (choice E).
A woman vegetarian is found to have a severe riboflavin deficiency. The function of which of the following enzymes in the citric acid cycle would be most directly affected by the riboflavin deficiency?
A. Aconitase
B. Citrate synthase
C. Isocitrate dehydrogenase
D. Malate dehydrogenase
E. Succinate dehydrogenase




The correct answer is E. To answer this question you need two separate pieces of information. First, riboflavin (one of the B vitamins) is used to make the flavin part of FAD (flavin adenine dinucleotide). Second, of the citric acid cycle enzymes listed, only succinate dehydrogenase (which catalyzes the conversion of succinate to fumerate) used FAD (which is converted to FADH2) as a cofactor.

Aconitase (choice A) converts citrate to isocitrate and does not require a cofactor.

Citrate synthetase (choice B) combines acetyl-CoA and oxaloacetate to make citrate with release of coenzyme A (which requires pantothenic acid for synthesis).

Isocitrate dehydrogenase (choice C) converts isocitrate to alpha-ketoglutarate and uses NAD+ (which is converted to NADH + H+). The NAD+ requires the vitamin niacin for synthesis.

Malate dehydrogenase (choice D) converts malate to oxaloacetate and uses NAD+ (which converted to NADH + H+). The NAD+ requires the vitamin niacin for synthesis.
Which of the following pairs of enzymes is required for the process of gluconeogenesis?
A. Fructose-1,6-bisphosphatase and pyruvate carboxylase
B. Glucose-6-phosphatase and phosphofructokinase-1
C. Glucose-6-phosphatase and pyruvate dehydrogenase
D. Phosphoenolpyruvate carboxykinase and glucokinase
E. Pyruvate kinase and pyruvate carboxylase




The correct answer is A. Gluconeogenesis is the process that results in the synthesis of glucose from non-carbohydrate precursors. This pathway is very important because the brain is highly dependent on glucose as the primary source of fuel. The three irreversible steps of glycolysis are catalyzed by hexokinase, phosphofructokinase-1 (choice B), and pyruvate kinase. In gluconeogenesis, other enzymes are needed to bypass these key steps. Pyruvate cannot be directly converted to phosphoenolpyruvate in gluconeogenesis. Therefore, pyruvate carboxylase (a mitochondrial enzyme; choice A) converts pyruvate to oxaloacetate, which can be converted to phosphoenolpyruvate by phosphoenolpyruvate carboxykinase (choice D), using two ATP equivalents per molecule of phosphoenolpyruvate. Fructose-1,6-bisphosphatase (choice A) is the enzyme that splits fructose-1,6-bisphosphate into fructose-6-phosphate and inorganic phosphate. It is also required for gluconeogenesis.

Glucose-6-phosphatase (choices B and C) is a liver enzyme that hydrolyzes glucose-6-phosphate to glucose. A deficiency of this enzyme leads to von Gierke disease, also known as glycogen storage disease type I.

Pyruvate dehydrogenase (choice C) is a mitochondrial enzyme that converts pyruvate to acetyl CoA. This enzyme requires thiamine pyrophosphate, lipoamide, and FAD as cofactors.

Glucokinase (choice D) is a liver enzyme that converts glucose to glucose-6-phosphate. Unlike hexokinase, it is specific for glucose and is unresponsive to the level of glucose-6-phosphate. Its function is to store excess glucose, so it has a very high Km (ie, a low affinity) for glucose, becoming active only when the concentration of glucose is very high.

Pyruvate kinase (choice E) catalyzes the conversion of phosphoenolpyruvate to pyruvate in the glycolytic pathway. It is activated by fructose-1,6-bisphosphate, the product of the committed step of glycolysis, and is allosterically inhibited by ATP, alanine, and acetyl CoA.
Cyanide poisoning will directly affect which anabolic process?
A. Breaking of covalent bonds in glucose molecules
B. Formation of carbon dioxide
C. Formation of GTP from GDP
D. Movement of hydrogen ions through channels in the respiratory enzymes
E. Splitting of water molecules into hydrogen and oxygen atoms




The correct answer is D. Cyanide binds to the heme Fe3+ of cytochrome a3 in the electron transport system, blocking the transfer of electrons to oxygen and consequently the synthesis of ATP. In other words, cyanide blocks the process of oxidative phosphorylation, which produces greater than 90% of the ATP used by the cells in our body. The major steps involved in this process occur within the "electron transport system (ETS)," or "respiratory chain," of the mitochondria. The steps at the end of the elctron transport system, where ATP is generated, are as follows: along the ETS, the repiratory enzymes continually pump hydrogen ions from the matrix of the mitochondria to the intermembrane space, which creates a large concentration gradient. At the end of the ETS, hydrogen ions pass through channels in the respiratory enzymes along the concentration gradient. As the hydrogen ions pass through these enzymes, the energy created is used to phophorylate ADP to ATP. Inhibition of mitochondrial respiration can lead to coma and death unless diagnosed and treated early.

The breaking of covalent bonds in glucose molecules (choice A), the formation of carbon dioxide in the TCA cycle (choice B), and the splitting of water molecules into hydrogen and oxygen atoms (choice E) all require energy; these processes do not create energy.

The formation of GTP from GDP (choice C) is a process that creates high energy phosphates in the tricarboxylic acid cycle.
An individual lacking the enzyme tyrosinase would be particularly predisposed to develop which of the following?
A. Glioblastoma multiforme
B. Hemangioblastoma
C. Hepatoma
D. Melanoma
E. Renal cell carcinoma




The correct answer is D. This question is simple if you know that tyrosinase is an enzyme in the biosynthetic pathway for melanin formation from tyrosine. A lack of tyrosinase causes one form of albinism; a second form is caused by defective tyrosine uptake. Patients with albinism are vulnerable to developing cancers of the skin of all types, including basal cell carcinoma, squamous cell carcinoma, and melanoma. The melanomas are unusual in that they are non-pigmented (amelanotic) rather than black, since the patients cannot form melanin. Malignant melanoma is the leading cause of death from skin disease. Tumor thickness (Breslow's Classification) is the single most important prognostic factor. These lesions are often flat or raised. The borders are typically irregular. Remember many cases of melanoma occur with lesions on the head and neck.
Thiamine is used by which coenzyme in order to prevent a lactic acid acidosis?
A. Lactate dehydrogenase
B. Pyruvate carboxylase
C. Pyruvate dehydrogenase
D. Pyruvate kinase
E. Transketolase




The correct answer is C. Thiamine is a water-soluble vitamin that is converted to the coenzyme thiamine pyrophosphate. This coenzyme is used by pyruvate dehydrogenase to convert pyruvate to acetyl coenzyme A. In the absence of thiamine, pyruvate accumulates and can be converted by lactate dehydrogenase to lactate, which is spilled in the blood causing lactic acidosis.

As a side note, thiamine deficiency is associated with the development of beri-beri - a neurological and cardiovascular disorder. The most common cause of beri-beri in the US today is seen in alcoholics. Damage to the nerves is expressed in terms of pain in the limbs and weakness of musculature. The heart can become enlarged and cardiac output can be decreased.

Lactate dehydrogenase (choice A) produces lactate from pyruvate but does not use thiamine pyrophosphate.

Some lactic acidosis might be produced by decreased pyruvate carboxylase activity (choice B), but the enzyme requires biotin rather than thiamine pyrophosphate.

Pyruvate kinase (choice D) makes pyruvate from phosphoenolpyruvate, but does not use thiamine pyrophosphate.

Transketolase (choice E) requires thiamine pyrophosphate, but operates in another pathway (pentose phosphate pathway). Decreased transketolase activity is not associated with the development of lactic acidosis.
A patient's lab studies demonstrate a deficiency of cytochrome C oxidase activity. A defect in which of the following subcellular organelles would cause this deficiency?
A. Golgi apparatus
B. Lysosomes
C. Mitochondria
D. Ribosomes
E. Smooth endoplasmic reticulum




The correct answer is C. Cytochrome C oxidase is an electron transport chain component, in the mitochondria of muscle and brain. Cytochromes and the electron transport chain are involved in oxidative respiration. Defects can produce mental retardation, weakness, ataxia or seizures.

The Golgi apparatus (choice A) is involved in packaging materials for secretion outside the cell. You should associate mucolipidosis I-cell disease with Golgi apparatus problems.

Lysosomes (choice B) are the organelles that degrade many cellular products. Defects can produce a wide variety of lysosomal storage diseases, including Hurler and Hunter syndromes.

Ribosomes (choice D) are the organelles that traslate mRNA into proteins. There are no known diseases of ribosomes, probably because their function is so crucial that any problems produce death in utero.

Hypertrophy of the smooth endoplasmic reticulum (choice E) in the liver is associated with conditions that stimulate the cytochrome p450 detoxification systems (e.g., barbiturate use and alcoholism).
A genetic mutation that results in abnormal stimulatory G protein (Gs) structure would adversely affect which of the following mechanisms?
A. Active transport
B. Facilitated diffusion
C. Pinocytosis
D. Receptor-mediated endocytosis
E. Signal transduction




The correct answer is E. G proteins are involved in signal transduction, which allows macromolecules to affect a cell's biological functions without crossing the plasma membrane. The binding of an agonist, such as a hormone, to a receptor on the plasma membrane causes a conformational change in the receptor, which then interacts with a stimulatory G protein (Gs), accompanied by the binding of GTP to the alpha subunit of the G protein. The alpha subunit of the G protein (with attached GTP) dissociates from the beta and gamma subunits, and interacts with intracellular effector molecules such as adenylate cyclase, which it activates.

Active transport (choice A) is the movement of molecules across a membrane from a region of low concentration to a region of high concentration; it requires both a carrier protein and the expenditure of energy (usually supplied by the hydrolysis of ATP by an ATPase).

Facilitated diffusion (choice B) is the movement of a molecule down its concentration gradient by means of a protein carrier. This type of diffusion is suited for molecules with low permeability resulting from inappropriate size or polarity.

Pinocytosis (choice C) is also known as fluid-phase endocytosis and refers to the uptake of molecules that are in solution. Receptors are not involved.

Receptor-mediated endocytosis (choice D) refers to the engulfment of a molecule that binds to a receptor on the surface of the cell, occurring commonly at the clathrin-coated pits on the plasma membrane.
Which of the following is the action of dietary fiber?
A. Increases the blood cholesterol
B. Decreases the blood cholesterol
C. Decreases bowel motility
D. Increases the exposure of gut to carcinogens




The correct answer is B. Dietary fiber lowers blood cholesterol. Dietary fiber consists of nondigestable carbohydrate, including cellulose, lignin, and pectin. Other actions include the increasing of bowel motility, reduced exposure of the gut to carcinogens, softer stools, and interference with mineral absorption.
Which of the following inhibits the activity of acetyl-CoA carboxylase?
A. Citrate
B. Glucagon
C. High-carbohydrate, low-fat diet
D. Insulin




The correct answer is B. The key thing to remember here is that acetyl-CoA carboxylase catalyzes the first and rate-limiting step of fatty acid synthesis. If you got that far, you could have figured out which of the choices would inhibit the synthesis of fatty acids. Certainly glucagon, a catabolic hormone released in response to low blood glucose, would be a likely candidate to inhibit the synthesis of fatty acids. In fact, glucagon inhibits fatty acid synthesis by a cAMP-dependent phosphorylation of acetyl-CoA carboxylase. Conversely, glucagon stimulates fatty acid oxidation.

Citrate (choice A) is a key player in fatty acid synthesis (citrate shuttle). Therefore, the presence of citrate would stimulate, not inhibit, acetyl-CoA carboxylase.

A high-carbohydrate, low-fat diet (choice C) would stimulate, not inhibit, the synthesis of fatty acids.

In contrast to glucagon, insulin (choice D) is an anabolic hormone that promotes fatty acid synthesis and therefore would stimulate acetyl-CoA carboxylase. It does so by dephosphorylating the enzyme.
Addition of which of the following exhaustively 14C labeled substrates would lead to evolution of 14CO2 from a cell-free suspension containing all the enzymes and substrates required for the synthesis of uridylic acid?
A. Aspartate
B. Carbamoyl phosphate
C. Glutamine
D. Glycine
E. N10-Formyltetrahydrofolate




The correct answer is A. In the first step of pyrimidine synthesis, carbamoyl phosphate condenses with aspartate to form carbamoyl aspartate, in a reaction catalyzed by aspartate transcarbamoylase. In subsequent steps, ring closure occurs with the loss of water, followed by oxidation to yield orotic acid. Addition of ribose-5-phosphate produces orotidylic acid, which is decarboxylated by orotidylate decarboxylase to yield uridylic acid. The carbon dioxide that is evolved is derived from the alpha carboxyl group of aspartate.

Carbamoyl phosphate (choice B) condenses with aspartate with the loss of inorganic phosphate to produce carbamoyl aspartate. The carbamoyl moiety of carbamoyl phosphate is retained.

Glutamine (choice C), glycine (choice D) and N10-formyltetrahydrofolate (choice E) are all used in purine synthesis. Glutamine also donates an amino group to UTP to form CTP, but this step occurs after the synthesis of uridylic acid is complete.
The receptors for the hormone that causes blood glucose level to quickly drop are located on which cellular component in target cells?
A. Mitochondrial matrix
B. Nuclear matrix
C. Nuclear membrane
D. Plasma membrane
E. Smooth endoplasmic reticulum




The correct answer is E. The hormone in question is insulin, which helps the body utilize glucose in the bloodstream. Circulating insulin binds to a receptor on the plasma membrane of target cells. The insulin receptor is a transmembrane protein that stimulates cytosolic tyrosine kinase activity when activated. One of the important proteins activated by the tyrosine kinase is an insulin receptor substrate, which in turn activates the wide variety of enzymes and transport molecules that produce the actual metabolic changes in sugar, protein, and fat metabolism seen in the presence of insulin.

The mitochondrial matrix (choice A) is not a significant receptor site for hormones.

The nuclear matrix (choice B), sometimes together with cytoplasm, contains receptors for steroid hormones such as cortisol.

The nuclear membrane (choice C) is not a significant receptor site for hormones.
Megaloblastic anemia, due to folate deficiency folate deficiency from dietary causes, is linked to which dietary problem?
A. Lack of leafy green vegetables
B. Lack of milk products
C. Lack of red meat
D. Lack of yellow vegetables
E. Overcooked food




The correct answer is E. Folates (pteroylglutamic acid and related compounds) are widely distributed in foodstuffs. Folic acid serves as an important mediator of one-carbon transfers Dietary deficiency is usually due to overcooked (folates are very labile) and old (folates rapidly decay with time) food. Folic acid deficiency is a macrocytic anemia that reveals macro-ovalocytes and hypersegmented neutrophils on the peripheral blood smear. Reduced folate levels can be seen in red blood cells or serum.
Absence of which of the following enzymes would impair the rate-limiting step of glycogenolysis?
A. α±-1,4-Glucan transferase
B. Glycogen phosphorylase
C. Glycogen synthase
D. Phosphoglucomutase
E. UDP-glucose pyrophosphorylase




The correct answer is B.A key to excelling in biochemistry on the NBDE part 1 is to master the most clinically important elements of metabolic pathways, e.g., rate-limiting steps, irreversible steps, and steps involving enzymes affected by genetic diseases. In this case, glycogen phosphorylase is the enzyme involved in the rate-limiting step of glycogenolysis:

Glycogen phosphorylase: (Glucose)n + Pi→ (Glucose)n–1 + Glucose-1-P

Note that this enzyme is activated in response to the binding of glucagon to liver cell receptors or the binding of epinephrine to muscle receptors, via signal transduction.

α±-1,4-Glucan transferase(choice A) is a debranching enzyme that removes three of four glucose units from a branch point and transfers them to the end of another chain. In this reaction, one α±-1,4 bond is cleaved and another is formed. The elongated chain then becomes a substrate for glycogen phosphorylase.

Glycogen synthase (choice C) is the enzyme involved in the rate-limiting step of glycogen synthesis:

Glycogen synthase: (Glucose)n + UDP-glucose → (Glucose) n+1 + UDP

Note that this enzyme is inactivated in response to the binding of glucagon to liver cell receptors or the binding of epinephrine to muscle receptors.

Phosphoglucomutase (choice D) is a key enzyme in glycogen synthesis that reversibly converts glucose-6-phosphate to glucose-1-phosphate.

UDP-glucose pyrophophorylase (choice E) is another key enzyme in glycogen synthesis:

UDP-glucose pyrophosphorylase: Glu-1-P + Uridine-P-P-P → UDP-Glu + PPi

Note that the linkage formed betrween UDP and glucose is a high-energy bond that can provide energy to many biosynthetic reactions.
Which of the following will be unchanged in a Lineweaver-Burk plot of an enzyme with and without a competitive inhibitor?
A. Km
B. Slope
C. x-Intercept
D. y-Intercept




The correct answer is D. It is worth taking the time to learn how to read a Lineweaver-Burk plot. Lineweaver-Burk plots are used to determine the Vmax and Km of an enzyme; they are also used to differentiate between competitive and noncompetitive inhibition.

Note that in a Lineweaver-Burk plot, the slope is Km/Vmax, the x-intercept is -1Km, and the y-intercept is 1/Vmax. In the presence of a competitive inhibitor, the Km(choice A), and therefore the slope (choice B), is increased. Similarly, if Km is increased, -1/Km will become less negative and the x-intercept (choice C) will shift to the right. Intuitively, this makes sense since a competitive inhibitor will increase the amount of substrate needed to reach half-maximal velocity (definition of Km). In contrast, the Vmax, and hence the y-intercept, is unchanged.
Which of the following structures is common to all sphingolipids?
A. Carnitine
B. Ceramide
C. Diacylglycerol
D. Sphingomyelin
E. Squalene




The correct answer is B. Sphingolipids are a class of lipids that are structural components of membranes. Specifically, sphingolipids are any lipid containing a long chain base like that of sphingosine (e.g., ceramides, cerebrosides, gangliosides). They are a common constituent of nerve tissue. Ceramide is a component of sphingolipids. Ceramide is composed of sphingosine, a long-chain amino alcohol with a saturated fatty acid linked to the amino group. Sphingolipids can be differentiated on the basis of the "X" group that is esterified to the terminal hydroxyl group of ceramide.

Carnitine (choice A) is involved in the oxidation of fatty acids. Carnitine is important in transferring fatty acids from the cytoplasm into the mitochondria (the carnitine shuttle).

Diacylglycerol (choice C) is the alcohol common to all phospholipids. The second alcohol (e.g., choline, ethanolamine, serine) contributes the polar head that distinguishes the different classes of phospholipids. Like sphingolipids, phospholipids are found in membranes.

Sphingomyelin (choice D) is a sphingolipid with phosphocholine as its "X" group. It is a component of the myelin sheath.

Squalene (choice E) is a 30-carbon intermediate in the synthesis of cholesterol.
Which of the following structures is common to all sphingolipids?
A. Carnitine
B. Ceramide
C. Diacylglycerol
D. Sphingomyelin
E. Squalene




The correct answer is B. Sphingolipids are a class of lipids that are structural components of membranes. Ceramide is a component of sphingolipids. Ceramide is composed of sphingosine, a long-chain amino alcohol with a saturated fatty acid linked to the amino group. Sphingolipids can be differentiated on the basis of the "X" group that is esterified to the terminal hydroxyl group of ceramide.

Carnitine (choice A) is involved in the oxidation of fatty acids. Carnitine is important in transferring fatty acids from the cytoplasm into the mitochondria (the carnitine shuttle).

Diacylglycerol (choice C) is the alcohol common to all phospholipids. The second alcohol (e.g., choline, ethanolamine, serine) contributes the polar head that distinguishes the different classes of phospholipids. Like sphingolipids, phospholipids are found in membranes.

Sphingomyelin (choice D) is a sphingolipid with phosphocholine as its "X" group. It is a component of the myelin sheath.

Squalene (choice E) is a 30-carbon intermediate in the synthesis of cholesterol.
A young child is well for the first 2 years of his life, and receives all his immunizations without any complications. Starting around his 2nd birthday he contracts frequent upper respiratory tract infections, and is hospitalized three times for pneumonia. Laboratory testing would most likely reveal a deficiency of which of the following immunoglobulins in this child?
A. IgA
B. IgD
C. IgG
D. IgM




The correct answer is A. Selective IgA deficiency (<5 mg/dL) is the most common of all the primary immunodeficiency diseases. The incidence reported in the US has ranged from 1:250 to 1:1000. IgA has two subclasses, IgA1 and IgA2. IgA1 predominates in the serum, while IgA2 predominates in mucosal secretions as a dimer bound together by a J chain with a secretory piece attached. Recurrent bacterial and viral infections of the respiratory tract can be attributed to a lack of secretory IgA (sIgA), the predominant immunoglobulin of the mucosal immune system. The most common types of infection include sinusitis, otitis and bronchitis. Atopic disease and autoimmune disorders can be associated with IgA deficiency.

IgD (choice B) has not been given any particular function other than to act as a receptor on the B cell. It can be found in very low levels in serum.

IgG (choice C) is the major immunoglobulin found in the humoral immune response. A patient with a low IgG will experience pyogenic infections.

IgM (choice D) is found in the early response to an antigen. If the patient was deficient in IgM he would have also been characteristically low in IgG and would have experienced recurrent pyogenic infection, usually commencing by the age of 5-6 months.
In the citric acid cycle, succinate thiokinase (succinyl-CoA synthetase) catalyzes the cleavage of the succinyl-CoA thioester bond with formation of a high-energy compound. This compound can then be used by the body in which of the following biochemical pathways?
A. Cysteine degradation
B. Elongation of the polypeptide chain
C. Epinephrine synthesis from tyrosine
D. Isopentyl pyrophosphate synthesis
E. Oxidative phosphorylation




The correct answer is B. In this question, you need to know that GTP is synthesized when CoASH is cleaved from succinyl-CoA to form succinate in the citric acid cycle. You also need to know that GTP, rather than ATP, is used as the energy source in protein synthesis, specifically in the formation of the activated elongation factor to which tRNA binds, and in the transfer of the elongating chain from the P to the A site in the ribosome.

Cysteine degradation (choice A) requires O2 and produces sulfite and then sulfate, which is secreted in urine.

Epinephrine synthesis from tyrosine (choice C) requires tetrahydrobiopterin, pyridoxal phosphate, oxygen, copper, and S-adenosylmethionine.

Isopentyl pyrophosphate synthesis from HMG CoA (choice D) requires NADPH and ATP, and releases CO2.

Oxidative phosphorylation (choice E) used NADH, FADH2, coenzyme Q, oxygen, and a variety of cytochromes to produce ATP.
A 25-year-old woman with sickle cell anemia is diagnosed with gallstones. Of which of the following compounds are these stones most likely composed?
A. Calcium bilirubinate
B. Calcium oxalate
C. Cholesterol
D. Cholesterol and calcium bilirubinate
E. Cystine




The correct answer is A. Bilirubin is a degradative product of hemoglobin metabolism. Bilirubin (pigment) stones are specifically associated with excessive bilirubin production in hemolytic anemias, including sickle cell anemia. Bilirubin stones can also be seen in hepatic cirrhosis and liver fluke infestation.

Calcium oxalate stones (choice B) and cystine stones (choice E) are found in the kidney, rather than the gallbladder.

Pure cholesterol stones (choice C) are less common than mixed gallstones, but have the same risk factors, including obesity and multiple pregnancies.

Mixed stones (choice D) are the common "garden variety" gallstones, found especially in obese, middle aged patients, with a female predominance.
A newborn with severe acidosis and vomiting has elevated serum levels of lactate and alanine. This suggests a deficiency in which enzyme?
A. Alanine aminotransferase
B. Glutamate dehydrogenase
C. Lactate dehydrogenase
D. Pyruvate carboxylase
E. Pyruvate dehydrogenase




The correct answer is E. Pyruvate dehydrogenase (PDH) catalyzes the irreversible conversion of pyruvate to acetyl-CoA. Thiamine pyrophosphate (TPP), lipomide and FAD serve as catalytic cofactors in addition to CoA and NAD+, the stoichiometric cofactors in this reaction. If PDH is absent, pyruvate will be used in other pathways instead. Pyruvate will be converted to alanine via alanine aminotransferase (choice A) and to lactate via lactate dehydrogenase (choice C).

Glutamate dehydrogenase (choice B) is involved in oxidative deamination, releasing ammonium ion for urea synthesis. Deficiency of this enzyme would not cause the symptoms described.

Pyruvate carboxylase (choice D) is a gluconeogenic enzyme that catalyzes the conversion of pyruvate to oxaloacetate. Deficiency of this enzyme would not cause the symptoms described.
Observation of a hematoxylin and eosin-stained microscope slide reveals that the nuclei are blue. What is the basis for this observation?
A. Eosin binds to carbohydrates
B. Eosin binds to lipids
C. Eosin binds to nucleic acids
D. Hematoxylin binds to lipids
E. Hematoxylin binds to nucleic acids




The correct answer is E. Blue hematoxylin binds to polyanions such as RNA and DNA. Nuclei contain large amounts of DNA and RNA, and they are consequently almost always blue. The nuclei of dysplastic and cancerous cells are often enlarged and hyperchromatic (e.g., darker blue) compared to normal cells of similar cell lines, because these altered cells often have extra DNA (are aneuploid) and/or RNA (are metabolically active).

Pink eosin binds relatively nonselectively to cellular components, particularly proteins. Cytoplasm of different cell lines can be pink, purple, or blue, depending principally on the number of ribosomes in the cytoplasm. Consequently, blue-tinged cytoplasm tends to suggest high synthetic activity (e.g., abundant ribosomes).
In achondroplasia, mutation results in overexpression of FGFR3 and excess inhibition of bone growth, particularly in long bones. Sons and daughters are equally affected. What is the mode of inheritance in achondroplasia?
A. Autosomal dominant
B. Autosomal recessive
C. X-linked dominant
D. X-linked recessive




The correct answer is A. Achondroplasia is an autosomal-dominant disease, typically seen in multiple generations of a pedigree. Men and women are affected in roughly equal frequencies.

Autosomal recessive diseases (choice B) are clinically seen only in the homozygous individual. Skipped generations are typically seen. Both men and women are affected.

An x-linked dominant disease phenotype (choice C) is seen in multiple generations. Both men and women are affected.

X-linked recessive diseases (choice D) are seen much more commonly in men than women. Skipped generations are commonly seen. Male-to-male transmission is not seen in X-linked inheritance.
Which of the following vitamins has antioxidant properties, reduces the incidence of heart attacks and deficiency of which can lead to hemolysis and neurological problems.
A. Vitamin A
B. Vitamin D
C. Vitamin E
D. Vitamin K




The correct answer is choice C. Vitamin E acts as antioxidant, and may prevent oxidation of LDL(low density lipoproteins). It protects membrane lipids from peroxidation. Oxidized LDL is thought to promote heart disease.

Choice A- Vitamin A is antioxidant, maintains vision and growth regulation. Consumption of beta-carotene in the diet deceases the incidence of lung and skin cancer. Vitamin A deficiency results in night blindness and follicular hyperkeratosis.

Choice B- In response to hypocalcemia, vitamin D helps normalize serum calcium level. Deficiency results in Rickets and osteomalacia.

Choice D- Vitamin K exposes calcium binding sites on several calcium dependent proteins. Deficiency leads to prolonged bleeding and easy bruising with normal bleeding time.
Aerobic glycolysis is utilized for energy during intense aerobic exercise. In which of the following forms will the carbons derived from glucose enter the citric acid cycle?
A. Acetyl-CoA
B. Citrate
C. Oxaloacetate
D. Pyruvate
E. Succinate




The correct answer is A. In aerobic glycolysis, glucose is degraded to pyruvate, which is then converted to acetyl-CoA, the form in which it actually enters the citric acid cycle. Two acetyl-CoAs, each containing two of the glucose's carbons, are produced from each glucose molecule. In addition, a total of two carbons from glucose are released as CO2 when each of the two pyruvates are converted to acetyl-CoA.

Citrate (choice B) is the product formed in the citric acid cycle when acetyl-CoA condenses with oxaloacetate.

Oxaloacetate (choice C) is the citric acid cycle intermediate that condenses with acetyl-CoA to form citrate.

Two pyruvates (choice D) are produced from degradation of glucose. These are then converted to the two acetyl-CoAs that enter the citric acid cycle.

Succinate (choice E) is another citric acid cycle intermediate that forms when coenzyme A is removed from succinyl-CoA.
A patient has an inherited disorder in which the administration of aspartame could be detrimental to her health. This patient most likely has which of the following genetic disorders?
A. Hyperornithinemia
B. Hyperuricemia
C. Hypervalinemia
D. Phenylketonuria
E. Wilson disease




The correct answer is D. The administration of any product that contains phenylalanine, such as aspartame (an artificial sweetener), to an individual with any of the hyperphenylalaninemias could be detrimental to his or her general health. The hyperphenylalaninemias result from an impaired conversion of phenylalanine to tyrosine. The most common and clinically important of these is phenylketonuria, which is characterized by an increased concentration of phenylalanine in blood, increased concentration of phenylalanine and its by-products (such as phenylpyruvate, phenylacetate, and phenyllactate) in urine, and mental retardation. Phenylketonuria is caused by a deficiency of phenylalanine hydrolase.

Hyperornithinemia (choice A) is an inherited disorder of amino acid metabolism that results from a defect of the enzyme ornithine decarboxylase. This condition is associated with mental retardation, neuropsychiatric dysfunction, and protein intolerance.

Hyperuricemia (choice B) is a condition associated with higher than normal blood levels of uric acid. Gout may be produced if the hyperuricemia persists.

Hypervalinemia (choice C) is a genetic disorder of amino acid metabolism that results from a defect of the enzyme valine aminotransferase. This condition is associated with mental retardation, neuropsychiatric dysfunction, and protein intolerance.

Wilson disease (choice E) is an autosomal recessive disorder associated with an abnormality of the hepatic excretion of copper, resulting in toxic accumulations of the metal in the brain, liver, and other organs.
What hormone is responsible for protein breakdown and increased gluconeogenesis?
A. Aldosterone
B. Cortisol
C. Estrogen
D. Testosterone




The correct answer is B. Cortisol increases gluconeogenesis and is responsible for protein breakdown. It also has anti-inflammatory action.

Aldosterone (choice A) stimulates renal reabsorption of sodium and excretion of potassium.

Estrogen (choice C) controls the menstrual cycle and promotes the development of female secondary sex characteristics.

Testosterone (choice D) stimulates spermatogenesis and promotes the development of male secondary sex characteristics.
A complete blood count with differential for a pregnant woman reveals a hematocrit of 30%, with hypersegmented neutrophils and large, hypochromic red cells. Deficiency of which of the following would be most likely to produce these findings?
A. Ascorbic acid
B. Calcium
C. Copper
D. Folate
E. Iron




The correct answer is D. The patient has a megaloblastic anemia, which can be due to deficiency of folate or B12. Pregnancy increases the need for folate and other nutrients used by both baby and mother, and may "unmask" a borderline dietary deficiency. For this reason, most obstetricians recommend vitamin supplements for pregnant women. Folic acid deficiency is a macrocyte anemia. Macro-ovalocytes and hypersegmented neutrophils are revealed on peripheral blood smear. Folic acid is the common term for pteroylmonoglutamic acid.

Ascorbic acid (choice A) is vitamin C, and its deficiency predisposes for capillary fragility and oral lesions.

Calcium deficiency (choice B) predisposes for osteoporosis/osteopenia.

Copper deficiency (choice C) is rare; when it occurs, it may cause a hypochromic anemia, neutropenia, osteoporosis, or hypotonia.

Iron deficiency (choice E) causes a microcytic, hypochromic anemia, with reduced mental and physical performance.
At which of the following sites is the characteristic triple helical structure of the collagen initially formed?
A. Extracellular space
B. Golgi body
C. Nucleus
D. Rough endoplasmic reticulum
E. Smooth endoplasmic reticulum




The correct answer is B. Collagen formation begins with transcription of mRNA from appropriate DNA genes in the nucleus. While still within the nucleus, the mRNA is spliced. It is then transported through the cytoplasm to the ribosomes on the rough endoplasmic reticulum. Individual chains are translated on the ribosomes, with the ends feeding into the endoplasmic reticulum lumen. Within the lumen, glycosylation of the individual chains occurs. The material then moves toward the Golgi bodies (whose lumens are connected to the endoplasmic reticulum) where the triple helices of procollagen form. The procollagen is then secreted into the extracellular space, where cleavage of pro-peptides and cross- linking of different triple helices occurs, maturing the collagen.

The extracellular space (choice A) is the site of procollagen cleavage and cross- linking.

The nucleus (choice C) is the site of mRNA transcription and splicing.

The rough endoplasmic reticulum (choice D) is the site of chain translation and glycosylation.

The smooth endoplasmic reticulum (choice E) does not participate in collagen synthesis.
The biochemical basis of a genetic disease is an inability to add the recognition marker mannose phosphate to enzymes. In which of the following organelles does this step usually occur?
A. Endoplasmic reticulum
B. Golgi apparatus
C. Lysosome
D. Mitochondria
E. Ribosome




The correct answer is B. The disease is I cell disease, which is a rare genetic disease that may receive disproportionate attention both because it is a mucolipidosis (a form of generalized lysosomal disorder) with accumulation of abnormal chemical material in lysosomes, and because the biochemical basis illustrates an interesting mechanism. Specifically, the Golgi apparatus in cells of these patients has an abnormal N-acetyl-glucosaminotransferase (N- acetylglucosamine-1-phosphotransferase), and is not able to add the necessary recognition marker mannose phosphate to enzymes usually destined to enter lysosomes. Physiologically, the Golgi apparatus is a cellular organelle consisting of a series of membranous plates that give rise to lysosomes and secretory vesicles. A complete deficiency of this enzyme (type I form of I cell disease) causes death early in life.

Endoplasmic reticulum (choice A) receives the growing peptide chain of the enzymes, but is not the site of addition of recognition markers.

Lysosomes (choice C) are the normal destination for the enzymes with the mannose phosphate marker, but do not receive them if the marker is not present.

Mitochondria (choice D) supply the ATP to drive these chemical reactions.

Ribosomes (choice E) are the site of synthesis of the amino acid chains of proteins, but not the site of addition of the destination marker.
A patient has been on a self-imposed "starvation diet" for four months, and has lost 60 pounds while consuming only water and vitamin pills. If extensive blood studies were performed, which of the following would be expected to be elevated?
A. Acetoacetic acid
B. Alanine
C. Bicarbonate
D. Chylomicrons
E. Glucose




The correct answer is A. Long-term starvation induces many biochemical changes. Much of the body's energy requirements are normally supplied by serum glucose, but in starvation are supplied by both glucose and lipid-derived ketone bodies, including acetoacetic acid and beta-hydroxybutyric acid. Glucose cannot be synthesized from lipids, and is instead made from amino acids such as alanine in the process of gluconeogenesis. Ketosis as seen in diabetic patients would also be expected.

Serum alanine (choice B) drops dramatically in starvation, due to its conversion to glucose.

Bicarbonate (choice C) levels drop as the bicarbonate buffers the hydrogen ions produced by the ketone bodies.

Chylomicrons (choice D) are the lipid form seen after absorption of dietary fat, and would drop because the person is not feeding.

Glucose (choice E) is maintained in the blood at a much lower than normal level during starvation.
A recent immigrant complains of fatigue, weight gain, constipation, and cold intolerance. Physical examination demonstrates a diffuse mass in the anterior aspect of the neck. Dietary deficiency of which of the following nutrients is most likely to have contributed to the patient's problem?
A. Copper
B. Iodine
C. Iron
D. Selenium
E. Zinc




The correct answer is B. Endemic goiter, such as in this patient, is due to dietary iodine deficiency. This disorder is common world-wide in mountainous areas (where fish are not available), although the use of iodized salt in the United States has limited its prevalence here. Frank symptoms of hypothyroidism may or may not be present, possibly because of the increased synthesis of the more potent triiodothyronine (T3) at the expense of thyroxine (T4). Goiters may become multinodular and grow to great size. Impaired cognition and hearing may be severe in congenital hypothyroidism.

Copper deficiency (choice A) can cause anemia, neutropenia, hypotonia, psychomotor retardation, osteoporosis, depigmentation of hair, and glucose intolerance.

Iron deficiency (choice C) can cause anemia, cognitive dysfunction, impaired immunity, impaired thermoregulation, and reduced levels of physical activity. Iron deficiency most commonly occurs during pathological bleeding, such as an ulcer.

Selenium deficiency (choice D) can cause congestive cardiomyopathy and skeletal muscle degeneration.

Zinc deficiency (choice E) causes rash, growth retardation, and impairments of immunity, wound healing, mentation, sexual function, and night vision. Zinc deficiency most commonly occurs in the elderly.
Histones can bind DNA tightly because they have a high isoelectric point. This is due to an enrichment in the amino acid
A. aspartate
B. glycine
C. lysine
D. proline
E. tyrosine




The correct answer is C. Histones are enriched in the amino acids lysine and arginine. They are positively charged and bind tightly to DNA, which is negatively charged from phosphate groups.

Proline (choice D) and glycine (choice B) are neutral, aliphatic amino acids. Aspartate (choice A) is negatively charged. Tyrosine (choice E) is a neutral, aromatic amino acid.
Which of the following amino acids is post-translationally hydroxylated in the cytoplasm of fibroblasts?
A. Cysteine
B. Glycine
C. Proline
D. Serine
E. Tyrosine




The correct answer is C. Hydroxylation of proline in fibroblasts generates the modified amino acid hydroxyproline. This is an example of post-translational modification. Hydroxyproline is involved in stabilizing the three-dimensional triple helix structure of collagen. Proline also has an alphatic side chain but differs from other amino acids in that its side chain is bonded to both the nitrogen and carbon atoms. Note: Proline contains a secondary rather than a primary amino group, which specifically makes it an imino acid.

Cysteine (choice A) is unique in its ability to form a covalent disulfide bond with another cysteine residue elsewhere in the protein molecule, thereby forming a cysteine bond. Such strong disulfide bonds stabilize the three-dimensional structure of the protein.

Glycine (choice B) is abundant in fibroblasts since it constitutes every third amino acid in the primary sequence of collagen. However, glycine is not hydroxylated.

Serine (choice D), tyrosine (choice E), and threonine can all be phosphorylated post-translationally to form phosphoserine, phosphotyrosine, and phosphothreonine, respectively. These phosphorylated amino acids are believed to play a role in signal transduction.
Which of the following will be unchanged in a Lineweaver-Burk plot of an enzyme with and without a competitive inhibitor?
A. Km
B. Slope
C. x-intercept
D. y-intercept




The correct answer is D. It is worth taking the time to learn how to read a Lineweaver-Burk plot. Lineweaver-Burk plots are used to determine the Vmax and Km of an enzyme; they are also used to differentiate between competitive and noncompetitive inhibition.

Note that in a Lineweaver-Burk plot, the slope is Km/Vmax, the x-intercept is -1/Km, and the y-intercept is 1/Vmax. In the presence of a competitive inhibitor, the Km(choice A) and therefore the slope (choice B) are both increased. Similarly, if Km is increased, -1/Km will become less negative and the x-intercept will shift to the right. Intuitively, this makes sense since a competitive inhibitor will increase the amount of substrate needed to reach half-maximal velocity (definition of Km). In contrast, the Vmax, and hence the y-intercept, is unchanged (choice D). It is important to note that it is convenient to transform the Michaelis-Menton equation into one that gives a straight line plot, which can be done by taking the reciprocal of both sides to

1/V = 1/Vmax + Km/Vmax + 1/[s]

A plot of 1/V versus 1/[s] is called the Lineweaver-Burk plot.
A patient has Lesch-Nyhan syndrome caused by an absence of hypoxanthine-guanine phosphoribosyl transferase. The patient's condition is caused by an enzyme deficiency in which of the following biochemical pathways?
A. Ganglioside metabolism
B. Monosaccharide
C. Purine metabolism
D. Pyrimidine metabolism
E. Tyrosine metabolism




The correct answer is C. The patient has a classic case of Lesch-Nyhan syndrome, an X-linked recessive disorder caused by severe deficiency of the purine salvage enzyme hypoxanthine-guanine phosphoribosyl transferase (HPRT). This defect is associated with excessive de novo purine synthesis, hyperuricemia. The clinical and symptoms include spasticity, writhing movements, compulsive biting of fingers and lips, and head-banging. At puberty, the child often develops arthritis and death from renal failure occurs at age 25 years. The biochemical basis of the often striking self-mutilatory behavior (which may require restraints and even tooth extraction) has never been established. Treatment with allopurinol inhibits xanthine oxidase and reduces gouty arthritis, urate stone formation, and urate nephropathy. It does not, however, modify the neurologic/psychiatric presentation. Mental deficiency, spasticity, and self-mutilation are the most common characteristics of this condition.
Patients with gout will have localized concentrations of needle shaped, negatively birefringent crystals of which substance?
A. Bile pigments
B. Calcium pyrophosphate
C. Cystine
D. Monosodium urate
E. Struvite




The correct answer is D. The patient has gout, which is due to precipitation of monosodium urate crystals in joint spaces (notably the great toe) and soft tissues (causing tophi, which are often found on the external ears). Gout is often an acute condition that is typically nocturnal and usually monarticular, often involving the first metatarsophalangeal joint. Hyperuricemia occurs in most cases. Identification of urate crystals in joint fluid or tophi is diagnostic.

Bile pigments (choice A) are found in some gallstones.

Calcium pyrophosphate (choice B) crystals are deposited in pseudogout, which classically affects the knee or other large joints.

Cystine (choice C) and struvite (choice E) can form kidney stones.
A young woman with anorexia nervosa consumes about 125g carbohydrate, 15g protein and 10 g fat daily. Her daily caloric intake is roughly equal to which of the following values?
A. 450 kcal/day
B. 650 kcal/day
C. 850 kcal/day
D. 1050 kcal/day
E. 1250 kcal/day




The correct answer is B. You may be asked to make this type of calculation on the NBDE part 1. You should know that 1 g of either protein or carbohyrate produces about 4 kcal = 4 Calories (kcal) of energy, and 1 g of fat produces 9 kcal = 9 Calories of energy. The calculation is then straightforward. The Calories from carbohydrates are 125 X 4 = 500; from protein are 15 X 4 = 60; and from fat are 10 X 9 = 90. The total is 500 + 60 + 90 = 650 kcal/day.
The activity of which of the following enzymes is directly affected by citrate?
A. Fructose-2,6-bisphosphatase
B. Isocitrate dehydrogenase
C. Phosphofructokinase I
D. Pyruvate carboxylase
E. 6-phosphogluconate dehydrogenase




The correct answer is C. Citrate is produced by citrate synthase from acetyl CoA and oxaloacetate. Note that this is the first step in the citric acid cycle. This reaction takes place in the mitochondria, but citrate can move freely from the mitochondria into the cytosol. When the citric acid cycle slows down, citrate accumulates. In the cytosol, it acts as a negative allosteric regulator of phosphofructokinase I, the enzyme that catalyzes the committed step of glycolysis.

Fructose-2,6-bisphosphatase (choice A) breaks down fructose-2,6-bisphosphate, a potent allosteric activator of phosphofructokinase I. Fructose-2,6-bisphosphatase is activated by cyclic AMP-dependent protein kinase.

Isocitrate dehydrogenase (choice B) converts isocitrate to alpha-ketoglutarate in the citric acid cycle. It is allosterically stimulated by ADP and inhibited by ATP and NADH. This reaction produces NADH and CO2.

Pyruvate carboxylase (choice D) is a mitochondrial enzyme that converts pyruvate to oxaloacetate. It is important in gluconeogenesis and replenishes the oxaloacetate in the citric acid cycle.

6-phosphogluconate dehydrogenase (choice E) converts 6-phosphogluconate to ribulose 5-phosphate in the pentose phosphate shunt pathway.
Which of the following enzymes is responsible for maintaining blood glucose levels by releasing glucose from its storage form in the liver?
A. Acetyl-CoA carboxylase
B. Glucose 6-phosphate dehydrogenase
C. Glycogen phosphorylase
D. Glycogen synthase
E. Thiolase




The correct answer is C. Glycogen is the storage form of glucose and a readily mobilizable fuel store. When individuals do not consume adequate quantities of carbohydrates, the body responds by breaking down glycogen stores in the liver to maintain normal blood glucose levels. The two enzymes involved in the degradation and synthesis of glycogen are glycogen phosphorylase (choice C) and glycogen synthase (choice D), respectively. The processes of glycogen synthesis and degradation are coordinated by a hormone-triggered cascade that ensures that when one enzyme is active, the other enzyme is inactive.

Acetyl-CoA carboxylase (choice A) is the key enzyme involved in fatty acid synthesis.

Glucose 6-phosphate dehydrogenase (choice B) is involved in the pentose phosphate pathway.

Thiolase (choice E) converts acetoacetyl-CoA into acetyl-CoA.
A cell biologist wants to examine the microstruture of an integral membrane protein. She solubilizes the protein by destabilizing its association with the membrane lipid bilayer. Which of the following techniques did she most likely use?
A. Alterations in pH
B. Detachment of protein phenyl groups
C. Dissociation of phospholipid polar head groups
D. Increase in ionic strength
E. Interruption of hydrophobic interactions




The correct answer is E. This question is asking you what forces are responsible for retaining integral membrane proteins within the lipid bilayer, i.e., hydrophobic interactions. Amphipathic agents, such as detergents, are used to solubilize integral membrane proteins; they do so by disrupting hydrophobic interactions between integral membrane proteins and other membrane constituents, such as phospholipids.

Choices A, B, C, and D would destabilize the association of peripheral membrane proteins with the membrane lipid bilayer.
Supplementation of calcium with which vitamin would reduce the risk for fracture caused by bone loss, particularly in elderly patients?
A. Calcitonin
B. Cholecalciferol
C. Carotene
D. Alpha tocopherol




The correct answer is B. Most of the calcium in the body is found as calcium phosphate crystals in the bones and teeth that contribute to the physical strength of these structures. Vitamin D (cholecalciferol) increases intestinal absorption of calcium and phosphate.

Choice A - Calcitonin is secreted by specialized parafollicular cells of the thyroid gland. It decreases the resorption of bone, reduces serum calcium, and improves bone architecture.

Choice C - Vitamin A (carotene) is converted to several active forms in the body and is associated with important functions, including maintenance of healthy epithelium and vision.

Choice D - Vitamin E (alpha tocopherol) prevents peroxidation of fatty acids in cell membranes, helping to maintain their normal fluidity.
Children with dry and wet beriberi typically have a deficiency of which vitamin?
A. Ascorbic acid
B. Retinol
C. Riboflavin
D. Thiamine
E. Vitamin K




The correct answer is D. Thiamine deficiency is most frequently encountered in alcoholics and in developing countries. Deficiency of this vitamin can take several forms: dilated cardiomyopathy (wet beriberi ), polyneuropathy (dry beriberi), and mamillary body degeneration (Wernicke-Korsakoff syndrome).

Ascorbic acid (choice A, Vitamin C) deficiency causes scurvy, associated with capillary fragility, bony abnormalities, and poor wound healing.

Retinol (choice B, Vitamin A) deficiency causes blindness and impaired immune responses.

Riboflavin (choice C) deficiency causes cheilosis, glossitis, and dermatitis.

Vitamin K (choice E) deficiency causes impaired blood clotting because of decreased production of factors II, VII, IX, and X.
Transcription of a prokaryotic gene by RNA polymerase yields an mRNA corresponding to a single polypeptide chain containing 64 amino acids. The mRNA has a 5' untranslated region of 120 nucleotides and a 3' untranslated region of 240 nucleotides. Approximately how many nucleotides are in the coding region of the mRNA?
A. 64
B. 128
C. 192
D. 424
E. 552




The correct answer is C. Regardless of how long the untranslated regions are, the number of nucleotides in the coding region of an mRNA is three times the number of amino acids, since three nucleotides are required to code for each amino acid, and 3 X 64 = 192. In reality, three nucleotides code for the first amino acid (formyl-methionine in prokaryotes, methionine in eukaryotes), which may be removed in posttranslational steps, and three nucleotides at the 3' end (are needed to terminate the process (i.e., a STOP codon), so the actual number would likely be slightly higher.
Which of the following enzymes does the neutrophil use to initiate the production of toxic oxygen compounds that kill bacteria abscesses and in other bacterial infections?
A. Hydrogen peroxide
B. Myeloperoxidase
C. NADPH oxidase
D. Peroxidase
E. Superoxide dismutase




The correct answer is C. The first step in killing bacteria is the production of superoxide ion, O2- , by the action of NADPH oxidase on NADPH and O2. The superoxide is then converted to hydrogen peroxide, either spontaneously or through the action of superoxide dismutase. The hydrogen peroxide can also be converted to the toxic HOCl. radical by the action of myeloperoxidase.

Hydrogen peroxide(choice A) is not an enzyme but is used as an antiseptic for dental procedures and for the treatment of apthous ulcers in the mouth.

Myeloperoxidase (choice B) is part of the killing pathway, but is active later in the sequence, when H2O2

Peroxidase (choice D) is also a heme enzyme. It catalyzes the reaction in which hydrogen peroxide is reduced to water.

Superoxide dismutase (choice E) is part of the killing pathway, but is active later in the sequence, when superoxide is converted to hydrogen peroxide. This enzyme is present in all aerobic organisms.
Which of the following amino acids is post-translationally hydroxylated in the cytoplasm of fibroblasts?
A. Cysteine
B. Glycine
C. Proline
D. Serine
E. Tyrosine




The correct answer is C. Hydroxylation of proline in fibroblasts generates the modified amino acid hydroxyproline. This is an example of post-translational modification. Hydroxyproline is involved in stabilizing the three-dimensional triple helix structure of collagen.

Cysteine (choice A) is unique in its ability to form a covalent disulfide bond with another cysteine residue elsewhere in the protein molecule, thereby forming a cystine residue. Such strong disulfide bonds stabilize the three-dimensional structure of the protein.

Glycine (choice B) is abundant in fibroblasts since it constitutes every third amino acid in the primary sequence of collagen. However, glycine is not hydroxylated.

Serine (choice D), tyrosine (choice E), and threonine can all be phosphorylated post-translationally to form phosphoserine, phosphotyrosine, and phosphothreonine, respectively. These phosphorylated amino acids are believed to play a role in signal transduction.
During DNA replication, which of the following enzymes produces a short strand of RNA complementary to the template DNA with a free 3'-OH end?
A. DNA ligase
B. Polymerase I
C. Polymerase III
D. Primase
E. Topoisomerase




The correct answer is D. The primer molecule required by DNA polymerase is a short strand of RNA (4-10 bases) complementary to the template strand of the DNA molecule. The primer is synthesized by a specific RNA polymerase known as primase. The growing end of the RNA primer is a free 3'-OH group. The primer RNA is excised at a later stage of replication. The primase does not itself require a primer for initiation of nucleotide synthesis.

DNA ligase (choice A) catalyzes the formation of a phosphodiester bond between the 3'-OH of one fragment of DNA and the 5'-monophosphate group of an adjacent DNA fragment.

Polymerase I (choice B) catalyzes the polymerization of nucleotides and also functions in processing and repair mechanisms.

Polymerase III (choice C) is a part of a multiprotein complex and is the major replicating enzyme in E. coli.

Topoisomerases (choice E) produce swivel points in the DNA molecule that relieve the strain induced by the replication fork. These enzymes cut and reseal the DNA.
Histones can bind DNA tightly because they have a high isoelectric point. This is due to an enrichment in the amino acid
A. aspartate
B. glycine
C. lysine
D. proline
E. tyrosine




The correct answer is C. Histones are enriched with the amino acids lysine and arginine. They are positively charged and bind tightly to DNA, which is negatively charged because of its phosphate groups.

Proline (choice D) and glycine (choice B) are neutral, aliphatic amino acids. Aspartate (choice A) is negatively charged. Tyrosine (choice E) is a neutral, aromatic amino acid.
Which of the following amino acids would most likely be found on the surface of a protein molecule?
A. Alanine
B. Arginine
C. Isoleucine
D. Leucine
E. Phenylalanine




The correct answer is B. This question requires two logical steps: first, you need to appreciate that the hydrophilic amino acids are more likely to appear on the surface of a protein molecule, while hydrophobic amino acids are most likely be found in its interior. Next, you need to figure out which of the amino acids listed is hydrophilic. If you recall that arginine is a basic amino acid that is positively charged at physiologic pH, you should be able to answer this question right away. In addition to arginine, lysine and histidine also have basic side chains. Structurally the planar outer part of arginine's side chain consists of 3 nitrogens bonded to a carbon atom, which is called a guanidium group.

All of the other choices have neutral side chains and are uncharged at physiologic pH. They would most likely be found in the hydrophobic core of the protein structure. Alanine (choice A), isoleucine (choice C), and leucine (choice D) all have aliphatic side chains; phenylalanine (choice E) has aromatic side chains.
A 25-year-old man with known history of drug abuse walks into the clinic. A routine screening test for HIV exposure is positive. To confirm HIV exposure, which one of these tests is ideal?
A. ELISA
B. Southern blot
C. Northern blot
D. Western blot




The correct answer is D. Western blot is used as the confirmatory test for HIV exposure and is highly specific. The combination of ELISA and Western blot has a positive predictive value greater than 99%. The Western blot primarily eliminates a small percentage of false positives found by the ELISA test alone.

Choice A - ELISA is used as the primary screening assay because it is very sensitive.

Choice B - Southern blotting is a technique that can be used to detect mutations in DNA.

Choice C - Northern blots analyze RNA extracted from a tissue and are typically used to determine which genes are being expressed.
A 25-year-old man with known history of drug abuse walks into the clinic. A routine screening test for HIV exposure is positive. To confirm HIV exposure, which one of these tests is ideal?
A. ELISA
B. Southern blot
C. Northern blot
D. Western blot




The correct answer is D. Western blot is used as the confirmatory test for HIV exposure and is highly specific. The combination of ELISA and Western blot has a positive predictive value greater than 99%. The Western blot primarily eliminates a small percentage of false positives found by the ELISA test alone.

Choice A - ELISA is used as the primary screening assay because it is very sensitive.

Choice B - Southern blotting is a technique that can be used to detect mutations in DNA.

Choice C - Northern blots analyze RNA extracted from a tissue and are typically used to determine which genes are being expressed.
A 30-year-old man has been fasting for several days. His blood glucose level is now about 60% of its normal value, but he does not feel lightheaded because his brain has reduced its need for serum glucose by using which of the following substances as an alternate energy source?
A. Apoprotein B
B. Beta-carotene
C. Beta-hydroxybutyrate
D. C-reactive protein
E. Coenzyme A




The correct answer is C. Ketone bodies, which include acetoacetate, beta-hydroxybutyrate, and acetone, are produced by the liver in the fasting state by beta-oxidation of fatty acids. They are then released into the blood stream, where they can be used as alternative energy sources for other organs, such as muscle, kidney, and brain. The brain specifically still requires a small amount of circulating glucose to function, but the amount required is reduced when ketone bodies are available.

Apoprotein B (choice A) is one of the proteins that hold lipoproteins together.

Beta-carotene (choice B) is a vitamin with antioxidant properties.

C-reactive protein (choice D) is a serum protein produced by the liver that rises during infections and in inflammatory states.

Coenzyme A (choice E) is found in mitochondria and carries acetyl groups into the citric acid, or tricarboxylic acid, cycle.
When alcohol is consumed, it is metabolized by the liver, with almost half the alcohol being oxidized to acetaldehyde. In which of the following sites does the reaction occur?
A. Golgi bodies
B. Lysosomes
C. Mitochondrial matrix
D. Peroxisomes
E. Ribosomes




The correct answer is D. Peroxisomes are cell organelles that are present in only small numbers in most mammalian cells. In the liver, however, these single membrane-bound organelles are present in large numbers and are import in detoxification and long chain fatty acid metabolism. The clinically important degradation of ethanol to (potentially toxic) acetaldehyde occurs in humans in both peroxisomes and the smooth endoplasmic reticulum (P450 system). Acetaldehyde is then oxidized to (non-toxic) acetate. This reaction, catalyzed by aldehyde dehydrogenase, occurs in the mitochondria.

Golgi bodies (choice A) are involved with packaging substances that are transported out of the cell.

Lysosomes (choice B) are sites of degradation of intracellular waste products.

The mitochondria (choice C) are involved with ATP production and contain both the electron transport chain and the citric acid (Krebs) cycle.

Ribosomes (choice E) are the sites of protein synthesis.
In the CNS, the drug levodopa is metabolized to dopamine, which can be subsequently converted by some neurons to which of the following substances?
A. Choline
B. Dihydroxyphenylalanine (DOPA)
C. Epinephrine
D. Norepinephrine (NE)
E. Tyrosine




The correct answer is D.Levodopa is used in the treatment of Parkinson disease, a condition caused by dopamine deficiency in the CNS. When levodopa enters the CNS, it is metabolized to dopamine, some of which is subsequently metabolized to norepinephrine (NE) in noradrenergic neurons. In other words, dopamine is the immediate precursor to NE. The synthesis of NE begins in the axoplasm of the terminal nerve endings of adrenergic fibers; however, its synthesis is completed inside the vesicles of these fibers.

With respect to the endogenous syntheses of NE, the basic steps are as follows: tyrosine (choice E) is converted to dihydroxyphenylalanine (DOPA) through the process of hydroxylation. Then, DOPA (choice B) undergoes decarboxylation to the neurotransmitter dopamine. Dopamine is then transported into the vesicles of the adrenergic fibers, where it undergoes hydroxylation to NE (choice D). In the adrenal medulla, NE is transofrmed into epinephrine (choice C) through the process of methylation.

Choline (choice A) is combined with acetyl-CoA to become acetylcholine.
The glycolytic degradation of ingested glucose commences with the action of which of the following enzymes?
A. Aldolase
B. Hexokinase
C. Phosphofructokinase
D. Phosphoglucose isomerase
E. Pyruvate kinase




The correct answer is B. The process of glycolysis is defined as the sequence of reactions that converts glucose into pyruvate with the concomitant production of ATP.

Glycolysis begins when glucose is converted by hexokinase to glucose-6-phosphate. When this compound interacts with the enzyme phosphoglucose isomerase (choice D), fructose-6-phosphate is formed. Fructose-6-phosphate is then converted by phosphofructokinase (choice C) to form fructose 1,6-biphosphate, which is subsequently converted to glyceraldehyde 3-phosphate by aldolase (choice A). After a number of enzymatic reactions, phosphoenolpyruvate is formed.

Phosphoenolpyruvate is converted to pyruvate by pyruvate kinase (choice E), and the glycolytic pathway is then completed.
Which type of dentin is most regular in structure?
A. Primary dentin
B. Secondary dentin
C. Reparative dentin
D. All have the same regular structure




The correct answer is A. All dentin is formed by odontoblasts located at the border of the pulp and dentin. It consists of tubules surrounding odontoblastic processes, as well as intertubular dentin. In primary dentin, the tubules are regular in pattern. In secondary dentin, formed later in life, tubules are less regular, less numerous and more wavy. Reparative dentin (sclerotic dentin) forms in response to caries, heat, deep fillings, etc. It is least regular and most wavy, and tubules are least numerous.
Alveolar bone is composed of:
A. cortical bone only
B. cancellous bone only
C. cortical bone surrounding cancellous bone
D. cancellous bone surrounding cortical bone




The correct answer is C. Alveolar bone is a combination of cortical (compact) bone and trabecular (spongy) bone. The cortical bone is on the outside surface, both next to the tooth root and also continuous with the cortical bone of the maxilla and mandible. The cortical bone adjacent to the tooth root is sometimes referred to as lamina dura. Nerves and blood vessels travel primarily through the more porous cancellous bone between the two cortical plates. Note that alveolar bone can be lost rapidly either in periodontal disease or following tooth extraction.
Which cusp becomes smaller and less conspicuous as you go from maxillary first to second to third molar?
A. Mesiobuccal
B. Mesiolingual
C. Distobuccal
D. Distolingual




The correct answer is D. The MB, ML, and DB cusps of the maxillary molars are the largest, and form the primary cusp triangle, or trigon, of the tooth. The distolingual cusp is the smallest, and is not part of the trigon. It is sometimes referred to as the talon or talon cusp. It is most noticeable in the first molar, less noticeable in the second, and often absent in the third molar. The maxillary third molar therefore often does not have the rhomboid appearance of a maxillary molar, and the occlusal aspect will appear triangular or heart-shaped instead.
Which cusp becomes smaller and less conspicuous as you go from maxillary first to second to third molar?
A. Mesiobuccal
B. Mesiolingual
C. Distobuccal
D. Distolingual




The correct answer is D. The MB, ML, and DB cusps of the maxillary molars are the largest, and form the primary cusp triangle, or trigon, of the tooth. The distolingual cusp is the smallest, and is not part of the trigon. It is sometimes referred to as the talon or talon cusp. It is most noticeable in the first molar, less noticeable in the second, and often absent in the third molar. The maxillary third molar therefore often does not have the rhomboid appearance of a maxillary molar, and the occlusal aspect will appear triangular or heart-shaped instead.
Which of the following is NOT considered an important reason for restoring proximal contact in dental restorations?
A. Contact protects the periodontal tissue
B. Contact prevents food impaction
C. Contact increases retention of the restorations
D. Contact prevents tooth drifting




The correct answer is C. Contact is re-established in proximal areas when Class II restorations are completed. This restoration of contact at the correct contact level prevents food from wedging in between teeth and causing buildup of plaque and debris. This, in turn, protects the periodontal ligament, because the chief etiology of periodontal bone loss is calculus and plaque accumulation. Stability of the arch is maintained and tooth drifting is also minimized. Without contact, teeth may drift mesially or distally into the space left by the faulty restoration. Proximal contact is NOT considered to be related to filling material retention. This is because each restoration's retention is supposed to depend solely on the retentive properties of that individual tooth, tooth preparation, and material. In other words, retention stands alone within the individual tooth. Adjacent teeth are never considered as part of restoration retention.
Which cusp on the primary mandibular first molar is highest and sharpest?
A. Mesiobuccal
B. Distobuccal
C. Mesiolingual
D. Distolingual




The correct answer is C. The primary first mandibular molar is a highly unusual tooth. It is molar-like in form, with two roots (mesial and distal), but has a very pronounced mesial development of the crown. The mesial section comprises two-thirds of the crown, and the distal portion is much reduced. The mesiolingual cusp is the highest and sharpest, although the mesiobuccal is overall largest as viewed from the occlusal. It is followed in size by the ML, DB, and DL cusps.
The masseteric sling is composed of the masseter and the:
A. medial pterygoid
B. lateral pterygoid
C. anterior digastric
D. temporalis




The correct answer is A. The masseteric sling is a powerful pair of muscles which wrap underneath the angle and ramus of the mandible and act to both support and close (elevate) the mandible. On the lateral surface of the mandible we find the masseter, while on the medial surface we find the medial pterygoid. Although the temporalis (choice D) also strongly elevates, it is not part of this structure. The lateral pterygoid (choice B) attaches to the condyle and TMJ disc, and pulls the condyles forward and laterally. The lateral pterygoid is not part of the masseteric sling.
Which cusp becomes smaller and less conspicuous as you go from maxillary first to second to third molar?
A. Mesiobuccal
B. Mesiolingual
C. Distobuccal
D. Distolingual




The correct answer is D. The MB, ML, and DB cusps of the maxillary molars are the largest, and form the primary cusp triangle, or trigon, of the tooth. The distolingual cusp is the smallest, and is not part of the trigon. It is sometimes referred to as the talon or talon cusp. It is most noticeable in the first molar, less noticeable in the second, and often absent in the third molar. The maxillary third molar therefore often does not have the rhomboid appearance of a maxillary molar, and the occlusal aspect will appear triangular or heart-shaped instead.
The anterior tooth with the most pronounced lingual ridge is the:
A. Maxillary central incisor
B. Mandibular lateral incisor
C. Maxillary canine
D. Mandibular canine




The correct answer is C. A lingual ridge runs from the incisal edge down the center of the lingual surface of the crown, toward the cervical end of the crown. It splits the lingual surface in half, and usually has a depression on either side (lingual fossae) which separate the lingual ridge from the mesial and distal marginal ridges. Lingual ridges are found on canines, but not on incisors (choices A and B). The lingual ridge is more pronounced on the maxillary canine than on the mandibular (choice D). This is in keeping with a general rule that the lingual anatomy of the maxillary canine is much more pronounced than that of the mandibular.
In centric occlusion, the mesiolingual cusp of the maxillary third molar will contact:
A. the central fossa of the mandibular second molar
B. the central fossa of the mandibular third molar
C. the distal marginal ridge of the mandibular first molar and mesial marginal ridge of the second molar
D. the distal marginal ridge of the mandibular second molar and mesial marginal ridge of the third molar




The correct answer is B. The mesiolingual cusp of a maxillary third molar is a holding cusp. The general rule for maxillary holding cusps is as follows: a maxillary holding cusp contacts the distal marginal ridge of its mandibular counterpart and the mesial marginal ridge of the mandibular tooth distal to its counterpart, EXCEPT FOR THE MESIOLINGUAL CUSPS OF THE MOLARS, WHICH CONTACT THE CENTRAL FOSSAE OF THEIR COUNTERPARTS. This should be the central fossa of the counterpart, the mandibular third molar.
In comparison to those of permanent molars, the common root trunks of primary molars are proportionately:
A. larger
B. smaller
C. relatively equal
D. variable, depending on the tooth




The correct answer is B. The common root trunk refers to that area of root apical to the crown but coronal to the bifurcation or trifurcation of the roots. It can be viewed as the combined root area from which the individual roots emerge. It is characteristic of primary molars that they have little or no common root trunk. That is, the roots appear to furcate almost immediately below the crown. This is due to the fact that the primary molar roots must flare widely in order to leave room for the developing crowns of the succedaneous premolars.
The attrition at the contact point between teeth is known to cause:
A. Reduced embrasures interproximally
B. Diastemas
C. Longer clinical crowns
D. Shorter clinical crowns




The correct answer is A. Over time, posterior teeth lose small amounts of enamel as the contact points wear against each other during chewing. The contacts become slightly broader, the teeth themselves become slightly closer, through drifting, and the embrasures between the teeth become slightly smaller. Diastemas (choice B) are not created, because the teeth drift to close any space created by the attrition. The process is at a microscopic level; the wear is very gradual, as is the accompanying drifting, so that contact is not lost. This process does not affect crown length (choices C and D), which is occluso-cervical in direction. It only affects the mesio-distal dimension.
The epithelium of the mucous membrane of the mouth is:
A. simple squamous
B. stratified squamous
C. simple columnar
D. stratified columnar
E. cuboidal




The correct answer is B. Squamous cells are flattened, pancake-like cells, and when in layers are known as stratified. The stratified squamous cells lining the mucosa of the oral cavity are similar to those lining the outer skin, although less keratinized. They do, however, flake off continuously into the mouth. Simple squamous tissue would be too thin and not allow for the constant flaking and replacement. Columnar epithelium is found in both the respiratory system and other organs of the digestive system.
The glenoid (articular) fossa in which the mandibular condyle articulates is a depression within which cranial bone?
A. Sphenoid
B. Zygomatic
C. Temporal
D. Parietal
E. A combination of more than one bone




The correct answer is C. The names for the fossa include mandibular, articular, glenoid, and temporal. This is due to the location of the fossa wholly within the temporal bone. The anterior border of the fossa is the articular eminence of the temporal bone, and the posterior border is the tympanic section of the temporal bone. Slightly more posterior is the mastoid process and associated styloid process. So the entire eminence is a temporal bone feature.
The mesiobuccal cusp of the maxillary first molar occludes in:
A. the mesial marginal ridge of the mandibular second molar
B. the distal marginal ridge of the mandibular first molar
C. the central groove of the mandibular first molar
D. the facial embrasure between the mandibular first and second molars
E. the buccal groove of the first mandibular molar




The correct answer is E. Buccal cusps of maxillary teeth are guiding cusps, not holding cusps. Therefore, they do not occlude on marginal ridges or central fossae. The general rule for maxillary buccal cusps is that they occlude in the facial embrasure between their mandibular counterpart and the tooth distal to their counterpart, EXCEPT for the mesiobuccal cusps of the molars, which occlude in the buccal grooves of their mandibular counterparts, and the distobuccal cusp of the first molar, which opposes the distobuccal groove of the mandibular first molar. In this question we are dealing with the exception of the mesiobuccal cusps of maxillary molars, which occlude with the buccal groove of their mandibular counterpart. In this case, the maxillary first molar mesiobuccal cusp contacts the mandibular first molar buccal groove.
In extraction of maxillary first premolars, the root anatomy guides the dentist to use a motion that is primarily:
A. buccal-lingual luxation
B. mesiodistal luxation
C. rotation
D. all of the above




The correct answer is A. One of the best reasons to know standard and unusual root shapes for the various maxillary and mandibular teeth is to use the correct motion on the forceps when performing extractions. In general, rounded roots are primarily rotated, including the maxillary central incisors and maxillary canines. All double- and triple-rooted teeth cannot be rotated and must be luxated in the buccal and lingual directions. Mesial-distal motion is normally ruled out because of the existence of adjacent teeth. It is used in rare cases where there is no mesial and distal adjacent tooth. The maxillary first premolar is invariably double rooted with a buccal and lingual root, and can never be rotated without breaking the crown off of the root.
How many teeth (out of 32) in the normal dentition oppose only ONE other tooth?
A.None
B. 1
C. 2
D. 4
E. 6




The correct answer is D. Because of the shift of the mandibular teeth in a mesial direction due to the smaller size of the anterior teeth, most teeth have two opposing teeth. That is, they oppose their counterpart in the other arch, and a tooth either mesial or distal to that counterpart. There are two classes of exceptions. One of the exceptions is the mandibular central incisors. These two teeth (#24 and #25) oppose only the maxilary centrals (#8 and #9). The other exception is the maxillary third molars (#1 and #16), which oppose only the mandibular third molars.

Note that the mandibular third molars oppose both the maxillary second and third molars.
Which premolar is considered to be most molar-like in form?
A. Maxillary first
B. Maxillary second
C. Mandibular first
D. Mandibular second




The correct answer is D. The maxillary premolars are considered to be much more alike than are the mandibular premolars. They are most premolar-like in form, with fairly similar sized buccal and lingual cusps and greater overall symmetry. The mandibular premolars, by contrast, are significantly different from each other. The first premolar is very canine-like with a small lingual cusp and relatively large buccal cusp. The mandibular second premolar, while more premolar-like than the first, has attributes of molars as well. The chief molar-like attribute of the mandibular second is the tendency toward three cusps rather than two. The most common form of this tooth has a large buccal cusp with two smaller lingual cusps making up the rest of the crown. The two lingual cusps are the mesiolingual and distolingual and are separated by a lingual groove. There is a less common two-cusped variety which is less molar-like in form.
The distal inclination in roots of the permanent mandibular first molar is:
A. more pronounced in either root, depending on the tooth
B. usually more pronounced in the mesial root
C. usually more pronounced in the distal root
D. usually equal in both roots




The correct answer is B. In the mandibular first molar, one of the major distinctions between roots is the greater distal curvature of the mesial root. The distal root, by contrast, is usually more straight, without significant curvature. Another distinction is the deep root concavity seen running the length of the mesial surface of the mesial root. The distal root, by comparison, will have no similar concavity or a very slight depression. This concavity will sometimes express itself as a partial split (bifurcation) of some small section of the apical end of the mesial root into two roots. This feature is rare to unknown on the distal root.
The greatest curvature of the cervical line interproximally is found on the:
A. Mesial of the maxillary central
B. Mesial of the maxillary lateral
C. Distal of the maxillary central
D. Distal of the maxillary lateral




The correct answer is A. When viewed from the proximal, all incisors show a distinct curvature of the cervical line as the line curves markedly upward (toward the incisal). It then returns to a much lower (apical) level on both the facial and lingual. It is most noticeable on the mesial of the central incisor. This is one of those most, biggest, longest, shortest type questions common in this section.
Which incisor commonly exhibits the most variety in anatomy in both crown form and root form?
A. Maxillary central
B. Maxillary lateral
C. Mandibular central
D. Mandibular lateral




The correct answer is B. It is commonly said that the third molars and the maxillary lateral incisor are the most variable teeth in the mouth. The common forms of maxillary lateral variation include the peg lateral, with an ice cream cone-shaped crown; the deep lingual pit, with a deep (often carious) invagination; and the dens-in-dente, a deep lingual invagination giving a "tooth-within-a-tooth" appearance on x-ray. By contrast, variations in the other incisors are minor and less pronounced.
The mesiobuccal cusp of the maxillary third molar occludes in:
A. the mesial marginal ridge of the mandibular second molar
B. the distal marginal ridge of the mandibular second molar
C. the mesial marginal ridge of the mandibular third molar
D. the facial embrasure between the mandibular second and third molars
E. the buccal groove of the third mandibular molar




The correct answer is E. Buccal cusps of maxillary teeth are guiding cusps, not holding cusps. Therefore, they do not occlude on marginal ridges or central fossae. The general rule for maxillary buccal cusps is that they occlude in the facial embrasure between their mandibular counterpart and the tooth distal to their counterpart, EXCEPT for the mesiobuccal cusps of the molars, which occlude in the buccal grooves of their mandibular counterparts, and the distobuccal cusp of the first molar, which opposes the distobuccal groove of the mandibular first molar. In this question we are dealing with the exception of the mesiobuccal cusps of maxillary molars, which occlude with the buccal groove of their mandibular counterpart. In this case, the maxillary third molar mesiobuccal cusp contacts the mandibular third molar buccal groove.
The mesiobuccal cusp of the mandibular second molar occludes with which maxillary tooth surfaces?
A. The mesial marginal ridge of the second molar and distal marginal ridge of the first molar
B. The distal marginal ridge of the second molar and mesial marginal ridge of the third molar
C. The embrasure between the first and second molars
D. The central fossa of the maxillary second molar




The correct answer is A. The mesiobuccal cusp of a mandibular molar is a holding (supporting) cusp. The general rule of occlusion of mandibular holding cusps is as follows: the holding cusps of the mandibular teeth occlude on the mesial marginal ridge of their maxillary counterpart, and the distal marginal ridge of the maxillary tooth mesial to their counterpart, EXCEPT distobuccal cusps of mandibular molars occlude with central fossae of their counterparts, the distal cusp of the mandibular first molar occludes with the distal triangular fossa of its counterpart, and the first premolar occludes only with the mesial marginal ridge of its counterpart (but not the canine). In this case, the maxillary counterpart is the maxillary second molar, and the tooth immediately mesial to it is the maxillary first molar.
The most prominent cingulum is found on which incisor?
A. Maxillary central
B. Maxillary lateral
C. Mandibular central
D. Mandibular lateral




The correct answer is B. The maxillary lateral is often the most distinct in almost any anatomical form. Its cingulum is usually more notable relative to its size than that of the central. It should be noted that maxillary cingula are almost always more prominent than their mandibular counterparts. In addition to the prominent cingulum, the maxillary lateral often features a deep lingual fossa, deep lingual pits, and tubercles on the cingulum.
Which premolar has a lingually inclined occlusal table?
A. Maxillary first
B. Maxillary second
C. Mandibular first
D. Mandibular second




The correct answer is C. The mandibular first premolar is very canine-like in form, with a much reduced lingual cusp, almost resembling a cingulum. The occlusal table thus inclines apically from buccal to lingual, as in a downward slope. This is important in operative dentistry, as the occlusal preparation for this tooth also is slanted to the lingual. All other premolars are essentially flat in their occlusal table, or possibly very slightly lingually leaning if the buccal cusp is slightly higher than the lingual. However, only the mandibular first premolar has this trait so pronounced that it is very different from the other three.
When viewed from the buccal, the tooth closest to vertical in the mesiodistal angulation of its main axis is the:
A. maxillary central incisor
B. maxillary lateral incisor
C. maxillary canine
D. maxillary first premolar




The correct answer is A. Another way of asking the question is: which tooth does not lean toward the mesial or distal? The maxillary central incisor is almost straight vertically, estimated to be about 2 degrees from vertical. The lateral tends to lean slightly mesially, the canine even more in a distal direction, and the first premolar slightly in a distal direction. Do not confuse this leaning with the buccolingual leaning (when viewed from the mesial or distal). These measurements differ significantly for the teeth listed, with the premolar being most vertical.
The distobuccal cusp of the mandibular first molar occludes with which maxillary tooth surfaces?
A. The mesial marginal ridge of the second molar and distal marginal ridge of the first molar
B. The central fossa of the first molar
C. The mesial marginal ridge of the first molar and the distal marginal ridge of the second premolar
D. The embrasure between the first and second molars




The correct answer is B. The distobuccal cusp of a mandibular molar is a holding (supporting) cusp. The general rule of occlusion of mandibular holding cusps is as follows: the holding cusps of the mandibular teeth occlude on the mesial marginal ridges of their maxillary counterparts, and the distal marginal ridges of the maxillary tooth mesial to their counterparts, EXCEPT distobuccal cusps of mandibular molars occlude with central fossae of their counterparts, the distal cusp of the mandibular first molar occludes with the distal triangular fossa of its counterpart, and the first premolar occludes only with the mesial marginal ridge of its counterpart (but not the canine). In this case, the maxillary counterpart is the maxillary first molar, and the cusp should occlude in the central fossa of this tooth.
Canaliculi and Haversian systems:
A. are found in acellular cementum
B. are found in cellular cementum
C. are found in both types of cementum
D. are not found in cementum




The correct answer is D. Cementum is often described as the dental tissue which most resembles bone. It has a similar mineral content, and has an organic matrix primarily composed of collagen fibers. However, the Haversian systems, concentric tissue layers, lacunae, canaliculi, etc., found in bone are not found in cementum. Remember that these features are not always found in bone. Only compact bone (rather than trabecular bone) has these features. Remember also that acellular cementum could not have such features as Haversian systems, because Haversian systems contain osteocytes (cells) within bony lacunae. However, neither acellular nor cellular cementum actually has these features.
At the cervical line, a cross section of the maxillary canine would show:
A. a round shape
B. an oval shape wider on the lingual than the labial
C. a triangular shape with a distinctly wider labial surface
D. an oval shape with a slightly wider labial surface




The correct answer is D. The maxillary canine, at the cervix, will have a cross section which is oval and flattened mesiodistally. It is generally slightly wider at the labial end than at the lingual. The pulp cavity at this point is lens-shaped, double convex. It is similar in cross-sectional shape to the mandibula canine.
An example of a guiding (nonsupporting) cusp is:
A. distolingual of #30
B. mesiobuccal of #18
C. distolingual of #14
D. lingual of #5
E. palatal of #13




The correct answer is A. Note that the holding (supporting, occluding) cusps of the posterior dentition are the lingual (palatal) cusps of the maxillary teeth and the buccal (facial) cusps of the mandibular teeth. Buccals of maxillary teeth and linguals of mandibular teeth are guiding cusps. Choice A is the lingual of a mandibular first molar, so it is not a holding cusp, and is a guiding cusp. Choice B is a buccal of a mandibular molar and is a holding cusp. Choice C is a lingual cusp of a maxillary molar and is a holding cusp. Choice D is the lingual of a maxillary premolar and is a holding cusp. Choice E is the palatal (lingual) of a maxillary premolar and is a holding cusp.
Which incisor is most commonly congenitally missing?
A. Maxillary central
B. Maxillary lateral
C. Mandibular central
D. Mandibular lateral




The correct answer is B. The maxillary lateral and the third molars show more variability than any other teeth. Besides a wide variety of unusual morphologies (peg lateral, etc.), the maxillary lateral incisor is fairly commonly congenitally missing. In these patients, the canines drift mesially toward the centrals. Also commonly congenitally missing are third molars and premolars.
A patient is congenitally missing the maxillary lateral incisors and mandibular second premolars. This individual is said to have the condition known as:
A. anodontia
B. oligodontia
C. hypodontia
D. hyperdontia




The correct answer is C. Anodontia (choice A) refers to the complete absence of tooth formation. In oligodontia (choice B), most but not all teeth are missing. Both of these conditions are often associated with ectodermal dysplasia. Hypodontia refers to a small number of congenitally missing teeth. This is fairly common, and is not usually associated with a systemic disease or disturbance.
Cervical bulges are found on which surfaces of primary molar teeth?
A. Mesial and distal
B. Buccal and lingual
C. Lingual only
D. Buccal only




The correct answer is D. Cervical bulges are overhangs of the crown sticking out above the cervical line of the primary teeth. In primary anteriors, the cervical bulge is found on both the buccal and lingual surfaces. In the molars, it is found only on the buccal. The most conspicuous cervical bulge in the primary molars is that of the primary mandibular first molar. It is often described as being potbelly when seen from the proximal view. The other molars are not as exaggerated in this area, but still show a pronounced buccal cervical bulge.
In rare cases, a permanent mandibular first molar may have a complete supernumerary third root. This root is usually:
A. a second mesial root
B. a second distal root
C. a buccal root
D. a lingual root




The correct answer is B. Despite the fact that the mandibular first molar has a distinct mesial root concavity on the mesial side of the mesial root, and that this root sometimes furcates partially upwards from the mesial, complete additional whole roots are usually second distal roots. Buccal and/or lingual additional roots are not found.
In centric occlusion, the distolingual cusp of the maxillary first molar will contact:
A. the mesial marginal ridge of the mandibular first premolar and distal marginal ridge of the second premolar
B. the distal marginal ridge of the mandibular second premolar and mesial marginal ridge of the first molar
C. the distal marginal ridge of the mandibular first molar and mesial marginal ridge of the second molar
D. the distal marginal ridge of the mandibular second molar and mesial marginal ridge of the first molar




The correct answer is C. The distolingual cusp of a maxillary molar is a holding cusp. The general rule for maxillary holding cusps is as follows: a maxillary holding cusp contacts the distal marginal ridge of its mandibular counterpart and the mesial marginal ridge of the mandibular tooth distal to its counterpart, EXCEPT FOR THE MESIOLINGUAL CUSPS OF THE MOLARS, WHICH CONTACT THE CENTRAL FOSSAE OF THEIR COUNTERPARTS. This should be the distal marginal ridge of the mandibular first molar and mesial marginal ridge of the second molar.
The mesiolingual cusp of the mandibular second molar contacts:
A. the lingual embrasure between the maxillary first molar and second molar
B. the distal marginal ridge of the maxillary second molar
C. the central fossa of the maxillary second molar
D. the lingual groove of the maxillary second molar
E. the lingual embrasure between the maxillary second molar and third molar




The correct answer is A. Lingual cusps of mandibular teeth are guiding cusps, not holding cusps. Therefore, they do not occlude on marginal ridges or central fossae. The general rule for mandibular lingual cusps is that they occlude in the lingual embrasures between their maxillary counterparts and the teeth mesial to their counterparts, EXCEPT for the distolingual cusps of the mandibular molars, which occlude in the lingual grooves of their maxillary counterparts. In this question we are not dealing with the exception of the distolingual cusps of mandibular molars, so in this case, the mandibular second molar mesiolingual cusp contacts the lingual embrasure between its counterpart (the maxillary second molar) and the tooth mesial to it (the maxillary first molar).
The epithelial root sheath of Hertwig is composed of:
A. inner enamel epithelium and stellate reticulum
B. stellate reticulum only
C. inner and outer enamel epithelium
D. all of the tissues listed above
E. none of the above




The correct answer is C. Only the original inner and outer enamel epithelium take part in root formation. There is no stellate reticulum in between them, as would be found in the original enamel organ. The root form is originally outlined by the epithelial root sheath of Hertwig, which then induces formation of odontoblasts, which then produce root dentin. Pieces of the root sheath, which do not disappear, remain as the epithelial rests of Malassez. These small groups of tissue are ectodermal in origin, as the original enamel epithelium is ectodermal.
At the cervical line, a cross section of the maxillary lateral incisor would show:
A. a round shape
B. an oval shape wider on the lingual than the labial
C. a triangular shape with a distinctly wider labial surface
D. an oval shape with a slightly wider labial surface
E. mesiodistal flattening




The correct answer is A. The maxillary lateral incisor exhibits an almost round root cross section shape at the cervical line. It is not flattened mesiodistally or labiolingually. The pulp chamber mimics the root shape by being round at this point as well. It is very similar in both root shape and pulp shape to those of the maxillary central. Note that as the pulp enters the coronal section, it will begin to widen mesiodistally (flatten buccolingually) in keeping with the crown shape.
Because of the staining effect, tetracycline is not given to children younger than approximately what age?
A. 2 years
B. 5 years
C. 8 years
D. 11 years
E. 14 years




The correct answer is C. This may not seem like dental anatomy, but it is actually a calcification question. In order to avoid tetracycline staining or any other intrinsic stain of enamel, we must avoid all ages where tooth crown calcification is occurring. The last tooth to calcify (and erupt) is the third molar. Most third molars erupt at 17 to 21 years, but calcification of enamel occurs mostly at about age 8. So pregnant mothers and children age 8 and younger are not given tetracycline.
Which structure is involved in pulling the disc of the TMJ forward?
A. Stylomandibular ligament
B. Medial pterygoid muscle
C. Mandibular condyle
D. TMJ meniscus
E. Lateral pterygoid muscle




The correct answer is E. The lateral pterygoid connects from the greater wing of the sphenoid and the lateral surface of the lateral pterygoid plate to the mandibular condyle, articular disc, and capsule. It is the only muscle that attaches to these structures. The stylomandibular ligament, sometimes given a vague protective function, extends from the styloid process to the lingula of the mandible. The medial pterygoid muscle is an elevator with no direct attachment to the TMJ. The mandibular condyle is separated from the glenoid (temporal) fossa by the articular (TMJ) disc. The condyle does not pull the disc. The disc is also called the meniscus; they are the same thing.
The largest cusp of a maxillary first molar is the:
A. mesiobuccal
B. mesiolingual
C. distobuccal
D. distolingual
E. Carabelli




The correct answer is B. The three largest cusps (trigon, or primitive cusp triangle) of the maxillary first molar are the MB, ML, and DB cusps. The DL cusp is smallest, and is known as the talon cusp (or talon). This cusp is also the one which is even less conspicuous as you go from first to second to third molar. Note also that the DB cusp decreases in relative size as you go from first to second and third molar. The order of size of the cusps in this tooth, then, is from largest to smallest: ML, MB, DB, DL, Carabelli.
Tooth #B is replaced by tooth #:
A. 2
B. 3
C. 4
D. 5
E. 6




The correct answer is D. Remember that primary molars are succeeded by permanent premolars. The permanent molars erupt distal to all primary teeth and do not replace them. So the maxillary replacements are 4 for A, 5 for B, 6 for C, 7 for D, 8 for E, 9 for F, 10 for G, 11 for H, 12 for I, and 13 for J. 1, 2, 3, 14, 15, and 16 do not replace primary teeth.
Which two molars can be considered to be five-cusped teeth?
A. Maxillary first and second
B. Mandibular first and second
C. Maxillary and mandibular first
D. Maxillary and mandibular second




The correct answer is C. Most maxillary molars are considered four-cusped teeth. However, in some cases, the Carabelli cusp (trait) can be large enough to be considered a fifth cusp on a maxillary first molar. The maxillary second molar is usually four-cusped. The mandibular first molar is normally five-cusped with an ML, MB, DL, and DB cusp and a fifth distal cusp. The mandibular second lacks the distal cusp and is normally a four-cusp tooth.
The shape of the pulp chamber in cross section cut at the cervical line in the maxillary canine is:
A. oval (flattened mesiodistally)
B. oval (flattened bucco-lingually)
C. round
D. figure eight




The correct answer is A. Only maxillary incisors have roundish pulp chamber cross sections when cut at the cervical line. This is true for both maxillary centrals and laterals. In general, mandibular centrals and both arch canines will have ovalish pulp chambers, flattened mesiodistally. (Imagine squashing a round pulp from both the mesial and distal sides at once, resulting in a squashed oval, pointing to the buccal and lingual). Remember that the shape of a pulp chamber is usually related to the shape of the surrounding root.
In the maxillary arch, the narrowest incisal or occlusal embrasure is located:
A. between first premolar and canine
B. between canine and lateral incisor
C. between lateral incisor and central
D. between central incisors




The correct answer is D. The best way to answer the question is to imagine contact points and proximal-incisal angles for the anterior teeth. A small incisal embrasure will be located wherever the contact points are high (incisal) and where proximal-incisal line angles are almost perpendicular. The best example of this arrangement is the central incisors. Their mesial incisal line angles are very square, and the contact is in the incisal third of the tooth. As you go further distally, the line angles become more rounded, especially for canines. These rounded line angles at the incisal, as well as the more apical contacts, dropping to the middle third as you reach the distal of the canine, ensure much larger incisal embrasures.
The inner enamel cuticle, found on the surface of the enamel of an erupting tooth crown, is:
A. also known as Nasmyth's membrane
B. also known as the stratum intermedium
C. cellular
D. formed by odontoblasts




The correct answer is A. As the tooth crown nears eruption, the ameloblasts produce their final product, known as the inner enamel cuticle. This material is acellular, and is firmly adherent to the enamel surface as the tooth erupts. Its older name is Nasmyth's membrane. This cuticle has on its outside a second acellular layer formed from the keratinized remnants of the dental sac. This adherent double layer may persist for some time after the tooth erupts but is eventually worn away, leaving the enamel exposed in the oral cavity.
Which ligament of the temporomandibular joint originates from a thin, pointed extension of the temporal bone?
A. Temporomandibular
B. Stylomandibular
C. Sphenomandibular
D. Lateral




The correct answer is B. The bony origins of the three TMJ ligaments are primarily from three different bones. The stylomandibular ligament originates from a thin, bony extension of the temporal bone, known as the styloid process. The spine of the sphenoid bone is the major origin of the sphenomandibular ligament. The lateral ligament (temporomandibular ligament) has its origin from the zygomatic process. This process is a fusion of the temporal and zygomatic bones.
Which of the following is the most coronally located periodontal fiber group?
A. Alveolar crestal
B. Apical
C. Horizontal
D. Oblique




The correct answer is A. We can eliminate choice B, as the apex is at the opposite end from the crown. The apical fibers are, of course, most apical, and run from the apex of the alveolar bone socket to the root tip. The oblique fibers are slightly more coronal. They run from the cementum near the apical end, obliquely coronally to alveolar bone. The horizontal fibers are more coronal and run horizontally from cementum to alveolar bone around mid-root. The alveolar crestal fibers are most coronal, and run from the most coronal part of the root to the alveolar crest of bone.
In a distal view of a permanent maxillary first molar, how many roots should be visible?
A. 1
B. 2
C. 3
D. 4




The correct answer is C. This may not be as obvious as it seems. Rule out 4, as the molar only has three roots: the palatal, mesiobuccal, and distobuccal. However, how should they look in a distal view? The palatal root will clearly be visible as the only root on the palatal side. It is massive, and often hooked. On the buccal end, a distal view should easily show the full length of the distobuccal root, so we have two roots so far. What about the mesiobuccal? In this case we can see this root mostly because it is larger than the distobuccal, so some, but not all, of the mesiobuccal root is blocked by the distobuccal. We should, however, see some small sections of it outside of the profile of the distobuccal root.
The heights of contour of the mandibular first premolar, when viewed from the mesial, are located within which third?
A. buccal in occlusal third, lingual in middle third
B. both in middle third
C. both in occlusal third
D. buccal in cervical third, lingual in occlusal third
E. both in cervical third




The correct answer is D. As a general rule, the buccal heights of contour of the premolars are located within the cervical thirds. The lingual heights of contour will normally be more occlusal. On the mandibular first premolar, it is especially occlusal, within the occlusal third.
Which of the following oral muscles is NOT innervated by CN V?
A. Masseter
B. Buccinator
C. Medial pterygoid
D. Lateral pterygoid
E. Mylohyoid




The correct answer is B. The general rule for innervation by CN V (the trigeminal) is that it innervates the muscles of mastication (temporalis, lateral pterygoid, medial pterygoid, masseter), the mylohyoid, the anterior digastric, and the two tensors (tensor tympani and tensor veli palatini).

Note that the buccinator is not included. The buccinator is in the group of muscles of facial expression, which are all innervated by CN VII (the facial nerve).
The contact of the permanent mandibular incisor with the canine occurs at which level of the lateral?
A. Incisal third
B. Junction of incisal and middle thirds
C. Middle third
D. Gingival third




The correct answer is A. Mandibular incisors are widest near the incisal tip, and their contacts (both central and lateral) are always near the incisal edge. The mandibular canine contact with the lateral incisor is thus in the incisal third. In the canine, the contact is also in the incisal third, but not as incisal as in the lateral. The distal contact of the canine is also in the incisal third, but near the junction of the incisal and middle thirds.
Dens-in-dente is most commonly noted in which tooth?
A. Maxillary central
B. Maxillary lateral
C. Maxillary canine
D. Mandibular central
E. Mandibular lateral




The correct answer is B. The maxillary lateral is the most highly variable tooth in the mouth. It can be normal incisor-shaped, peg-shaped, or congenitally missing, or have a wide variety of invaginations, from small pits to deep pits to dens-in-dente, or tooth-within-a-tooth. This is the case when the invagination is so deep that the radiographic appearance appears to show a small tooth inside the maxillary lateral.
Dentinal tubules which enter into enamel are known as:
A. enamel spindles
B. enamel tufts
C. enamel lamellae
D. none of the above




The correct answer is A. There are a number of somewhat confusing anomalies at the dentinoenamel junction. Enamel spindles are odontoblastic processes and dentinal tubules which extend a short way into enamel. Enamel lamellae, on the other hand, extend from enamel into dentin. They are pieces of uncalcified organic enamel material. Enamel tufts are hypocalcified enamel rods. They are individual entities; however, they appear as groups (tufts) when a group of them are viewed laterally.
Guiding cusps normally contact:
A. mesial marginal ridges
B. distal marginal ridges
C. central fossae
D. embrasures




The correct answer is D. In occlusion, cusps are defined as either holding (supporting) cusps or guiding cusps. Holding cusps, in central occlusion, make contact with the opposing arch, establish vertical dimension of occlusion, and support the forces of occlusion. They are the lingual cusps of the maxillary arch and buccal cusps of the mandibular. When the teeth are in centric occlusion, these cusps are in contact. The other cusps (guiding) are not. The other cusps are either lingual to the contact (lingual cusps of the mandibular arch) or buccal to the contact (buccal cusps of the maxillary arch). Holding cusps generally occlude in the marginal ridge and central fossa areas of the opposing arch. Guiding cusps generally lie in embrasures between teeth, or between cusps of the lingual or buccal surfaces of the opposing arch.
Which premolar, when viewed from the facial, has a longer mesial cusp ridge than distal cusp ridge?
A. Maxillary first
B. Maxillary second
C. Mandibular first
D. Mandibular second




The correct answer is A. The question depends on the fact that the maxillary first premolar has an asymmetric positioning of its two cusps. Rather than having the buccal and lingual cusps in a straight line, the buccal cusp is displaced slightly distally, and the lingual cusp is displaced slightly mesially. This gives the occlusal view of the tooth a slightly twisted appearance. Cusp ridges run from the cusp tip to the proximal edge of the tooth at the marginal ridge. Therefore, if you look facially at the maxillary first premolar, you will see the buccal cusp shifted toward the distal. This makes the cusp tip farther from the mesial, and thus there will be a longer mesial cusp ridge on this tooth.
In centric occlusion, the distolingual cusp of the maxillary second molar will contact:
A. the central fossa of the mandibular second molar
B. the distal marginal ridge of the mandibular first molar and mesial marginal ridge of the second molar
C. the distal marginal ridge of the mandibular second molar and mesial marginal ridge of the first molar
D. the distal marginal ridge of the mandibular second molar and mesial marginal ridge of the third molar




The correct answer is D. The distolingual cusp of a maxillary second molar is a holding cusp. The general rule for maxillary holding cusps is as follows: a maxillary holding cusp contacts the distal marginal ridge of its mandibular counterpart and the mesial marginal ridge of the mandibular tooth distal to its counterpart, EXCEPT FOR THE MESIOLINGUAL CUSPS OF THE MOLARS, WHICH CONTACT THE CENTRAL FOSSAE OF THEIR COUNTERPARTS. This should be the distal marginal ridge of the mandibular second molar and mesial marginal ridge of the third molar.
Of the following premolars, the one which can never be rotated during extraction is the:
A. maxillary first
B. maxillary second
C. mandibular first
D. mandibular second




The correct answer is A. For the most part, rotation of a tooth during extraction works best on teeth with rounded roots, such as the maxillary central incisor and maxillary canine. It can be used in conjunction with buccal-lingual luxation on teeth with single ovalish roots. It can never be used on double-rooted teeth. Of the premolars above, the maxillary first is always double rooted. Of the others, the maxillary second is sometimes double rooted and sometimes single. For the mandibular premolars, the mandibular first is the most likely to have a single ovalish, but close to round, root. It is theMOST likely premolar on which rotation may be part of the extraction.
In centric occlusion, the lingual cusp of the maxillary second premolar contacts:
A. the mesial marginal ridge of the mandibular first premolar and distal marginal ridge of the second premolar
B. the distal marginal ridge of the mandibular first premolar and mesial marginal ridge of the second premolar
C. the mesial marginal ridge of the mandibular first molar and distal marginal ridge of the second premolar
D. the distal marginal ridge of the mandibular second premolar and mesial marginal ridge of the first molar




The correct answer is D. The mesiolingual cusp of a maxillary second premolar is a holding cusp. The general rule for maxillary holding cusps is as follows: a maxillary holding cusp contacts the distal marginal ridge of its mandibular counterpart and the mesial marginal ridge of the mandibular tooth distal to its counterpart, EXCEPT FOR THE MESIOLINGUAL CUSPS OF THE MOLARS, WHICH CONTACT THE CENTRAL FOSSAE OF THEIR COUNTERPARTS. This should be the distal marginal ridge of the mandibular second premolar and mesial marginal ridge of the first molar.
The maxillary molar most likely to have fused roots is the:
A. first
B. second
C. third
D. fused roots are rare for all three molars




The correct answer is C. While all three maxillary molars commonly have three roots, the roots and their arrangement differ distinctively from one to the other. Most notable in the first molar is the pliers-like appearance of the two buccal roots. The mesiobuccal hooks distally and the distobuccal hooks mesially. The second molar does not have this curvature, and the two buccal roots are more parallel, with both roots inclined distally. The third molar often has three fused roots forming a cone-like structure, or sometimes all distally inclined. These factors are important in exodontia of maxillary molars. Maxillary third molars, in particular, can sometimes be elevated out distally in a rotating manner because of the fused cone-like distally facing roots.
The distobuccal cusp of the mandibular second molar occludes with which maxillary tooth surfaces?
A. The mesial marginal ridge of the second molar and distal marginal ridge of the first molar
B. The distal marginal ridge of the second molar and mesial marginal ridge of the third molar
C. The embrasure between the first and second molars
D. The central fossa of the maxillary second molar




The correct answer is D. The distobuccal cusp of a mandibular molar is a holding (supporting) cusp. The general rule of occlusion of mandibular holding cusps is as follows: the holding cusps of the mandibular teeth occlude on the mesial marginal ridge of their maxillary counterpart, and the distal marginal ridge of the maxillary tooth mesial to their counterpart, EXCEPT distobuccal cusps of mandibular molars occlude with central fossae of their counterparts, the distal cusp of the mandibular first molar occludes with the distal triangular fossa of its counterpart, and the first premolar occludes only with the mesial marginal ridge of its counterpart (but not the canine). In this case, the maxillary counterpart is the maxillary second molar, and the mandibular cusp will occlude in its central fossa.
The major blood supply of the PDL is from:
A. pulpal vessels
B. gingival vessels
C. periosteal vessels
D. periapical vessels




The correct answer is C. All of the vessels listed supply blood to the PDL, as all of them are in the vicinity of the PDL and all anastomose or send branches there. Out of this group, however, the MAJOR source is vessels branching from the periosteum surrounding the alveolar bone.

Note that both lymphatics and nerves follow the path of the blood vessels in the PDL.
Which tooth below is MOST likely to be caries-resistant?
A. Maxillary first premolar
B. Mandibular second premolar
C. Mandibular lateral incisor
D. Mandibular second molar
E. Maxillary central incisor




The correct answer is C. Look for a mandibular anterior tooth to be your answer here. You will often notice clinically that partially edentulous patients often retain mandibular anteriors despite loss of most other teeth. This is due to the following three factors: 1) they have few significant pits or fissures; 2) they are bathed in saliva most of the time, which resists plaque buildup; and 3) they are cleaned by the normal movements of the tongue. Other teeth cannot fit all three of these categories. Note that mandibular canines are often the last teeth left. This is because, although the mandibular incisors are rarely lost to caries, they are often lost to periodontal disease, partially due to their short roots and easy calculus buildup.
A patient presents for examination with both mandibular first molars distal to the maxillary first molars. This occlusion is known as:
A. Class I, distocclusion
B. Class II, mesiocclusion
C. Class II, distocclusion
D. Class III, mesiocclusion
E. Class III, distocclusion




The correct answer is C. The normal (Class I) jaw relationship will result in the mandibular first molar being located one cusp (one-half tooth) mesial to the maxillary first molar. If the mandibular molar is located distal to the maxillary, it indicates a small mandible (micrognathy) or another problem causing the maxillary teeth to be too far mesial and the mandibular too far distal. This is distocclusion is Angles Class II. Mesiocclusion is Angles Class III, where the mandibular first molar is more than one-half tooth mesial to the maxillary first molar.
Which of the following is NOT a periodontal fiber?
A. Oblique
B. Transseptal
C. Apical
D. Horizontal




The correct answer is B. Periodontal fibers must run from the cementum of the tooth into alveolar bone. The main periodontal fiber types are alveolar crestal, horizontal, oblique, and apical, named for either their position or alignment. The transseptal fiber is classified with the gingival group, as it does not enter alveolar bone. Instead, as the name suggests, it travels from one tooth to another tooth, crossing over the alveolar crest. It attaches directly from the cementum of one tooth into the cementum of another, but does not attach to bone.
The anterior tooth with the most pronounced lingual ridge is the:
A. Maxillary central incisor
B. Mandibular lateral incisor
C. Maxillary canine
D. Mandibular canine




The correct answer is C. A lingual ridge runs from the incisal edge down the center of the lingual surface of the crown, toward the cervical end of the crown. It splits the lingual surface in half, and usually has a depression on either side (lingual fossae) which separate the lingual ridge from the mesial and distal marginal ridges. Lingual ridges are found on canines, but not on incisors (choices A and B). The lingual ridge is more pronounced on the maxillary canine than on the mandibular (choice D). This is in keeping with a general rule that the lingual anatomy of the maxillary canine is much more pronounced than that of the mandibular.
Oblique ridges are found on which molar teeth?
A. Maxillary first and second
B. Mandibular first and second
C. Maxillary and mandibular first
D. Maxilary and mandibular second
E. First and second molars in both arches




The correct answer is A. Oblique ridges are characteristic of maxillary molar teeth. They connect the mesiolingual and distobuccal cusps. The oblique ridge is also considered to be the distal end of the trigon, the major cusp area of the maxillary molar, composed of the ML, MB, and DB cusps. The DL cusp is often reduced and is referred to as the talon (as opposed to the trigon). The trigon is evolutionarily the primitive cusp triangle of the molar crown. Oblique ridges are not found on mandibular molars or on any other teeth.
The most highly mineralized oral or dental tissue is:
A. enamel
B. dentin
C. bone
D. cementum




The correct answer is A. Enamel is the most mineralized and hardest tissue in the human body. Most estimates place the mineral content of enamel at about 95 to 98%. Dentin, though highly mineralized, still has more organic content than does enamel. Most estimates place the mineral content of dentin at about 80%. Cementum is least mineralized, and is most similar to bone. There are varying estimates of its mineral content, but most place it at about 60 to 65%.
The embrasures which surround the contact points of anterior teeth are:
A. mesial, distal, lingual, buccal
B. mesial and distal only
C. buccal and lingual only
D. buccal, lingual, cervical, gingival
E. incisal, cervical, buccal, lingual




The correct answer is E. The best way to answer the question is to imagine that the embrasures are spaces surrounding a small point of contact between two incisors. This small contact point will have space above it, below it, in front of it, and in back of it. These spaces in dental anatomy will be, respectively, incisal, cervical (gingival), buccal (facial), and lingual (palatal). Note that there is no space mesial or distal to the contact point. Immediately mesial or distal to the contact would be tooth structure.
The distolingual cusp of the mandibular second molar contacts:
A. the lingual embrasure between the maxillary first molar and second molar
B. the lingual groove of the maxillary second molar
C. the central fossa of the maxillary second molar
D. the lingual groove of the maxillary first molar
E. the lingual embrasure between the maxillary second molar and third molar




The correct answer is B. Lingual cusps of mandibular teeth are guiding cusps, not holding cusps. Therefore, they do not occlude on marginal ridges or central fossae. The general rule for mandibular lingual cusps is that they occlude in the lingual embrasures between their maxillary counterparts and the teeth mesial to their counterparts, EXCEPT for the distolingual cusps of the mandibular molars, which occlude in the lingual grooves of their maxillary counterparts. In this question we are dealing with the exception of the distolingual cusps of mandibular molars, so in this case, the mandibular second molar distolingual cusp contacts the lingual groove of its counterpart (the maxillary second molar).
The distobuccal cusp of the maxillary second molar occludes in:
A. the mesial marginal ridge of the mandibular second molar
B. the buccal groove of the mandibular second molar
C. the central groove of the mandibular third molar
D. the facial embrasure between the mandibular first and second molars
E. the facial embrasure between the mandibular second and third molars




The correct answer is E. Buccal cusps of maxillary teeth are guiding cusps, not holding cusps. Therefore, they do not occlude on marginal ridges or central fossae. The general rule for maxillary buccal cusps is that they occlude in the facial embrasures between their mandibular counterparts and the teeth distal to their counterparts, EXCEPT for the mesiobuccal cusps of the molars, which occlude in the buccal grooves of their mandibular counterparts, and the distobuccal cusp of the first molar, which opposes the distobuccal groove of the mandibular first molar. In this question, we are NOT dealing with the exception of the mesiobuccal cusps of the maxillary molars or the distobuccal cusp of the maxillary first molar, so this cusp should follow the general rule and occlude with the facial embrasure between its mandibular counterpart (the mandibular second molar) and the mandibular molar distal to it (the mandibular third molar).
The mandibular incisors of a patient are worn down so that dentin is visible on the incisal edge. This is due to grinding of these edges against the natural maxillary incisors. This loss of tooth structure can be termed:
A. attrition
B. abrasion
C. erosion
D. internal resorption




The correct answer is A. Attrition is the mechanical wearing of teeth due to physiologic processes, including chewing and bruxism. Abrasion (choice B) is the mechanical wearing away of tooth structure due to some outside object, such as toothbrush abrasion at the cervical lines of teeth due to hard sideways brushing, or from habitually holding bobby pins or nails with the teeth. Erosion (choice C) is the chemical dissolving of tooth structure. It can be caused by a number of factors, including sucking on lemons and other acidic fruit, excessive intake of acidic beverages (cola), or excessive vomiting, as in bulimia. Bulimics often exhibit normal facial surfaces with severely eroded lingual surfaces.
In centric occlusion, the lingual cusp of the maxillary first premolar contacts:
A. the mesial marginal ridge of the mandibular first premolar and distal marginal ridge of the second premolar
B. the distal marginal ridge of the mandibular first premolar and mesial marginal ridge of the second premolar
C. the mesial marginal ridge of the mandibular first molar and distal marginal ridge of the second premolar
D. the distal marginal ridge of the mandibular second premolar and mesial marginal ridge of the first molar




The correct answer is B. The lingual cusp of a maxillary premolar is a holding cusp. The general rule for maxillary holding cusps is as follows: a maxillary holding cusp contacts the distal marginal ridge of its mandibular counterpart and the mesial marginal ridge of the mandibular tooth distal to its counterpart, EXCEPT FOR THE MESIOLINGUAL CUSPS OF THE MOLARS, WHICH CONTACT THE CENTRAL FOSSAE OF THEIR COUNTERPARTS. This should be the distal marginal ridge of the mandibular first premolar and mesial marginal ridge of the second premolar.
When a fourth canal is found in the permanent maxillary first molar, it is found in the:
A. mesiobuccal root
B. distobuccal root
C. palatal root
D. mesiolingual root




The correct answer is A. Note that the three-rooted maxillary first molar does not have a mesiolingual root. The maxillary first molar most often has three roots: mesiobuccal, distobuccal, and palatal. The palatal root is largest, and its canal is largest and widest. In the case of a fourth canal, it is invariably found in the mesiobuccal root. The distobuccal and palatal roots are invariably one-canalled.
Odontoblasts laying down dentin matrix would be expected to contain large numbers of well-developed:
A. nuclei
B. lysosomes
C. rough ER
D. smooth ER
E. mitochondria




The correct answer is C. This is a case of a best answer with a reasonable second-best answer. The best answer is rough endoplasmic reticulum. The rough ER contains numerous ribosomes, and is known for producing proteins for export. Collagen fibers of the dentin matrix would be good examples of protein made for export (outside of the cell). The second-best answer is probably mitochondria, in that the synthesis of collagen would be energy-consuming. Odontoblasts are single nuclei cells (unlike osteoclasts, which are multinucleated). Note that smooth ER lacks ribosomes, and that the digestive enzyme activity of lysosomes would not be needed for this process.
A mandibular permanent first molar is likely to have a deep root concavity running apically down:
A. both roots
B. the mesial root
C. the distal root
D. neither root




The correct answer is B. One of the distinctions between the mandibular first-molar mesial and distal roots is the deep root concavity seen running the length of the mesial surface of the mesial root. The distal root, by comparison, will have no similar concavity or a very slight depression. Other differences include the greater likelihood of the mesial root having a distal curvature, and the greater likelihood of the mesial root being partly split by a partial furcation running partway up from the apical end.
The distobuccal cusp of the mandibular third molar occludes with which maxillary tooth surfaces?
A. The mesial marginal ridge of the third molar and distal marginal ridge of the second molar
B. The central fossa of the third molar
C. The mesial marginal ridge of the third molar only
D. The distal marginal ridge of the third molar only




The correct answer is B. The distobuccal cusp of a mandibular molar is a holding (supporting) cusp. The general rule of occlusion of mandibular holding cusps is as follows: the holding cusps of the mandibular teeth occlude on the mesial marginal ridge of their maxillary counterpart, and the distal marginal ridge of the maxillary tooth mesial to their counterpart, EXCEPT distobuccal cusps of mandibular molars occlude with central fossae of their counterparts, the distal cusp of the mandibular first molar occludes with the distal triangular fossa of its counterpart, and the first premolar occludes only with the mesial marginal ridge of its counterpart (but not the canine). In this case, the maxillary counterpart is the maxillary third molar, and the cusp should occlude in the central fossa of this tooth. Remember that the mandibular third molar is located one-half tooth mesial to the maxillary.
Damage to the right lateral pterygoid results in:
A. the mandible turning right on protrusion
B. inability to elevate the mandible
C. the mandible turning left on protrusion
D. none of the above




The correct answer is A. As a rule, the mandible will move toward the side of injury when the lateral pterygoid is damaged. In this case, the right condyle will not move because of the muscle damage. The left condyle moves forward, but because there is no movement on the right side, the left condyle moves out and rotates to the right, with the nonmoving right condyle acting as a pivot point. Damage to the lateral pterygoid will not affect elevation (closing), because the lateral pterygoid is not an elevator. The elevators are the temporalis, medial pterygoid, and masseter.
The distal cusp of the mandibular first molar occludes with which maxillary tooth surfaces?
A. The mesial marginal ridge of the first molar and distal marginal ridge of the second molar
B. The distal marginal ridge of the second molar and mesial marginal ridge of the first molar
C. The embrasure between the first and second molars
D. The distal triangular fossa of the first molar




The correct answer is D. The distal cusp of the mandibular first molar is a holding cusp. The general rule of occlusion of mandibular holding cusps is as follows: the holding cusps of the mandibular teeth occlude on the mesial marginal ridge of their maxillary counterpart, and the distal marginal ridge of the maxillary tooth mesial to their counterpart, EXCEPT DISTOBUCCAL CUSPS OF MANDIBULAR MOLARS OCCLUDE WITH CENTRAL FOSSAE OF THEIR COUNTERPARTS, THE DISTAL CUSP OF THE MANDIBULAR FIRST MOLAR OCCLUDES WITH THE DISTAL TRIANGULAR FOSSA OF ITS COUNTERPART, AND THE FIRST PREMOLAR OCCLUDES ONLY WITH THE MESIAL MARGINAL RIDGE OF ITS COUNTERPART (BUT NOT THE CANINE).

Note that mesiobuccal cusps of mandibular molars and buccal cusps of premolars contact marginal ridges.
Hypocalcified enamel rods which enter into enamel from the DEJ are known as:
A. enamel spindles
B. enamel tufts
C. enamel lamellae
D. none of the above




The correct answer is B. There are a number of somewhat confusing anomalies at the dentinoenamel junction. Enamel spindles are odontoblastic processes and dentinal tubules which extend a short way into enamel. Enamel lamellae, on the other hand, extend from enamel into dentin. They are pieces of uncalcified organic enamel material. Enamel tufts are hypocalcified enamel rods. They are individual entities; however, they appear as groups (tufts) when a group of them are viewed laterally.
The mesiolingual cusp of the mandibular third molar contacts:
A. The lingual embrasure between the maxillary second molar and third molar
B. the distal marginal ridge of the maxillary second molar
C. the central fossa of the maxillary second molar
D. the lingual groove of the maxillary third molar
E. the buccal embrasure between the maxillary second molar and third molar




The correct answer is A. Lingual cusps of mandibular teeth are guiding cusps, not holding cusps. Therefore, they do not occlude on marginal ridges or central fossae. The general rule for mandibular lingual cusps is that they occlude in the lingual embrasures between their maxillary counterparts and the teeth mesial to their counterparts, EXCEPT for the distolingual cusps of the mandibular molars, which occlude in the lingual grooves of their maxillary counterparts. In this question we are not dealing with the exception of the distolingual cusps of mandibular molars, so in this case, the mandibular third molar mesiolingual cusp contacts the lingual embrasure between its counterpart (the maxillary third molar) and the tooth mesial to it (the maxillary second molar).
Based on root morphology, the tooth most likely to be successfully rotated during exodontia is the:
A. Maxillary central
B. Mandibular central
C. Mandibular lateral
D. Mandibular canine




The correct answer is A. During extraction, rotation can be used on teeth with rounded conical roots. Two primary examples are maxillary centrals (especially) and maxillary laterals. Rotation may also be used somewhat with maxillary canines. If roots are flattened mesiodistally, then buccal-lingual movement is used instead of rotation, as rotation may cause crown fracture. Mandibular centrals, laterals, and canines (choices B, C, and D) are all flattened mesiodistally, and therefore are usually not rotated during extraction. Mandibular canines also may occasionally have two roots (buccal and lingual), which will prohibit rotation.
As compared to the cross section of the maxillary central incisor at the cervical line, the cross section of the mandibular central is:
A. more flattened mesiodistally
B. more flattened faciolingually
C. rounder
D. almost identical




The correct answer is A. As a general rule, maxillary incisors have round root sections and round pulp chamber cross sections. Mandibular incisors have more oval ones, and are flattened mesiodistally. This is in keeping with the general root shape of each. Maxillary incisor roots are round, and mandibular ones are flattened to a greater degree. Note that in both arches, as the pulp moves coronally it will flatten buccolingually, in keeping with the coronal shape of all incisors.
The marrow space inside alveolar bone surrounding teeth:
A. does not exist
B. is usually red marrow
C. is usually hematopoietic marrow
D. is usually yellow marrow




The correct answer is D. Two types of marrow space are found within bone, red (hematopoietic), which is the source of blood cells; and yellow (fatty), which does not produce blood cells. The alveloar bone is similar to most bone in the body in having a compact outer layer of lamellar bone and an inner layer of spongy bone. This spongy bone contains marrow space, usually of the yellow (fatty) type, although some red marrow exists, especially in the ramus and condyle of the mandible.
In the Posselt envelope of motion, protruded contact position (PCP) is:
A. achieved with the posterior teeth in occlusion
B. an extreme border position
C. the same as edge-to-edge
D. the same as an Angle Class II relationship




The correct answer is B. The Posselt envelope of motion is a tracing of the extreme border movements of the mandible. It shows how far the mandible can protrude, retrude, open, and close, and all motions connecting these points. Protruded contact position is caused by protruding the mandible as far as possible. This will normally push the mandible past edge-to-edge, until the mandible appears to be in a Class III relationship. As the mandible protrudes, anterior tooth contact causes separation of the teeth out of occlusion.
Rests of Malassez are composed of:
A. ectodermal derivatives
B. former enamel epithelium cells
C. former root sheath of Hertwig cells
D. all of the above




The correct answer is D. The epithelial rests of Malassez are found in the periodontal ligament and are leftovers from the root formation process. The root is originally outlined by the epithelial root sheath of Hertwig, which is composed of the inner and outer enamel epithelium tissues, without a stellate reticulum in between. The root sheath induces formation of odontoblasts, which then produce root dentin. Pieces of the root sheath, which do not disappear, remain as the epithelial rests of Malassez. These small groups of tissue are ectodermal in origin, as the original enamel epithelium is ectodermal.
When compared to the buccal roots of the maxillary first molar, the buccal roots of the maxillary second molar are:
A. more parallel, with a pliers-like appearance
B. less parallel, lacking a pliers-like appearance
C. more parallel, with both having a mesial tip orientation
D. more parallel, with both having a distal tip orientation




The correct answer is D. A characteristic curve of the two buccal root tips of the maxillary first molar toward each other is sometimes referred to as a pliers-handled appearance or pincer appearance. In effect, the roots grow apically and then turn toward each other, forming a U shape. This shape helps explain the stability of the tooth, especially when combined with the tripod-like arrangement of the palatal root. Note that the same roots of the second maxillary molar often both point distally, distinguishing one molar from the other. These roots in the second molar are also more parallel than those in the first molar. In buccal view they seem to sway distally, with the palatal root in the midline and slightly mesially tilted.
Which of the following permanent teeth is MOST likely to be missing a distolingual cusp?
A. Mandibular first molar
B. Maxillary first molar
C. Mandibular second premolar
D. Maxillary second molar




The correct answer is C. The mandibular first molar is invariably a five-cusped tooth, with MB, ML, DB, DL, and distal cusps. Maxillary first and second molars are normally four-cusped teeth. The distolingual cusp is usually less pronounced on the second molar. It is often absent on the maxillary third molar, but that answer is not listed. The mandibular second premolar is either a two- or three-cusped tooth. The two-cusped variety has a single buccal cusp and a single lingual cusp, while the three-cusped version has a buccal cusp and paired mesiolingual and distolingual cusps. However, both varieties are common, so it is common to find this tooth lacking the distolingual cusp.
The buccal cusp of the maxillary second premolar occludes in:
A. the mesial marginal ridge of the mandibular first premolar
B. the distal marginal ridge of the mandibular second premolar
C. the mesial marginal ridge of the mandibular second premolar
D. the facial embrasure between the mandibular first and second premolars
E. the facial embrasure between the mandibular second premolar and first molar




The correct answer is E. Buccal cusps of maxillary teeth are guiding cusps, not holding cusps. Therefore, they do not occlude on marginal ridges or central fossae. The general rule for maxillary buccal cusps is that they occlude in the facial embrasure between their mandibular counterpart and the tooth distal to their counterpart, EXCEPT for the mesiobuccal cusps of the molars, which occlude in the buccal grooves of their mandibular counterparts, and the distobuccal cusp of the first molar, which opposes the distobuccal groove of the mandibular first molar. In this question, the contact is the embrasure between second mandibular premolar (the counterpart) and the first molar (the tooth distal to the counterpart).
In occlusion, guiding cusps are defined as:
A. lingual cusps of maxillary and mandibular arches
B. bucccal cusps of maxillary and mandibular arches
C. buccal cusps of the maxillary arch and lingual cusps of the mandibular arch
D. lingual cusps of the maxillary arch and buccal cusps of the mandibular arch




The correct answer is C. In occlusion, cusps are defined as either holding (supporting) cusps or guiding cusps. Holding cusps, in central occlusion, make contact with the opposing arch, establish vertical dimension of occlusion, and support the forces of occlusion. They are the lingual cusps of the maxillary arch and buccal cusps of the mandibular. When the teeth are in centric occlusion, these cusps are in contact. The other cusps (guiding) are not. The other cusps are either lingual to the contact (lingual cusps of the mandibular arch) or buccal to the contact (buccal cusps of the maxillary arch).
Protrusive movement causes the condyle of the mandible to move:
A. backward and downward
B. backward and upward
C. forward and downward
D. forward and upward




The correct answer is C. Protrusive movement is defined as moving the mandible outward, away from the head. If you are in centric occlusion, you will protrude to move toward an edge-to-edge position. The lateral pterygoid moves both condyles forward when it contracts, so the initial protrusive movement is forward. When the condyle contacts the articular eminence in the glenoid fossa, it cannot move directly forward anymore and begins to glide downward along the surface of the fossa.
As compared to those of the permanent mandibular first molar, the roots of the second molar are:
A. more mesially inclined
B. straighter
C. more distally inclined
D. very similar in inclination to the first molar




The correct answer is C. In the permanent mandibular first molar, the mesial root is usually slightly curved distally, and the distal root is straighter but pointing toward the distal. In the second molar, both roots are usually significantly inclined toward the distal and curved distally at the end. They tend to resemble each other more than the two roots of the first molar do. They are also closer together. The roots in the first molar are more widely spread.
Which of the following fiber sets does NOT attach to cementum?
A. Alveolar crestal
B. Dento-gingival
C. Circular
D. Oblique




The correct answer is C. The attachment fibers are usually divided into two groups, the gingival group and the periodontal group. Gingival fibers attach tissues, not including alveolar bone. For example, circular fibers run only within gingiva, encircling the tooth. Dento-gingival fibers run from cementum to gingiva. Dento-periosteal fibers run from cementum to periosteum. Periodontal fibers run from alveolar bone to cementum, and include alveolar crestal, oblique, horizontal, and apical.
The mesiobuccal cusp of the maxillary second molar occludes in:
A. the buccal groove of the second mandibular molar
B. the facial embrasure between the mandibular first and second molars
C. the central groove of the mandibular second molar
D. the distal marginal ridge of the mandibular first molar
E. the buccal groove of the mandibular third molar




The correct answer is A. Buccal cusps of maxillary teeth are guiding cusps, not holding cusps. Therefore, they do not occlude on marginal ridges or central fossae. The general rule for maxillary buccal cusps is that they occlude in the facial embrasures between their mandibular counterparts and the teeth distal to their counterparts, EXCEPT for the mesiobuccal cusps of the molars, which occlude in the buccal grooves of their mandibular counterparts, and the distobuccal cusp of the first molar, which opposes the distobuccal groove of the mandibular first molar. In this question we are dealing with the exception of the mesiobuccal cusps of maxillary molars, which occlude with the buccal grooves of their mandibular counterparts. In this case, the maxillary second molar mesiobuccal cusp contacts the mandibular second molar buccal groove (its counterpart).
The shape of the pulp chamber in cross section cut at the cervical line in the maxillary central incisor is:
A. oval (flattened mesiodistally)
B. oval (flattened bucco-lingually)
C. round
D. figure eight




The correct answer is C. Only maxillary incisors have roundish pulp chamber cross sections when cut at the cervical line. Laterals are egg-shaped with the widest portion labial, not round. This is true for both maxillary centrals and laterals, which are egg-shaped with the widest portion of the labial not round. In general, mandibular incisors and both arch canines will have oval-shaped pulp chambers, flattened mesiodistally. (Imagine squashing a round pulp from both the mesial and distal sides at once, resulting in a squashed oval, pointing to the buccal and lingual). Remember that the shape of a pulp chamber is usually related to the shape of the surrounding root.
The appearance of a second root in a canine is:
A. never found
B. found more often in the maxillary than the mandibular
C. found more often in the mandibular than the maxillary
D. found about equally in both arches




The correct answer is C. Two roots are a fairly rare occurrence in canines, but are occasionally found in the mandibular canine. They are almost unknown in the maxillary. If they are found in the mandibular canine, they will be buccal and lingual, similar to those found in premolars.
The submucosa of gingival masticatory mucosa is:
A. similar in thickness to that of lining mucosa
B. much thicker than that of lining mucosa
C. found between the epithelium and lamina propria
D. thin or absent




The correct answer is D. The gingival masticatory mucosa is characterized by keratinization, thin or absent submucosa, and a firm, direct binding of the lamina propria to the periosteum. In general, the submucosa is thicker in the lining mucosa. Note that submucosa, when present, is always found beneath lamina propria, and is never between epithelium and lamina propria. If all four tissues are present in a digestive mucosa, the order from the outside will be: epithelium, lamina propria, muscularis mucosae, submucosa.
In centric occlusion, the distolingual cusp of the maxillary third molar will contact:
A. the central fossa of the mandibular second molar
B. the central fossa of the mandibular third molar
C. the distal marginal ridge of the mandibular first molar and mesial marginal ridge of the second molar
D. the distal marginal ridge of the mandibular second molar and mesial marginal ridge of the first molar
E. None of the above




The correct answer is E. The distolingual cusp of a maxillary third molar is theoretically a holding cusp, however it is often missing on this tooth. The general rule for maxillary holding cusps is as follows: a maxillary holding cusp contacts the distal marginal ridge of its mandibular counterpart and the mesial marginal ridge of the mandibular tooth distal to its counterpart, EXCEPT FOR THE MESIOLINGUAL CUSPS OF THE MOLARS, WHICH CONTACT THE CENTRAL FOSSAE OF THEIR COUNTERPARTS. This should be the distal marginal ridge of the mandibular third molar and no mesial marginal ridge, because there is no tooth distal to the third molar. So the answer is either the cusp does not exist, or it contacts only the distal marginal ridge of the mandibular third molar.
Collagen fibers inserting into compact bone of the alveolar socket are called:
A. Tomes processes
B. Sharpey's fibers
C. Von Ebner lines
D. Contour lines of Owen
E. None of the above




The correct answer is B. The fibers of the periodontal ligament are collagenous and insert into the compact, lamellar outer layer of the tooth socket. This is similar to fibers of a tendon inserting into bone. The appearance of the bone with the collagen fibers causes it to be named bundle bone. The fibers themselves are called Sharpey's fibers. Tomes processes (choice A) are projections of ameloblasts into developing enamel. Von Ebner lines (choice C) are found in dentin and show the incremental laying down of dentin tissue. Lines of Owen (choice D) are found in dentin as well, and are exaggerated Von Ebner lines found in areas of trauma during dentin formation.
The most numerous cell type of the dental pulp is the:
A. neuron
B. fibroblast
C. leukocyte
D. macrophage
E. ameloblast




The correct answer is B. The dental pulp is a connective tissue characterized by multiple collagen fibers running in all directions, and large numbers of fibroblasts, which produce the fibers. There are also capillaries, neurons, lymphatic channels, and different types of leukocytes, including macrophages, neutrophils, and eosinophils. However, the predominant cell type is the fibroblast. Note that the pulp is lined by the dentin-forming odontoblasts, but there are no ameloblasts, which form enamel.
Which two tissues are formed by the same part of the tooth bud?
A. Enamel and dentin
B. Dentin and cementum
C. Dentin and pulp
D. Pulp and cementum
E. None of the above




The correct answer is C. The three sections of the tooth bud are the enamel organ, the dental sac, and the dental papilla. The enamel organ, not surprisingly, produces enamel and contains the inner enamel epithelium, stratum intermedium, stellate reticulum, and outer enamel epithelium. The dental papilla, a mesodermal derivative, produces both dental pulp and dentin. The dental sac, also mesodermal, produces both cementum and the periodontal ligament.
Which ligament of the temporomandibular joint inserts into the lingula of the mandible?
A. Temporomandibular
B. Stylomandibular
C. Sphenomandibular
D. Lateral




The correct answer is C. The origins and insertions of the TMJ ligaments are as follows: The sphenomandibular ligament arises from a spine on the sphenoid bone and runs forward and downward to insert on the lingula and deep ramus of the mandible. The stylomandibular ligament arises from the spine of the temporal bone known as the styloid process and inserts on the lower ramus and angle of the mandible. The lateral ligament is also known as the temporomandibular ligament. It descends from the lower border and tubercle of the zygoma to the posterior lateral condyle. Its fibers merge with those of the articular capsule.
The most common arrangement of canals in the roots of a permanent maxillary first molar is:
A. ML, MB, DL, DB
B. ML, MB, and DB only
C. ML, MB, and palatal
D. MB, DB, and palatal
E. none of the above




The correct answer is D. Most often, the maxillary first molar has three canals: a mesiobuccal, a distobuccal, and a palatal. The palatal is usually largest and widest. Occasionally (30%) the first molar may have a fourth canal, located within the mesiobuccal root, slightly lingual to the mesiobuccal canal. When found, it is the smallest and thinnest of all canals in the tooth.
In a left working movement:
A. the left side moves laterally, and the right side medially
B. the left side moves medially, and the right side laterally
C. both sides move medially
D. both sides move laterally




The correct answer is A. In a left working movement, the mandible shifts to the left. From a central position, the left side of the mandible is moving away from center (laterally). The right side of the mandible, although also moving left, is moving toward the center (medially). The left TMJ purely rotates when the right is rotating and translating mechanically.

Note that it is impossible for both sides of the mandible to move either laterally or medially at the same time.
Alveolar bone is composed of:
A. cortical bone only
B. cancellous bone only
C. cortical bone surrounding cancellous bone
D. cancellous bone surrounding cortical bone




The correct answer is C. Alveolar bone is a combination of cortical (compact) bone and trabecular (spongy) bone. The cortical bone is on the outside surface, both next to the tooth root and also continuous with the cortical bone of the maxilla and mandible. The cortical bone adjacent to the tooth root is sometimes referred to as lamina dura. Nerves and blood vessels travel primarily through the more porous cancellous bone between the two cortical plates. Note that alveolar bone can be lost rapidly either in periodontal disease or following tooth extraction.
When you move the mandible from centric occlusion (maximum intercuspation) to protruded contact (edge-to-edge) position:
A.vertical dimension decreases
B. horizontal overlap increases
C. vertical overlap increases
D. None of the above




The correct answer is D. As you move from centric occlusion to edge-to-edge position, the mandibular teeth separate from the maxillary and a space is created between the two arches. Try this yourself, as you protrude and slide the mandible forward. The anterior teeth act as a guide as you slowly separate from occlusal contact until only incisal edges of incisors contact. At this point, the increased space between the arches adds to vertical dimension. There is no vertical overlap (overbite; choice C) nor is there any horizontal overlap (overjet; choice B) at this point. So protrusion to edge-to-edge increases vertical dimension while decreasing both vertical and horizontal overlap.
From a labial view, which incisors are bilaterally symmetrical?
A. Both mandibular central and lateral
B. Neither mandibular central nor lateral
C. Mandibular central only
D. Mandibular lateral only




The correct answer is C. The mandibular central incisor is often described as the most symmetrical tooth when viewed from the labial. The incisal edge is horizontal, the mesial and distal contacts are at the same height, and the mesioincisal and distoincisal line angles are equal. In addition, the tooth is not rotated. The mandibular lateral incisor crown is rotated, and its distal contact point is more apical than the mesial.
The Golgi apparatus in odontoblasts producing dentin can be expected to be:
A. absent
B. less developed than in an average cell
C. developed to the same degree as that of an average cell
D. more highly developed than that of an average cell




The correct answer is D. Remember that the function of Golgi apparatus is to modify and package proteins produced for export by the cell. In this way, an odontoblast would be similar to any other protein-producing secretory cell. Proteins produced by ribosomes on the rough ER will enter the ER and travel to the Golgi apparatus. Here, within the flattened sacs of the Golgi, the collagen and other fibers will be modified and packaged to be sent to the edge of the cell for secretion as dentin matrix. So the large production demands of producing collagen and other proteins for predentin should result in odontoblasts having large, numerous, and active Golgi bodies.
The longest tooth in the mouth (apico-incisally) is the:
A. Maxillary central
B. Maxillary canine
C. Mandibular canine
D. Maxillary second premolar




The correct answer is B. This is one of the biggest, shortest, roundest, flattest type questions that NBDE Part 1 loves so much. The maxillary canine is the longest tooth in the mouth, measured cusp tip to root tip. It is about 3 mm longer than the maxillary central (choice A), 1 mm longer than the mandibular canine (choice C), and 4 mm longer than the maxillary second premolar (choice D). Note that the mandibular central is the narrowest tooth in the mouth.
The least likely area to find caries on teeth is the:
A. lingual surface of maxillary molars
B. occlusal surface of mandibular molars
C. lingual surface of mandibular molars
D. buccal surface of mandibular molars
E. occlusal surface of mandibular premolars




The correct answer is C. Caries is most commonly found in grooves, pits, and fissures of teeth. Sucrose and other substrates collect and S. mutans secretes lactic acid to dissolve enamel. All posterior occlusal surfaces are susceptible to caries, as they contain pits, fissures, and grooves. Maxillary molars have distinct lingual pits or grooves which may become carious. Likewise, buccal pits are common on mandibular molars. Note that lingual surfaces of mandibular molars are generally smooth, without pits and fissures.
When extracting maxillary teeth, the root tip most likely to be forced into the maxillary sinus is that of a:
A. canine
B. first premolar
C. second premolar
D. first molar
E. second molar




The correct answer is D. Anatomically, roots of the maxillary first molar are closest to the sinus, which dips downward in this area. Periapical x-rays of the area sometimes reveal that the roots of the first molar border right on the sinus margin. Although the first molar is the MOST likely tooth to have a root enter the sinus, roots of the second premolar and second molar can occasionally end up there as well.
The class of teeth considered to have a long axis which is most vertical in a buccolingual direction (as viewed from the mesial or distal) is the:
A. incisors
B. canines
C. premolars
D. molars




The correct answer is C. Premolars do not vary far from a vertical line when viewed from the mesial or distal. Both maxillary and mandibular incisors tip their roots greatly toward the lingual, and canines less so. Mandibular molar roots lean buccally while maxillary molar roots lean palatally.
The lingual surfaces of most maxillary teeth of a patient are worn down so that dentin is visible through the remaining enamel on the lingual side. This is due to the dissolving of the mineral of the enamel by strong acid. This loss of tooth structure can be termed:
A. attrition
B. abrasion
C. erosion
D. internal resorption




The correct answer is C. Erosion is the chemical dissolving of tooth structure. It can be caused by a number of factors, including sucking on lemons and other acidic fruit, excessive intake of acidic beverages (cola), or excessive vomiting, as in bulimia. Bulimics often exhibit normal facial surfaces with severely eroded lingual surfaces. This is due to the effect of strong mineral acid from the stomach (hydrochloric acid at pH 2). Attrition (choice A) is the mechanical wearing of teeth due to physiologic processes, including chewing and bruxism. Abrasion (choice B) is the mechanical wearing away of tooth structure due to some outside object, such as toothbrush abrasion at the cervical lines of teeth due to hard sideways brushing, or from habitually holding bobby pins or nails with the teeth. Internal resorption (choice D) is the loss of dentin from pathologic dissolving by processes inside the pulp chamber. It can follow tooth trauma.
The formation of dual teeth with combined enamel and dentin crowns but with only one root is known as:
A. gemination
B. dilaceration
C. concrescence
D. dens-in-dente
E. taurodont




The correct answer is A. Gemination is a form of fusion, where a single root attempts to form two crowns, and these crowns share dentin and enamel. A dilaceration is a sharply bent root, especially near the apex. Concrescence is joining of two teeth by cementum union. Taurodonts are teeth, usually molars, with short roots relative to crown size, and large pulpal chambers, giving rise to a bull-like appearance. Dens-in-dente refers to a tooth-within-a-tooth appearance and is due to invagination, usually in a maxillary lateral incisor.
The buccal cusp of the mandibular second premolar occludes with which maxillary tooth surfaces?
A. The mesial marginal ridge of the first molar and distal marginal ridge of the second premolar
B. The mesial marginal ridge of the second premolar and distal marginal ridge of the first premolar
C. The embrasure between the first and second molars
D. The embrasure between the second premolar and first molar




The correct answer is B. The buccal cusp of a mandibular premolar is a holding (supporting) cusp. The general rule of occlusion of mandibular holding cusps is as follows: the holding cusps of the mandibular teeth occlude on the mesial marginal ridge of their maxillary counterpart, and the distal marginal ridge of the maxillary tooth mesial to their counterpart, EXCEPTdistobuccal cusps of mandibular molars occlude with central fossae of their counterparts, the distal cusp of the mandibular first molar occludes with the distal triangular fossa of its counterpart, and the first premolar occludes only with the mesial marginal ridge of its counterpart (but not the canine). In this case, the maxillary counterpart is the maxillary second premolar, and the tooth mesial to it is the first premolar.
In cases of TMJ dislodgement, where the condyle becomes locked due to movement beyond the articular (temporal) fossa, the dentist should move the condyle in which direction initially to return the joint to its normal position?
A. Anteriorly
B. Posteriorly
C. Inferiorly
D. Superiorly




The correct answer is C. This may seem counterintuitive, as excess motion of the condyle results in the mandible being locked in a forward and open position. However, anatomically, what is happening is that the condyle has moved forward, down the surface of the articular eminence, past the tip of the eminence, and past the bulk of the articular tubercle. It is now past a large, inferiorly projecting mound of bone (the tubercle). To return the condyle to normal position, a downward (inferior) force is needed to positon the condyle once again below the articular tubercle and eminence. The inferior directed force is also needed to counteract the elevating forces of the medial pterygoid, masseter, and temporalis muscles. On repositioning, the mandible often snaps strongly into normal position, due to these strong elevations.
Which pair of angles of a maxillary first molar are acute when the tooth is viewed from the occlusal?
A. MB and ML
B. DB and DL
C. MB and DB
D. ML and DL
E. None of the above




The correct answer is E. If you view the maxillary first molar from the occlusal, its shape is that of a rhombus, or equal-sided parallelogram. In any parallelogram, one pair of opposite corners will be obtuse, while the other pair of opposite corners will be acute. In the maxillary first molar, the opposite pair of MB and DL are acute, while the pair ML and DB are obtuse.
Which characteristic most distinguishes permanent mandibular central incisors from lateral incisors?
A. Mesiodistal length
B. Buccolingual length
C. Root length
D. Rotation of the crown around the root axis
E. None of the above




The correct answer is D. The mandibular central and lateral incisors can be viewed as almost twins. They are similar in dimension both mesiodistally and buccolingually. (The lateral may be slightly, but not significantly, larger). Their roots are similar in length, although the lateral may have a more pronounced distal inclination. The crowns are of similar anatomy. However, the lateral has a twisted or rotated crown, as if you took a central and then slightly rotated the crown around the long axis of the root. The root may appear to face forward, while the incisal edge is curved in keeping with the normal curve of cusp tips of the mandibular arch.
The lingual cusp of the mandibular first premolar contacts:
A. the mesial marginal ridge of the maxillary first premolar
B. the distal marginal ridge of the maxillary second premolar
C. the central fossa of the maxillary second premolar
D. the lingual embrasure between the maxillary first and second premolars
E. the lingual embrasure between the maxillary first premolar and canine




The correct answer is E. Lingual cusps of mandibular teeth are guiding cusps, not holding cusps. Therefore, they do not occlude on marginal ridges or central fossae. The general rule for mandibular lingual cusps is that they occlude in the lingual embrasures between their maxillary counterparts and the teeth mesial to their counterparts, EXCEPT for the distolingual cusps of the mandibular molars, which occlude in the lingual grooves of their maxillary counterparts. In this question we are not dealing with the exception of the distolingual cusps of mandibular molars, so in this case, the mandibular first premolar lingual cusp contacts the lingual embrasure between its counterpart (the maxillary first premolar) and the tooth mesial to it (the maxillary canine).
The lingual cusp of the mandibular first premolar contacts:
A. the mesial marginal ridge of the maxillary first premolar
B. the distal marginal ridge of the maxillary second premolar
C. the central fossa of the maxillary second premolar
D. the lingual embrasure between the maxillary first and second premolars
E. the lingual embrasure between the maxillary first premolar and canine




The correct answer is E. Lingual cusps of mandibular teeth are guiding cusps, not holding cusps. Therefore, they do not occlude on marginal ridges or central fossae. The general rule for mandibular lingual cusps is that they occlude in the lingual embrasures between their maxillary counterparts and the teeth mesial to their counterparts, EXCEPT for the distolingual cusps of the mandibular molars, which occlude in the lingual grooves of their maxillary counterparts. In this question we are not dealing with the exception of the distolingual cusps of mandibular molars, so in this case, the mandibular first premolar lingual cusp contacts the lingual embrasure between its counterpart (the maxillary first premolar) and the tooth mesial to it (the maxillary canine).
Alveolar bone is composed of:
A. compact bone
B. spongy bone
C. bundle bone
D. all of the above
E. none of the above




The correct answer is D. The alveloar bone is similar to most bone in the body in having a compact outer layer of lamellar bone and an inner layer of spongy bone. This spongy bone contains marrow space, usually of the yellow (fatty) type, although some red marrow exists. Collagen fibers of the periodontal ligament insert into the outer cortical compact layer, and are known as Sharpey's fibers. Compact bone with Sharpey's fibers is also known as bundle bone.
The TMJ structure consists of:
A. One synovial compartment
B. An upper synovial and lower nonsynovial compartment
C. Two synovial compartments
D. Two nonsynovial compartments




The correct answer is C. The TMJ has an upper and lower compartment, separated by the articular disc, which is composed of fibrous connective tissue. Both the upper and lower compartments are synovial; that is, they produce synovial fluid. Synovial fluid aids in lubricating the articular surfaces of the joint, and is found in many movable joints. Note that the synovial membrane lines the inside of the joint but not the actual articulating surfaces. These surfaces are fibrous connective tissue over hyaline cartilage.
Odontoblasts are noted for highly developed:
A. smooth ER
B. rough ER
C. lysosomes
D. mitochondria
E. flagella




The correct answer is B. The odontoblast function is to lay down the protein matrix for the formation of dentin. Protein matrix is an exported protein, and therefore the cell must have highly developed machinery for production and export of protein. This usually involves ribosomes attached to ER (rough ER) and Golgi bodies. The ribosomes translate RNA messages into assembled proteins and send them in the ER to the Golgi apparatus, where they are modified and packaged for export. Odontoblasts do not have flagella, and there is no special reason to have highly developed lysosomes for internal digestion. As metabolically active cells they would have elevated mitochondria levels, but this would not be as notable as the extensive protein producing apparatus.
When primary mandibular incisors are retained too long and the permanent incisors erupt with the primaries still in place, the permanents usually erupt in what position relative to the primaries?
A. Mesially
B. Distally
C. Lingually
D. Buccally




The correct answer is C. This is a fairly common occurrence. A child of about 6 years will often present to the dental clinic with a parent, and the parent will be concerned about a double row of teeth. In these cases, the permanent mandibular incisors are erupting before the primaries have completely exfoliated. An x-ray usually confirms partial resorption of the primary roots, and the primaries are allowed to exfoliate. No further treatment is usually indicated. In rare cases where the permanents erupt buccally, extraction of the primaries and tooth movement of the permanents is in order, however this is highly unusual.
Which of the following is NOT typical of masticatory mucosa?
A. Simple squamous epithelium
B. Highly keratinized
C. Poorly developed submucosa
D. Thick epithelium
E. Covers the hard palate




The correct answer is A. Both types of oral epithelium (masticatory and lining) are stratified, not simple, squamous epithelium. In masticatory epithelium it is highly keratinized and thickened. This type of mucosa covers the hard palate and gingival areas. The submucosa of these tissues is usually poorly developed or absent. Lining mucosa is generally thin and nonkeratinized, and has a glandular lamina propria and a well developed submucosa.
The buccal cusp of the mandibular first premolar contacts which surfaces on maxillary teeth?
A. The mesial marginal ridge of the first premolar and distal marginal ridge of the canine
B. The mesial marginal ridge of the second premolar and distal marginal ridge of the first premolar
C. The distal marginal ridge of the first premolar and the mesial marginal ridge of the canine
D. None of the above




The correct answer is D. While theoretically it may appear that the correct answer is choice A, most authors state that there is no contact with the maxillary canine distal marginal ridge, and the canine remains slightly out of contact but near the area of the buccal cusp of the mandibular first premolar. The general rule of occlusion of mandibular holding cusps is as follows: the holding cusps of the mandibular teeth occlude on the mesial marginal ridge of their maxillary counterpart, and the distal marginal ridge of the maxillary tooth mesial to their counterpart, EXCEPT DISTOBUCCAL CUSPS OF MANDIBULAR MOLARS OCCLUDE WITH CENTRAL FOSSAE OF THEIR COUNTERPARTS, THE DISTAL CUSP OF THE MANDIBULAR FIRST MOLAR OCCLUDES WITH THE DISTAL TRIANGULAR FOSSA OF ITS COUNTERPART, AND THE FIRST PREMOLAR OCCLUDES ONLY WITH THE MESIAL MARGINAL RIDGE OF ITS COUNTERPART (BUT NOT THE CANINE).
The buccal cusp of the mandibular canine occludes with which maxillary tooth surfaces?
A. The mesial marginal ridge of the first premolar and distal marginal ridge of the canine
B. The mesial marginal ridge of the canine and distal marginal ridge of the lateral incisor
C. The embrasure between the canine and first premolar
D. None of the above




The correct answer is D. The buccal cusp of a mandibular canine is not considered to be a holding (supporting) cusp. Therefore, the general rule of occlusion of mandibular holding cusps does not apply. The canine, as an anterior tooth, will normally be slightly out of contact between its maxillary counterpart and the tooth immediately mesial to it. In this case, the maxillary counterpart is the maxillary canine, and the tooth mesial to it is the lateral incisor.
The lamina propria is:
A. epithelial tissue
B. connective tissue
C. adipose tissue
D. muscle tissue




The correct answer is B. The lamina propria is a loose connective tissue located within the mucosal layer, just underneath the epithelium. In the oral cavity, it will be found below the outer layer of stratified squamous epithelium. The lamina propria in the oral cavity often forms wedge-like extensions into concavities in the epithelium, known as connective tissue papillae. The corresponding epithelial extensions into the connective tissue are known as rete pegs.
The lamina propria is:
A. epithelial tissue
B. connective tissue
C. adipose tissue
D. muscle tissue




The correct answer is B. The lamina propria is a loose connective tissue located within the mucosal layer, just underneath the epithelium. In the oral cavity, it will be found below the outer layer of stratified squamous epithelium. The lamina propria in the oral cavity often forms wedge-like extensions into concavities in the epithelium, known as connective tissue papillae. The corresponding epithelial extensions into the connective tissue are known as rete pegs.
Which of the following is not normally found in cases of occlusal trauma?
A. Resorption of alveolar bone
B. Increased mobility
C. Thicker cementum (hypercementosis)
D. Widened periodontal ligament space




The correct answer is C. Increased cementum production (hypercementosis) is a common x-ray finding. Its cause is not completely known, but it seems to occur more often in teeth that have lost function and/or are supererupted, rather than in teeth with excessive function (occlusal trauma). Occlusal trauma is recognized clinically by heavy contact (using articulating paper), and high mobility. It is recognized radiographically by a widened PDL space and possibly with accompanying bone resorption.

Note that in loss of function, teeth often have a narrowed PDL.
The earliest succedaneous tooth in the mouth to erupt is the:
A. Primary mandibular central incisor
B. Primary mandibular lateral incisor
C. Permanent mandibular first molar
D. Permanent maxillary first molar
E. Permanent mandibular central incisor




The correct answer is E. The question depends entirely on the definition of succedaneous. A succedaneous tooth succeeds (replaces) another tooth. So only permanent teeth can be succedaneous. Note, however, that permanent molars are never succedaneous, as they erupt distal to all primary teeth (choices C and D). Only permanent incisors, canines, and premolars can be succedaneous (refer to choices A and B). Of all succedaneous teeth, the permanent mandibular central incisor is usually first, appearing at about age 6, close to the eruption of the nonsuccedaneous permanent mandibular first molar.
The time between eruption of a tooth crown and root completion is often closest to:
A. 6 months
B. 1 year
C. 2-3 years
D. 4-5 years
E. 10 years or more




The correct answer is C. Although the time varies, 2 to 3 years is a common average time. Remember that as a crown erupts, the root is still forming, and x-rays of newly erupted teeth show varying levels of root formation. This has important implications in endodontics, when we may try to cause a tooth to complete its root formation in order to better manage it for restoration. A few examples of typical time lags are: maxillary central incisor, eruption 7-8 years, root completion 10 years; maxillary canine, eruption 11-12 years, root completion 14 years; mandibular second molar, eruption 11-13 years, root completion 14-15 years.
A mandibular permanent first molar is more likely to have a partially bifurcated:
A. mesial root
B. distal root
C. both roots are equally likely to be bifurcated
D. neither root is bifurcated




The correct answer is A. One of the distinctions between the mandibular first-molar mesial and distal roots is the deep root concavity seen running the length of the mesial surface of the mesial root. The distal root, by comparison, will have no similar concavity or a very slight depression. This concavity will sometimes express itself as a partial split (bifurcation) of some small section of the apical end of the mesial root into two roots. This feature is rare to unknown on the distal root. Other differences include the greater likelihood of the mesial root having a pronounced distal curvature.
The mesial and distal heights of contour of the maxillary canine are, respectively:
A. incisal third, incisal third
B. junction of incisal/ middle third, junction of incisal/middle third
C. junction of incisal/middle third, middle third
D. middle third, middle third
E. middle third, cervical third




The correct answer is C. One way to eliminate answers is to note that the labial view of the maxillary canine is not symmetrical. The mesial cusp ridge is shorter than the distal cusp ridge. This means that the distance from the cusp tip to the mesial contact is shorter than that from the cusp tip to the distal contact. Another way to look at it is that the distal cusp ridge dips lower (more cervical). Therefore, the distal height of contour will be more cervical than the mesial. The mesial height of contour is at about the junction of incisal and middle thirds, while the distal is at the middle of the middle third. Do not consider choice E, even though it follows the pattern discussed above. These contacts (middle, cervical) are too cervical for an anterior tooth.
Sharpey's fibers in the periodontal ligament:
A. enter only alveolar bone
B. enter only cementum
C. are collagenous in composition
D. enter cancellous bone to form bundle bone




The correct answer is C. Sharpey's fibers is an older name for the collagenous bundles of the periodontal ligament which connect the tooth to the tooth socket. The two tissues entered by these fibers are cementum on the tooth, and the cortical bone plate of the alveolar bone. When they enter the cortical plate, the resulting bone, when viewed under the microscope, is termed bundle bone. The fibers do not penetrate the bone enough to reach the cancellous inner layers of the alveolar bone.
The most common arrangement of canals in the roots of a permanent mandibular first molar is:
A. two mesial, two distal
B. two mesial, one distal
C. one mesial, two distal
D. one mesial, one distal
E. none of the above




The correct answer is B. Although some variation exists, the most common arrangement of canals in the mandibular first molar is two canals in the mesial root (MB and ML), and one in the distal. The largest is normally the distal, followed by the MB, then the ML. Approximately 25% of mandibular first molars will have four canals: MB, ML, DB, and DL.
The lingual fossa is normally deepest on which incisor?
A. Maxillary central
B. Maxillary lateral
C. Mandibular central
D. Mandibular lateral




The correct answer is B. The maxillary lateral is noted for its variability, and one area of great variety is the lingual fossa. It is normally more pronounced than that of any other incisor and often contains a pit, from narrow and shallow to deep and extensive. Extremely deep pits may give a dens-in-dente appearance. In operative dentistry the maxillary lateral is the only incisor in which you will occasionally see lingual pit amalgams. The lingual surface is often marked by a lingual marginal groove and tubercles on the cingulum. In general, expect any type of unusual variety to be most commonly found on maxillary laterals.
The buccal cusp of the maxillary first premolar occludes in:
A. the mesial marginal ridge of the mandibular first premolar
B. the distal marginal ridge of the mandibular second premolar
C. the mesial marginal ridge of the mandibular second premolar
D. the facial embrasure between the mandibular first and second premolars
E. the facial embrasure between the mandibular first premolar and canine




The correct answer is D. Buccal cusps of maxillary teeth are guiding cusps, not holding cusps. Therefore, they do not occlude on marginal ridges or central fossae. The general rule for maxillary buccal cusps is that they occlude in the facial embrasure between their mandibular counterpart and the tooth distal to their counterpart, EXCEPT for the mesiobuccal cusps of the molars, which occlude in the buccal groove of their mandibular counterpart, and the distobuccal cusp of the first molar, which opposes the distobuccal groove of the mandibular first molar. In this question, the contact is the embrasure between first and second mandibular premolars.
In restoring a mandibular first molar, lingual cusps are important for which movement?
A. Centric occlusion
B. Protrusive
C. Retrusive
D. Working
E. Nonworking




The correct answer is D. To answer this question, place your own teeth in centric occlusion (maximum intercuspation). Notice that the occlusion is held in place by the lingual cusps of maxillary teeth and the buccal cusps of mandibular teeth. Mandibular lingual cusps are not in contact. In protrusion, the anterior teeth act to separate the maxilla and mandible as the mandible moves forward. The anterior teeth act as guides in retrusion as well (try it). So far the mandibular lingual cusps are not involved. If you make a left working movement (slide your mandible left), notice that the mandibular lingual cusps on the left, drag against the lingual side of the maxillary lingual cusps. In the nonworking side (in this case, the right side), the mandibular lingual cusps move AWAY from contact.
The mucosa found on the surface of the hard palate is known as:
A. lining mucosa
B. masticatory mucosa
C. specialized mucosa
D. none of the above




The correct answer is B. Masticatory mucosa consists of the gingiva and lining of the hard palate. Most of the other areas of the oral cavity are lined by lining mucosa. Specialized mucosa is found on the dorsum of the tongue and contains taste buds and various papillae. The characteristics of masticatory mucosa are keratinization, thin or absent submucosa, and tight binding of the lamina propria to underlying periosteum. Lining mucosa is generally nonkeratinized, with a thin lamina propria. Near the teeth, the gingiva and lining mucosa (alveolar mucosa) meet at the mucogingival junction.
Which primary teeth MOST differ from their permanent counterparts?
A. Incisors
B. Canines
C. First molars
D. Second molars




The correct answer is C. For the most part, despite an overall size difference and some differences in ratio (crown:root, crown height:mesiodistal length), the primary teeth generally resemble the permanents for all teeth EXCEPT the first molars. Primary central and lateral incisors and canines are fairly close to small versions of the permanents. Primary second molars very closely resemble permanent first molars. However, primary first molars are unique. The primary maxillary first is premolar-like in crown form but possesses three roots. The primary mandibular molar is molar-like in form, but with an unusually pronounced mesial section and a much less developed distal section. It does not resemble any permanent tooth.
The premolar with the buccal cusp most distally placed relative to the lingual is the:
A. maxillary first
B. maxillary second
C. mandibular first
D. mandibular second




The correct answer is A. An unusual aspect of the maxillary first premolar is the mesial-distal placement of the cusps. The buccal cusp is off center toward the distal, while the lingual cusp is off center toward the mesial. If you look at the tooth from the lingual, therefore, you will always see two distinct cusp tips, one slightly to the mesial or distal of the other. This placement, when viewed occlusally, is said to give the tooth a twisted appearance. This situation is not found in the second premolar, where the cusps are almost equal in size and straight in alignment.
Thin organic structures which extend from enamel at the DEJ into the dentin are known as:
A. enamel spindles
B. enamel tufts
C. enamel lamellae
D. none of the above




The correct answer is C. There are a number of somewhat confusing anomalies at the dentinoenamel junction. Enamel spindles are odontoblastic processes and dentinal tubules which extend a short way into enamel. Enamel lamellae, on the other hand, extend from enamel into dentin. They are pieces of uncalcified organic enamel material. Enamel tufts are hypocalcified enamel rods. They are individual entities; however, they appear as groups (tufts) when a group of them are viewed laterally.
In the oral cavity, red marrow is found:
A. in all spongy bone of tooth sockets
B. in the condyle
C. in the lamellar layers
D. in most of the hard palate




The correct answer is B. Two types of marrow space are found within bone: red (hematopoietic), which is the source of blood cells; and yellow (fatty), which does not produce blood cells. The alveloar bone is similar to most bone in the body in having a compact outer layer of lamellar bone and an inner layer of spongy bone. This spongy bone contains marrow space, usually of the yellow (fatty) type, although some red marrow exists. The chief places where red marrow is found in the oral cavity are the condyle, the angle of the mandible, and the maxillary tuberosity.
Which tooth is LEAST likely to have a divided pulp canal?
A. Mandibular canine
B. Maxillary first premolar
C. Maxillary central incisor
D. Maxillary lateral incisor




The correct answer is C. Some teeth almost invariably have a divided pulp canal, such as the maxillary first premolar. This tooth normally has two roots, although the level at which the division occurs can vary. The other three teeth listed are normally single rooted with single undivided pulp canals. However the maxillary central incisor almost never varies from a single rounded root. In contrast, you can sometimes, although rarely, find divided roots, especially near the apex, for the mandibular lateral and canine.
The distal contact point and height of contour of the maxillary lateral incisor is located:
A. at the incisal edge
B. in the incisal third
C. in the middle third
D. in the cervical third




The correct answer is C. As is generally the case, the distal contact of this tooth is more cervical than is the mesial contact. On the mesial side, where it contacts the central incisor, the contact will be near the junction of incisal and middle thirds. However, on the distal side, it is slightly more cervical, usually around the middle of the middle third. This is the contact with the maxillary canine. On the canine, this height of contour is at the junction of the incisal and middle third.
The distobuccal cusp of the maxillary first molar occludes in:
A. the mesial marginal ridge of the mandibular second molar
B. the distal marginal ridge of the mandibular first molar
C. the central groove of the mandibular first molar
D. the facial embrasure between the mandibular first and second molars
E. the distobuccal groove of the first mandibular molar




The correct answer is E. Buccal cusps of maxillary teeth are guiding cusps, not holding cusps. Therefore, they do not occlude on marginal ridges or central fossae. The general rule for maxillary buccal cusps is that they occlude in the facial embrasures between their mandibular counterparts and the teeth distal to their counterparts, EXCEPT for the mesiobuccal cusps of the molars, which occlude in the buccal grooves of their mandibular counterparts, and the distobuccal cusp of the first molar, which opposes the distobuccal groove of the mandibular first molar. In this question we are dealing with the exception of the distobuccal cusps of the maxillary first molar, which occludes with the distobuccal groove of its mandibular counterpart, the mandibular first molar.
Which teeth exhibit isomorphy with each other?
A. Primary maxillary first and second molars
B. Primary maxillary second and primary mandibular second molars
C. Primary mandibular first molars and primary maxillary first molars
D. Primary maxillary second molars and permanent maxillary first molars
E. None of the above




The correct answer is D. Isomorphy refers to a close similarity in morphology and crown shape between two teeth. The most notable examples of isomorphy are the primary second molars with the permanent first molars IN BOTH ARCHES. Thus, by looking at a primary second molar in either arch, you can very accurately predict the morphology of the permanent first molar in that same arch. Isomorphy does not exist between primary first and second molars, as both first molars are very unusual in shape and both second molars closely resemble permanent first molars.
Synovial fluid-producing tissue in the TMJ:
A. is found throughout all surfaces
B. is not found within the joint
C. is found only on articular surfaces
D. is found only on non-articular surfaces




The correct answer is D. The TMJ is a synovial joint, similar to most other movable joints. It is different in having a fibrous articular disc. The surface of the disc is fibrous connective tissue, and the surface of the articulating surfaces of the glenoid fossa are fibrous connective tissue overlying hyaline cartilage. These surfaces are not covered with synovial tissue; however, their smooth functioning is dependent on the production of synovial fluid elsewhere in the joint. This production occurs in all non-articulating surfaces of the joint, which are lined with synovial tissue.
The primary maxillary first molar is often said to have a premolar shaped crown. This tooth will invariably have how many roots?
A. One
B. Two
C. Three
D. Four




The correct answer is C. Despite a somewhat premolar-shaped crown, the primary maxillary first molar is consistent in root form with all maxillary molars, primary or permanent, in having three roots. They are the mesiobuccal, distobuccal, and palatal. Premolars will routinely have either one or two roots. Primary molars lack a common root trunk, and have widespread roots which allow space for the developing premolar crown underneath.
Mesial-occlusal-distal (MOD) cavity preparations are most difficult in which of the following teeth?
A. Permanent mandibular first premolar
B. Permanent maxillary second molar
C. Primary mandibular second molar
D. Primary mandibular first molar




The correct answer is D. MOD preparations can be difficult because tooth structure must be removed from three tooth surfaces. The remaining tooth structure can be weak, and pulp horns may be exposed. They are especially hard in small teeth with large pulp horns. The primary mandibular first molar is a good example. It has a very narrow distal end, is small overall, and has very high mesial pulp horns. Additionally, little tooth structure remains after the MOD is cut. Permanent mandibular first premolars are small, but not like the primaries. They should have a slanted occlusal floor, but an MOD is possible. Permanent maxillary second molars are generally large and would be the best of this group for making an MOD preparation. The primary mandibular second molar, while small, is larger than the first molar, and has the same shape as a permanent first molar, making it not as difficult to perform an MOD preparation.
Which cell type of the enamel organ is widely separated by liquid between the cells?
A. Outer enamel epithelium
B. Stellate reticulum
C. Stratum intermedium
D. Inner enamel epithelium




The correct answer is B. Both the inner and outer enamel epithelium are solid lines of cells with no space between the cells in each line. Between these two layers of cells we find the stratum intermedium, close to the inner enamel epithelium; and the stellate reticulum, filling the space between the stratum intermedium and outer enamel epithelium. The stratum intermedium is generally a few layers of squamous epithelium. The stellate reticulum is a reticulum (net-like network) of widely separated cells (think of stars in the sky). These cells are surrounded by a clear liquid matrix and are connected by cell processes.
The primary second maxillary molar is usually exfoliated between which ages?
A. 4-5 years
B. 6-7 years
C. 8-9 years
D. 10-11 years
E. 12-13 years




The correct answer is choice D. Eruption and exfoliation questions are common NBDE Part 1 topics. Answers are approximate, of course, and subject to variation. However, you should know the most common average ages for all teeth. For maxillary primaries, the centrals exfoliate at year 7-8, laterals at 8-9, canines at 11-12, first molars at 10-11, and second molars at 11-12. Remember that these are averages. Always eliminate wildly wrong answers first.
In a mandibular canine, which surface is usually most parallel to the long axis of the tooth?
A. Buccal
B. Lingual
C. Mesial
D. Distal




The correct answer is C. Reviewing the surfaces, the lingual surface of the canine contains the bulbous cingulum. The surface of the lingual is thus slanted at roughly 45 degrees from the long axis (as represented by the root). Likewise, the facial surface slants inward as you go from cervical to incisal. The distal surface slants lingually as the tooth narrows coronally. Our answer is mesial. The mesial surface is vertical overall, with no large bulge. Notice that the distal is always more rounded and bulbous than the mesial, which is relatively straight and, if extended, would roughly parallel the root.
The mesiolingual cusp of the mandibular first molar contacts:
A. the lingual embrasure between the maxillary first molar and second premolar
B. the distal marginal ridge of the maxillary second molar
C. the central fossa of the maxillary first molar
D. the lingual groove of the maxillary first molar
E. the lingual embrasure between the maxillary first molar and second molar




The correct answer is A. Lingual cusps of mandibular teeth are guiding cusps, not holding cusps. Therefore, they do not occlude on marginal ridges or central fossae. The general rule for mandibular lingual cusps is that they occlude in the lingual embrasures between their maxillary counterparts and the teeth mesial to their counterparts, EXCEPT for the distolingual cusps of the mandibular molars, which occlude in the lingual grooves of their maxillary counterparts. In this question we are not dealing with the exception of the distolingual cusps of mandibular molars, so in this case, the mandibular first molar mesiolingual cusp contacts the lingual embrasure between its counterpart (the maxillary first molar) and the tooth mesial to it (the maxillary second premolar).
Which muscle of mastication controls movement of the articular disc of the TMJ?
A. Medial pterygoid
B. Lateral pterygoid
C. Temporalis
D. Masseter




The correct answer is B. The lateral pterygoid muscle has two heads. The superior head arises from the greater wing of the sphenoid bone, and the inferior head from the lateral plate of the lateral pterygoid bone. They extend posteriorly to the mandibular condyle, TMJ capsule, and TMJ disc. Fibers from the upper head enter the capsule and merge with the fibrous connective tissue of both the capsule and disc. This is related to the function of the muscle in protruding the mandible, and pulling the disc with the condyle during protrusion, depressing (opening), and lateral movements. No other muscle enters the TMJ.
The inclinations of the root tips of the mesiobuccal and distobuccal roots of the maxillary first molar are, respectively:
A. mesial, mesial
B. mesial, distal
C. distal, mesial
D. distal, distal
E. straight, straight




The correct answer is C. A characteristic curve of the two buccal root tips of the maxillary first molar toward each other is sometimes referred to as a pliers-handled appearance or pincer appearance. In effect, the roots grow apically and then turn toward each other, forming a U shape. This shape helps explain the stability of the tooth, especially when combined with the tripod-like arrangement of the palatal root. Note that the same roots of the second maxillary molar often both point distally, distinguishing one molar from the other. These roots in the second molar are also more parallel than those in the first molar.
The predentin:
A. is more mineralized than dentin
B. contains the odontoblasts
C. contains the ameloblasts
D. is primarily collagen
E. None of the above




The correct answer is D. Predentin would be found in between odontoblasts and true dentin. The odontoblasts lay down predentin, which is primarily protein matrix composed almost entirely of collagen. As it mineralizes farther away from the odontoblastic layer, it becomes true dentin, with its high mineral content. Ameloblasts form enamel and are not involved in dentin or predentin production.
Which premolar exhibits H, Y, and U occlusal patterns?
A. Maxillary first
B. Maxillary second
C. Mandibular first
D. Mandibular second




The correct answer is D. The mandibular second premolar has a variety of occlusal appearances due mostly to its two-cusped or three-cusped nature. There is a slightly larger buccal cusp area, and then either an approximately equal-sized lingual cusp, or a pair of lingual cusps, known as the mesiolingual and distolingual cusps. Together, these two form an area slightly smaller than the buccal cusp. In the event of the three cusps, an occlusal Y groove pattern is common. On the two-cusped variety, either an H (more common) or U (less common) pattern may be found.
Which fiber group is bound on both sides by cementum?
Which fiber group is bound on both sides by cementum?
Which of the following permanent teeth is NOT succedaneous?
A. Maxillary lateral incisor
B. Mandibular first molar
C. Maxillary second premolar
D. Mandibular canine
E. Maxillary canine




The correct answer is B. Succedaneous refers to a tooth that succeeds another tooth; that is, a tooth that replaces a previous tooth. Remember that permanent incisors and canines replace primary incisors and canines (choices A, D, and E). Permanent premolars replace primary molars (choice C). Thus, all the permanent teeth mentioned so far are succedaneous. Permanent molars are not succedaneous as they do not replace primary teeth. The permanent first molar develops distal to all primary teeth, and later the second and third molars do likewise. Molars do not replace any primary teeth and are therefore not succedaneous.
Which of the following is not a fiber of the gingival unit?
A. Dento-gingival
B. Dento-periosteal
C. Alveolar crestal
D. Circular




The correct answer is C. The alveolar crestal fibers are examples of fibers of the periodontal ligament, rather than gingival group fibers. The distinction is that periodontal fibers attach on one end into cementum, and on the other into alveolar bone. Gingival fibers do not attach to bone. The alveolar crest fibers attach from cementum to alveolar bone at the alveolar crest. Dento-gingival fibers attach from cementum to gingiva, dento-periosteal fibers from cementum to periosteum, and circular fibers run circularly around the tooth within the gingiva.
Of the following anterior teeth, the one most likely to be double rooted is the:
A. Maxillary lateral
B. Mandibular lateral
C. Maxillary canine
D. Mandibular canine




The correct answer is D. Although none of the teeth listed is commonly double rooted, the mandibular canine is the most likely of this group. Maxillary laterals (choice A) are highly variable and are often the most variable teeth in the mouth. However, their variability does not usually include double-rooted appearance. Mandibular centrals and laterals (choice B) often have proximal root grooves. If these grooves are deep, there may be an appearance of becoming double rooted, but they rarely are. Maxillary canines (choice C) are invariably single rooted. Mandibular canines are occasionally double rooted. The point of separation into two roots may be toward the cervical of the root, or almost at the apex. This double-rooted possibility is significant for both exodontia and endodontics.
Which periodontal fibers are most in line with the long axis of the root?
A. Alveolar crestal
B. Apical
C. Horizontal
D. Oblique




The correct answer is B. If we arbitrarily call the long axis of the root vertical, only the apical periodontal fibers are nearly parallel (vertical). The apical fibers run vertically from the apex of the alveolar bone socket to the root apex cementum. More coronally, the next fibers, the oblique, run obliquely (slanted) from near apical cementum to more coronal alveolar bone. Horizontal fibers run horizontally from cementum to bone, which is thus perpendicular to the root axis. The most coronal alveolar fibers run obliquely from coronal cementum, apically slanted toward the alveolar crest of bone.
In protrusive movements, the lingual surface of the maxillary lateral incisor will contact which mandibular teeth?
A. Central incisor only
B. Central and lateral
C. Lateral incisor only
D. Lateral and canine
E. Canine only




The correct answer is D. Remember that the mandibular teeth, being smaller, are set one-half tooth mesial to the corresponding maxillary tooth. That is, the maxillary central contacts the mandibular central and half of the mandibular lateral. The maxillary lateral contacts both the mandibular lateral and canine. Note that the maxillary canine cusp tip sits in the embrasure between the mandibular canine and first premolar, although the incisors are the main contacts in protrusive movement.
The Carabelli cusp (trait) is found attached to which cusp?
A. Mesiobuccal
B. Mesiolingual
C. Distobucal
D. Distolingual




The correct answer is B. The Carabelli cusp (trait) is a variable cusp or extension of the mesiolingual cusp of the maxillary first molar. Its expression varies from that of a full cusp, at its largest, to a small protuberance at its smallest. Its expression varies among various races and ethnic groups. Some anatomists describe it as a cusp, making the maxillary first molar a five-cusped tooth. Others consider it a variety or trait of a four-cusped tooth. It is sometimes missing, and a groove or pit is found in the normal Carabelli cusp region of the mesiolingual cusp.
If vertical dimension of occlusion is VDO, and vertical dimension of rest is VDR, and freeway space is FS, then:
A. VDO + VDR = FS
B. VDO + FS = VDR
C. VDR + FS = VDO
D. None of the above




The correct answer is B. The vertical dimension of occlusion (VDO) is a vertical measurement on the front of the face when the teeth are in full occlusion (centric occlusion). When the face is at rest, the teeth are slightly apart, and the vertical dimension of the front of the face is slightly longer. This is the vertical dimension of rest (VDR). The distance between the teeth at this point is the freeway space (FS). So when we take the smaller VDO and add the few millimeters of the FS, we get the slightly longer VDR. VDR is generally 2 to 5 mm more than VDO.
The shape of the pulp chamber in cross section cut at the cervical line in the mandibular central incisor is:
A. oval (flattened mesiodistally)
B. oval (flattened bucco-lingually)
C. round
D. figure eight




The correct answer is A. Only maxillary incisors have roundish pulp chamber cross sections when cut at the cervical line. This is true for both maxillary centrals and laterals. In general, mandibular incisors and both arch canines will have ovalish pulp chambers, flattened mesiodistally. (Imagine squashing a round pulp from both the mesial and distal sides at once, resulting in a squashed oval, pointing to the buccal and lingual). Remember that the shape of a pulp chamber is usually related to the shape of the surrounding root.
Enamel is formed from which embryonic germ layer?
A. Endoderm
B. Mesoderm
C. Ectoderm
D. None of the above




The correct answer is C. Of the tissues comprising the tooth structure, only enamel is formed from ectoderm. It is made by ameloblasts, which are ectodermal derivatives. Cementum, produced by cementoblasts, and dentin, produced by odontoblasts, are both mesodermal derivatives (choice B). Note also that pulpal tissue is mesodermal. No tooth structural tissue is endodermal (choice A).
In a noncarious, unrestored, and fully erupted tooth, dentin continues to form at the border of the pulp and dentin. This dentin is known as:
A. primary dentin
B. secondary dentin
C. tertiary dentin
D. reparative dentin
E. none of the above




The correct answer is B. Dentin forms the bulk of both the crown and root of the tooth. In the crown, it underlies the outer enamel. In the root, it underlies a thin layer of cementum. The original dentin of the crown and root is primary dentin and is the most regular in structure. The pulp continues to form dentin later in life at the dentin-pulp border. This results in smaller pulp chambers with age. This dentin is secondary dentin, and is less regular in structure than is primary dentin. Reparative dentin is formed as a defense against caries, irritation, deep restorations, etc. It is the least regular in structure, and is alo known as sclerotic dentin. There is no tertiary dentin.
The primary crown most UNLIKE any permanent tooth is the:
A. central incisor
B. lateral incisor
C. canine
D. first molar
E. second molar




The correct answer is D. The primary first molars, both maxillary and mandibular, are unique in morphology and bear little resemblance to any permanent teeth. Primary incisors and canines resemble their permanent counterparts closely, except in certain proportional ways. Likewise, primary second molars are notable for closely resembling their permanent FIRST molar counterparts. The primary maxillary first molar is a little like a premolar in shape, but generally has one minor and two major cusps. It is both small and squat in appearance. The mandibular primary first molar is four-cusped but has a distinctly larger development of the mesial end. The MB and ML cusps generally comprise two-thirds of the area of the crown.
Osteons, Haversian systems, and concentric bone layers:
A. are not found in alveolar socket bone
B. are found in the outer layer of alveolar bone
C. are found in the inner layer of alveolar bone
D. are found throughout alveolar bone




The correct answer is B. The alveloar bone is similar to most bone in the body in having a compact outer layer of lamellar bone and an inner layer of spongy bone. This spongy bone contains marrow space, usually of the yellow (fatty) type, although some red marrow exists. Haversian systems, with concentric rings of bone with osteocytes and canaliculi, which surround a canal with artery, vein, and nerve, are found only in compact (lamellar) bone. These systems do not exist within the inner spongy layers of bone, whether we are looking at alveolar bone or any other bone in the body.
Which muscle of mastication both strongly elevates and strongly retrudes the mandible?
A. Masseter
B. Medial pterygoid
C. Lateral pterygoid
D. Temporalis
E. Anterior digastric




The correct answer is D. The temporalis is a strong, broad, flat muscle which can both elevate (close) and retrude (pull back) the mandible. This is due to the fact that it contains both vertical and nearly horizontal muscle fibers. The vertical fibers pull the mandible straight upward (elevation), while the horizontal fibers pull straight backward (retrusion). The most anterior fibers of the temporalis are the vertical. As you move posteriorly, they become first diagonal, and then horizontal. NBDE questions in the past have referred to this muscle as being an elevator in the anterior and an elevator/retruder in the posterior.
Which incisor is most commonly found doubled (supernumerary)?
A. Maxillary central
B. Maxillary lateral
C. Mandibular central
D. Mandibular lateral




The correct answer is B. As is often the case, the maxillary lateral incisor is the answer for weirdest or most variable tooth. In this case, it is the most common supernumerary (doubled incisor), as it is also the most common missing, oversized, or undersized incisor. Note that the most distal tooth (lateral incisor, second premolar, third molar) in a set is always more likely to be missing or have a supernumerary. Note also that the most common supernumerary is the mesiodens. This tooth is found in the maxillary midline. It is not, however, considered an additional central. It has a morphology unique to itself and different from centrals.
The primary mandibular central incisor usually exfoliates between which ages?
A. 4-5 years
B. 6-7 years
C. 8-9 years
D. 10-11 years
E. 12-13 years




The correct answer is B. Eruption and exfoliation questions are common in this section. The mandibular primary centrals are often the first teeth lost, most commonly at age 6. Often they exfoliate as the permanent first molars are erupting. For mandibular primaries, the centrals exfoliate at year 6-7, the laterals at year 7-8, the canine at year 9-10, the first molar at year 10-11, and the second molar at age11-12. Remember that all exfoliation questions are approximations of average ages. The wrong answers must be fairly far off the mark.
In centric occlusion, the mesiolingual cusp of the maxillary second molar will contact:
A. the central fossa of the mandibular second molar
B. the central fossa of the mandibular first molar
C. the distal marginal ridge of the mandibular first molar and mesial marginal ridge of the second molar
D. the distal marginal ridge of the mandibular second premolar and mesial marginal ridge of the first molar




The correct answer is A. The mesiolingual cusp of a maxillary second molar is a holding cusp. The general rule for maxillary holding cusps is as follows: a maxillary holding cusp contacts the distal marginal ridge of its mandibular counterpart and the mesial marginal ridge of the mandibular tooth distal to its counterpart, EXCEPT for the mesiolingual cusps of the molars, which contact the central fossae of their counterparts. This should be the central fossa of the counterpart, the mandibular second molar.
When assigning types of mandibular motion to the compartments of the TMJ, it is generally agreed that which compartment is associated with which motion type?
A. Anterior: rotation, posterior: translation
B. Anterior: translation, posterior: rotation
C. Superior: rotation, inferior: translation
D. Superior: translation, inferior: rotation




The correct answer is D. First, eliminate choices A and B, as the disc divides the joint compartment into upper (superior) and lower (inferior) compartments only. In the upper compartment, the upper surface of the disc acts as a cushion as the mandibular condyle slides forward. This cushion slides against the articular eminence in the movement of the condyle in a forward and downward direction. When rotating about an axis during opening, the condyle slides against the lower surface of the disc which conforms to the shape of the condylar head. So we say that translation occurs in the upper compartment and rotation in the lower.
The mesial height of contour of the mandibular central incisor is:
A. near the incisal edge
B. at the junction of the incisal and middle thirds
C. in the middle third
D. at the junction of the middle and cervical thirds




The correct answer is A. Mandibular central incisors contact at the most incisal point found on any tooth. The contact is located just below the incisal edge. On the distal surface, it is still incisal and near the edge, but is slightly more cervical than the contact found on the mesial side of the tooth.
When compared to that of permanent teeth, the ratio of mesio-distal diameter to crown height of primary teeth is:
A. greater
B. less
C. the same
D. variable, depending on the individual tooth




The correct answer is A. Primary teeth are often said to have a short or squat appearance of their crowns compared to those of permanent teeth. This is because these teeth are often relatively wide mesiodistally and short incisocervically. This results in a LARGE ratio of mesiodistal diameter to incisocervical crown height. This ratio is smaller in the permanent teeth, which lack the short, squat appearance of primaries. One example: for maxillary central incisors, the primary tooth ratio is 1.083 (the tooth is just slightly bigger mesioincisally than in crown height), while for the corresponding permanent incisor it is .809 (the mesiodistal diameter is only 80% of the crown height).
When compared to that of permanent teeth, primary teeth color tends to be:
A. whiter
B. darker
C. similar in shade
D. variable, depending on the individual tooth




The correct answer is A. As a general rule, primary teeth are whiter than their permanent counterparts. This shade difference is often noticed by parents. It is common for parents to notice, for example, that newly erupted permanent mandibular incisors are a few shades darker or more yellowish than the primary maxillary incisors that may remain as the mandibular permanent incisors erupt. In the mixed dentition (normal, unstained, not heavily carious, and no tetracycline effects), you can often pick out the primary versus permanent teeth by color alone.
Which is the narrowest anterior tooth in a mesiodistal direction?
A. Maxillary lateral
B. Mandibular central
C. Mandibular lateral
D. Mandibular canine




The correct answer is B. In general, canines are wider mesiodistally than lateral incisors(both arches), so we should eliminate the canine (choice D). In general, maxillary incisors are wider mesiodistally than their mandibular counterparts, so we should eliminate the maxillary lateral (choice A). So it should only be a question of mandibular lateral versus central. On the average, the central is slightly smaller, about 0.5 mm smaller, and this gives it the distinction of narrowest anterior tooth mesiodistally. The Dental Board exam really loves longest, shortest, roundest, fattest, etc., type questions. Note that the maxillary canine is the longest tooth in the mouth.
Which of the following angles in a maxillary incisor is sharpest?
A. Mesioincisal of central
B. Mesioincisal of lateral
C. Distoincisal of central
D. Distoincisal of lateral




The correct answer is A. In looking at a maxillary central incisor from the facial, you will see that the mesioincisal angle is a sharp, nearly right angle. This is especially noticeable where the two centrals meet and a very small embrasure is present. The distoincisal angle, by contrast, is more rounded (choice C). On the lateral, the mesioincisal (choice B) is again sharper than the distoincisal (choice D), which is very rounded. However the angle on the lateral does not approach the sharpness of that on the central.
As you go from mandibular first to second to third molars, mesiodistal crown length:
A. increases, then decreases
B. increases
C. decreases
D. decreases, then increases




The correct answer is C. While the mandibular third molar is often so variable that it is hard to make useful predictions about it, these generalities apply to mandibular first, second, and third molars as you move distal in the arch: 1) they decrease in mesiodistal length, and 2) their roots decrease in length. Third-molar crowns may resemble normal molars or may be missing whole cusps. In rare cases they are roundish or ovoid, but they are always smaller than their second-molar counterparts, which are always smaller than the first molars. Note that buccolingual length is always shortest in the third molar but about equal in the first and second.
Which of the following permanent teeth is MOST likely to be missing a distolingual cusp?
A. Mandibular first molar
B. Maxillary first molar
C. Maxillary third molar
D. Maxillary second molar




The correct answer is C. The mandibular first molar is invariably a five-cusped tooth, with MB, ML, DB, DL, and distal cusps. Maxillary first and second molars are normally four-cusped teeth. The distolingual cusp is usually less pronounced on the second molar. It is often absent on the maxillary third molar. This is an arch trait: the gradual diminishment in size of the distolingual cusp as you go posteriorly in the maxilla. In the lower arch, the mandibular second premolar can also be viewed as possibly lacking a distolingual cusp. This tooth is either two- or three-cusped. The two-cusped variety has a single buccal cusp and a single lingual cusp, while the three-cusped version has a buccal cusp and paired mesiolingual and distolingual cusps. However, both varieties are common, so it is common to find this tooth lacking the distolingual cusp as well, although that answer is not listed here.
The protein matrix of enamel is:
A. composed of collagen
B. composed of keratin
C. composed of protein which is not collagen or keratin
D. absent, there is no protein matrix




The correct answer is C. Chemical analysis of the enamel matrix has not adequately identified the protein involved, but it is clear that it is neither keratin, as was once thought, nor collagen, which is the matrix of both dentin and bone. The protein is extremely high in proline, but makes up less than 1% of the enamel mass.
Cervical bulges are found on which surfaces of primary anterior teeth?
A. Mesial and distal
B. Buccal and lingual
C. Lingual only
D. Buccal only




The correct answer is B. Primary teeth are notable for cervical bulges; that is, extension of the crown at the cervical line. In the primary anteriors, these bulges extend over both the buccal and lingual surfaces, as though the cervical line was a tightened belt and was tightened further to make the crown overhang in both front and back. In the primary molars, by contrast, the cervical bulge is buccal only.
Which of the following is a function of pulpal tissue?
A. Sensory
B. Formation of dentin
C. Nutritive
D. Defensive
E. All of the above




The correct answer is E. It should be noted that the MAJOR function of pulp is the production of dentin, as dentin is the major structural tissue of both crown and root. Any question on the dental boards that asks for the "most important" function will have dentin formation as the answer. However, the pulp is also sensory (clearly). All sensory fibers in the pulp carry pain sensation, regardless of whether the original stimulus is hot, cold, or touch. The nutritive function of pulp consists of providing materials to the dentin through the odontoblastic processes. The defensive function of the pulp consists of a limited ability to produce a defensive inflammatory response, and the formation of reparative dentin when the tooth is damaged.
Functions attributed to the ligaments of the temporomandibular joint include all of the following EXCEPT:
A. increasing stability of the joint
B. aiding in forward movement of the condyle
C. increasing the strength of the joint
D. limiting extreme motion of the joint
E. All of the above are ligament functions




The correct answer is B. The ligaments, being fibrous connective tissue in composition, play no role in movement of the condyle. Muscular tissue is necessary for this movement, specifically fibers from the upper head of the lateral pterygoid muscle. There is disagreement among anatomy sources as to the function or role of the ligaments. None give the ligaments an extremely important function, but most claim that they help limit motion of the mandible beyond a certain point, or act as barriers to excessive forward or downward motion. Others give the ligaments a general strengthening function which adds stability to the joint. Most importantly, the Dental Boards usually agree with the general notion of adding strength, stability, and limits of movement to the joint.
Which permanent mandibular molar is most likely to be marked by supplemental occlusal grooves, crenulations, and unpredictable placement of pits?
A. First molar
B. Second molar
C. Third molar
D. All molars about equally




The correct answer is C. The question may be answered on general principle, in that the third molars are often the most unpredictable teeth in either arch. The specific patterns referred to here are the pit-groove patterns. In third molars, you will often find supplemental grooves at right angles to the main grooves, and additional pits and fissures not normally seen in first and second molars. The crenulated pattern refers to a highly grooved overall occlusal surface with grooves running in all directions, leaving a lacy, nook-and-cranny occlusal surface.
Which cell type is NOT found within the periodontal membrane?
A. Cementoblasts
B. Osteoblasts
C. Odontoblasts
D. Osteoclasts




The correct answer is C. Odontoblasts form dentin and surround the dental pulp. The dentin forms the bulk of both the tooth crown and root. However, dentin is not one of the opposing surfaces within the periodontal ligament. Within the ligament, the alveolar bone of the tooth socket lies next to the cementum surface of the tooth root. On the cemental side of the ligament, cementum-forming cementoblasts are found. On the bone side, bone-forming osteoblasts and bone-remodeling osteoclasts are both found. In addition, numerous collagen fiber bundles insert in both the cementum and alveolar bone, forming the connection that keeps the tooth in place in the socket.
The Carabelli cusp (trait) is found on which molar tooth?
A. Maxillary first
B. Mandibular first
C. Maxillary second
D. Mandibular second




The correct answer is A. The Carabelli cusp (trait) is a variable cusp or extension of the mesiolingual cusp of the maxillary first molar. Its expression varies from that of a full cusp, at its largest, to a small protuberance at its smallest. Its expression varies among various races and ethnic groups, including an increased incidence in people of Asian descent. Some anatomists describe it as a cusp, making the maxillary first molar a five-cusped tooth. Others consider it a variety or trait of a four-cusped tooth. It is sometimes missing, and a groove or pit is found in the normal Carabelli cusp region of the mesiolingual cusp.
In a left lateral (working movement):
A. the left mandibular teeth are the working side and move to the left
B. the left mandibular teeth are the working side and move to the right
C. the left mandibular teeth are the balancing side and move to the left
D. the left mandibular teeth are the balancing side and move to the right




The correct answer is A. By definition, a working movement occurs when the mandibular teeth on one side move laterally across the surfaces of the maxillary teeth toward their own side. In other words, in a left working movement, left mandibular cusps move laterally left across the cusps of the maxillary left teeth. The other side is known as the nonworking (balancing) side. In this case, the balancing side is the right side. In a left working movement, the balancing side also moves left, as the mandible cannot move both left and right at the same time.
Of the following incisors, the one which can most often be rotated during extraction is the:
A. maxillary central
B. maxillary lateral
C. mandibular central
D. mandibular lateral




The correct answer is A. For the most part, rotation of a tooth during extraction works best on teeth with rounded roots, such as the maxillary central incisor and maxillary canine. It can be used in conjunction with buccal-lingual luxation on teeth with single ovalish roots. It can never be used on double-rooted teeth. Of the incisors above, the maxillary central is most likely to have a roundish root. The others tend to have roots which are more ovalish rather than round and are luxated primarily, with some rotation. In general, maxillary incisor roots are more rounded than are mandibular roots.
In the permanent mandibular third molar, when viewed from the occlusal
A. the mesial end is larger than the distal
B. the distal end is larger than the mesial
C. the mesial and distal ends are the same size
D. it is impossible to generalize about mandibular third molars




The correct answer is A. Beware of choice D. Although mandibular third molars are notoriously variable in both crown and root form, there are some good generalizations. Almost invariably, the mesial (trigonid) end of the crown is wider, larger, and more distinct than the distal (talonid). The most common pattern is a four-cusp tooth, with MB and ML being the largest cusps and the whole occlusal surface tapering toward the distal.
When viewed from the meisal, the tooth closest to vertical in the buccolingual angulation of its main axis is the:
A. maxillary central incisor
B. maxillary lateral incisor
C. maxillary canine
D. maxillary first premolar




The correct answer is D. Another way of asking the question is: which tooth does not lean toward the buccal or lingual? The maxillary first premolar is almost straight vertically, estimated to be about 5 degrees from vertical. Premolars, as a class, have the least bucco-lingual angulation. The central tends to lean greatly toward the lingual (28 degrees), the lateral about the same (26 degrees), and the canine slightly less. Do not bother to remember degree measures, which are mentioned only for comparison. Do not confuse this leaning with the mesiodistal leaning (when viewed from the buccal or lingual). These measurements differ significantly for the teeth listed, with the central incisor being most vertical.
Tooth #23 replaces which primary tooth?
A. K
B. L
C. M
D. N
E. O




The correct answer is D. Both 23 and N are mandibular left incisors. Remember that A through J are maxillary, while K through T are mandibular. The mandibular replacements are 20 for K, 21 for L, 22 for M, 23 for N, 24 for O, 25 for P, 26 for Q, 27 for R, 28 for S, and 29 for T. Remember that permanent molars 17, 18, 19, 30, 31, and 32 do not replace primary teeth.
The mesiobuccal cusp of the mandibular third molar occludes with which maxillary tooth surfaces?
A. The mesial marginal ridge of the third molar and distal marginal ridge of the second molar
B. The distal marginal ridge of the third molar and mesial marginal ridge of the second molar
C. The mesial marginal ridge of the third molar only
D. The central fossa of the maxillary third molar




The correct answer is A. The mesiobuccal cusp of a mandibular molar is a holding (supporting) cusp. The general rule of occlusion of mandibular holding cusps is as follows: the holding cusps of the mandibular teeth occlude on the mesial marginal ridge of their maxillary counterpart, and the distal marginal ridge of the maxillary tooth mesial to their counterpart, EXCEPT distobuccal cusps of mandibular molars occlude with central fossae of their counterparts, the distal cusp of the mandibular first molar occludes with the distal triangular fossa of its counterpart, and the first premolar occludes only with the mesial marginal ridge of its counterpart (but not the canine). In this case, the maxillary counterpart is the maxillary third molar, and the tooth immediately mesial to it is the maxillary second molar.
In the primary mandibular first molar, the most noticeable cusps are:
A. MB and DB
B. MB and ML
C. DB and DL
D. MB and DL




The correct answer is B. The primary mandibular first molar is a highly unusual tooth. Although it technically contains four cusps, in typical molar fashion it has a great variety in the prominence of those cusps. The MB and ML cusp are most prominent, and the mesial section of the crown comprises two-thirds of the crown area. Both the DB and DL cusps are much reduced. The range of cusp size, in decreasing order is: MB, ML, DB, DL. The ML cusp is notable for a pointy, cone-like shape.
The central fossa of a maxillary first molar is NOT bordered by which cusp?
A. Mesiobuccal
B. Mesiolingual
C. Distobuccal
D. Distolingual




The correct answer is D. The central fossa, containing a central pit, is located within the main cusp triangle (trigon) of the maxillary first molar. The three major cusps: MB, ML, and DB, surround the fossa and form the trigon, or primitive cusp triangle. The DL cusp (talon) is not part of the trigon, does not surround the central fossa, and is the smallest cusp of the tooth (excluding the Carabelli cusp or trait).
Which position is achieved entirely by soft tissue, without guidance from teeth?
A. Postural (physiologic rest)
B. Centric occlusion
C. Centric relation
D. Protruded contact




The correct answer is A. In postural rest position, the mandible stays a comfortable distance from the maxilla, with the teeth slightly separated. The freeway space between the teeth is often 2-5 mm. There is no tooth contact, and the position is determined and maintained by the muscles of occlusion and related tendons and ligaments. Centric occlusion involves full tooth contact. Centric relation is normally a few millimeters distal to centric occlusion and is achieved by moving distal from centric occlusion. This procedure involves tooth contact. Protruded contact involves contacting guidance of the maxillary and mandibular teeth during protrusion.
The faciolingual dimension of a mandibular first molar, when compared to the mesiodistal dimension, is:
A. smaller
B. larger
C. the same
D. larger or smaller, depending on the tooth




The correct answer is A. Mandibular first molars are the largest teeth in the mouth, in mesiodistal dimension. While they are also large buccolingually, they are invariably larger mesiodistally by about 0.5 to 1.0 mm. This is in contrast to the maxillary first molar, which is slightly larger buccolingually than mesiodistally, usually by about 1.0 mm. The mandibular second molar is about equal in both directions and is thus more symmetrical than the first molar.
In a left working movement:
A. the left side moves laterally, and the right side medially
B. the left side moves medially, and the right side laterally
C. both sides move medially
D. both sides move laterally




The correct answer is A. In a left working movement, the mandible shifts to the left. From a central position, the left side of the mandible is moving away from center (laterally). The right side of the mandible, although also moving left, is moving toward the center (medially). The left TMJ purely rotates when the right is rotating and translating mechanically.

Note that it is impossible for both sides of the mandible to move either laterally or medially at the same time.
The premolar which is most often double rooted is the:
A. maxillary first
B. maxillary second
C. mandibular first
D. mandibular second




The correct answer is A. Maxillary first premolars are almost always double rooted. When single rooted, they always have two separate root canals. The level of division of the two roots varies, from complete division up to the crown, to only a small separation apically. The mandibular first premolar is most often singlerooted, and the mandibular second premolar is almost always single rooted (double roots are rarer than in the mandibular first). The maxillary second premolar is most often single rooted. The extreme likelihood that the maxillary first premolar will have two roots has implications for both endodontics and exodontia of this tooth. It is also reported that some small percentage (5% in one study) are actually triple rooted.
In centric occlusion, the mesiolingual cusp of the maxillary first molar will contact:
A. the mesial marginal ridge of the mandibular first premolar and distal marginal ridge of the second premolar
B. the distal marginal ridge of the mandibular first molar and distal marginal ridge of the second molar
C. the distal marginal ridge of the mandibular second premolar and mesial marginal ridge of the first molar
D. the central fossa of the mandibular first molar




The correct answer is D. The mesiolingual cusp of a maxillary molar is a holding cusp. The general rule for maxillary holding cusps is as follows: a maxillary holding cusp contacts the distal marginal ridge of its mandibular counterpart and the mesial marginal ridge of the mandibular tooth distal to its counterpart, EXCEPT FOR THE MESIOLINGUAL CUSPS OF THE MOLARS, WHICH CONTACT THE CENTRAL FOSSAE OF THEIR COUNTERPARTS. This should be the central fossa of the counterpart, the mandibular first molar.
A rule regarding congenitally missing teeth is:
A. A mandibular tooth is more likely to be missing than its maxillary counterpart
B. A distal tooth of a type is more likely to be missing than the mesial tooth of the same type
C. A mesial tooth of a type is more likely to be missing than a distal tooth of the same type
D. None of the above is true




The correct answer is B. In any set of teeth (incisor, premolar, molar), the distal tooth is more likely to be congenitally missing than the mesial. So lateral incisors are missing more than centrals, second premolars are missing more than first premolars, and third molars are more commonly missing than first or second. Of all of these, the MOST commonly missing is the maxillary lateral incisor. Note that choice A is therefore incorrect.
The crowns of the incisors when viewed from the incisal
A. are wider mesiodistally than buccolingually in both arches
B. are wider buccolingually than mesiodistally in both arches
C. are wider mesiodistally in the maxilla, and wider buccolingually in the mandible
D. are wider mesiodistally in the mandible and wider mesiodistally in the maxilla




The correct answer is C. This is known as an arch trait, as it is true for both incisors of each arch. Both incisors of the maxilla are wider mesiodistally. Both incisors of the mandible are wider buccolingually. The maxillary central, in particular, is known for the greatest assymetry in this regard. It is much wider mesiodistally than buccolingually, and that difference, expressed as a ratio, is greatest for that incisor. It is also the largest incisor, in both dimensions, in absolute size.
Which of the following will cause reduction of the size of the pulp chamber in mid-life?
A. Formation of primary dentin
B. Formation of secondary dentin
C. Internal resorption
D. External resorption
E. Pulp fibrosis




The correct answer is B. Primary dentin (choice A) surrounds the pulp chamber when the tooth is first formed and erupts. It is the initial dentin that makes up the base of both the crown and root. If the young tooth has a given size pulp chamber, further dentin formation, secondary dentin, later in life, will begin to fill the chamber with dentin, making it smaller. Internal resorption (choice C) is loss of dentin, resulting in a larger pulp chamber. External resorption (choice D) involves the exterior of the root and does not change the pulp chamber size. Pulp fibrosis (choice E) would change the histology of the cells in the chamber, but not the chamber's size.
The largest root of the maxillary first molar is the:
A. mesiobuccal
B. distobuccal
C. palatal
D. varies depending on the individual tooth




The correct answer is C. The palatal or lingual root of the maxillary first molar is the most massive by far. It is normally straight but palatally inclined, giving the three roots a tripod-like appearance. The mesiobuccal root (choice A) is second in size, and the distobuccal (choice B) is smallest. The mesiobuccal is often curved distally and the distobuccal curved mesially. This pliers-like appearance is not found in the maxillary second and third molars.
Which is the first primary tooth to erupt?
A. Mandibular central
B. Mandibular lateral
C. Maxillary central
D. Maxillary lateral




The correct answer is A. If you see enough small children in your practice, you'll often notice that a 6-month-old child will have only two teeth: the two mandibular centrals. These are commonly the first teeth to erupt, and most commonly do so at 6 months. Mandibular laterals are often seen at about 9 months. Maxillary centrals and maxillary laterals average about 8 months. First molars then precede canines in both arches, and second molars are last in both arches. The last primary is usually the maxillary second molar at about 24 months. These times may vary considerably. Know the most common order: central, lateral, first molar, canine, second molar.
In performing a left working movement:
A. Only the left lateral pterygoid contracts
B. Only the right lateral pteyrgoid contracts
C. Both lateral pterygoids contract
D. Neither lateral pterygoid contracts




The correct answer is B. In a left working movement, the mandible moves toward the left. The left side is then known as the working side and the right side as the nonworking (balancing) side. The lateral pterygoids pull the mandibular condyles forward, so the pull of both together results in protrusion. For only a left-sided movement, the left lateral pterygoid does not contract and the left condyle stays in a relatively unchanging position (it does rotate slightly). The right lateral pterygoid contracts and pulls the right side of the mandible outward, and then it turns left, as the left side of the mandible is stationary. Remember that the right lateral pterygoid moves the mandible left, and the left lateral pterygoid moves the mandible right.
On which premolar can you see more of the occlusal table from the mesial than from the distal?
A. Maxillary first
B. Maxillary second
C. Mandibular first
D. Mandibular second




The correct answer is C. The mandibular first premolar is the exception to premolar form in many ways. It is canine-like with a reduced lingual cusp, resembling a cingulum. Its occlusal table tilts downward from buccal to lingual in a slanted fashion. It is also slightly higher on the distal side than the mesial, and its mesial marginal ridge is less prominent than its distal marginal ridge. This means that in a mesial view you can see some of the occlusal table because it slants toward you. From the distal view, this is not true. The higher distal end blocks any occlusal view from that direction.
At the cervical line, a cross section of the mandibular canine would show:
A. a round shape
B. an oval shape wider on the lingual than the labial
C. a triangular shape with a distinctly wider labial surface
D. an oval shape with a slightly wider labial surface




The correct answer is D. The mandibular and maxillary canines exhibit similar cross sectional shapes at the cervical line. The mandibular canine cross section is oval and flattened mesiodistally. It is generally slightly wider at the labial end than at the lingual. The pulp cavity at this point is lens-shaped, double convex.
In a mandibular first molar, which pulp horn is likely to be smallest?
A. Mesiobuccal
B. Mesiolingual
C. Distobuccal
D. Distolingual
E. Distal




The correct answer is E. There are two facts or concepts involved in correctly answering this question. The first is that the mandibular first molar is a five-cusped tooth, named corresponding to the cusps listed in the answer choices. The second is that the pulpal anatomy reflects the cuspal anatomy (or vice versa). Tall, large cusps are likely to have tall, large pulp horns. Small cusps generally have small associated pulp horns. In this case, the distal cusp is generally the smallest of the cusps of the mandibular first molar, so its pulp horn is likely to be smallest as well.
The premolar most likely to have a three-cusped appearance when viewed from the occlusal is the:
A. maxillary first
B. maxillary second
C. mandibular first
D. mandibular second




The correct answer is D. The premolars are usually considered to be two-cusped teeth (bicuspids). The exception to the rule, to a small extent, is the mandibular second premolar. Often, the lingual cusp area is divided into a mesiolingual cusp and a distolingual cusp. This gives the tooth a three-cusped appearance. A two-cusped variety also exists, with a large lingual cusp instead of the two smaller ones. In the three-cusped variety, a lingual groove separates the two lingual cusps. Note that the two-cusped and three-cusped varieties are not grossly different in size, as the single lingual cusp is about the same size as a mesioloingual and distolingual combined. The other premolars exist primarily in double-cusped varieties only.
The distolingual cusp of the mandibular first molar contacts:
A. the lingual embrasure between the maxillary first molar and second premolar
B. the lingual groove of the maxillary second molar
C. the central fossa of the maxillary first molar
D. the lingual groove of the maxillary first molar
E. the lingual embrasure between the maxillary first molar and second molar




The correct answer is D. Lingual cusps of mandibular teeth are guiding cusps, not holding cusps. Therefore, they do not occlude on marginal ridges or central fossae. The general rule for mandibular lingual cusps is that they occlude in the lingual embrasures between their maxillary counterparts and the teeth mesial to their counterparts, EXCEPT for the distolingual cusps of the mandibular molars, which occlude in the lingual grooves of their maxillary counterparts. In this question we are dealing with the exception of the distolingual cusps of mandibular molars, so in this case, the mandibular first molar distolingual cusp contacts the lingual groove of its counterpart (the maxillary first molar).
Which of the premolars is most likely to have a pronounced mesial marginal groove extending into a pronounced mesial root concavity?
A. Maxillary first
B. Maxillary second
C. Mandibular first
D. Mandibular second




The correct answer is A. The mesial marginal groove, extending from the marginal ridge, along the mesial side of the crown into the proximal area (mesial concavity) and down into the mesial root concavity, is a characteristic of many maxillary first premolars. This has implications for both calculus removal (scaling and root planing) and restoration (placement of well adapted matrix bands). It is not found in the other premolars, although they may sometimes have some less pronounced root concavities. Root concavities are less common and less pronounced in the mandibular premolars.
At age 6 1/2, an average child is MOST likely to have how many erupted permanent teeth? (Assume that you average the teeth present in a sample of 100 children).
A. 4-8
B. 9-12
C. 13-16
D. 17-20
E. 21-24




The correct answer is A. This can vary considerably, but using average eruptions, all permanent first molars usually erupt at around age 6 to 7, so four first molars are likely. Rule out all premolars, which generally erupt between age 9 and 12. Likewise, canines usually erupt between age 9 and 12. For the incisors, usually the mandibular centrals are in at about age 6, so that makes eight so far. Mandibular laterals and maxillary centrals and laterals usually erupt between 7 and 9 years. So the average 6-year-old has all first molars and mandibular centrals as erupted permanent teeth.
The major function of contact points in the dentition is to:
A. protect the incisal surface
B. protect the periodontium
C. protect the alveolar mucosa
D. protect restorations from poor retention
E. none of the above




The correct answer is B. Contacts ensure that food does not get lodged between teeth and into the sulcus. Food, debris, and plaque in that area will lead to inflammation, tissue destruction, and bone loss. Contacts do not protect the incisal surface, as they are below (cervical to) that area. They do protect the gingiva (see above), but not especially the alveolar mucosa, which is more cervical and basically below the tooth region. The alveolar mucosa begins at the mucogingival junction. Contact is not involved in restoration retention. All restorations must be retained by retention within the individual tooth itself.
The maxillary canine has:
A. a mesial marginal ridge only
B. a distal marginal ridge only
C. neither a mesial nor distal marginal ridge
D. both a mesial and distal marginal ridge




The correct answer is D. We often do not think of anterior teeth as having marginal ridges, as they are less distinct than on the posterior teeth; and in the posterior teeth, they are often points of occlusal contact. However, the maxillary canine, as well as other anterior teeth, does have both a mesial and distal marginal ridge. The lingual anatomy of the maxillary canine has a cusp tip, with mesial and distal marginal ridges sloping down from it. Centrally, a lingual ridge splits the lingual surface in two. In between the marginal ridge and the lingual ridge, we find two depressions, the mesiolingual fossa and the distolingual fossa.
As the tooth bud develops into a tooth, the ameloblasts and odontoblasts:
A. become farther apart
B. become closer together
C. stay approximately the same distance apart
D. are always immediately next to each other




The correct answer is A. Early in tooth bud formation, the tissues of the enamel organ-outer enamel epithelium, stellate reticulum, stratum intermedium, and inner enamel epithelium-form adjacent to the dentin-forming tissues of the dental papilla. In fact, at this stage, the ameloblasts and future odontoblasts are in contact along the length of the tooth bud. The dentin production begins first, and predentin is laid down by odontoblasts. This predentin production induces the ameloblasts to begin enamel production by cells of the inner enamel epithelium. Thus, dentin is produced above the odontoblasts while enamel is formed below the inner enamel epithelium. Both dentin and enamel are deposited between these two layers, and the former site of the junction of the two layers becomes the DEJ, or dento-enamel junction. By tooth completion, the enamel organ is at the surface of the gingiva while the odontoblasts are producing dentin for the tooth root. Thus, these cell layers are very far apart at this point.
In extraction of maxillary central incisors, the root anatomy guides the dentist to use a motion that is primarily:
A. buccal-lingual luxation
B. mesio-distal luxation
C. rotation
D. all of the above




The correct answer is C. One of the best reasons to know standard and unusual root shapes for the various maxillary and mandibular teeth is to use the correct motion on the forceps when performing extractions. In general, rounded roots are primarily rotated, including the maxillary central incisors and maxillary canines. All double and triple rooted teeth cannot be rotated and must be luxated in the buccal and lingual directions. Mesial-distal motion is normally ruled out because of the existence of adjacent teeth. It is used in rare cases where there is no mesial and distal adjacent tooth. The maxillary central is invariably single rooted with a rounded root, and can almost always be rotated without breaking the crown off of the root.
The most common number of mammelons on an incisor is:
A. 1
B. 2
C. 3
D. 4




The correct answer is C. Mammelons are bumps on the incisal edge of incisors. They are most commonly found on central incisors of both arches, and if found on the laterals, they are less prominent than those found on the centrals. There are normally three, but this is variable. The three are termed mesial, central, and distal. They are often worn off on adult teeth through years of occlusion.
As you go from mandibular first to second to third molars, overall root length:
A. increases, then decreases
B. increases
C. decreases
D. decreases, then increases




The correct answer is C. While the mandibular third molar is often so variable that it is hard to make useful predictions about it, these generalities apply to mandibular first, second, and third molars as you move distally in the arch: 1) they decrease in mesiodistal length, and 2) their roots decrease in length. Third-molar roots may resemble normal molars or may be missing roots or have additional roots. Roots are often partially or totally fused. In some cases they form a cone-like, fused mass, making exodontia much simpler in these cases. In all cases, however, their roots are shorter than those of their second-molar counterparts, which are always smaller than those of the first molars.
Incisal contact between the maxillary and mandibular incisors is seen normally:
A. in centric occlusion
B. in working movements
C. in protrusive movements
D. in retrusive movements
E. never




The correct answer is C. Posterior contact is normal during centric occlusion and maintains the vertical dimension of occlusion. In a normal dentition, however, the incisors are slightly out of contact during centric occlusion. When the mandible moves forward (protrusion), the incisal edges of the mandibular anteriors meet the lingual surfaces of their maxillary counterparts as the teeth separate during opening. (Try it in your mouth!). Note that during retrusion (moving the mandible from centric occlusion to a more distal position), no such contact occurs. During working (lateral) movements, only posterior teeth and canines contact (group function), or canines contact alone (canine/cuspid protected guidance).
Severing the left lateral pterygoid will cause the mandible to move in what direction if the patient attempts to protrude?
A. Posteriorly
B. Closure (elevation)
C. Right
D. Left
E. Downward (depression)




The correct answer is D. If you like rules, the rule is that if a lateral pterygoid is injured, the mandible moves toward the injured side. If you wish to understand this rule better, imagine two intact lateral pterygoids protruding the mandible normally. Now cut the left lateral pterygoid, and imagine that the left side stays in place but the right side continues to protrude. In effect, the mandible will rotate around the axis of the left side. The left side stays in place while the right side rotates left. Lateral pterygoids have little effect on closing.
Which characteristic is NOT typical of lining mucosa?
A. Nonkeratinized
B. Lines the surface of the hard palate
C. Freely moveable
D. Contains a well developed submucosa
E. Epithelium thin




The correct answer is B. Lining mucosa covers most of the oral cavity except for the hard palate and gingival areas. It is usually loose, freely movable, and has a well-developed submucosa. The lamina propria is often highly glandular. The epithelium is thin and nonkeratinized. Epithelial ridges and rete pegs are uncommon and short.
Which ligament of the temporomandibular joint merges with the fibers of the articular capsule?
A. Lateral
B. Stylomandibular
C. Sphenomandibular
D. None of the above




The correct answer is A. The lateral ligament, also known as the temporomandibular ligament, is found extending downward from the lateral surface of the zygoma. The zygoma is formed from the contact of the zygomatic section of the temporal bone and the temporal portion of the zygomatic bone. The ligament is roughly triangular in shape, and on its medial side is found the articular capsule and joint. Fibrous connective tissue composes both the ligament and capsule, and there is a fibrous connection between them.
The central portion of the articular disc of the TMJ is best described as:
A. convex and avascular
B. convex and vascular
C. concave and avascular
D. concave and vascular




The correct answer is C. The articular disc of the TMJ is composed of fibrous connective tissue. Some sources state that chondrocytes are found within it, so it is sometimes referred to as fibrous connective tissue with an underlying base of fibrocartilage. Its shape is biconcave, somewhat like that of an erythrocyte, with a raised outer portion and depressed middle. The central portion is avascular, and only the periphery, especially the posterior periphery is vascularized.
The tissue located between the inner and outer enamel epithelium is known as:
A. stellate reticulum
B. dental sac
C. dental papilla
D. enamel organ




The correct answer is A. The stellate reticulum is an ectodermal tissue located between the inner and outer enamel epithelium. The stellate, or star-like, appearance of the tissue is caused by the arrangement of few cells within a network, separated by intracellular fluid. The inner enamel epithelium will differentiate into ameloblasts and form enamel. The outer enamel epithelium eventually merges with the gingiva as the tooth erupts. A small cell layer between the inner enamel epithelium and stellate reticulum is referred to as the stratum intermedium. Its presence is necessary for proper ameloblast function. The dental sac is mesodermal and forms cementum and the periodontal ligament. The dental papilla is mesodermal and forms pulp and dentin. The enamel organ consists of inner and outer enamel epithelium, stellate reticulum and stratum intermedium and forms enamel. It is ectodermal in origin.
Which ligament of the temporomandibular joint inserts into the ramus and angle of the mandible?
A. Temporomandibular
B. Stylomandibular
C. Sphenomandibular
D. Lateral




The correct answer is B. The origins and insertions of the TMJ ligaments are as follows: The stylomandibular ligament arises from the spine of the temporal bone known as the styloid process and inserts on the lower ramus and angle of the mandible. The sphenomandibular ligament arises from a spine on the sphenoid bone and runs forward and downward to insert on the lingula and deep ramus of the mandible. The lateral ligament is also known as the temporomandibular ligament. It descends from the lower border and tubercle of the zygoma to the posterior lateral condyle. Its fibers merge with those of the articular capsule.
The premolar with a root cross section and pulp chamber shape at the cervical line which is most similar to that of a canine is the:
A. mandibular first
B. mandibular second
C. maxillary first
D. maxillary second




The correct answer is A. You could answer this on general principal, that the mandibular first premolar is the most canine-like premolar tooth in almost every way. However, in detail, the mandibular first premolar has a round root cross section and a roundish pulp chamber at the cervix. The second premolar has a generally more flattened shape in the mesiodistal direction and a correspondingly oval pulp chamber. It may be constricted at the center with a corresponding hourglass shape. The maxillary first premolar root is flatttened mesiodistally, as is the pulp chamber. It is usually constricted at the center, indicating the two roots and two canals found in the tooth. The second maxilllary premolar, with one root, is less likely to be constricted into an hourglass shape.
The mucosa found on the surface of the dorsum of the tongue is known as:
A. lining mucosa
B. masticatory mucosa
C. specialized mucosa
D. none of the above




The correct answer is C. Specialized mucosa is found on the dorsum of the tongue and contains taste buds and various papillae, including circumvallate, foliate, fungiform, and filiform. Masticatory mucosa consists of the gingiva and covering of the hard palate. Most of the other areas of the oral cavity are lined by lining mucosa. The characteristics of masticatory mucosa are keratinization, thin or absent submucosa, and tight binding of the lamina propria to underlying periosteum. Lining mucosa is generally nonkeratinized, with a thin lamina propria. Near the teeth, the gingiva and lining mucosa (alveolar mucosa) meet at the mucogingival junction.
The mucosa found on the surface of the dorsum of the tongue is known as:
A. lining mucosa
B. masticatory mucosa
C. specialized mucosa
D. none of the above




The correct answer is C. Specialized mucosa is found on the dorsum of the tongue and contains taste buds and various papillae, including circumvallate, foliate, fungiform, and filiform. Masticatory mucosa consists of the gingiva and covering of the hard palate. Most of the other areas of the oral cavity are lined by lining mucosa. The characteristics of masticatory mucosa are keratinization, thin or absent submucosa, and tight binding of the lamina propria to underlying periosteum. Lining mucosa is generally nonkeratinized, with a thin lamina propria. Near the teeth, the gingiva and lining mucosa (alveolar mucosa) meet at the mucogingival junction.
The mucosa found on the surface of the dorsum of the tongue is known as:
A. lining mucosa
B. masticatory mucosa
C. specialized mucosa
D. none of the above




The correct answer is C. Specialized mucosa is found on the dorsum of the tongue and contains taste buds and various papillae, including circumvallate, foliate, fungiform, and filiform. Masticatory mucosa consists of the gingiva and covering of the hard palate. Most of the other areas of the oral cavity are lined by lining mucosa. The characteristics of masticatory mucosa are keratinization, thin or absent submucosa, and tight binding of the lamina propria to underlying periosteum. Lining mucosa is generally nonkeratinized, with a thin lamina propria. Near the teeth, the gingiva and lining mucosa (alveolar mucosa) meet at the mucogingival junction.
Extreme bruxism over time may cause masseteric hypertrophy. It can be noted on the face as:
A. Swelling in the area of the temporal fossa
B. Swelling in the lateral mandibular ramus area
C. Swelling on the medial surface of the jaw
D. Loss of muscle substance near the ramus
E. Loss of muscle mass above and below the lips




The correct answer is B. Masseteric hypertrophy means gain in muscle substance of the masseter muscle. The masseter is located on the lateral part of the ramus and angle, and its growth in thickness can be seen as a squareness of the lower face on frontal view. People who brux (express tension by clenching their teeth) often develop this larger-than-normal size masseter. It can sometimes be confused with parotid swelling, although the parotids are slightly higher in location. Remember that the medial pterygoid is medial, not lateral, to the ramus. Loss of muscle substance would be atrophy rather than hypertrophy.
The mesiobuccal cusp of the mandibular first molar occludes with which maxillary tooth surfaces?
A. The mesial marginal ridge of the first molar and distal marginal ridge of the second premolar
B. The distal marginal ridge of the first molar and mesial marginal ridge of the second molar
C. The embrasure between the first and second molars
D. The distal triangular fossa of the first molar




The correct answer is A. The mesiobuccal cusp of a mandibular molar is a holding (supporting) cusp. The general rule of occlusion of mandibular holding cusps is as follows: the holding cusps of the mandibular teeth occlude on the mesial marginal ridge of their maxillary counterpart, and the distal marginal ridge of the maxillary tooth mesial to their counterpart, EXCEPT DISTOBUCCAL CUSPS OF MANDIBULAR MOLARS OCCLUDE WITH CENTRAL FOSSAE OF THEIR COUNTERPARTS, THE DISTAL CUSP OF THE MANDIBULAR FIRST MOLAR OCCLUDES WITH THE DISTAL TRIANGULAR FOSSA OF ITS COUNTERPART, AND THE FIRST PREMOLAR OCCLUDES ONLY WITH THE MESIAL MARGINAL RIDGE OF ITS COUNTERPART (BUT NOT THE CANINE). In this case, the maxillary counterpart is the maxillary first molar, and the tooth mesial to it is the second premolar.
Which is the most common supernumerary tooth?
A. Mesiodens
B. Second lateral incisor
C. Third mandibular
D. Fourth maxillary molar




The correct answer is A. A mesiodens is a supernumerary located midline in the maxilla. It has a wide variety in morphology and is usually extracted. It is the most common and frequently causes a diastema between the central incisors. The other relatively common supernumeraries are the second maxillary lateral incisor, third mandibular premolar, and fourth maxillary molar.
Which cusp of the maxillary molar is NOT part of the trigon (primitive cusp triangle)?
A. Mesiolingual
B. Mesiobuccal
C. Distolingual
D. Distobuccal




The correct answer is C. In evolution, three cusps of the maxillary molar are considered to be the primary cusp triangle, and the original three cusps of the evolving molar. They form the trigon, and are the MB, ML, and DB cusps. The fourth cusp, the DL, is the smallest, is considered to be a minor cusp, and is known evolutionarily as the talon, or talon cusp. This cusp is less conspicuous as you go from first to second to third maxillary molar.
Which factor does NOT affect pulp cavity size in a tooth?
A. Caries
B. Age of tooth
C. Action of dentin-forming ameloblasts
D. Abrasion




The correct answer is C. The answer depends only on the fact that ameloblasts form enamel, not dentin. Clearly, odontoblasts, as dentin formers, influence pulp size because the pulp is surrounded by dentin. Caries may cause deposition of reparative or secondary dentin. The same is true for other factors, such as abrasion, heavy occlusal wear, erosion, etc. Pulp tends to become smaller and more calcified with age, so age is also a factor. Be careful when reading questions on your test! Occasionally you will find a careful-reading question like this one.
A rule regarding congenitally missing teeth is:
A. A mandibular tooth is more likely to be missing than its maxillary counterpart
B. A distal tooth of a type is more likely to be missing than the mesial tooth of the same type
C. A mesial tooth of a type is more likely to be missing than a distal tooth of the same type
D. None of the above is true




The correct answer is B. In any set of teeth (incisor, premolar, molar), the distal tooth is more likely to be congenitally missing than the mesial. So lateral incisors are missing more than centrals, second premolars are missing more than first premolars, and third molars are more commonly missing than first or second. Of all of these, the MOST commonly missing is the maxillary lateral incisor. Note that choice A is therefore incorrect.
An unvaccinated dental student has a needle-stick accident involving an empty syringe that had been previously used on a patient with a known hepatitis B infection. Which of the following is the most probable outcome for the medical worker?
A. Acute hepatitis followed by recovery
B. "Healthy" carrier
C. Persistent infection followed by recovery
D. Persistent infection progressing to chronic hepatitis
E. Subclinical disease followed by recovery




The correct answer is E. Hepatitis B infection can produce a wide variety of clinical outcomes. The most common outcome (60% to 65%), however, turns out to be subclinical disease followed by complete recovery. The other choices listed show other possible outcomes, and their statistical impact.

Approximately 20% to 25% of infected persons develop acute hepatitis (choice A), which is followed in 99% of these cases by recovery and in about 1% of cases by fulminant hepatitis.

Approximately 5% to 10% of cases become "healthy" carriers (choice B).

Approximately 4% of cases develop persistent infection, 67% to 90% of which then recover (choice C) and 10% to 33% of which have chronic hepatitis (choice D).
Biopsy of an ulcerated gastric lesion on a 60-year-old smoker demonstrates the presence of a gastric carcinoma. If the patient noted that he had severe "heart burn" for the past several years, with which of the following infectious agents has this type of lesion been most strongly associated?
A. Epstein-Barr virus
B. Helicobacter pylori
C. Human papilloma virus
D. Molluscum contagiosum virus
E. Schistosoma haematobium




The correct answer is B. The patient has gastric carcinoma, which has been strongly linked, in at least some studies, to prior gastric infection with Helicobacter pylori. H. pylori has also been implicated in the etiologies of gastric peptic ulcer, chronic gastritis, and (questionably) gastric lymphoma. It is believed that treatment of the H. pylori infection with a combination antibiotic treatment regimen, such as amoxicillin + metronidazole + proton pump inhibitor + bismuth subsalicylate will decrease the incidence of this type of carcinoma.

Epstein-Barr virus (choice A) has been linked to African Burkitt lymphoma and nasopharyngeal carcinoma.

Human papilloma virus (choice C) has been linked to a variety of warts, condyloma, and genital cancers.

Molluscum contagiosum virus (choice D) is a poxvirus that causes small tumor-like papules of the skin.

Schistosoma haematobium(choice E) has been linked to bladder cancer.
The biological attribute of influenza A virus, which allows the sudden appearance of dramatically new genetic variants, is also present in a limited number of other viral families. Which of the following viruses also possesses this biological attribute?
A. Coronavirus
B. HIV
C. Measles virus
D. Rotavirus
E. Rubella virus




The correct answer is D. Pandemics of influenza A can be caused by the ability of the virus to undergo dramatic genetic changes of type by reassortment of its segmented RNA genome, a trait called genetic shift. The only virus on the list that possesses a segmented genome is the rotavirus, in the reovirus family, which possesses 10-11 segments in its genome.

Coronavirus (choice A) is not segmented and is a cause of the common cold.

HIV (choice B) is not segmented and is known for its genetic drift (minor mutational changes over time caused by an error-prone polymerase), not genetic shift.

Measles virus (choice C) is not segmented and is controlled largely by vaccination. The virus is known as a paramyxovirus.

Rubella virus (choice E) is not segmented. The togavirus causes arthralgia (primarily in young women), fever, malaise, coryza, lymphadenopathy, and a fine maculopapular rash.
Three months after a needle-stick exposure to blood from a patient with hepatitis B, a nurse is evaluated for infection with the virus. Laboratory results reveal:

HBsAg absent anti-HBs antibody absent IgM anti-HBc present IgG anti-HBc absent HBeAg absent

On the basis of these results, which of the following most accurately describes the nurse's hepatitis B status?
A. She had been effectively vaccinated against hepatitis B before the needle-stick exposure occurred
B. She has mounted an inappropriate antibody response to hepatitis B as a result of an immunocompromised state
C. She is a carrier of hepatitis B
D. She is actively infected with hepatitis B
E. She was not infected with hepatitis B




The correct answer is choice D. The nurse's elevated IgM anti-HBc indicates that she was infected with hepatitis B. Formerly, HBsAg (surface antigen) and anti-HBsAg (antibody to surface antigen) were used exclusively to determine this. Typically, HBsAg is positive for up to 6 months, and anti HBsAg is positive for years after that. Unfortunately, this simple scheme has the disadvantage that many patients have a 2-week to 4-month "window" period, when the surface antigen (HBsAg) and the antibody (anti-HBs) are not detectable. Presumably, for a relatively brief period, HBsAg production exactly matches antibody production, and the two coprecipitate such that neither free species is present in adequate concentration to be detectable. This problem can be circumvented by concurrent measurements of other antigens and antibodies, including HBeAg, anti HBe, and anti-HBc (HBcAg is not reliable). During the window period, IgM anti-HBc may be the only marker of recent HBV infection, as it is in this nurse.

If the nurse had been effectively vaccinated for hepatitis B (choice A), she would have had an elevated anti-HBs antibody level and no HBsAg present in the serum. Anti-HBc antibody would have been absent as well.

The antibody response to hepatitis B infection was appropriate in this person, which argues against immunocompromise (choice B).

Carriers (choice C) have elevated anti-HBs and may have persistently elevated HBsAg (in approximately 10% of cases). IgG anti-HBc (not IgM) predominates in these chronic patients.

Had she not been infected with hepatitis B at all (choice E), IgM anti-HBc would be absent.

A. antigen) were used exclusively to determine this. Typically, HBsAg is positive for up to 6 months, and anti HBsAg is positive for years after that. Unfortunately, this simple scheme has the disadvantage that many patients have a 2-week to 4-month "window" period, when the surface antigen (HBsAg) and the antibody (anti-HBs) are not detectable. Presumably, for a relatively brief period, HBsAg production exactly matches antibody production, and the two coprecipitate such that neither free species is present in adequate concentration to be detectable. This problem can be circumvented by concurrent measurements of other antigens and antibodies, including HBeAg, anti HBe, and anti-HBc (HBcAg is not reliable). During the window period, IgM anti-HBc may be the only marker of recent HBV infection, as it is in this nurse.

If the nurse had been effectively vaccinated for hepatitis B (choice A), she would have had an elevated anti-HBs antibody level and no HBsAg present in the serum. Anti-HBc antibody would have been absent as well.

The antibody response to hepatitis B infection was appropriate in this person, which argues against immunocompromise (choice B).

Carriers (choice C) have elevated anti-HBs and may have persistently elevated HBsAg (in approximately 10% of cases). IgG anti-HBc (not IgM) predominates in these chronic patients.

Had she not been infected with hepatitis B at all (choice E), IgM anti-HBc would be absent.
An important factor in determining the likelihood of converting to HIV+ status after a needlestick injury is which of the following?
A. Depth of the needlestick
B. Volume of blood in the needle
C. Type of needle
D. Health status of source patient
E. All of the above




The correct answer is E. If a healthcare worker is accidently stuck with an HIV-infected bloody sharp, the worker may or may not contract HIV disease. Conversion following needlestick accidents is low overall, sometimes estimated at 0.3% (3 per thousand). Several factors, however, have been identified that make it MORE likely that the accident will transfer the virus to the worker. They include a large volume of blood, a hollow-bore needle, visible blood on the needle, a deep (not superficial) stick into deep skin or muscle, and a source patient in late stages of AIDS. Followup of needlestick injuries is a rapidly evolving field, so be sure to use your most recent notes and test reviews to stay on top of this subject.
A 16-year-old boy with sickle cell disease is hospitalized for a severe infection. His symptoms include fever, chills, cough, and chest pain. Bacteria from the patient's sputum yield optochin-sensitive organisms with a positive Quellung reaction. Which of the following is the most likely pathogen?
A. Escherichia coli
B. Haemophilus influenzae
C. Klebsiella pneumoniae
D. Neisseria gonorrhoeae
E. Streptococcus pneumoniae




The correct answer is E. The combination of optochin sensitivity and positive Quellung reaction is characteristic of a single organism, Streptococcus pneumoniae (diplococcus). The Quellung reaction is a capsular swelling caused by contact with specific capsular antisera. The other encapsulated organisms that have Quellung-positive reactions are Haemophilus influenzae (choice B), Neisseria meningitidis, and Klebsiella pneumoniae (choice C). None of these organisms, however, are optochin sensitive.

The other choices, Escherichia coli (choice A) and Neisseria gonorrhoeae (choice D), are not encapsulated.
A 33-year-old woman has recently returned from overseas and presents with severe, acute, right upper quadrant abdominal pain, bloody diarrhea, and tenesmus. CT scan of the liver demonstrates lesions identified as abscesses. Which of the following organisms is the most likely cause of her illness?
A. Ascaris lumbricoides
B. Entamoeba histolytica
C. Enterobius vermicularis
D. Salmonella typhi
E. Shigella species




The correct answer is B. The patient probably has hepatic amebiasis, a life-threatening complication of intestinal infection with Entamoeba histolytica. E. histolytica is transmitted by way of the fecal-oral route. Intestinal colonization (which may be asymptomatic) always precedes infection of the liver. The abscesses generally contain necrotic debris, with amoebae located along the edges of the abscess. Patients often present initially with frequent bloody, small-volume stools, often associated with fever, abdominal cramps, tenesmus, and fecal urgency. When untreated, this condition can result in the development of hepatic amebiasis.

Ascaris lumbricoides(choice A) can cause intestinal obstruction, but does not usually cause hepatic abscesses.

Enterobius vermicularis(choice C) is the pinworm, which inhabits the rectum.

Bloody diarrhea can also be seen with some strains of Salmonella (choice D) and Shigella (choice E), but these organisms do not usually cause hepatic abscesses. Bacterial causes of hepatic abscesses include E. coli, Klebsiella, Streptococcus, Staphylococcus, Bacteroides, and Pseudomonas.
A 65-year-old man presents with fever, severe headache, and nuchal rigidity. Lumbar puncture reveals cloudy cerebrospinal fluid (CSF) with elevated neutophils, elevated protein, and decreased glucose. Which of the following is the most probable etiologic agent of this condition in a patient of this age?
A. Arbovirus
B. Herpesvirus
C. Mycobacterium tuberculosis
D. Streptococcus pneumoniae
E. Neisseria meningitidis




The correct answer is D. The clinical manifestations (fever, headache, and nuchal rigidity), along with the CSF findings (increased neutrophils, elevated protein, and reduced glucose), strongly indicate acute pyogenic (bacterial) meningitis as the underlying condition. Of the microorganisms listed, either Neisseria meningitidis or Streptococcus pneumoniae can cause this form of meningitis; however, Streptococcus pneumoniae is by far the most frequent organism causing acute meningitis in elderly patients.

Arboviruses and herpesviruses (choices A and B) can cause an encephalitis characterized by lymphocytic infiltration of the brain parenchyma and leptomeninges. In this case, CSF findings would include an increased number of lymphocytes and a normal glucose concentration, although the protein level in CSF would be increased.

Mycobacterium tuberculosis(choice C) may cause a chronic meningoencephalitis, with a prolonged clinical course. It is characterized pathologically by a dense granulomatous infiltrate of the base of the brain. Associated CSF findings include increased lymphocytes and normal or slightly decreased glucose.
A child has a history of recurrent infections with organisms having polysaccharide antigens (i.e., Streptococcus pneumoniae and Haemophilus influenzae). This susceptibility can be explained by a deficiency of
A. C3 nephritic factor
B. C5
C. IgG subclass 2
D. secretory IgA




The correct answer is C. IgG is the predominant antibody in the secondary immune response. IgG subclass 2 is directed against polysaccharide antigens and is involved in the host defense against encapsulated bacteria.

C3 nephritic factor (choice A) is an IgG autoantibody that binds to C3 convertase, making it resistant to inactivation. This leads to persistently low serum complement levels and is associated with Type II membranoproliferative glomerulonephritis.

C5 (choice B) is a component of the complement system. C5a is an anaphylatoxin that effects vasodilatation in acute inflammation. It is also chemotactic for neutrophils and monocytes and increases the expression of adhesion molecules. A deficiency of C5a would affect the acute inflammatory response against any microorganism or foreign substance.

Secretory IgA (choice D) is the immunoglobulin associated with mucous membranes. Selective IgA deficiency is the most common hereditary immunodeficiency. In this disorder, there is failure of the B cell to switch the heavy chain class from IgM to IgA. Patients have an increased incidence of sinopulmonary infections, diarrhea, allergies, and autoimmune diseases.
A sexually active 25-year-old man with epididymitis and orchitis demonstrates a prominent leukocytic infiltrate with numerous neutrophils on biopsy. Which of the following organisms is the most likely cause of this man's infection?
A. Escherichia coli
B. Mycobacterium tuberculosis
C. Neisseria gonorrhoeae
D. Pseudomonas sp.
E. Treponema pallidum




The correct answer is C. Acute epididymitis and orchitis with prominent neutrophils in a sexually active male are most likely caused by infection with Neisseria gonorrhoeae or Chlamydia trachomatis. N. gonorrhoeae can produce a nonspecific pattern of acute inflammation (nonspecific epididymitis and orchitis) or can be sufficiently severe as to cause frank abscesses within the epididymis.

Escherichia coli(choice A) is an important cause of nonspecific epididymitis and orchitis in children with congenital genitourinary abnormalities and in older men.

Mycobacterium tuberculosis(choice B) can cause tuberculosis of the epididymis and testes, characterized by granuloma formation.

Pseudomonas sp.(choice D) has been implicated as an important cause of nonspecific epididymitis and orchitis in older men.

Treponema pallidum(choice E), the causative agent of syphilis, can cause testicular involvement with (in later stages) gumma formation, endarteritis, or a prominent plasma cell infiltrate.
A 5-year-old child develops a febrile disease with cough, a blotchy rash, and cervical and axillary lymphadenopathy. Also noted is an erythematous, maculopapular rash behind the ears and along the hairline, involving the neck and, to a lesser extent, the trunk. Examination of this patient's oropharynx would likely reveal which of the following lesions?
A. Adherent thin, whitish patch on gingiva
B. Cold sores on the lips
C. Curdy white material overlying an erythematous base on the oral mucosa
D. Large shallow ulcers on the oral mucosa
E. Multiple small white spots on the buccal mucosa




The correct answer is E. The disease described is measles (rubeola), which has the typical presentation described in the question stem. Measles is caused by a Morbillivirus, an RNA virus belonging to the Paramyxovirus family. Koplik's spots, which are pathognomonic for measles, are small, bluish-white spots on the buccal mucosa in the early stages of measles. These lesions appear just before the onset of the characteristic rash which is a brick red irregular maculopapular rash that onsets 3-4 days after the onset of the prodrome (which can also involve the extremities) and fade as the rash develops. This is usually a prodrome of fever, coryza, cough, conjunctivitis, malaise, irritibility and photophobia as well as koplik's spots.

Leukoplakia is a premalignant condition characterized by adherent whitish patches on the gingiva (choice A) and other sites in the oral cavity. Histologically, they are similar to hyperkeratoses.

Cold sores of the lips (choice B) are due to infection with herpes viruses.

Candida infection (thrush) produces curdy white material loosely attached to an erythematous base (choice C). When removed there is typically a painful lesion that may bleed.

Aphthous ulcers are large shallow ulcers of the oral mucosa (choice D), commonly known as canker sores. No systemic involvement is seen.
A 38-year-old woman vacationing in Connecticut is bitten by a tick and develops chronic arthritis of the knee and hip joints and paralysis of the left facial muscles. A physical examination during the early stages of the disorder would most likely have revealed
A. aphthous ulcers in the mouth
B. erythema chronicum migrans
C. flaccid paralysis of limb flexors
D. purpuric lesions in a bathing trunk distribution
E. spastic paralysis of limb extensors




The correct answer is B. Lyme disease should be suspected in a patient who is bitten by a tick in the northeastern United States. Lyme disease was named after a township in eastern Connecticut where the disease was endemic. The disease is spread by way of a tick vector that transmits a spirochete that causes a systemic illness. Erythema chronicum migrans is usually the first sign of the illness. This is a large red patch on the buttocks or chest that slowly expands as the center blanches. Generally, patients also have constitutional symptoms, such as fever and chills, during this phase. Stiff neck may develop, along with other signs of meningeal irritation, because of an aseptic meningitis. Other neurologic complications of Lyme disease include Bell palsy caused by involvement of branches of the facial nerve. Arthritis is a prominent feature in approximately half the patients with Lyme disease. It tends to appear several months after the infection but may persist for several years. The course of the chronic arthritis shows exacerbations and remissions; the most commonly affected joints are the knees and hips. Cardiac abnormalities in Lyme disease include pericarditis and heart block.

Skin manifestations do not include aphthous ulcers (choice A).

Flaccid or spastic paralysis of limbs (choices C and E) does not accompany Lyme disease; neurologic involvement is generally limited to cranial nerves and meningitis.

Purpura (choice D) is associated with vasculitis and does not occur in Lyme disease.
A 31-year-old HIV-positive man develops a severe pneumonia. Lower respiratory tract secretions stained with methenamine silver stain demonstrate cup-shaped cysts with sharply outlined walls. Which of the following organisms is the most likely pathogen in this case?
A. Candida albicans
B. Giardia lamblia
C. Haemophilus influenzae
D. Pneumocystis carinii
E. Streptococcus pneumoniae




The correct answer is D. The organism described is Pneumocystis carinii, which is an opportunistic parasite that seems to be more closely related to fungi than to protozoa. Its cyst form, when stained with silver stains, has the distinctive appearance described in the question stem, and is typically found in frothy material that occupies the lumen of alveoli. Pneumocystis pneumonia is a common infection among AIDS patients, and is very uncommon in other clinical settings. Formerly, many AIDS patients died with Pneumocystis pneumonia, but the combination of early drug treatment (with trimethoprim/sulfamethoxazole or pentamidine) and prophylaxis (usually with trimethoprim/sulfamethoxazole) has decreased the number of fatal infections. In severe cases, Pneumocystis infection can sometimes be demonstrated in extrapulmonary sites.

Candida albicans(choice A) can infect the lung and stain with methenamine silver, but the respiratory tract secretions would probably include fungal hyphae and yeast forms.

Giardia lamblia (choice B) causes diarrhea, rather than pneumonia.

Haemophilus influenzae(choice C) and Streptococcus pneumoniae(choice E) are bacteria that would not stain with silver stains and are not especially common in HIV-positive patients.
A neonate is born in very poor condition, with a severe, generalized encephalitis. If fever is noted as well as temporal lobe involvement, which of the following viruses is the most likely pathogen in this setting?
A. Eastern equine encephalitis virus
B. Herpes simplex type II
C. Herpes zoster-varicella virus
D. Poliomyelitis virus
E. St. Louis encephalitis virus




The correct answer is B. Viral causes of neonatal encephalitis include three members of the herpes family of viruses: herpes simplex I, herpes simplex II, and cytomegalovirus. All three types can have devastating effects on the neonate, with extensive CNS damage leading to mental retardation, seizures, and focal neurologic problems. Acyclovir may be of some help in modifying these infections, but both treatment and prognosis remain very problematic. Herpes simplex encephalitis presents with nonspecific symptoms: a flu-like prodome followed by headache, fever, behavioral and speech disturbances. A distinguishable feature is a propensity to involve the temporal lobe with mass effect on imaging studies and temporal lobe seizure foci on EEG's.

Eastern equine encephalitis virus (choice A) and St. Louis encephalitis virus (choice E) are causes of epidemic encephalitis but are not the most likely cause of neonatal encephalitis.

Herpes zoster-varicella virus (choice C), unlike herpes simplex, is not usually a cause of neonatal encephalitis.

Poliomyelitis virus (choice D) is a gastrointestinally transmitted virus that is not usually encountered in neonates (or anyone else in the U.S. currently).
A 23-year-old woman with a history of sickle cell disease presents with fever and severe bone pain localized to her left tibia. An x-ray film reveals a lytic lesion, and blood cultures reveal infection. A bone culture grows gram-negative rods. Which of the following best describes the infecting organism?
A. It is comma-shaped and sensitive to acidic pH
B. It is a facultative intracellular parasite
C. It is motile and does not ferment lactose
D. It is motile and oxidase positive
E. It is a nonmotile facultative anaerobe




The correct answer is C. The presence of sickle cell disease in a question stem is usually a significant clue. This question tests the fact that patients with sickle cell anemia are more susceptible to osteomyelitis caused by Salmonella. The patient's fever, bone pain, and x-ray results indicate osteomyelitis. Note, however, that Staphylococcus aureus (gram-positive coccus) is the most common cause of osteomyelitis in sicklers and nonsicklers. If it has not been ruled out on bone culture, you should have looked for it in the answer choices. Notice that you were required to know more than just the organism's name; you needed to know its distinguishing features. Choice C describes Salmonella (a gram-negative rod) accurately. Salmonella exists in more than 1,800 serotypes and is known to contaminate poultry.

A comma-shaped organism that is sensitive to acidic pH (choice A) is Vibrio cholerae, a gram-negative rod that causes severe enterotoxin-induced diarrhea, with "rice water" stools and dehydration. The toxin acts by stimulating adenylyl cyclase to overproduce cAMP in the brush border of the small intestine.

A facultative intracellular parasite (choice B) is Legionella, a catalase-positive gram-negative rod. It contaminates air-conditioning cooling towers and causes Legionnaire disease (a type of pneumonia).

A motile and oxidase-positive organism (choice D) is Pseudomonas, a gram-negative rod with pili that sometimes produces a polysaccharide slime layer. P. aeruginosa is the prototype and commonly colonizes the lungs of patients with cystic fibrosis. It is associated with blue-green pus.

A nonmotile, facultative anaerobe (choice E) is Shigella, a gram-negative rod that does not produce H2S. All Shigella contain an endotoxic lipopolysaccharide. The organism causes bacillary dysentery, with abdominal cramps, fever, and mucoid, bloody diarrhea.
A patient suffers an infarct in the territory of the middle cerebral artery. Pathologic examination of the patient's brain would likely show
A. caseous necrosis
B. coagulative necrosis
C. enzymatic fat necrosis
D. gangrenous necrosis
E. liquefactive necrosis




The correct answer is E. Liquefactive necrosis occurs in brain or other neural tissues and in pancreatic tissue. In this type of necrosis, the tissue appears liquefied under the microscope, without preservation of cell outlines. Liquefactive necrosis can also be seen in some bacterial infections, especially those caused by pyogenic Staphylococci, Streptococci, or certain coliform bacteria.

Caseous necrosis (choice A) is generally an indication of infection by Mycobacterium tuberculosis. The term caseous refers to the appearance of the tissue, i.e., soft, white necrotic areas that have a cheese-like appearance. Microscopically, the necrotic areas are lightly eosinophilic (stain light pink), with little or no discernible cellular detail. The eosinophilia reflects staining of residual cellular proteins.

Coagulative necrosis (choice B) is a more common type of necrosis, characteristic of anoxic injury and most infarcts. Cellular outlines are preserved, but proteins are denatured, and the cells stain in an eosinophilic manner.

Enzymatic fat necrosis (choice C) is seen primarily with pancreatic injury when pancreatic lipases are released and digest fat to form free fatty acids. These fatty acids complex with calcium, resulting in the production of calcium soaps (saponification) in the pancreatic tissue or in extrapancreatic fatty tissues (eg, omentum).

In gangrene (choice D), bacterial infection is superimposed on a background of massive necrosis and putrefaction.
A 4-year-old girl presents with a maculopapular rash on her hands and feet and painful ulcers distributed anteriorly on her lips, palate, tongue, and buccal mucosa. Systemic features and lymphadenopathy are absent. Which of the following viruses is most likely to have caused this disorder?
A. Coronavirus
B. Coxsackievirus type A16
C. Herpes simplex virus type 1
D. Parainfluenza type 3
E. Rhinovirus




The correct answer is B. Hand-foot-and-mouth disease is characterized by the appearance of ulcers in the mouth and a maculopapular or vesicular rash on the hands and feet. It is most frequently caused by coxsackievirus type A16, although other coxsackieviruses have occasionally been implicated. The disease usually affects young children. Systemic features and lymphadenopathy are absent, and recovery is uneventful.

Coronavirus (choice A) is a cause of the common cold.

Herpes simplex virus type 1 (choice C) causes a variety of diseases, including gingivostomatitis, pharyngotonsillitis, herpes labialis, genital herpes, keratoconjunctivitis, and encephalitis.

Parainfluenza virus (choice D) is responsible for croup. Croup, or acute laryngotracheobronchitis, is an acute febrile illness with stridor, hoarseness, and cough.

Rhinovirus (choice E) is a member of Picornaviridae. It is the most common cause of the common cold.
Evaluation of an adult third world immigrant to this country demonstrates chronic headaches, chronic mild nuchal rigidity, and chronic inflammatory infiltrate of the CSF with lymphocytes, plasma cells, macrophages, and fibroblasts. If an acid-fast organism is identified, which of the following is the most likely etiologic agent?
A. Herpes virus
B. Mumps virus
C. Mycobacterium tuberculosis
D. Neisseria meningitidis
E. Streptococcus pneumoniae




The correct answer is C. Although this may seem to be a difficult clinical question, by simply knowing that mycobacteria are acid-fast organisms, one can answer the question. Nuchal rigidity suggests meningitis. It is convenient to classify meningitis based on the cerebrospinal fluid (CSF) findings: (1) acute pyogenic meningitis if neutrophils are markedly increased, (2) acute lymphocytic meningitis if lymphocytes (alone) are markedly increased, and (3) chronic meningitis if lymphocytes, plasma cells, macrophages, and fibroblasts are increased. This patient has chronic meningitis. The classic cause of chronic meningitis is tuberculosis, whose etiologic agent is Mycobacterium tuberculosis. Other causes include other indolent meningeal infections such as syphilis, brucellosis, and chronic fungal infections. The granulomas that are characteristic findings in other tissues may or may not be present in the meningeal tissue, and are usually not recognizable in CSF. Tubercular meningitis is now uncommon in this country. In immigrants from third world countries, a history of pulmonary tuberculosis may be distant, undiagnosed, or deliberately concealed.

Herpes virus (choice A) and mumps virus (choice B) are causes of acute lymphocytic meningitis. These organisms are not acid-fast.

Neisseria meningitidis(choice D) is a gram-negative bacteria. Streptococcus pneumoniae(choice E) is a gram-positive bacteria. Both are causes of acute pyogenic meningitis. Neither is acid-fast.
A 45-year-old white man with a history of alcohol abuse and periodontal disease has a spiking fever, chills, and lung consolidation. A chest x-ray shows a cavity in the right lower lobe that has an air/fluid level. Based on the clinical presentation, which of the following would be the most likely cause of the symptoms?
A. Anaerobic bacteria
B. Aspergillus fumigatus
C. Entamoeba histolytica
D. Staphylococcus aureus
E. Streptococcus pyogenes




The correct answer is A. Anaerobic bacteria derived from the oral flora in the clinical setting of periodontal disease are the most common isolates from lung abscesses. Single lung abscesses are the most common pattern, with the superior segment of a lower lobe or the posterior segment of an upper lobe being affected most often. Prominent members of the oral anaerobic flora include fusobacteria, prevotella, and aerobic spirochetes.

Aspergillus fumigatus(choice B) presents in the lung as hemorrhagic infarctions, aspergillomas (fungus balls) in cavitary tuberculosis cavities, or as allergic bronchopulmonary aspergillosis.

Entamoeba histolytica(choice C) is associated with pulmonary abscesses as an extension of a liver abscess across the diaphragm.

Staphylococcus aureus(choice D) usually presents as multiple lung lesions in noncontiguous sites because the spread is embolic. The source of the infection is usually tricuspid endocarditis in intravenous drug abusers.

Streptococcus pyogenes(choice E) typically produces a bronchopneumonia pattern following an upper respiratory infection.
A young mother takes her infant to the pediatrician, who notices the infant's teeth are yellow and brownish striped. The antibiotic this mother most likely took during pregnancy
A. inhibits aminoacyl-tRNA binding
B. inhibits peptidyl transferase
C. interferes with cell wall synthesis
D. is a large, cyclic lactone-ring structure




The correct answer is A. This question relates to an NDBE favorite side effect: the teeth mottling that occurs when a child is exposed to tetracycline in utero. You should remember that tetracycline is contraindicated in pregnancy and early childhood up to approximately age 8 years. Tetracycline is a bacteriostatic drug that binds to the 30s subunit of ribosomes, preventing aminoacyl-tRNA from binding with complementary mRNA. This inhibits peptide bond synthesis. Resistance is plasmid-mediated.

Inhibition of peptidyl transferase (choice B) occurs with chloramphenicol, a broad-spectrum bacteriostatic agent that binds to the 50s subunit of ribosomes. Resistance is plasmid-mediated. It has high toxicity (gastrointestinal disturbances, aplastic anemia, and gray baby syndrome), so it is used mainly in severe infections or as a topical agent.

Interference with cell wall synthesis (choice C) occurs with penicillins and cephalosporins, the beta-lactam antibiotics. Resistance to these drugs appears in organisms that have developed beta-lactamases (penicillinases), enzymes that destroy the beta-lactam ring of these medications. The wider spectrum ampicillin, amoxicillin, ticarcillin, and carbenicillin are particularly penicillinase susceptible.

Large, cyclic, lactone-ring structures (choice D) describe the macrolides: erythromycin, azithromycin, and clarithromycin. They inhibit bacterial protein synthesis by reacting with the 50s ribosomal subunit and preventing the release of the uncharged tRNA. Resistance is plasmid-mediated. Common side effects include gastrointestinal irritation, skin rashes, and eosinophilia. Erythromycin is a popular choice for patients with penicillin hypersensitivity. It is a cytochrome p450 inhibitor and therefore must be used with caution in patients taking other drugs.
A 38-year-old woman complains of cold and painful fingertips, as well as difficulty swallowing and indigestion. Physical examination shows a thickened, shiny epidermis over the entire body, with restricted movement of the extremities, particularly the fingers, which appear claw-like. Which of the following autoantibodies will likely be found in this patient's serum?
A. Anti-DNA topoisomerase I (anti-Scl-70)
B. Anti-double-stranded DNA (ds DNA)
C. Anti-IgG
D. Anti-Sm
E. Anti-SS-A




The correct answer is A. This patient has systemic sclerosis, also called scleroderma. Antibodies to topoisomerase I (anti-Scl-70) occur in up to 70% of patients with diffuse systemic sclerosis, but only rarely in other disorders. Systemic sclerosis is characterized intitially by excessive fibrosis and edema of the skin, especially the hands and fingers, producing sclerodactyly (characteristic changes in the fingers, which resemble claws). Raynaud phenomenon is common. The diffuse type of systemic sclerosis generally spreads to include visceral organs, such as the esophagus (producing dysphagia), the lungs (producing pulmonary fibrosis), the heart (leading to heart failure or arrhythmia), and the kidneys (renal failure causes 50% of scleroderma deaths). A more restricted variant of systemic sclerosis with a somewhat more benign course is CREST syndrome (Calcinosis, Raynaud syndrome, Esophageal dysmotility, Sclerodactyly, and Telangiectasia), characterized by the presence of anti-centromere antibodies (although 10% of CREST patients have antitopoisomerase antibody also).

Anti-ds DNA (choice B) and anti-Sm (Smith antigen; choice D) are characteristic of systemic lupus erythematosus (SLE) but are not common in patients with systemic sclerosis.

Rheumatoid factor is an autoantibody directed against IgG (choice C). It is found in patients with rheumatoid arthritis.

Anti-SS-A (choice E) is typically seen in Sjogren syndrome (although it may also be seen in SLE).
A young patient presents with bilateral swelling of the parotid gland, accompanied by swelling of one testicle. Which of the following is the most likely diagnosis?
A. Gonorrhea
B. Mumps
C. Nonspecific epididymitis and orchitis
D. Syphilis
E. Tuberculosis




The correct answer is B. Mumps, once a common childhood disease, is now much less common because of immunization. It still should be considered as a potential cause of parotitis in medically underserved populations, however. It is caused by the paramyxovirus and is spread by droplet infection. Although testicular involvement by mumps in school-aged children is extremely uncommon, postpubertal males who develop this viral infection have a 20-30% chance of subsequently developing acute interstitial orchitis, usually in one testis. Microscopically, the inflammatory reaction consists of lymphocytes, plasma cells, and macrophages. Although the process can be locally destructive, sterility does not usually develop because typically only one testicle is involved. The incubation period is 14-21 days before the onset of symptoms. Patients initially present with painful, swollen salivary glands, usually the parotid. In unvaccinated individuals, other tissues frequently become involved. These tissues include the testes, pancreas, and meninges.

Neglected gonorrhea infection (choice A) of the epididymis and testis occurs in the setting of previous sexual activity and would not usually cause parotitis.

Nonspecific epididymitis and orchitis (choice C) occur in children with genital urinary congenital abnormalities (gram-negative rods), in sexually active young adults (Chlamydia and gonorrhea), and in older men (gram-negative rods and Pseudomonas).

Syphilis (choice D) can cause orchitis with either gumma formation or a leukocytic infiltrate with prominent plasma cells.

Tuberculosis (choice E) can cause granulomatous involvement with acid-fast organisms of the epididymis and testis.
Which of the following cell surface markers is required for lysis of IgG-coated target cells (antibody-dependent, cell-mediated cytotoxicity, or ADCC) by natural killer cells?
A. CD3
B. CD16
C. CD19
D. CD21
E. CD56




The correct answer is B. CD16 is a cell surface marker used to identify natural killer (NK) cells (lymphocytes lacking most T- and B-cell markers). CD16 is an Fc receptor for IgG, allowing the NK cells to bind to the coated target cell during ADCC, facilitating lysis.

CD3 (choice A) is a five-polypeptide cluster that represents the nonvariable part of the T-cell receptor complex. The variable part is able to rearrange itself to adapt to specific antigens. NK cells are CD3-negative.

CD19 (choice C) is a B-cell marker. It is a signal-transducing molecule that is expressed in early B-cell differentiation. NK cells are negative for CD19.

C21 (choice D) is also a B-cell marker. It is a complement receptor, and is also the same receptor the Epstein-Barr virus uses to bind to cells during infection.

CD56 (choice E) is an NK cell marker, but is not involved with ADCC.
Which of the following organisms would most likely cause infection after a partial sterilization procedure that killed vegetative cells but did not kill spores?
A. Chlamydia
B. Clostridium
C. Escherichia
D. Pseudomonas
E. Streptococcus




The correct answer is B. Although bacterial spores are extensively discussed in microbiology courses, you should be aware that only Bacillus (aerobic) and Clostridium (anaerobic) species produce spores. This means that the list of diseases related to bacterial spore formation is also small: anthrax (Bacillus anthracis), some forms of food poisoning (Bacillus cereus), botulism (Clostridium botulinum), tetanus (Clostridium tetani), gas gangrene (Clostridium perfringens and others), and pseudomembranous colitis (Clostridium difficile). This is one of the reasons why some clostridium infections are difficult to treat. With respect to dentistry, clostridium difficile induced colitis can be seen following treatment with antibiotics, especially clindamycin. Remember, clindamycin is commonly used to treat a variety of dental-related infections because it has excellent anaerobic coverage. Note also that spores (bacillus stearothermophilus) are used to test autoclave function.
An elderly man develops a vesicular rash localized to a narrow circumferential band on one side of his chest. The rash is very painful and the vesicles are confluent with some ulceration. No other significant findings are demonstrated on physical examination. Which of the following diagnoses is most likely to be correct?
A. Chicken pox
B. Herpes simplex I infection
C. Herpes simplex II infection
D. Measles
E. Shingles




The correct answer is E. This is shingles, the recurrent form of herpes zoster infection, which is usually (except in the case of immunosuppressed patients) confined to a single, unilateral dermatome. Isolated vesicles may be seen outside the dermatome. The primary herpes zoster infection (chicken pox) precedes the development of shingles by years or decades; the prevalence of shingles rises steadily with age, to the point that 1% of people older than 80 years of age have the condition. Shingles lesions are infections and should be considered an infectious hazard in the hospital setting. Acyclovir can ameliorate the condition.

Varicella, or chicken pox (choice A), is the primary form of herpes zoster infection and affects face and trunk diffusely. It is generally seen in young patients. The macules evolve to papules and vesicles then crust over; all stages are simultaneously present.

Herpes simplex I (choice B) affects oral and perioral sites.

Herpes simplex II (choice C) primarily affects genital sites, producing a painful set of beefy red lesions.

Measles (choice D) causes a blotchy rash, rather than a dermatomal one. Patients present with a maculopapular rash that is brick red. It begins on the head and neck and spreads downward and outward. Koplik spots often appear on the buccal mucosa.
A 35-year-old man who recently traveled to a third world country develops chronic, severe dysentery with ulceration of the cecum. Biopsy reveals 15-40 micron amoebae with ingested erythrocytes and small nuclei with distinctive tiny central karyosomes. If hepatomegaly is a late complication of this patient's condition, which of the following organisms is the most likely culprit?
A. Isospora belli
B. Cryptosporidium parvum
C. Entamoeba histolytica
D. Giardia lamblia
E. Microsporidia spp.




The correct answer is C.Entamoebahistolytica is the usual cause of intestinal amebiasis, and has the microscopic features described in the question stem. A particularly helpful (but not always present) feature of this organism is the presence of ingested red blood cells within the amoebae. These amoebae cause flask-shaped ulceration of the intestinal mucosa and submucosa, with a particular propensity for involving the cecum and ascending colon. The disease manifestations range from none (asymptomatic carriers) to mild chronic diarrhea, to severe, purging dysentery. In symptomatic cases, the liver may develop destructive amoebic liver abscesses that tend to become secondarily (and potentially life-threateningly) infected by bacteria. Patients initially present with recurrent diarrhea and abdominal cramps. As the condition worsens severe colitis may be present. Then hepatic involvement may occur in fulminant cases.

Isospora belli (choice A), Cryptosporidium parvum (choice B), and Microsporidia spp. (choice E) are commonly seen in AIDS patients. These organisms can cause severe diarrhea. None of these organisms are amebae.

Giardia lamblia(choice D) is a small intestinal protozoa with a distinctive pear-shaped morphology that appears to have a "face."
Which of the following characteristics would help to differentiate Streptococcus agalactiae from Streptococcus pneumoniae?
A. Alpha-hemolysis
B. Carbohydrate capsule
C. Cytochrome enzyme system
D. Growth in bile
E. Oxacillin sensitivity




The correct answer is A. Streptococci are usually initially speciated by their hemolytic capacity on sheep blood agar. Beta-hemolytic streptococci include groups A, B, and D. S. agalactiae is the classic group B streptococcus. The non-beta-hemolytic streptococci consist principally of the pneumococci and the viridans group. These groups are both alpha-hemolytic. Streptococcus is a genus of gram-positive facultative cocci occurring in pairs or chains. The genus is separable into the pyrogenic group, viridans group, enterococcus group, and lactic group. Because streptococci are so common, it is essential that the dental student understand how to differentiate all the different species. Note that alpha hemolysis produces a green halo around the colony in RBC culture, whereas beta hemolysis produces a clear halo.

Both S. agalactiae and pneumococcus have a carbohydrate capsule (choice B), an important virulence factor and means of subtyping streptococcal species.

None of the streptococci use cytochrome enzymes (choice C). They derive all of their energy from the fermentation of sugars to lactic acid.

Neither pneumococcus nor S. agalactiae can grow in bile (choice D). This ability is specific for the enterococcus group (group D) of streptococci.

Both pneumococci and S. agalactiae are usually treated with penicillin-type antibiotics, although group B streptococci require a penicillinase-resistant type such as oxacillin (choice E).
A 73-year-old woman with a history of diabetes presents with left ear pain and drainage of pus from the ear canal. She has swelling and tenderness over the left mastoid bone. Which of the following microorganisms is the most likely causative agent?
A. Haemophilus influenzae
B. Klebsiella pneumoniae
C. Mucor sp.
D. Pseudomonas aeruginosa
E. Streptococcus pyogenes




The correct answer is D.Pseudomonas aeruginosa causes malignant otitis externa, which is a severe necrotizing infection of the external ear canal. Infection tends to spread to the mastoid bone, temporal bone, sigmoid sinus, base of the skull, meninges, and brain. Patients at increased risk include the elderly, those with diabetes, and the immunocompromised.

Haemophilus influenzae(choice A) produces a variety of clinical syndromes. H. influenzae is the third most common cause of meningitis in children aged 1 month to 18 years. It is the most common cause of acute epiglottitis, the most common cause of purulent bacterial conjunctivitis, and the second most common cause of otitis media. Patients at risk include those with COPD and cystic fibrosis, alcoholics, splenectomized patients, and young patients.

Klebsiella pneumoniae(choice B) is a gram-negative organism that produces a necrotizing pneumonia in diabetic patients and alcoholics. Patients typically present with an abrupt onset of fever, shaking chills, and purulent, foul-smelling sputum.

Mucor (choice C) is a fungal infection that is particularly severe in the diabetic or immunocompromised patient. In the acidotic diabetic, the fungus produces a life-threatening, invasive rhinocerebral infection. The infection begins in the nasal passages, extends into the paranasal sinuses, and spreads through the cribiform plate to the frontal lobes of the brain. Patients typically complain of headache, facial pain, and orbital swelling.

Streptococcus pyogenes(choice E) causes bacterial pharyngitis, otitis media, and sinusitis. It is also associated with toxin-related diseases and skin infections.
A poor African community is experiencing an epidemic of severe hepatitis. The mortality among pregnant women is particularly high. Which of the following viruses is the most likely cause of the epidemic?
A. Cytomegalovirus (CMV)
B. Hepatitis A virus (HAV)
C. Hepatitis C virus (HCV)
D. Herpes simplex I
E. Hepatitis E virus (HEV)




The correct answer is E. Although hepatitis E is not generally seen in this country, it is essential that the student have an understanding of all forms of hepatitis. Hepatitis E is an important, and until recently, unrecognized cause of epidemics of enterically transmitted (fecal-oral) acute hepatitis. Hepatitis E is caused by a virus that occurs primarily in India, Asia, Africa, and Central America. Infection is associated with a 10-20% mortality among pregnant women. This type of hepatitis needs to be considered in patients traveling to and from endemic areas.

CMV (choice A) can cause acute hepatitis, but the disease is usually mild and often goes unrecognized, except in profoundly immunosuppressed patients.

Hepatitis A virus (choice B) is the major cause of epidemics of enterically transmitted viral hepatitis, especially in the United States, but is not a significant cause of mortality in pregnant women.

Hepatitis C virus (choice C) is usually transmitted parenterally rather than enterically and is not a significant cause of mortality in pregnant women.

Herpes simplex (choice D) usually causes significant hepatitis only in profoundly immunosuppressed patients.
Several students ate lunch at a restaurant at which they all were served pork with vegetables and fried rice. All of the students developed nausea, vomiting, abdominal pain, and diarrhea within 4 hours of eating lunch. Which of the following is the most likely cause of these symptoms?
A. Bacillus cereus
B. Clostridium botulinum
C. Clostridium perfringens
D. EHEC (enterohemorrhagic escherichia coli)
E. Staphylococcus aureus




The correct answer is A. Bacillus cereus produces a self-limited diarrhea caused by ingestion of the preformed enterotoxin in contaminated fried rice (especially when reheated) and seafood. The incubation period is typically approximately 4 hours. The degree of vomiting is usually greater than that of diarrhea. B. cereus is also associated with keratitis, producing a corneal ring abscess.

Clostridium botulinum (choice B) produces a neurotoxin that blocks the release of acetylcholine, resulting in a symmetric descending paralysis that may lead to respiratory complications causing death. Symptoms include blurred vision, photophobia, dysphagia, nausea, vomiting, and dysphonia. Most cases are associated with the ingestion of contaminated home-canned food.

Clostridium perfringens (choice C) produces a severe diarrhea with abdominal pain and cramping (sometimes called "church picnic" diarrhea). The incubation period is 8-24 hours after ingesting contaminated meat, meat products, or poultry. The meats have usually been cooked, allowed to cool, and then warmed, which causes germination of the clostridial spores.

EHEC, enterohemorrhagic escherichia coli (choice D), produces a bloody, noninvasive diarrhea caused by the ingestion of verotoxin found in undercooked hamburger at fast food restaurants. Some patients develop a life-threatening complication called hemolytic-uremic syndrome.

Staphylococcus aureus (choice E) produces a self-limited food poisoning syndrome with nausea, vomiting, and abdominal pain followed by diarrhea beginning 1-6 hours after ingestion of the enterotoxin. The organism is found in foods such as potato salad, custard, milk shakes, and mayonnaise.
Which of the following organisms is most likely to be implicated as a cause of urethritis that persists after antibiotic therapy for gonorrhea?
A. Actinomyces
B. Chlamydia
C. Mycobacteria
D. Nocardia
E. Rickettsia




The correct answer is B.Chlamydia, unlike the other choices, is a type of sexually transmitted disease, as is gonorrhea. Gonococcal infections typically produce profuse urethal discharge, especially in men, yielding a positive smear. Fever, rash and arthritis can occur with disseminated disease. Chlamydia, Mycoplasma, and Ureaplasma are not effectively treated by penicillins and cephalosporins, and are important causes of post-gonococcal urethritis. Chlamydial urethritis can be diagnosed by using fluorescent antibodies to identify inclusions in epithelial cells.

Actinomyces(choice A) is a mouth commensal that rarely causes a deeper oral infection.

Mycobacteria(choice C) cause chronic granulomatous diseases such as tuberculosis and leprosy.

Nocardia(choice D) can cause necrotizing pneumonia and disseminated disease.

Rickettsia(choice E) cause typhus and Rocky Mountain spotted fever.
Which of the following organisms would be most likely to cause an outbreak of enteritis in a day care center in the United States?
A. Helicobacter jejuni
B. Salmonella typhi
C. Shigella species
D. Vibrio cholerae
E. Yersinia enterocolitica




The correct answer is E. Enteritis is an inflammation of the intestine, especially the small intestine. Yersinia enterocolitica is an important cause of mini-epidemics of pediatric diarrhea. Adults can also be affected, but less commonly than children. Some diarrheal cases are severe (and occasionally fatal) and may be complicated by severe dysentery, appendicitis, or chronic relapsing ileocolitis that may require antibiotics to shorten the course. Yersinia is a non-motile ovoid or rod shaped, nonencapsulated gram-negative bacteria.

Helicobacter jejuni (choice A) is an important cause of hospital-acquired diarrhea, especially in immunocompromised individuals.

Salmonella typhi (choice B) causes typhoid fever. It is a gram-negative bacteria.

Shigella species (choice C) causes epidemics of dysentery in military camps and other close quarter areas. It is a gram-negative bacteria.

Vibrio cholerae (choice D) causes cholera. It is a gram-negative bacteria. Gastroenteritis can occur in the United States upon eating improperly cooked seafood from other countries.
In a closed system, spores are formed during which of the following phases of bacterial growth?
A. Decline phase
B. Exponential phase
C. Lag phase
D. Log phase
E. Stationary phase




The correct answer is E. Spore formation usually occurs during the stationary phase, when cell growth ceases because of a developing lack of nutrients or accumulation of toxins.

During the phase of decline (choice A), the lack of nutrients and the accumulation of toxin become so severe that any viable organisms usually die before they can form spores.

The exponential phase (choices B) and log phase (choice D) are descriptors for active growth occurring after the lag phase and before the stationary phase. Many antibiotics are most effective in this period.

The lag phase (choice C) is the initial period of adaptation, prior to growth, which occurs when organisms are introduced to a new environment.
On physical examination, a man has several disfiguring lesions on his face and loss of cutaneous sensation to fine touch, pain, and temperature. An acid-fast organism is observed in scrapings from a skin lesion. Which of the following organisms is the most likely cause of this patient's disease?
A. Bartonella henselae
B. Listeria monocytogenes
C. Mycobacterium avium-intracellulare
D. Mycobacterium leprae
E. Nocardia asteroides




The correct answer is D. The disease in question is leprosy, or Hansen disease. The key feature in the description is the fact the organism is acid-fast. These acid-fast bacilli are commonly found in skin lesions or nasal scrapings. Both of the mycobacteria, M. avium-intracellulare and M. leprae are strongly acid-fast, that is, they retain the carbol fuchsin dye in the face of acid-alcohol decolorization. M. leprae has a predilection for the skin and cutaneous nerves, thereby producing the symptoms of depigmentation and anesthetic cutaneous lesions. This loss of peripheral nerve function leads to many of the disfiguring features of the disease; because patients do not have normal pain sensation, they sustain repeated injuries. In addition, the organism attacks cartilage and causes granuloma formation in the skin, leading to some of the facial disfigurement. Patients often have a history of residence in an endemic area in childhood.

Bartonella henselae(choice A) is a very small, gram-negative bacterium that is closely related to the rickettsia, although it is able to be cultured on lifeless media. It is the cause of cat-scratch disease, a local, chronic lymphadenitis most commonly seen in children.

Listeria monocytogenes(choice B) is a ubiquitous microbe that causes disease in more than 100 animal species. Although it is best known as an agent of meningitis in the newborn, it is a cause of multiple other diseases. A characteristic feature of these infections is the development of granulomas at the site of the infection. The organism is not acid-fast.

M. avium-intracellulare(choice C) causes tuberculosis-like pulmonary disease in the immunosuppressed, primarily in HIV infected individuals.

Nocardia asteroides(choice E) primarily produces pulmonary infections in humans. The organism is considered to be "weakly" acid-fast.
A 28-year-old HIV-positive man complains of pain on swallowing. Physical examination shows white plaque-like material on his tongue and buccal mucosa, which is biopsied. The man is diagnosed with acquired immunodeficiency syndrome (AIDS). With which of the following agents is the man most likely infected?
A. Candida albicans
B. Cytomegalovirus
C. Herpes simplex I
D. Human herpesvirus 8
E. Human papilloma virus




The correct answer is A. The most common early form of oral lesions in HIV infected individuals includes thrush, hairy leukoplakia, and aphthous ulcers. Candida albicans produces oral thrush, an AIDS-defining lesion, which is common in acute HIV disease, and becomes increasingly common as the CD4 + cell count falls, especially as the CD4 decreases to less than 300. As the CD4 count decreases to less than 100, the esophagus, trachea, bronchi, or lungs may be infected. The lesions are usually painless. Diagnosis is by demonstration of pseudohyphae using a wet smear with confirmation by culture.

Although cytomegalovirus (choice B) is associated with numerous clinical scenarios in the AIDS population, including odynophagia (painful swallowing), it would not produce white plaques on the oral mucosa.

Herpes simplex I (choice C) produces vesicular lesions occurring in clusters in the oral cavity. There is an increased risk for herpes infections in the AIDS group, but the lesions do not resemble those described in the question. These lesions are commonly red and extremely painful.

Human herpesvirus 8 (choice D) is the causative agent of Kaposi sarcoma, a malignancy arising from endothelial cells that appears as hemorrhagic nodules in different organ systems. It is the most common cancer in the HIV infected population.

Human papilloma virus (choice E) is associated with a variety of lesions, including warts and intraepithelial neoplasias of the vulva and cervix. It is associated with anal condyloma, which can occur in the AIDS population.
A viral organism was isolated from a painful blister on the lip of a teenage girl. The agent was found to double-stranded, linear enveloped DNA. The patient had a similar sore approximately 2 months ago following a week long trip to the beach. Which of the following is the most likely causative organism?
A. Adenovirus
B. Coxsackie virus
C. Herpes simplex type 1 virus
D. Herpes zoster virus
E. Papilloma virus




The correct answer is C. Herpes simplex often causes recurrent grouped vesicles on an erythematous base. The Tzanck smear is positive for multi-nucleated epithelial giant cells. Herpes simplex is an enveloped, linear DNA virus that is a very common infectious agent; most adults will have anti-Herpes simplex antibodies in their serum, although many may never have had any clinical signs of disease. The hallmark of this disease is these painful skin vesicles, often called "cold" sores, or "fever" blisters to denote the precipitating event that preceded the appearance of the lesions. The virus has a propensity to become latent in the host's nervous tissue. Activation of the infection occurs following mild trauma (e.g., a visit to the dentist), hormonal changes (e.g., menses), immunosuppression (e.g., following organ transplantation), or may follow minor infections, stress or sun exposure. Other, more serious, manifestations of disease include encephalitis, pneumonia, and hepatitis; these are particularly likely to be seen in immunodeficient patients such as those with AIDS.

Adenoviruses (choice A) are naked, linear, double-stranded DNA viruses that cause acute, usually self-limiting, influenza-like illnesses occurring in the fall and winter. The symptoms include pharyngitis, fever, cough, and general malaise. Epidemic pharyngoconjunctivitis and pneumonia can occur in closed populations such as military installations.

Coxsackie viruses (choice B) are naked, single-stranded, polycistronic viruses with an RNA genome. They are divided into groups A and B based on their virulence. Coxsackie A causes herpangina and hand-foot-and-mouth disease, while Coxsackie B is seen in patients with pleurodynia, myocarditis, and pericarditis. Both groups cause upper respiratory infections, febrile rashes, and meningitis.

Herpes zoster (choice D), the varicella virus, is an enveloped, double-stranded DNA virus that is a very common infectious agent in children. Chickenpox is a mild, self-limiting illness in children that is evidenced as a fever followed by a macular rash that progresses to papules, then vesicles of the skin and mucous membranes. Shingles is a recurrence of a latent varicella infection in which the virus has taken refuge in sensory ganglia of spinal or cranial nerves. Various factors that decrease the immune status of the patient contribute to the exacerbation of the infection. Severe dermatomal pain occurs with a vesicular eruption, fever, and malaise.

Papilloma viruses (choice E) are members of the Papovavirus family. They are non-enveloped and possess a double-stranded, circular DNA genome. They cause skin, plantar, and genital warts.
Electron micrographs of glomeruli reveal prominent deposits between the podocytes and the basement membrane of the glomerular capillaries. If the patient was noted to have a skin infection before this condition, these findings are most likely related to prior infection with which of the following genera?
A. Escherichia
B. Klebsiella
C. Neisseria
D. Pseudomonas
E. Streptococcus




The correct answer is E. The disease is postinfectious glomerulonephritis, which is usually related to prior sore throat or skin infection by Streptococcal species; Staphylococcus also causes some cases. The prominent deposits are also known as subepithelial humps, and consist of immunoglobulin and complement. Postinfectious glomerulonephritis commonly occurs after impetigo. Onset occurs within 1-3 weeks after infection (average 7-10 days). Other causes of postinfectious glomerulonephritis include bacteremic states, such as systemic staphylococcus infections.

Gram-negative rods such as Escherichia(choice A), Klebsiella (choice B), and Pseudomonas (choice D) have not been implicated as significant causes of postinfectious glomerulonephritis.

The gram-negative cocci Neisseria(choice C) have not been implicated in postinfectious glomerulonephritis.
Which of the following interleukins is produced by macrophages and stimulates fever production by its action on hypothalamic cells?
A. IL-1
B. IL-2
C. IL-3
D. IL-4
E. IL-5




The correct answer is A. IL-1 is produced by macrophages and other antigen-presenting cells. It has several actions, including stimulation of T cells to secrete IL-2, chemotactic activity for neutrophils and monocytes, increased expression of intercellular adhesion molecules (ICAMs) on vascular endothelial cells, and activation of macrophages and natural killer (NK) cells. It is also pyrogenic (fever inducing) because of a direct effect on the hypothalamus.

IL-2 (choice B) is produced by activated T helper cells. It stimulates the proliferation of other T cells as well as activated B cells. It also activates NK cells and stimulates lymphokine secretion.

IL-3 (choice C) stimulates all stem cells to produce hematopoietic cells; it is also known as multilineage colony stimulating factor (CSF).

IL-4 (choice D) is produced by T helper 2 (Th 2) cells and mast cells. It has several functions, including inducing cells to express class II major histocompatibility complex (MHC) antigens and stimulation of B-cell proliferation. It is involved in the induction of atopic allergies by its mitogenic activity for mast cells and its enhancement of immunoglobulin class switching to IgG and IeE.

IL-5 (choice E) is secreted by activated T helper cells. It promotes B-cell proliferation, production of eosinophils, and stimulates B-cell class switching to IgA.
A patient received second- and third-degree burns over his body. He later developed a wound infection with a bluish-green exudate. Treatment with chloramphenicol and tetracycline was unsuccessful. A gram-negative, motile organism was isolated. Which of the following organisms was most likely isolated?
A. Candida albicans
B. Clostridium perfringens
C. Escherichia coli
D. Pseudomonas aeruginosa




The correct answer is D. Although this is a clinically-based question, it is essentially asking about the microbiology of each bacteria. Pseudomonas aeruginosa is a very common opportunist in burn patients and patients on ventilators, in whom it classically causes secondary wound infections and septicemia. It may also cause cystitis in patients with urinary catheters and pneumonia in patients with cystic fibrosis. The organism is found in water and usually gains access to the body as a contaminant in the water used in respirators or in water baths, etc., used to cleanse wounds. This organism is a nonfermenter, that is, it does not metabolize sugars by classic pathways. It produces a blue-green, water-soluble pigment (pyocyanin), and has a fruity odor when grown on laboratory media. It has a propensity for developing antibiotic resistance; current therapy uses the synergistic combination of an aminoglycoside, such as amikacin, with a cell wall synthesis inhibitor (carbenicillin, ticarcillin, or piperacillin).

Candida albicans (choice A) is a normal flora yeast that appears as large, weakly gram-positive, spherical to ovoid organisms with budding daughter cells in Gram-stained preparations. Candidiasis is an opportunistic infection in individuals with a compromised immune system. The fungus usually causes mucocutaneous lesions, but in severely compromised individuals like patients with AIDS, systemic disease may occur. Oral candidiasis appears as creamy white patches of exudate that can be scraped off an inflamed tongue or buccal mucosa to reveal a painful lesion that may bleed.

Clostridium perfringens(choice B) is a gram-positive, spore-forming, anaerobic rod. It is a common cause of gas gangrene when introduced into a wound. The organism produces a variety of toxins and enzymes that enable it to destroy muscle tissue and spread through the soft tissues of the body.

Escherichia coli(choice C) is a lactose-fermenting, gram-negative rod seen as normal flora of the intestine. It is the most common cause of urinary bladder infections, pyelonephritis, and sepsis in patients with indwelling urinary catheters. It is also the major cause of traveler's diarrhea and is a very important pathogen in neonates, who become infected during passage through the birth canal.
An 8-year-old girl is bitten in the leg by a neighbor's cat. She presents the next day with fever and bone pain localized to her right calf. An x-ray film reveals a lytic lesion of the right tibia. Which of the following is the most likely pathogen?
A. Brucella melitensis
B. Eikenella corrodens
C. Francisella tularensis
D. Pasteurella multocida
E. Yersinia pestis




The correct answer is D. This patient has osteomyelitis due to a cat bite that penetrated the periosteum. Whenever you see dog or cat bites in a question stem, consider Pasteurella multocida as a primary cause of wound infection. This organism is a short, encapsulated, gram-negative rod demonstrating bipolar staining. Rapidly arising cellulitis is particularly indicative of this organism.

Brucella melitensis (choice A) is a small, acapsular, gram-negative rod that causes brucellosis (undulant fever) and is associated with contact with goats or sheep. Brucella abortus and Brucella suis are variants associated with contact with cows and pigs, respectively. The organisms enter the body through the skin or through contaminated dairy products, such as unpasteurized imported goat's milk or cheeses.

Eikenella corrodens(choice B) is commonly found in human bites. It is a gram-negative rod that is part of the normal flora of the human mouth.

Francisella tularensis(choice C) is a small, pleomorphic, gram-negative rod that causes tularemia (rabbit fever). It occurs most commonly in rural areas. In the U.S., rabbits are the main reservior for this organism, which is transmitted to humans by the Dermacentor tick or by contact with infectious animal tissues.

Yersinia pestis(choice E) is responsible for bubonic plague, which has been known to occur in the western U.S. Its main reservoir is the prairie dog, and its vector is the rat flea.
A 27-year-old IV drug user presents with difficulty swallowing. Examination of the oropharynx reveals white plaques along the tongue and the oral mucosa. Which of the following best describes the microscopic appearance of the microorganism responsible for this patient's illness?
A. Budding yeast and pseudohyphae
B. Encapsulated yeast
C. Mold with nonseptate hyphae
D. Mold with septate hyphae




The correct answer is choice A. The patient has Candida esophagitis. Any time a patient presents with dysphagia or odontophagia, along with whilte plaques in the oropharynx (thrush), you can assume that the Candida is affecting the esophagus as well. The fact that the patient is an IV drug user make an opportunistic infection such as Candida more likely. Candida appears as budding yeast with pseudohyphae in vivo.

The other answer choices represent the morphology of other important opportunistic fungi:

Cryptococcus are encapsulated yeast (choice B). You should think about Cryptococcus neoformans when you're presented with an immunocompromised patient with neurologic symptoms. The classic clue is the presence of encapsulated organisms observable in an India ink preparation.

Mucor and Rhizopus are molds with nonseptate hyphae (choice C). You should think about Mucor when you are presented with a diabetic (especially ketoacidotic) or a leukemic patient with a severe sinus infection.

Aspergillus is a mold with septate hyphae (choice D). In immunocompromised patients, aspergillosis can present with acute pneumonia, often with a cavitation (aspergillomas = fungus balls in the lungs).
A 32-year-old woman with increased frequency of urination, suprapubic pain, and dysuria for the past 3 days comes to the emergency department. She has no fever, nausea, or vomiting. A Gram stain reveals gram-negative rods. Which of the following is the most likely pathogen?
A. Escherichia coli
B. Neisseria gonorrhoeae
C. Shigella dysenteriae
D. Streptococcus pneumoniae
E. Treponema pallidum




The correct answer is A. This patient has the symptoms of a urinary tract infection (UTI). Escherichia coli is the leading cause of community-acquired UTIs. The proximity of the urinary tract to the anus facilitates colonization of the tract by fecal flora.

Other gram-negative rods causing UTIs include Enterobacter cloacae, Klebsiella pneumoniae, Serratia marcescens, Proteus mirabilis, and Pseudomonas aeruginosa.

None of the other choices listed cause UTIs.
A 24-year-old AIDS patient develops chronic abdominal pain, low-grade fever, diarrhea, and malabsorption. Oocysts are demonstrated in the stool. Which of the following organisms is most likely to be the cause of the patient's diarrhea?
A. Chlamydia psittaci
B. Entamoeba histolytica
C. Giardia lamblia
D. Isospora belli
E. Microsporidia




The correct answer is D. All of the organisms listed are protozoa. There are two intestinal protozoa specifically associated with AIDS that can cause transient diarrhea in immunocompetent individuals but can cause debilitating and potentially life-threatening chronic diarrhea in AIDS patients. These organisms are Isospora belli, treated with trimethoprim-sulfamethoxazole (or other folate antagonists) and Cryptosporidium parvum (no treatment presently available).

Chlamydia psittaci (choice A) is associated with the development of fever, chills, cough, and atypical pneumonia with slightly delayed appearance of signs of pneumonitis.

Entamoeba histolytica(choice B) and Giardia lamblia(choice C) are both causes of diarrhea, but they are not specifically associated with AIDS. Giardiasis, for example, can cause mild to severe bulky, greasy, frothy, malodorous stools, free of blood and pus.

Microsporidia (choice E) are a protozoan cause of diarrhea, but produce spores rather than oocysts.
A middle-aged man presents with fever, anorexia, and jaundice. He is diagnosed with hepatitis caused by an enveloped DNA virus. What is the most likely causative organism?
A. Hepatitis A
B. Hepatitis B
C. Hepatitis C
D. Hepatitis E




The correct answer is B. Hepatitis B is a hepadnavirus, which is enveloped DNA.

Hepatitis A (choice A) is a picornavirus and is a naked-capsid RNA.

Hepatitis C (choice C) is a flavivirus, which is RNA enveloped.

Hepatitis E (choice D) is a Calicivirus, which is a naked-capsid RNA.
A 6 month-old child who has been breast-fed since birth develops voluminous, non-bloody, watery diarrhea and vomiting. Which of the following viruses is the most likely cause of the child's diarrhea?
A. Coronavirus
B. Lymphocytic choriomeningitis virus
C. Norwalk agent
D. Orbivirus
E. Rotavirus




The correct answer is E. Rotavirus is the major cause of diarrhea in infants and children under the age of 2. It is a major cause of diarrheal morbidity worldwide due to dehydration. The virus replicates in the intestinal mucosa, producing a profuse, watery, non-bloody diarrhea, often coupled with nausea and vomiting. Transmission is by the fecal-oral route. Treatment is symptomatic with fluid and electrolyte replacement.

Coronaviruses (choice A) usually cause cold-like illnesses.

Lymphocytic choriomeningitis virus (choice B) can cause headache, malaise, myalgia, conjunctivitis, and, occasionally, meningitis.

Norwalk agent (choice C) can also cause diarrhea, but usually affects patients older than 2 years. This causes profuse vomiting and diarrhea and is often transmitted by food, especially shellfish.

Orbivirus (choice D) is the cause of Colorado tick fever, which is the only tick-borne viral disease in the United States.
A 6 month-old child who has been breast-fed since birth develops voluminous, non-bloody, watery diarrhea and vomiting. Which of the following viruses is the most likely cause of the child's diarrhea?
A. Coronavirus
B. Lymphocytic choriomeningitis virus
C. Norwalk agent
D. Orbivirus
E. Rotavirus




The correct answer is E. Rotavirus is the major cause of diarrhea in infants and children under the age of 2. It is a major cause of diarrheal morbidity worldwide due to dehydration. The virus replicates in the intestinal mucosa, producing a profuse, watery, non-bloody diarrhea, often coupled with nausea and vomiting. Transmission is by the fecal-oral route. Treatment is symptomatic with fluid and electrolyte replacement.

Coronaviruses (choice A) usually cause cold-like illnesses.

Lymphocytic choriomeningitis virus (choice B) can cause headache, malaise, myalgia, conjunctivitis, and, occasionally, meningitis.

Norwalk agent (choice C) can also cause diarrhea, but usually affects patients older than 2 years. This causes profuse vomiting and diarrhea and is often transmitted by food, especially shellfish.

Orbivirus (choice D) is the cause of Colorado tick fever, which is the only tick-borne viral disease in the United States.
With respect to the bacterial spectrum of coverage, which of the following antibiotics has the broadest activity against gram-negative organisms?
A. Cefuroxime (second generation cephalosporin)
B. Cephalexin (first generation cephalosporin)
C. Ceftriaxone (third generation cephalosporin)
D. Metronidazole
E. Vancomycin




The correct answer is C. For NBDE Part 1, it is important to know general properties of antibiotic families. Ceftriaxone is a third generation cephalosporin commonly used in the treatment of a variety of life-threatening infections caused by gram-negative organisms. The agent has moderate gram-positive activity. A general rule to remember about the cephalosporins is that when progressing from first generation to third generation, the gram-negative activity is increased and gram-positive activity is decreased. In other words, first generation cephalosporins have the most activity against gram-positive organisms and third generation cephalosporins have the most activity against gram-negative organisms.

Metronidazole (choice D) is an antibiotic with excellent anaerobic bacterial coverage.

Vancomycin (choice E) is an antibiotic with excellent gram-positive coverage; it is generally used in the treatment of life-threatening infections.
If a newborn child develops an infection with Chlamydia trachomatis, how does infection with this organism cause blindness?
A. Cataract formation
B. Hemorrhage into the anterior chamber
C. Hemorrhage into the posterior chamber
D. Retinal detachment
E. Scarring of the cornea




The correct answer is E.Chlamydiae are obligate intracellular parasites. Studies in the United States demonstrate that 5-25% of pregnant women have C. trachomatis infections of the cervix. In these women, approximately one half to two thirds of infants are exposed and if untreated can develop inclusion conjunctivitis. The lesions begin with formation of lymphoid follicles in the conjunctiva. With disease progression, there is tissue necrosis, granulation tissue deposition, and scar formation, leading to lacrimal duct obstruction and distortion of the eyelids. With the loss of an adequate tear system, the cornea becomes vulnerable to dehydration and opacification. Also, the vigorous inflammatory response can directly involve the cornea, with resulting opacity.
A 25-year-old man with a history of IV drug abuse presents with a high fever and generalized malaise. Physical examination reveals a systolic murmur, and echocardiography shows bulky vegetations attached to the tricuspid valve leaflets. Which of the following microorganisms will be most likely be isolated?
A. Candida albicans
B. Hemophilus influenzae
C. Staphylococcus aureus
D. Staphylococcus epidermidis
E. Viridans (α±-hemolytic) streptococci




The correct answer is C. The patient has a fever and is extremely ill. The most important clue to the diagnosis is the presence of bulky vegetations on the tricuspid valve, indicating that he has infective endocarditis. On the basis of the rapid clinical course, this is likely a case of acute infective endocarditis. The diagnosis of this condition must be confirmed by blood cultures, which are also necessary to determine bacterial antibiotic sensitivity. S. aureus (commonly present on the skin) is the most frequent etiologic agent of infective endocarditis in intravenous drug abusers. It commonly affects the tricuspid valve. Because of its high virulence, S. aureus-related endocarditis follows an acute course and may lead to death within a few days.

The causative agents of infective endocarditis differ depending on host factors. Fungal organisms, such as Candida albicans(choice A), may cause infective endocarditis in severely immunosuppressed patients, such as those with AIDS.

A minority of cases of infective endocarditis are caused by a number of normal commensals in the oral cavity, i.e., the "HACEK" group: Hemophilus(choice B), Actinobacillus, Cardiobacterium, Eikenella, and Kingella.

S. epidermidis(choice D) and other coagulase-negative staphylococci tend to produce endocarditis in recipients of prosthetic valves.

Viridans streptococci (choice E are the most frequent agents causing endocarditis in previously abnormal valves, such as those damaged by rheumatic disease, or congenitally abnormal valves. Coagulase-negative staphylococci and viridans (α±-hemolytic) streptococci are less virulent than S. aureus and are thus associated with a subacute (more prolonged) clinical course and a better prognosis.
Which of the following is a feature of gram-positive bacteria rather than gram-negative bacteria?
A. Lipid A-containing lipopolysaccharide
B. Lipoprotein in periplasmic space
C. Outer membrane
D. Peptidoglycan in periplasmic space
E. Thick peptidoglycan cell wall




The correct answer is E. Most of the features listed are those of gram-negative bacteria, which have a complex cell envelope consisting of a cytoplasmic (inner) membrane, a periplasmic space containing peptidoglycan (choice D) and lipoprotein (choice B), an outer membrane (choice C), and sometimes a capsule. The outer membrane contains lipopolysaccharide (choice A) which is a major component of endotoxin. The peptidoglycan cell wall of the gram-negative bacteria is thin, while that of the gram-positive bacteria is thick. Other features of gram- positive bacteria include a fairly simple surface of cytoplasmic membrane, peptidoglycan, cell wall, and sometimes, an outer capsule. The cell wall contains lipoteichoic acids. Gram-positive bacteria retain the stain or resist decolorizationby alcohol in Gram's method of staining. This is primarily characteristic of bacteria whose cell wall is composed of peptidoglycan or teichoic acid.
A diabetic patient has chronic sinusitis, which is eventually treated by evacuation of the contents of the maxillary and ethmoid sinuses. Mucor species are found when the material is examined histologically. The pathologist should notify the clinician immediately because Mucor can cause a virulent
A. meningitis
B. pneumonia
C. septicemia
D. skin infection
E. urinary tract infection




The correct answer is choice A. Mucormycosis, to which diabetics are particularly vulnerable (perhaps because of the high glucose content of the diabetic nasal secretions), typically involves the nasal sinuses. It is especially dangerous because the organism can erode into the bones of the cranium, causing life-threatening meningitis and/or encephalitis that is very difficult to treat.

Mucor can cause pneumonia (choice B), but this is not the most immediate threat.

Mucor can cause septicemia (choice C), but this usually occurs in more seriously ill patients and is not an immediate risk in this patient.

Mucor can cause a skin infection (choice D), but this generally occurs in conjuncition with severe burns.

Mucor does not usually cause urinary tract infections (choice E).
In trying to sterilize some dental materials, the dentist notices that one of the materials melts at 115 C. Which of the following sterilization methods is most appropriate for this material?
A. Steam Autoclave
B. Dry heat autoclave
C. Glutaraldehyde (Cidex) soak (6 hours)
D. Ethylene oxide
E. Ethyl alcohol




The correct answer is choice DNote firstly that ethyl alcohol and glutaraldehyde are chemicals that can only high level disinfect, and cannot be used for sterilization. In particular, they are not sporicidal. Choices A, B and D are the three most common methods of instrument sterilization. However, the steam autoclave operates at 121 C, and the dry heat sterilizer at 160 C, both too hot for the material which melts at 115 C. So we can only use a non-heat sterilization method. The only non-heat method is ethylene oxide, at 8-12 hours of contact time.
Global eradication of Lyme disease is unlikely because
A. Borrelia burgdorferi can be maintained in nature indefinitely by a tick vector
B. Borrelia burgdorferi is resistant to antibiotics and disinfectants
C. Borrelia burgdorferi is resistant to environmental stresses
D. human disease may reactivate after the primary infection (Brill-Zinsser disease)
E. humans are the primary reservoir for Borrelia burgdorferi




The correct answer is A. Borrelia burgdorferi can be maintained in nature indefinitely by a tick vector. This organism is the tick-transmitted spirochete that causes Lyme disease. The tick, Ixodes dammini, can infect the white-footed mouse and large mammals such as deer during its life cycle, making them reservoirs. The tick itself is a reservoir, however, because it acquires the disease through transovarial passage of the organism. Together, these factors make Lyme disease an endemic infection with little hope for eradication. Lyme disease is characterized by erythema migrus, a flat or slightly raised red lesion that expands with central clearing. Headache or stiff neck are common. Arthritis is common, chronic, and recurrent.

Borrelia burgdorferi is not resistant to antibiotics and disinfectants (compare with choice B). The spirochete can be successfully treated with penicillins, tetracycline, and ceftriaxone.

Borrelia burgdorferi is a delicate spirochete and is not resistant to environmental stresses (compare with choice C).

Brill-Zinsser disease (choice D) is the reactivation of epidemic typhus infection caused by Rickettsia prowazekii. It can occur many years after an infection that was not treated with antibiotics.

Humans are incidental hosts, rather than the primary reservoir, for Borrelia burgdorferi(choice E). The primary reservoirs are ticks, mice, and large mammals.
A chef cut his finger. After a week, the site of the injury is warm, red, and swollen, and begins draining pus. He contaminates some pastries with drainage from the lesion which later were eaten by patrons of the restaurant. Within 4 hours they developed diarrhea and vomiting with no fever. Which of the following organisms would be most likely to cause these symptoms?
A. Bacillus cereus
B. Clostridium perfringens
C. Escherichia coli
D. Shigella sonnei
E. Staphylococcus aureus




The correct answer is E. Cellulitis is an acute infection involving the epidermis and dermis. Cellulitis can lead to the development of an abscess, as seen in this patient. The most common types of skin infections (cellulitis or abscess) usually involve gram-positive organisms such as staphylococcus and streptococci. The chef had a staphylococcal abscess on his finger. S. aureus, produces enterotoxin A, which was likely transferred to the pastries. When ingested, the toxin causes severe nausea and vomiting within a few hours (the average incubation time is 3-6 hours). There is little diarrhea associated with this type of food poisoning outbreak.

Bacillus cereus(choice A) is a gram-positive spore-forming rod that is associated with food poisoning outbreaks following the ingestion of fried rice. The time of onset and symptoms would mimic staphylococcal disease; the major differentiating feature is the food involved. The organism survives the boiling of the rice because it is a spore-former. It germinates as the rice cools, grows, and elaborates an enterotoxin that is responsible for the nausea and vomiting characteristic of the disease.

Clostridium perfringens(choice B) is a gram-positive spore-forming anaerobe that can cause a longer incubation (18-24 hour) food poisoning, typically with marked diarrhea. Once again, the spores allow the organism to survive the heating process used in the preparation of the food. Both Clostridium perfringens and Clostridium botulinum are associated with home-canned vegetable and sausages.

The symptoms of Escherichia coli(choice C) food poisoning are usually watery diarrhea (traveler's diarrhea) with minimal nausea and vomiting, or a bloody diarrhea caused by enteroinvasive strains of the agent. Also, E. coli would be an unlikely cause of the primary infection in the chef.

Shigella sonnei (choice D) causes enterocolitis characterized by fever, cramps, and diarrhea after an incubation period of one to four days. Transmission is fecal-oral, associated with poor hygiene. A wide range of foods has been implicated.
A Native American man is brought to a rural hospital in New Mexico with severe bronchopneumonia, chills, fever, and headache. One day later, the man complains of chest pain and difficulty breathing, and coughs up blood-tinged sputum. Chest x-ray reveals patchy infiltrates and segmental consolidation. Which of the following organisms is the most likely cause of this man's pneumonia?
A. Poliomyelitis virus
B. Clostridium perfringens
C. Paramyxovirus
D. Listeria monocytogenes
E. Yersinia pestis




The correct answer is E. Any previously healthy person in the southwestern United States who develops septic shock or severe pulmonary disease should be evaluated for plague. Plague is not an extinct disease, but is still encountered in sporadic cases in various places, including Asia, Africa, parts of Europe, and the American Southwest. The causative organism is Yersinia pestis, which is endemic in many wild animal populations, and can be transmitted to humans either by direct contact or by arthropod bite. Human plague may take many forms, including pestis minor (mild lymphadenopathy); bubonic plague (prominent lymphadenopathy); pneumonic plague (as described above); and septicemic plague. Antibiotics are most effective if given within the first 24 hours, which can be problematic if medical staff do not suspect the disease. Because plague is rare in the United States, a high degree of clinical suspicion is required to make a rapid diagnosis and to institute timely treatment. If the diagnosis is missed, the mortality rate is high.

Poliomyelitis virus (choice A) is an enterovirus that causes muscle weakness, headache, stiff neck, sore throat, fever, nausea, and vomiting. Lower motor neuron lesions (flaccid paralysis) can be seen as well as deep tendon reflexes and muscle wasting.

Clostridium perfringens(choice B) causes gas gangrene and gastroenteritis. This is the most common cause of gangrene.

Paramyxovirus (choice C) is the causative organism of the mumps. In this condition, exposure to paramyxovirus occurs 14-21 days before onset of symptoms. Patients present with painful swollen salivary glands, usually the parotids. Frequent involvement of the other tissues is common, such as the testes, pancreas, and meninges.

Listeria monocytogenes(choice D) causes listeriosis. Infection during pregnancy may result in sepsis, abortion, or premature delivery. Infection in the neonate may produce meningitis. In immunocompromised adults, either meningitis or sepsis may occur.
A Native American man is brought to a rural hospital in New Mexico with severe bronchopneumonia, chills, fever, and headache. One day later, the man complains of chest pain and difficulty breathing, and coughs up blood-tinged sputum. Chest x-ray reveals patchy infiltrates and segmental consolidation. Which of the following organisms is the most likely cause of this man's pneumonia?
A. Poliomyelitis virus
B. Clostridium perfringens
C. Paramyxovirus
D. Listeria monocytogenes
E. Yersinia pestis




The correct answer is E. Any previously healthy person in the southwestern United States who develops septic shock or severe pulmonary disease should be evaluated for plague. Plague is not an extinct disease, but is still encountered in sporadic cases in various places, including Asia, Africa, parts of Europe, and the American Southwest. The causative organism is Yersinia pestis, which is endemic in many wild animal populations, and can be transmitted to humans either by direct contact or by arthropod bite. Human plague may take many forms, including pestis minor (mild lymphadenopathy); bubonic plague (prominent lymphadenopathy); pneumonic plague (as described above); and septicemic plague. Antibiotics are most effective if given within the first 24 hours, which can be problematic if medical staff do not suspect the disease. Because plague is rare in the United States, a high degree of clinical suspicion is required to make a rapid diagnosis and to institute timely treatment. If the diagnosis is missed, the mortality rate is high.

Poliomyelitis virus (choice A) is an enterovirus that causes muscle weakness, headache, stiff neck, sore throat, fever, nausea, and vomiting. Lower motor neuron lesions (flaccid paralysis) can be seen as well as deep tendon reflexes and muscle wasting.

Clostridium perfringens(choice B) causes gas gangrene and gastroenteritis. This is the most common cause of gangrene.

Paramyxovirus (choice C) is the causative organism of the mumps. In this condition, exposure to paramyxovirus occurs 14-21 days before onset of symptoms. Patients present with painful swollen salivary glands, usually the parotids. Frequent involvement of the other tissues is common, such as the testes, pancreas, and meninges.

Listeria monocytogenes(choice D) causes listeriosis. Infection during pregnancy may result in sepsis, abortion, or premature delivery. Infection in the neonate may produce meningitis. In immunocompromised adults, either meningitis or sepsis may occur.
A patient with large, penetrating vegetations on his mitral and aortic valves develops severe headaches. Acute bacterial endocarditis is diagnosed. Which of the following organisms is the most likely cause of the patient's disorder?
A patient with large, penetrating vegetations on his mitral and aortic valves develops severe headaches. Acute bacterial endocarditis is diagnosed. Which of the following organisms is the most likely cause of the patient's disorder?
A can of disinfectant spray states that it kills HIV virus. This statement:
A. is important because of the difficulty in killing this virus
B. shows that the disinfectant will kill most other pathogens
C. illustrates the use of a "benchmark" organism
D. is not a good indication of disinfectant strength




The correct answer is D. HIV virus is notoriously easy to kill on most environmental surfaces. Many disinfectants and other common physical and chemical exposures will kill the virus fairly easily. Therefore, ability to kill HIV is NOT an important criterion in choosing disinfectants. Mycobacterium tuberculosis is chosen as the "benchmark organism" (the standard against which the disinfectant is compared). It is very resistant to surface disinfectants, partially because of its waxy cell wall. Another highly resistant pathogen on environmental surfaces is Hepatitis A virus.
A 35-year-old, sexually active man presents with a painless penile vesicle and inguinal lymphadenopathy. The infecting organism is definitively diagnosed and is known to exist in distinct extracellular and intracellular forms. Which of the following is the most likely pathogen?
A. Calymmatobacterium granulomatis
B. Chlamydia trachomatis
C. Haemophilus ducreyi
D. Neisseria gonorrhoeae
E. Treponema pallidum




The correct answer is choice B. This patient has lymphogranuloma venereum caused by Chlamydia trachomatis (type L1, 2, or 3). Chlamydia exhibit distinct infectious and reproductive forms. The extracellular infectious form is known as the elementarly body (EB), which cannot reproduce. It attaches to the host cell and enters through endocytosis. Once inside the cell, the EB is transformed into the reticulate body (RB) within the endosome. The RB is capable of binary fission and divides within the endosome; fusion with other endosomes occurs to form a single large inclusion. Eventually, the RBs undergo DNA condensation and disulfide bond bridgings of the major outer membrane protein, forming EBs. The EBs are then released. Note that C. trachomatis is responsible for several sexually or perinatally transmitted diseases, including ocular trachoma (types A, B and C), neonatal conjunctivitis, nongonococcal urethritis, cervicitis, and pelvic inflammatory disease (types D-K).

Calymmatobacterium granulomatis (choice A) is a gram-negative rod that causes superficially ulcerated genital or inguinal papules that coalesce to form substantial lesions. The appearance of Donovan bodies in histiocytes is diagnostic of this infection.

Haemophilus ducreyi (choice C) is a gram-negative rod that causes a soft, painful penile chancre, unlike that of a chlamydial or syphilitic lesion. This infection is common in the tropics.

Neisseria gonorrhoeae (choice D) is a gram-negative diplococcus responsible for gonorrhea. Patients typically present with purulent penile discharge, not genital lesion.

Treponema pallidum (choice E) is the spirochete responsible for syphilis. It may cause a firm, painless ulcer as a manifestation of primary syphilis, but the organism does not exist in distinct extracellular and intracellular forms as does Chlamydia. Secondary syphilis is associated with the appearance of condyloma lata -- flat, gray wart-like lesions.
A 23-year-old man develops explosive watery diarrhea with blood, fecal leukocytes, and mucus approximately 3 days after eating chicken that was improperly cooked. Comma-shaped organisms were also found in the fecal smear. Which of the following pathogens is the most likely cause of these symptoms?
A. Campylobacter jejuni
B. Enterotoxigenic E. coli
C. Shigella sonnei
D. Staphylococcus aureus
E. Vibrio cholera




The correct answer is A. This is another question in which knowing the microbiological characteristics is essential. Campylobacter jejuni is a pathogen causing an invasive enteric infection associated with ingestion of raw or undercooked food products, or through direct contact with infected animals. In the United States, ingestion of contaminated poultry that has not been sufficiently cooked is the most common means of acquiring the infection. The patients typically have bloody diarrhea, abdominal pain, and fever. The presence of fecal leukocytes indicates an invasive infection. Campylobacter are microaerophilic, motile, gram-negative "comma-shaped" rods.E. coliare not comma-shaped.

Enterotoxigenic E. coli(choice B) causes the classic traveler's diarrhea. The infection is noninvasive and is acquired by way of the fecal-oral route through consumption of unbottled water or uncooked vegetables. The major manifestation is a copious outpouring of fluid from the gastrointestinal tract presenting as explosive diarrhea. This is caused by the action of one of two types of enterotoxins on the gastrointestinal tract mucosa.

Shigella sonnei(choice C) produces a syndrome very similar to C. jejuni. The organism, however, appears as a gram-negative rod on Gram's stain. It does not have a comma shape. Transmission is from person to person by way of the fecal-oral route. Infection requires a low infective dose because the organism is fairly resistant to gastric acidity.

Staphylococcus aureus(choice D) produces food poisoning through the ingestion of a preformed enterotoxin. The organism is present in food that is high in salt content, such as potato salad, custard, milk shakes, and mayonnaise. The patient presents with nausea, vomiting, and abdominal pain, followed by diarrhea beginning 1-6 hours after ingestion of the enterotoxin.S. aureus are seen as clusters of coli and are not comma-shaped.

Vibrio cholerae(choice E) produces a secretory diarrhea caused by increases in cAMP in the intestinal cells. The organism is comma-shaped, but is not invasive. The patient presents with the sudden onset of painless, watery diarrhea that becomes voluminous, followed by vomiting. The stool appears nonbilious, gray, and slightly cloudy with flecks of mucus, no blood, and a sweet odor.
A 3-year-old boy presents with a 1-day history of loose stools, fever, abdominal cramping, headache, and myalgia. He has no blood in the stool. A careful history reveals that he has had several pet turtles. Which of the following is the most likely pathogen?
A. Chlamydia psittaci
B. Entamoeba histolytica
C. Salmonella spp.
D. Staphylococcus aureus
E. Yersinia enterocolitica




The correct answer is C. Salmonella spp., including S. enteriditis and S. typhimurium, produce a gastroenteritis or enterocolitis. Patients with decreased gastric acidity, sickle cell disease, or defects in immunity, and children younger than 4 years of age have a more severe course of disease. Salmonella spp. are carried in nature by animal reservoirs such as poultry, turtles, cattle, pigs, and sheep. The incubation period is 8-48 hours after ingestion of contaminated food or water.

Chlamydia psittaci (choice A) produces an interstitial pneumonitis accompanied by headache, backache, and a dry, hacking cough. A pale, macular rash is also found on the trunk (Horder spots). Patients at risk include pet shop workers, pigeon handlers, and poultry workers.

Entamoeba histolytica (choice B) produces a diarrhea (frequently bloody or heme-positive), right lower quadrant crampy abdominal pain, and fever. Patients frequently have weight loss and anorexia. There is usually a history of travel outside the United States. Most cases are chronic. Complications include liver abscesses.

Staphylococcus aureus (choice D) produces a self-limited gastroenteritis caused by the production of preformed, heat-stable enterotoxins. The incubation period is 16 hours. The toxins enhance intestinal peristalsis and induce vomiting by a direct effect on the CNS.

Yersinia enterocolitica (choice E) usually produces a chronic enteritis in children. These patients have diarrhea, failure to thrive, hypoalbuminemia, and hypokalemia. Other findings include acute right lower quadrant abdominal pain, tenderness, nausea, and vomiting. The infection mimics appendicitis or Crohn disease.
A 48-year-old presents with malaise, loss of appetite, nausea, moderate fever, and jaundice. Laboratory tests indicate a marked increase in serum transaminases and presence of HBsAg, HBc IgM antibody, and HCV antibody. Antibody tests for HBsAb and HAV are negative. The results indicate:
A. A dual infection of HBV and HAV
B. Chronic hepatitis A infection
C. Chronic hepatitis B infection
D. Hepatitis C infection
E. The presence of an acute HBV infection.




The correct answer is E.The presence of hepatitis B surface antigen (HBsAg) along with hepatitis B core IgM antibody (HBc IgM Ab), and the absence of hepatitis B surface antibody (HBsAb) indicates the presence of the early stages of an acute infection with Hepatitis B. The presence of antibody to Hepatitis C (HCV) only indicates exposure, but not a specific time of exposure; however, 85% of patients who are infected with HCV develop chronic infections, indicating that this patient has an 85% chance of having a dual infection with HBV and HCV.

A dual infection of HBV and HAV (choice A) is not plausible since the IgM anti-HAV serology is negative.

Hepatitis A does not cause chronic disease (choice B).

Chronic HBV infection (choice C) is unlikely because the patient has HBc IgM Ab, which is characteristic of an acute infection, rather than a chronic infection.

Hepatitis C infection (choice D) is not confirmed by these data because the presence of HCV Ab only indicates exposure to the virus, and not the state of infection. This could be caused by exposure at some earlier time (the elevated serum transaminases might be due to HBV infection). An active or chronic HCV infection can only be confirmed by PCR.
A 47-year-old man presents with a nonproductive cough and rales. A chest x-ray film suggests atypical pneumonia. The pneumonia resolves after treatment with azithromycin. A diagnosis of psittacosis is established by the presence of complement fixing antibodies against Chlamydia psittaci in the convalescent serum. Which of the following is the most likely occupation of this patient?
A. Cat breeder
B. Florist
C. Homeless shelter worker
D. Poultry farmer
E. Slaughterhouse worker




The correct answer is D. When you see Chlamydia psittaci, one word should come to mind: birds! Infection with this organism is an occupational hazard for anyone who works with birds (e.g., veterinarians, pet store employees), including poultry farmers.

Individuals who work with cats (choice A) would be at an increased risk for infection with Pasteurella multocida (which is acquired primarily through cat bites), Bartonella henselae (cat-scratch fever), and the protozoa Toxoplasma gondii (which can be acquired from ingestion of food contaminated with cat feces).

Florists (choice B) are at increased risk for infection with the fungus Sporthrix schenckii, a primary pathogenic fungus acquired by inoculation (e.g., a rose thorn puncturing the skin).

A person who works in a homeless shelter (choice C) would be at increased risk for infection with Mycobacterium tuberculosis, because this organism is spread through the air and is prevalent in the homeless community.

Slaughterhouse workers (choice E) are at an increased risk for infection with Brucella, a bacterium that is acquired by handling infected animals.
A 3-year-old child with cystic fibrosis presents with weight loss, irritability, and a chronic productive cough. On physical examination, he is febrile and wheezing with rhonchi and rales. Chest x-ray demonstrates patchy infiltrates and atelectasis, and Gram stain of the sputum reveals slightly curved, motile, gram-negative rods that grow aerobically. The microorganism responsible for this child's pneumonia is also the most common cause of which of the following diseases?
A. Croup
B. Epiglottitis
C. Meningitis
D. Otitis externa
E. Otitis media




The correct answer is D. This is an example of a two-step question. Step 1: what is the organism that causes pneumonia in cystic fibrosis? Then, this organism is commonly seen in cases of...? Anytime you see pneumonia in a cystic fibrosis patient you should suspect Pseudomonas aeruginosa. The Gram stain revealing aerobic, gram-negative rods confirms your suspicion in this case. Now the question is: which of the diseases listed is also caused by Pseudomonas? The answer is otitis externa. P. aeruginosa is often found in the external ear, especially if the conditions are moist ("swimmer's ear") and there is any sort of inflammation. External otitis is usually a benign process with the only symptoms being an itchy, painful ear. If, however, the organism penetrates the epithelium and invades the soft tissue, cartilage, and cortical bone, the process becomes malignant otitis externa, which can progress to osteomyelitis leading to cranial nerve palsies. This condition is most common in diabetics.

Croup (choice A), also called laryngotracheobronchitis, is a respiratory disease of children that presents with a characteristic "barking" cough. Croup is caused by parainfluenza virus.

Epiglottitis (choice B) is a potentially fatal infection in children, caused by H. influenzae, which presents with drooling, difficulty breathing, and stridor. The incidence of this disease has dropped dramatically with the introduction of the H. influenzae type b (Hib) vaccine.

Meningitis (choice C) is caused by numerous different bacteria, depending on the age of the patient. The most common causes include S. pneumoniae (elderly patients), H. influenzae (unvaccinated children), Group B strep and E. coli (neonates) and N. meningitidis (1 month to adult). Although P. aeruginosa can cause meningitis, it is not a common cause.

The most common causes of otitis media (choice E) include S. pneumoniae and H. influenzae. Even in cases of external ear infections with P. aeruginosa, the middle ear is typically spared.
A combined chemical/heat autoclave (Chemiclave) uses what set of chemicals/conditions?
A. Alcohol at 121 C
B. Ethylene oxide at 160 C
C. Alcohol/formaldehyde at 132 C
D. Glutaraldehyde at 121 C
E. Glutaraldehyde/formaldehyde at 160 C




The correct answer is C. Besides the steam autoclave, dry heat autoclave, and ethylene oxide sterilizer, there is a heat/chemical combination known as a Chemiclave or heat/chemical sterilizer. Its conditions are formaldehyde and alcohol at a temperature of 132 C for periods of 20-30 minutes at 20-40 psi (pounds per square inch). The chemical mixture is provided by the manufacturer and is re-collected for disposal.
A combined chemical/heat autoclave (Chemiclave) uses what set of chemicals/conditions?
A. Alcohol at 121 C
B. Ethylene oxide at 160 C
C. Alcohol/formaldehyde at 132 C
D. Glutaraldehyde at 121 C
E. Glutaraldehyde/formaldehyde at 160 C




The correct answer is C. Besides the steam autoclave, dry heat autoclave, and ethylene oxide sterilizer, there is a heat/chemical combination known as a Chemiclave or heat/chemical sterilizer. Its conditions are formaldehyde and alcohol at a temperature of 132 C for periods of 20-30 minutes at 20-40 psi (pounds per square inch). The chemical mixture is provided by the manufacturer and is re-collected for disposal.
A 10-year-old boy develops an itchy, vesicular rash, which is maximal on his face and trunk. Physical examination demonstrates a mixture of lesions, with macules, papules, vesicles, and crusted lesions. The mother reports that the lesions seem to be occurring in crops. Which of the following is the most likely diagnosis?
A. Herpes simplex I
B. Herpes simplex II
C. Measles
D. Shingles
E. Varicella




The correct answer is E. This is varicella (chicken pox), which is the primary form of infection by the herpes zoster (varicella-zoster) virus. Recurrence due to virus harbored in neurons tends to be dermatomal in distribution and is called shingles. Fever, malaise, headache, and myalgia may also be present, particularly in the prodromal phase. The rash is pruritic, papular, changing to vesicular ("dew drops on a rose petal"), pustular and finally crusting. Tzanck smear of the base of a vesicle may demonstrate multinucleated giant cells. Immunocompromised patients can be treated with acyclovir to prevent dissemination. Chicken pox may be complicated by secondary bacterial infection, pneumonia and systemic spread (immunosuppressed patients).

Herpes simplex I (choice A) causes oral vesicles and ulcers. The lesions are often very painful. A rash is not seen with herpes simplex I.

Herpes simplex II (choice B) causes genital vesicles and ulcers. The lesions are described as being beefy red and very painful.

Measles (choice C) causes a blotchy, nonvesicular rash. Prodrome consists of fever, coryza, malaise, photophobia and Koplik's spots. The rash is brick red, irregular and maculopapular.

Shingles (choice D) is the recurrent form of herpes zoster infection and usually is localized to a single dermatome. It is typically seen in the elderly.
Which of the following organisms is the most common cause of community-acquired pneumonia?
A. Chlamydia pneumoniae
B. Haemophilus influenzae
C. Mycoplasma pneumoniae
D. Staphylococcus aureus
E. Streptococcus pneumoniae




The correct answer is E. The most common bacterium implicated in community-acquired pneumonia is the pneumococcus Streptococcus pneumoniae. When community-acquired pneumonia occurs in elderly patients or patients with comorbidity, aerobic gram-negative bacilli and Staphylococcus aureus are added to the list.

The organisms listed in choices A, B, and C are important causes of community-acquired pneumonia without comorbidity in patients younger than 60 years of age, but are not the most frequent causes.

Staphylococcus aureus (choice D) is an important cause of community-acquired pneumonia (particularly in the elderly and in patients with comorbidity) but is not the most frequent cause.
A 21-year-old man presents with cough, fever, and hemoptysis. Blood tests show significantly elevated BUN and creatinine. Immunofluorescent microscopy reveals a diffuse linear pattern of fluorescence along the basement membranes of alveolar septa and glomerular capillaries. Which type of hypersensitivity is associated with this disease?
A. I
B. II
C. III
D. IV




The correct answer is B. This patient has Goodpasture syndrome, which affects both the renal and pulmonary systems. In the kidney, it causes a rapidly progressive glomerulonephritis associated with antibodies directed against a collagen component of the glomerular basement membrane (anti-GBM antibody a classic clue to this diagnosis). They are also active against the basement membrane of respiratory alveoli, accounting for the pulmonary component of the disease. These antibodies create a linear pattern on immunofluorescence.

Autoimmune reactions such as those found in Goodpasture syndrome, certain drug allergies, blood transfusion reactions, and hemolytic disease of the newborn, are classified as Type II hypersensitivities (antibody-mediated cytotoxicity). IgG or IgM antibody reacts with membrane-associated antigen on the surface of cells, causing activation of the complement cascade and, ultimately, cell destruction.

Type I (choice A) reactions (immediate, atopic, or anaphylactic) require an initial (sensitizing) exposure to an antigen. On re-exposure to the antigen, cross-linking of IgE receptors occurs on the surface of basophils and mast cells. The mast cells then release a variety of mediators, including histamine. Clinical syndromes include asthma, atopic dermatitis, eczema, hives, and allergic rhinitis.

Type III (choice C) hypersensitivity (immune complex-mediated hypersensitivity) is caused by antibodies to foreign antigens. Immune complexes of IgG or IgM with the antigen activate complement. This results in the generation of C3b, which promotes neutrophil adherence to blood vessel walls. The complexes also generate C3a and C5a (anaphylatoxins), which lead to inflammation and tissue destruction. The hallmark signs of Type III sickness, which occur 7-14 days after exposure to the offending antigen, include urticaria, angioedema, fever, chills, malaise, and glomerulonephritis. Clinical syndromes include serum sickness (e.g., penicillin, streptomycin, sulfonamide, phenylbutazone hypersensitivity) and the Arthus response. Immune complexes are also observed in systemic lupus erythematosus (SLE). Type III glomerulonephritis (e.g., poststreptococcal glomerulonephritis) is characterized by a lumpy bumpy appearance on immunofluorescence using labeled antibody specific for immunoglobulin or complement.

Type IV (choice D) is also known as delayed-type hypersensitivity (DTH). Unlike the other types, which are mediated by antibody, DTH depends on TDTH cells that have been sensitized to a particular antigen. T cells react with antigen in association with MHC class I gene products and release lymphokines. Examples include tuberculin skin sensitivity and contact dermatitis (e.g., poison ivy rash).
Which of the following is found only in gram-negative microorganisms?
A. Cell envelope
B. Exotoxin
C. Peptidoglycan
D. Periplasmic space
E. Teichoic acids




The correct answer is D. This question requires you to appreciate the key structural difference between gram-positive and gram-negative microorganisms. Most gram-negatives (other than exceptional microorganisms, such as Mycoplasma, that lack a cell wall) have a more complex cell envelope than gram-positive microorganisms. It includes both a cytoplasmic membrane and an outer membrane. Between these two membranes is the periplasmic space, which contains enzymes such as phosphatase and penicillinase, binding proteins for the transport of various nutrients, and peptidoglycan, as well as a portion of the lipoprotein that firmly anchors the outer membrane to the peptidoglycan. Gram-positives do not have outer membranes.

Choice A, the cell envelope, is incorrect because both gram-positive and gram-negative microorganisms have this structure, which is defined as all the layers that enclose the cytosol of the bacterium. It is the composition of the envelope that differs between gram-positive and gram-negative microorganisms.

Choice B, exotoxin, is not exclusive to gram-negative microorganisms, but is also found in some gram-positives. By contrast, endotoxin (lipopolysaccharide; LPS) is found exclusively in gram-negatives.

Choice C, peptidoglycan, is found in the cell walls of both gram-positive and gram-negative microorganisms. Note that there is a larger amount of peptidoglycan in gram-positive microorganisms.

Choice E, teichoic acids, are found exclusively in gram-positive organisms.
A 6-week-old infant has a 10-day history of coughing and choking spells. The child is gasping for breath and experiencing paroxysms of coughing. Encapsulated, gram-negative rods are cultured. The organism is most likely
A. Bordetella pertussis
B. Haemophilus influenzae type b
C. Klebsiella pneumoniae
D. Legionella pneumophila
E. Mycoplasma pneumoniae




The correct answer is A. The child has whooping cough, or more accurately if one goes by the chronology of the symptoms, "coughing whoop," as the patient is subject to a series of coughing episodes that are followed by a rapid inspiration of air through a narrow airway (the "whoop"). The infection of the respiratory tract by Bordetella pertussis causes a hyperreactivity of the respiratory apparatus; even the slightest stimulus will trigger a coughing episode. The offending pathogen excretes adenylate cyclase and also produces an exotoxin that inactivates the inhibitory subunit of the G-protein complex, thus activating adenylate cyclase within the cells of the respiratory system. The symptoms of pertussis last about 6 weeks and are divided into 3 stages:

1) Catarhal stage: Insidious onset with lacrimation, sneezing, anorexia, coryza and a hacking cough at night

2) Paroxysmal stage: Characterized by rapid bursts of consecutive coughs followed by a deep, high-pitched inspiration (whoop)

3) Convalesent stage: Begins 4 weeks after onset and is associated with a reduction in frequency and severity of paroxysms of cough

Haemophilus influenzae type b (choice B) was the major cause of infant meningitis before the Hib conjugate vaccine nearly eradicated this pathogen from the United States. Nonencapsulated strains cause otitis media in children and pneumonia in adults.

Klebsiella pneumoniae (choice C) is a gram-negative, encapsulated rod that is a significant pulmonary pathogen in individuals with a compromised respiratory apparatus. It is a common cause of aspiration pneumonia and pulmonary abscesses in alcoholics and patients with chronic obstructive pulmonary disease.

Legionella pneumophila (choice D) is another fastidious, gram-negative, respiratory pathogen that may cause either a fulminating disease or a mild "walking pneumonia-like" condition (i.e., an atypical pneumonia).

Mycoplasma pneumoniae (choice E) is the most common cause of primary atypical pneumonia. The disease is considered "atypical" because the patients have a very mild disease with low-grade fever, little in the way of constitutional signs, and a non-productive cough. These organisms are fastidious and are difficult to grow in the laboratory.
An AIDS patient develops symptoms of pneumonia, and Pneumocystis carinii is suspected as the causative organism. Which of the following stains would be most helpful in demonstrating the organism's cysts?
A. ELISA
B. Hematoxylin and eosin
C. Methenamine silver
D. Prussian blue
E. Western blot




The correct answer is C. The appropriate stain is methenamine silver, since routine hematoxylin and eosin does not adequately demonstrate the organisms. The cysts, when stained with methenamine silver, have a characteristic cup or boat shape; the trophozoites are difficult to demonstrate without electron microscopy. It is also worth knowing that sputum samples are not nearly as effective as bronchial washes in demonstrating the organisms.

ELISA (choice A) is enzyme-linked immunosorbent assay. It is the primary test for HIV antibody. It is not a stain.

Hematoxylin and eosin (choice B) is the routine tissue stain used in pathology laboratories.

Prussian blue (choice D) is good for demonstrating iron.

The western blot (choice E) is the confirmatory test of choice for diagnosing HIV infection. The western blot test is generally performed once the ELISA test has been shown to be positive.
A sexually active man presents to a dermatologist because of a severe mucocutaneous rash that involves most of his body, including his palms and soles. These mucous patches are described as being painless, silvery ulcerations of mucous membrane with surrounding erythema. If iritis and arthritis are also seen, which of the following is the most likely causative agent of this rash?
A. Herpes simplex I
B. Herpes simplex II
C. HIV
D. Neisseria gonorrhoeae
E. Treponema pallidum




The correct answer is E. The rash described is that of secondary syphilis, caused by Treponema pallidum. Involvement of palms and soles by a rash is unusual, and secondary syphilis should come to mind. Not all patients with secondary syphilis have a severe form of the rash, and consequentially some cases are missed. Primary syphilis takes the form of a painless, button-like mass called a chancre. Tertiary syphilis, which is now rare, has a propensity for involving the aorta and central nervous system and can also cause "gummas" (granulomatous-like lesions) in many sites, notably including liver and bone. The mucous patches are described as being painless silvery ulcerations of mucous membrane with surrounding erythema. Other symptoms include cranial neuropathies (II - VIII), iritis, glomerulonephritis, arthritis and periositis.

Herpes simplex I (choice A) usually causes perioral vesicular lesions.

Herpes simplex II (choice B) usually causes genital vesicular lesions.

HIV (choice C) does not itself cause a rash, although co-infection with other organisms can result in a rash.

Neisseria gonorrhoeae(choice D) does not typically cause a rash but does not cause urethral discharge and dysuria.
An African child develops massive unilateral enlargement of his lower face in the vicinity of the mandible. Biopsy demonstrates sheets of medium sized blast cells with admixed larger macrophages. This type of tumor has been associated with which of the following?
A. Epstein-Barr virus
B. Hepatitis B
C. Herpesvirus
D. HIV
E. Human papillomavirus




The correct answer is A. The patient has Burkitt lymphoma. This high-grade B-cell lymphoma occurs endemically in Africa (it is the most common neoplasm in children in an equatorial belt that includes Africa and New Guinea) and sporadically in the U.S. and Europe. The sporadic form is often in an abdominal site and occurs in young adults. The African form of Burkitt lymphoma has been strongly associated with antibodies directed against Epstein-Barr virus; the association is weaker in sporadic cases.

Hepatitis B (choice B) is associated with hepatocellular carcinoma.

Herpesvirus (choice C) type 8 is associated with Kaposi sarcoma.

HIV (choice D) is linked to AIDS. Some patients also develop primary lymphomas (not usually Burkitt).

Human papillomavirus (choice E) is linked with common warts, genital condylomata, and genital cancers.
A 4-month-old infant presents with failure to thrive, progressive muscular weakness, poor head control, dysphagia, and dry mouth. The infant often eats soy-based formula sweetened with honey. Which of the following organisms is most likely to be responsible for the child's presentation?
A. Clostridium botulinum
B. Clostridium difficile
C. Clostridium perfringens
D. Clostridium tetani
E. Corynebacterium diphtheriae




The correct answer is A. The child has infant botulism (floppy baby syndrome)that is caused by germination of Clostridium botulinum spores (found in honey) in the baby's gastrointestinal tract. Patients improve when honey is removed from the diet. This disorder is most common in children under the age of 6 months; older children and adults do not seem to be vulnerable to this form of botulism, but are susceptible to botulism caused by ingestion of preformed toxin. Patients usually present with a sudden onset of dry mouth, diplopia, dysphagia, dysphonia, and muscle weakness progressing to respiratory paralysis. Pupils are often fixed and dilated.

Clostridium difficile(choice B) causes pseudomembranous colitis, especially after antibiotic therapy. It causes severe diarrhea through the production of toxin from these bacteria.

Clostridium perfringens(choice C) causes gas gangrene and gastroenteritis, and it is not associated with ingestion of honey.

Clostridium tetani(choice D) causes tetanus and does not cause a food-borne illness in infants.

Corynebacterium diphtheriae(choice E) causes diphtheria in susceptible individuals.
A 66-year-old man with an untreated urinary tract infection develops a spiking fever, tachypnea, and severe hypoxemia. If adult respiratory distress syndrome is caused as a result of the urinary tract infection, blood cultures would most likely reveal
A. Gram-negative diplococci
B. Gram-negative rods
C. Gram-positive cocci
D. Gram-positive diplococci
E. Gram-positive rods




The correct answer is B. Escherichia coli is a very common pathogen associated with urinary tract infections and is a common cause of cystitis. In fact, more than 90% of all "first" infections are caused by Escherichia coli. It is part of the normal flora of the gastrointestinal tract. Patients with cystitis can develop bacteremia and subsequent septic shock and adult respiratory distress syndrome (ARDS).E. coli is a gram-negative rod. The key to this question is knowing that most urinary tract infections are caused by E. coli.

Gram-negative diplococci (choice A) might be Neisseria spp. or Moraxella catarrhalis. Neisseria gonorrhea is the cause of gonorrhea, a sexually transmitted disease that presents with urethritis or may be asymptomatic. Patients are usually young, sexually active males. Moraxella spp. are gram-negative cocci that can cause a wide variety of infections, usually implicated as a cause of otitis media and sinusitis in children, or as a cause of purulent tracheobronchitis and pneumonia in a population of patients who are over 50 years of age and have underlying obstructive lung disease.

Gram-positive cocci (choice C), such as Staphylococcus aureus, S. epidermidis and Streptococci, rarely cause cystitis. Staphylococcus saprophyticus causes urinary tract infections, but the patients are typically young, sexually active women.

Gram-positive diplococci (choice D) would be a description of Streptococcus pneumoniae, which is the most common cause of community-acquired pneumonia. This is rarely a cause of UTIs.

Gram-positive rods (choice E) would include members of the following genera: Clostridium, Bacillus, Listeria, and the coryneform bacteria. The only significant member of this group to produce urinary tract infections is Corynebacterium urealyticum.The organism creates an alkaline urine environment with the potential for stone formation. Patients are usually immunocompromised or have had recurrent urinary tract infections.
Which organism is LEAST likely to be involved in lesions of chronic adult periodontitis?
A. Spirochetes
B. Bacteroides melaninogenicus
C. Capnocytophaga
D. Streptococcus mitis
E. Fusobacteria




The correct answer is choice D. Periodontal disease pathogens are usually anaerobic or capnophilic. The term capnophilic means "carbon-dioxide loving". In the anaerobic environment deep in the sulcus, the following genera are often said to contribute to the tissue destruction of periodontal disease: Bacteroides, Porphyromonas gingivalis, Borelia, Spirochetes,Capnocytophaga, Fusobacteria and Eichenella. Streptococci (mutans, mitis, sanguis, salivarius) are all gram positive facultative anaerobes that do not require anaerobic conditions to survive. They are Viridans streptococci, generally non-pathogenic, and not linked to periodontal disease, although S. mutans is clearly linked to caries.
If a dentist were to administer a bacteriostatic antibiotic, such as a macrolide, and a bactericidal antibiotic, such as a cephalosporin, concomitantly to a patient with a severe intraoral infection, the most likely pharmacological result for the patient would be which of the following?
A. A small extension of bacterial coverage between the two agents
B. Antagonism of the antibacterial effects of both agents
C. Profound toxicity for the patients taking both agents
D. Synergism of the antibacterial effects of both agents
E. The appearance of mild antifungal activity




The correct answer is B. The concomitant administration of bacteriostatic and bactericidal antibiotics would result in antagonism of the antibacterial effects of both agents. The reason for the antagonism between the two is that each agent interferes with the mechanism of action of the other agent. By definition, a bacteriostatic agent is one that prohibits the growth and development of bacteria and a bactericidal agent is one that directly causes the death of a microbe during the growth and development stages.

An extension of bacterial coverage between two agents (choice A) is typically seen when two bacteriostatic or two bactericidal agents are given together.

The administration of a macrolide and a cephalosporin may cause some discomfort for the patient, such as nausea or stomach cramps; however, profound toxicity for the patients taking both agents (choice C) is very unlikely.

Synergism (choice D) is the correlated action of two agents that results in a final effect that is greater than that of each agent acting separately. A classical example of synergism is seen when an aminoglycoside, such as gentamicin, is administered concomitantly with extended spectrum penicillin, such as piperacillin, for the treatment of a pseudomonas infection.

The appearance of mild antifungal activity (choice E) would not be seen when bacteriostatic and bactericidal antibiotics are given together, if neither have any antifungal activity.
Which of the following viruses is capable of replication in enucleated cells?
A. Adenovirus
B. Cytomegalovirus
C. Influenza virus
D. JC virus
E. Poliovirus




The correct answer is E. Most RNA viruses (eg, poliovirus) replicate in the cytoplasm and therefore can replicate in enucleated cells. Poliovirus belongs to the family Picornaviridae. These viruses are nonenveloped and have an icosahedral nucleocapsid that contains positive-sense RNA.

The exception to the rule regarding RNA viruses is the family Orthomyxoviridae, the influenza viruses (choice C). Orthomyxoviruses undergo transcription and RNA replication in the nucleus of the host cell because they need to cannibalize the capped 5' termini of cellular RNAs for use as primers for viral mRNA transcription.

Adenoviruses (choice A) are nonenveloped and have an icosahedral nucleocapsid that contains a double-stranded linear DNA genome.

Cytomegalovirus (choice B) is a member of family Herpesviridae. It is an enveloped virus with an icosahedral nucleocapsid that contains a double-stranded linear DNA genome.

JC virus (choice D) belongs to family Papovaviridae. It is nonenveloped and has an icosahedral nucleocapsid that contains a double-stranded circular DNA genome.

For most DNA viruses, transcription and DNA replication occur in the nucleus of the host cell. The exception to this observation is the family Poxviridae, which carries out its replication in the cytoplasm. Poxviridae includes variola virus, vaccinia virus, molluscum contagiosum, and orf virus.
Blood cultures for a patient are positive for alpha-hemolytic gram-positive diplococci. Immunity to the causative organism is based on
A. alternative complement pathway activation
B. antibody to an alpha-helical coiled fimbria
C. IgA antibodies to C carbohydrate
D. IgG antibodies to C carbohydrate
E. IgG antibodies to a surface acidic polysaccharide




The correct answer is E. The patient in this question has pneumococcal pneumonia. It is caused by Streptococcus pneumoniae, an alpha-hemolytic, gram-positive coccus that grows in chains. It can be easily distinguished from other alpha-hemolytic streptococci because it is exquisitely sensitive to bile and bile-like compounds, such as optochin. It is the most common cause of community-acquired pneumonia and the most common cause of community-acquired meningitis in adults older than 30 years of age. The only recognized virulence factor of S. pneumoniae is its carbohydrate capsule (which contains acidic polysaccharides). Antibody to a specific capsule type is necessary to overcome infection. More than 80 capsule types have been recognized. The 23 types that most commonly cause disease are contained in a vaccine (pneumovax) that is recommended for high-risk groups, including the elderly and those undergoing splenectomy. Increased susceptibility is also found in patients with Hodgkin disease, chronic lymphocytic leukemia, and myeloma. The vaccine should still be given to patients with these conditions, but it is less successful.

The alternative complement pathway (choice A) is important in clearing Neisseria infections. Individuals with deficiencies in C5 through C8 are at increased risk for disease from Neisseria.

The fimbria of Streptococcus pyogenes (Group A beta-hemolytic streptococcus) is composed of an alpha-helically coiled M protein. Antibody against a specific M type (choice B) will prevent infection. Raising antibodies to M proteins, however, can lead to rheumatic fever, so strep throat infections are routinely treated with penicillin to prevent an antibody response.

The C carbohydrate is an antigen of beta-hemolytic streptococci used to divide them into different groups. Antibody against C carbohydrate (choices C and D) is not protective.
A microbiology laboratory reports growth of Staphylococcus aureus from pus drained from a breast abscess. What is the most likely condition predisposing the patient to the development of a breast abscess?
A. Breast feeding
B. Endocarditis
C. Inflammatory breast carcinoma
D. Menopause
E. Paget disease of the breast




The correct answer is A. Acute mastitis, frequently complicated by breast abscess formation, typically develops in the postpartum period when the nipples are first subjected to the physical stresses of breast feeding. Bacteria enter the breast by way of cracks in the nipple and flourish in the microenvironment of the lactating breast. Acute mastitis causes redness, pain, and swelling in the affected breast; S. aureus is the most common pathogen. There is a very apparent cellulitis and fever may be present. Penicillin antibiotics are generally administered because they are effective and safe to use during breast-feeding.

Although endocarditis (choice B) could send septic emboli to the breast, S. aureus endocarditis is more commonly associated with bacteremia and fevers, proliferative glomerulonephritis, valvular dysfunction, and emboli to the brain, kidneys, heart, and gut.

Inflammatory breast carcinoma (choice C) does not invoke an inflammatory response, and it is not associated with an infection. Inflammatory breast carcinoma is characterized by tumor spread into the dermal lymphatics, producing diffuse induration and skin tenderness with the typical peau d'orange (orange peel) appearance.

Mastitis arising in the perimenopausal period (choice D) is usually a chronic mastitis caused by obstruction of ducts from inspissated secretions. Chronic mastitis is sterile. The breast tissue shows lymphocytes and plasma cells surrounding dilated ducts filled by cellular debris.

Paget disease (choice E) presents as skin changes on the breast or nipple in association with ductal carcinoma in the underlying breast that is percolating out into the epidermis. Paget disease produces an eczematous skin lesion that may be crusted or weeping, but it is not related to bacterial infection.
A 3-year-old child has multiple isolated lesions on his face and neck. The lesions are up to 4 cm in diameter with golden crusts, whereas in other sites small blisters and weeping areas are seen. Which of the following is the most likely diagnosis?
A. Aphthous ulcers
B. Erysipelas
C. Herpes simplex I
D. Impetigo
E. Measles




The correct answer is D. This is impetigo, which is typically seen in preschool children with poor hygiene, particularly in the summer in warm climates. The characteristic lesion has a large golden crust. Most cases are caused by Staphylococcus aureus; Streptococcus pyogenes is occasionally implicated. Impetigo is highly infectious, and mini-epidemics can occur in daycare settings. The initial treatment is typically with penicillins and topical preparations, primarily muciprocin ointment. Methicillin-resistant strains are presently rare in this setting, but can occur. Itching is the only symptom. The lesions consist of macules and vesicles.

Aphthous ulcers (choice A), commonly known as "canker sores," are painful, shallow ulcers of the oral cavity.

Erysipelas (choice B) is a different type of skin infection, often caused by Streptococcus pyogenes (also sometimes others including Staphylococcus), and is characterized by large erythematous patches.

Herpes simplex I (choice C) causes tiny oral and perioral vesicles, but not large golden crusts.

Measles (choice E) causes a blotchy erythematous rash on the face and all over the body. These lesions typically do not "weep" as seen with impetigo.
A woman delivers a full term baby with possible toxoplasmosis, but no obvious signs of infection with the protozoan parasite. The best test to diagnose acute infection in the newborn would be to look for:
A. IgA
B. IgE
C. IgG 1
D. IgG 4
E. IgM




The correct answer is E. IgM immunoglobulin directed against Toxoplasma would provide evidence of infection in the newborn baby. IgM is the only antibody that a baby can form with an acute infection. It is the first antibody that is formed in an infection and it is also the antibody that is present on the surface of immature and mature B cells.

IgA (choice A) is the antibody that is produced in response to mucosal infections. It cannot cross the placenta. The baby's immune system is still not well developed at birth and cannot produce IgA immunoglobulin. In adults or older children, IgA exists in serum in a monomeric form but is present in seromucous secretions as a dimer.

IgE (choice B) is the antibody that is produced in response to an allergen. It cannot cross the placenta, and is not present in the baby at birth or for some time after birth. The baby's immune system is still not well developed at birth and can initially only form IgM immunoglobulin.

IgG 1 (choice C) and IgG4 (choice D) immunoglobulins might be present in the baby, but would not indicate infection in the infant. The presence of these isotypes would indicate that the mother was infected with the organism and produced antibody that was transported across the placenta in utero. IgG is the only immunoglobulin that can cross the placenta, providing protection to the baby during the first few months of life.
Two weeks after birth, a neonate develops sepsis, skin vesicles, and conjunctivitis. Over the next several days, the infant's condition deteriorates with development of seizures, cranial nerve palsies, and lethargy. The infant dies approximately 1 week after onset of symptoms. Which of the following infectious agents would most likely cause this clinical presentation?
A. Cytomegalovirus
B. Herpes simplex
C. Rubella
D. Syphilis
E. Toxoplasmosis




The correct answer is B. All of the agents listed can cause devastating congenital infections with high mortality and often major organ malformation (the TORCH agents: Toxoplasma, other, rubella, cytomegalovirus, herpes simplex). It is herpes simplex type II, however, typically acquired during delivery, that causes the devastating neonatal encephalitis described in the question stem. The mortality rate for neonatal herpes is approximately 65%. Only 10% of these infants escape without neurologic sequelae. A distinguishing feature is the propensity to involve the temporal lobe, with mass effect on imaging studies and temporal lobe seizure foci on EEGs. A point worth remembering is that adult herpes encephalitis (in nonimmunosuppressed individuals) is usually caused by herpes simplex I, whereas neonatal and congenital herpes are usually caused by herpes simplex II.
A young woman presents with a confluent maculopapular rash that began on her face then spread downward over her trunk. She also has fever and headache, with bilateral neck pain and joint pain. Which of the following diseases does she most likely have?
A. Infectious mononucleosis
B. Lyme disease
C. Roseola
D. Rubella
E. Rubeola




The correct answer is D. Rubella, also called German measles or 3-day measles, is a disease caused by a Togavirus, which is a small, enveloped, single-stranded (+) linear RNA virus. Approximately 40% of patients are asymptomatic or have mild symptoms. In symptomatic patients, the clinical presentation typically consists of an erythematous rash that begins on the head and spreads downward to involve the trunk, and lasts for approximately 3 days. In addition, symptoms include fever, posterior cervical lymphadenopathy, and arthralgias. The greatest danger from rubella is to the fetus. If clinical rubella develops or seroconversion is demonstrated, there is a high risk for congenital abnormalities or spontaneous abortion. Women of childbearing age should be warned not to become pregnant within 2-3 months from the time of immunization. Mild arthralgias and other symptoms may develop in 25% of immunized women. Enteroviral rashes may mimic rubella and rubeola.

Infectious mononucleosis (choice A) is caused by the Epstein-Barr virus, a herpesvirus. Classic findings include fever, exudative pharyngitis, generalized lymphadenopathy, severe malaise (most common complaint), and hepatosplenomegaly. A rash is not a characteristic feature unless the patient has been treated with ampicillin.

Lyme disease (choice B) is caused by the spirochete Borrelia burgdorferi. The disease is transmitted by the bite of the tick Ixodes dammini. The initial lesion is an annular rash with central clearing and a raised red border (erythema chronicum migrans) at the bite site. Patients also have fever, malaise, myalgias, arthralgias, headache, generalized lymphadenopathy, and, occasionally, neurologic findings.

Roseola (choice C) is caused by human herpesvirus 6. Other names include exanthem subitum or sixth disease. Children have a febrile period of 3-5 days with rapid defervescence followed by an erythematous maculopapular rash lasting 1-3 days.

Rubeola (choice E), or regular measles, is a disease caused by a paramyxovirus. Patients present with an upper respiratory prodrome and characteristic oral lesions (Koplik spots) that precede the rash. The nonpruritic maculopapluar rash begins on the face and spreads to the trunk and extremities, including palms and soles. The incubation period is 10-14 days. Patients also have a posterior cervical lymphadenopathy. The virus is not associated with risk for a fetus.
A 29-year-old woman AIDS has had a progressive blurring of vision in her right eye. Which of the following is the most appropriate therapy for this patient?
A. Acyclovir
B. Amantadine
C. Flucytosine
D. Ganciclovir
E. Zidovudine




The correct answer is choice D. The best drug treatment for CMV this infection of the retina is ganciclovir.

Acyclovir (choice A) is not effective in CMV infections. It is used more for HSV type 1 and 2 infections.

Amantadine (choice B) is used either therapeutically or prophylactically for the influenza A virus.

Flucytosine (choice C) is an antifungal agent.

Zidovudine (choice E) is a reverse transcriptase inhibitor and is used as a first-line drug for the treatment of AIDS. The drug by itself is ineffective against CMV retinitis.
The causative organism of gonorrhea has which of the following characteristics?
A. Both hyphae and spores
B. Can live within neutrophils
C. Cannot make ATP
D. No true cell wall
E. Stains with silver stains




The correct answer is B. The causative organism is Neisseria gonorrhoeae, which is a sexually transmitted gram-negative coccus that can live in phagocytic vacuoles in neutrophils and macrophages. The organism is the most common cause of septic arthritis in otherwise healthy, sexually active adults. Cervicitis in women with purulent discharge or women who are asymptomatic, yielding a positive culture, is a common charactristic. Fever, rash, tenosynovitis and arthritis occur with disseminated disease.

Hyphae and spores (choice A) would be features of Candida, which usually causes arthritis as a rare complication of systemic candidiasis in immunocompromised patients. Furthermore, there would be the appearance of other classic characteristics, such as the appearance of creamy white lesions.

Chlamydia is an intracellular organism that cannot make ATP (choice C) and does not usually cause septic arthritis.

Mycoplasma do not have true cell walls (choice D) and do not usually cause septic arthritis, but are commonly associated with respiratory conditions, such as mycoplasma tuberculosis.

Syphilis is caused by a spirochete that stains with silver stains (choice E). Secondary syphilis can (uncommonly) cause an acute arthritis, but the interval between acquiring the infection and developing the arthritis is usually months to years.
A large lung abscess is biopsied from which pure Fusobacterium nucleatum is cultured. What is the most likely source of the Fusobacterium?
A. Blood
B. Infected aerosols
C. Oral cavity
D. Stomach
E. Upper respiratory tract




The correct answer is C. Anaerobic bacteria make up most of the normal human oral flora. For example, prominent members of the normal microbial flora of the mouth include anaerobic spirochetes, fusobacteria, and prevotella. Anaerobic lung abscesses commonly arise from aspirated oral secretions in patients with compromised neurologic status (e.g., anesthesia, coma, intoxication), or in individuals with a depressed cough reflex. The most common anaerobic organisms isolated from lung abscesses include Bacteroides, Fusobacterium, and Peptococcus species, all of which are typical oral flora.

Blood-borne pathogens (choice A) producing lung abscesses probably originate as emboli from vegetations on the right heart. The most likely pathogens in endocarditis are Staphylococcus and Streptococcus species; Fusobacterium would be very unusual.

Fusobacterium usually is not pathogenic and lives in a commensal relationship with the host. Infective aerosols (choice B) generally contain organisms that are intrinsically pathogenic (e.g., mycobacteria, influenza virus).

It is the acidity of the gastric aspirate that most seriously injures the lung, not the gastric bacteria. Fusobacterium do inhabit the gastrointestinal tract, but viable bacteria are much more likely to be transmitted from the oral secretions than from the stomach (choice D).

The upper respiratory tract (choice E) is generally sterile, although COPD is frequently complicated by chronic bronchitis. Numerous bacterial and viral pathogens can be involved in chronic bronchitis; however, Fusobacterium is not an upper respiratory tract pathogen.
A 67-year-old woman in a skilled nursing facility complains of flu-like symptoms of high fever, dyspnea, and a productive cough. A chest x-ray shows a cavitary lesion in her left lung which is confirmed to be an abscess. If a gram-positive bacteria is noted, which of the following organisms would most likely be identified from examination of her sputum?
A. Candida albicans
B. Klebsiella pneumoniae
C. Pneumocystis carinii
D. Staphylococcus aureus
E. Streptococcus pneumoniae




The correct answer is D. The woman has developed a pulmonary abscess, as evidenced by the chest radiograph. Of the organisms listed, Staph. aureus is the most likely cause of bacterial pneumonia complicated by abscess formation. Bacteria commonly producing pneumonia developing in the context of influenza include Staphylococcus aureus, Haemophilus influenzae, and Streptococcus pneumoniae, but Streptococcus pneumoniae is not a frequent cause of lung abscesses.

Staph. aureus pneumonia/abscess are more common in patients in chronic care facilities, nosocomial infections, influenza epidemics and cystic fibrosis.

Candida albicans(choice A) would not be a likely cause of this woman's abscess, or of the pneumonia that preceded it.Candida pneumonia in a nonimmunosupressed individual is rare. Note also that Candida is a fungus.

Klebsiella pneumoniae(choice B) is more likely in men who are alcoholic or diabetic, or suffering from chronic obstructive lung disease. It is also a plump gram-negative encapsulated rod.

Pneumocystis carinii(choice C) is associated with pneumonia in immunocompromised hosts, especially AIDS patients, but there is no indication of immune compromise in this patient. Furthermore, it would not show as a gram-positive organism, as it is a fungus.

Streptococcus pneumoniae(choice E) is only rarely associated with lung abscess. It commonly causes pneumonia in chronic lung diseases. It is a gram-positive diplococcius.
Following a barbecue hosted by a hunter who served "bear hamburgers," several guests develop abdominal pain, diarrhea, fever, myalgia, periorbital edema, and petechial hemorrhages in the conjunctiva. A peripheral blood smear shows an increased eosinophil count. Which of the following is the most likely cause of these symptoms?
A. Anthrax
B. Bacillus cereus food poisoning
C. Botulism
D. Escherichia coli gastroenteritis
E. Trichinosis




The correct answer is E. Although inadequately cooked pork and trichinosis are commonly associated, the quality of purchased pork and the degree of cooking in this country are sufficiently high that pork-related trichinosis is uncommon. The more common scenario is actually the one described in the question stem, for two reasons. First, bear meat is not inspected, and it may often be served "rare," or relatively uncooked. The infected meat contains cysts that are released in the gastrointestinal tract after ingestion. The Trichinella spiralis larvae penetrate the intestinal mucosa and develop into adult worms in 30-40 hours. After reaching adulthood, they mate, and the females produce larvae that grow to maturity and seek out muscle in which to encyst (often in the orbital muscles) within approximately 5-8 days. If you didn't know the answer to this question, you still might have noticed the presence of eosinophilia, which should have clued you in to the presence of a parasitic infestation, making trichinosis the only plausible answer.

Anthrax (choice A) after eating bear meat is very unlikely. In addition, anthrax would not produce the described symptoms; instead, cutaneous lesions (95% of cases) or respiratory disease culminating in death (5% of cases; wool sorter disease) would occur.

Bacillus cereus food poisioning (choice B) and Escherichia coli gastroenteritis (choice D) might lead to similiar gastrointestinal symptoms but would not cause the eye findings.

Botulism (choice C) causes flaccid paralysis, not the symptoms described.
An alcoholic patient is brought to the emergency department in respiratory distress. A chest x-ray film demonstrates lobar consolidation of the right lower lung. Which of the following organisms should be highest on the differential diagnosis?
A. Klebsiella pneumoniae
B. Legionella spp.
C. Mycoplasma pneumoniae
D. Pneumocystic carinii
E. Staphylococcus aureus




The correct answer is A. Lobar pneumonia, in which an entire lobe of the lung becomes rapidly affected with pneumonia, is acutally a relatively uncommon pattern for pneumonia. Common causative organisms include Streptococcus pneumoniae (pneumococcus), Haemophilus influenzae, and Klebsiella pneumoniae. K. pneumoniae is specifically associated with alcohol abuse, diabetes mellitus, and nosocomial infections.

Legionella spp. (choice B) can cause either a patchy or lobar consolidation but are not specifically associated with alcoholism. Instead, Legionella infection is associated with inspiration of aerosolized contaminated water.

Mycoplasma pneumoniae (choice C) causes extensive patchy infiltrates rather than lobar pneumonia. It is the most common cause of pneumonia in young adults.

Pneumocystis carinii (choice D) causes diffuse interstitial and alveolar infiltrates, typically in patients immunosupressed by AIDS, cytotoxic drug therapy, or cancer.

Staphylococcus aureus (choice E) produces patchy infiltrates. Staphylococcal pneumonia may complicate influenza during epidemics, or it may be a nosocomial infection. It is associated with empyema formation.
An otherwise healthy patient who has just received a prosthetic aortic valve develops postoperative fever. Blood cultures are done, and she is given broad-spectrum antibiotics. Two days later she is still febrile and clinically deteriorating. Which of the following is the most likely pathogen?
A. Actinomyces israelii
B. Candida albicans
C. Histoplasma capsulatum
D. Nocardia asteroides
E. Trichophyton rubrum




The correct answer is B. The patient likely has candidal infection of the prosthetic aortic valve. That is why she did not respond favorably to antibacterial therapy, which is known to promote fungal infection. Note that Candida tends to colonize foreign bodies, such as intravenous and Foley catheters, prosthetic valves, and ventricular shunts.

Actinomyces israelii (choice A) is known to cause cervicofacial infection in patients undergoing dental work.

Histoplasma capsulatum (choice C) causes histoplasmosis, a pulmonary infection common in the midwestern river valleys. Multiorgan involvement is usually seen only in the immunocompromised. Transmission of the organism occurs through the inhalation of airborne microconidia (infectious) spores. The organism is found in bird and bat droppings and in the soil.

Nocardia asteroides (choice D) is an actinomycete that causes a chronic lobar pneumonia that may metastasize to the brain. It is more common in immunocompromised patients. It is found in the soil and in aquatic environments.

Trichophyton rubrum (choice E) is one of the organisms that commonly produces a variety of cutaneous mycoses, including tinea corporis (ringworm), tinea cruris (jock itch), and tinea pedis (athlete's foot).
A formula-fed, 1-month-old boy is exposed to his sister, who has chickenpox. He does not develop signs of varicella. His mother had the infection 5 years ago. Which class of immunoglobulins did he acquire from his mother in utero that protected him from this virus?
A. IgA
B. IgD
C. IgE
D. IgG
E. IgM




The correct answer is D. This infant is exhibiting passive immunity acquired from his mother in utero. IgG is the only class of immunoglobulin that can cross the placenta. As such, IgG molecules diffuse into the fetal circulation, providing immunity. This circulating maternal IgG protects the newborn during the first 4-6 months of life. Note that IgG is also capable of opsonization and complement activation (a feature shared with IgM).

IgA (choice A) functions in the secretory immune response. The secretory form of this immunoglobulin (sIgA) is found in tears, colostrum, saliva, breast milk, and other secretions. It is produced by the plasma cells in the lamina propria of the gastrointestinal and respiratory tracts.

IgD (choice B) functions as a cell surface antigen receptor on undifferentiated B cells.

IgE (choice C) is involved in the allergic response and immediate hypersensitivity reactions. The Fc region of IgE binds to the surface of basophils and mast cells. Antigen binding to two IgE molecules leads to mast cell degranulation and the release of leukotrienes, histamine, eosinophil chemotactic factors, and heparin.

IgM (choice E) is the first antibody detected in serum after exposure to antigen. IgM circulates as a pentamer and thus has five Fc regions. This structure makes it especially effective in fixing complement.
Which of the following organisms is a natural transformer?
A. Escherichia coli
B. Neisseria gonorrhoeae
C. Plasmodium vivax
D. Pseudomonas aeruginosa
E. Staphylococcus aureus




The correct answer is B. Transformation is the uptake and integration of naked DNA by a bacteria from the environment. Transformation can be induced in the laboratory (a technique used to introduce gene-carrying plasmids into bacteria) or, relatively uncommonly, occurs naturally. Only a few medically important species undergo natural transformation: Haemophilus species, Streptococcus species, Neisseria gonorrhoeae, and Helicobacter pylori.
A patient presents with chronic, symmetric polyarthritis involving the knees. He gives a history of having developed an extensive rash after a deer hunting trip in Connecticut after which he felt "sick," and developed knee pain that prevented him from climbing stairs for several months, but then it partially resolved. Which of the following organisms is most likely etiologically related to the patient's arthritis?
A. Fungus
B. Gram-negative cocci
C. Gram-negative rod
D. Gram-positive cocci
E. Spirochete




The correct answer is E. The history is characteristic for Lyme disease, caused by the spirochete Borrelia burgdorferi. The clues are deer, Connecticut, rash, knees, and acute arthritis followed by chronic arthritis. Patients with Lyme disease initially have erythema migrus, a flat or slightly raised lesion that expands with central clearing. Headache and stiff neck are also common. Arthralgias, arthritis, and myalgias are seen. The arthritis is often chronic and recurrent.

Fungal arthritis (choice A) can be caused by coccidioidomycosis, blastomycosis, sporotrichosis, and candidiasis.

N. gonorrhoeaeare gram-negative cocci (choice B) that can cause septic arthritis. Septic arthritis is associated with acute arthritis, usually monarticular, most often in large weightbearing joints. Previous joint damage or intravenous drug abuse are common risk factors.

Haemophilus (in young children) and Salmonella (in sickle cell patients) are gram-negative rods (choice C) that can cause septic arthritis.

Staphylococcus are gram-positive cocci (choice E) that can cause septic arthritis.
What is the mechanism of action of the antibiotic ciprofloxacin?
A. It inhibits dihydrofolate reductase
B. It inhibits DNA-dependent RNA polymerase
C. It inhibits protein synthesis by binding to the 30s ribosomal subunit
D. It inhibits protein synthesis by binding to the 50s ribosomal subunit
E. It inhibits topoisomerase II (DNA gyrase)




The correct answer is E. Antibiotic mechanism questions are NBDE Part 1 favorites. Ciprofloxacin and norfloxacin belong to a category of antibiotics called the fluroquinolones. They are bactericidal and work by inhibiting topoisomerase II (DNA gyrase). They are effective against gram-negative rods and are the only oral agents effective against Pseudomonas

Inhibition of dihydrofolate reductase (choice A) is the mechanism of action of trimethoprim, which is typically used in combination with sulfonamides which inhibit an earlier step in folate synthesis (dihydropteroate synthase). Trimethoprim-sulfa is used in the treatment of Shigella, Salmonella, recurrent urinary tract infections, and Pneumocystis carinii pneumonia.

Inhibition of DNA-dependent RNA polymerase (choice B) is the mechanism of action of rifampin. Rifampin is used (along with other drugs) in the treatment of tuberculosis.

Inhibition of the 30s ribosomal subunit (choice C) is the mechanism of action of two important classes of antibiotics - the tetracyclines (tetracycline, doxycycline, democlocycline) and the aminoglycosides (gentamycin, tobramycin, streptomycin, etc.). The tetracyclines inhibit the attachment of the aminoacyl-tRNA to the ribosome, whereas the aminoglycosides inhibit the formation of the intitiation complex.

Inhibition of the 50s ribosomal subunit (choice D) is the mechanism of action of the macrolides (e.g., erythromycin), the lincosamides (e.g., lincomycin, clindamycin), and chloramphenicol. Chloramphenicol inhibits the 50s peptidyl transferase, whereas erythromycin blocks translocation.
An injection of lipopolysaccharide (LPS) into the vascular system will rapidly produce myocardial dysfunction, hypotension, disseminated intravascular coagulation, and coma. This sequence of events most closely mimics what type of shock?
A. Anaphylactic
B. Cardiogenic
C. Hypovolemic
D. Neurogenic
E. Septic




The correct answer is E. Since lipopolysaccharide can be seen in gram-negative bacteria, it is important to understand how it will affect the body. Septic shock is a complex, multisystem organ failure that can be produced either by LPS (which is present in the cell wall of all gram-negative bacteria) or certain toxins released by gram-positive bacteria and fungi. LPS binds to a serum protein and stimulates CD14 receptors on endothelial cells and circulating inflammatory cells, eliciting a broad range of end-organ responses.

Anaphylactic shock (choice A) is brought about by an exaggerated Type I hypersensitivity reaction mediated by IgE antibodies bound to mast cells and basophils. The resulting degranulation produces massive histamine and adenosine release, which produces constriction of the bronchi and pulmonary circulation.

Cardiogenic shock (choice B) reflects the inability of the heart to maintain arterial pressure sufficiently to perfuse the systemic vasculature. Cardiogenic shock is intrinsic to the heart and usually a consequence of ischemia, arrhythmia, or obstruction.

Hypovolemic shock (choice C) occurs when blood volume decreases to a point at which it is inadequate to maintain arterial pressure in the vital organs. Hypovolemic shock is due to hemorrhage, fluid loss from burns, or severe diarrhea and vomiting.

Neurogenic shock (choice D) is an unusual form of shock that occurs in catastrophic nervous system injuries that cause diffuse vasodilation and hypotension.
A 28-year-old male intravenous drug user presents with fever, chills, weakness, dyspnea, cough, arthralgia, diarrhea, and abdominal pain. On examination, a heart murmur is present, and small, tender nodules are found on the finger and toe pads, along with small hemorrhages on the palms and soles. Infective endocarditis, caused by which organism, should be suspected?
A. Enterococci
B. Pseudomonas aeruginosa
C. Staphylococcus aureus
D. Streptococcus pneumoniae
E. Streptococcus pyrogenes




The correct answer is C. The question is essentially asking "Which of the following is the most common cause of infective endocarditis (IE) in intravenous drug users?" Being an intravenous drug user places him at a very high risk for the development of IE. The factors that determine the clinical presentation of IE include the nature of the organism, the valve infected, and the route of the infection. Although the clinical characteristics of IE can vary, most patients present with a febrile illness lasting several days to 2 weeks. This illness is often accompanied by a variety of nonspecific signs and symptoms, such as chills, weakness, dyspnea, cough, arthralgia, diarrhea, and abdominal pain. Heart murmurs occur in approximately 90% of all patients, but may be absent in patients with right-sided infections. Other clinical signs include Osler nodules (purplish or erythematous subcutaneous papules or nodules on the pads of the fingers and toes), Janeway lesions (hemorrhagic painless plaques on the palms and soles), petechiae (small erythematous painless hemorrhagic lesions that may appear anywhere), and splinter hemorrhages (thin linear hemorrhages found under the nail beds of fingers and toes). The diagnosis of IE is dependent on positive blood cultures and echocardiographic evidence of "valvular vegetation" or valvular injury. Staphylococci, Streptococci, and Enterococci are possible pathogens; however, Staphylococcus aureus accounts for most cases of acute IE.

Enterococci(choice A) cause a minority of cases of valvular endocarditis, but are not usually involved in endocarditis associated with intravenous drug abuse.

Gram-negative organisms such as Pseudomonas aeruginosa(choice B) are rarely the cause of IE. They primarily cause pneumonia in cystic fibrosis patients and those on ventilators.

Streptococcus pneumoniae(choice D) and Streptococcus pyogenes(choice E) are not common causes of IE. They are, however, common causes of pneumonia and pharyngitis, respectively.
A construction worker with sustained trauma later develops tetanus. What is the causative organism?
A. Gram-negative anaerobic rod
B. Gram-positive cocci
C. Gram-positive anaerobic rod
D. Gram-positive aerobic rod




The correct answer is C. The causative organism of tetanus is Clostridium tetani , which is a gram-positive, spore-forming anaerobic rod.

Bacteroides is gram-negative anaerobic rod (choice A).

Examples of gram-positive cocci (choice B) are staphylococci and streptococci.

The causative organism of anthrax, Bacillus anthracis , is a gram-positive, spore-forming aerobic rod (choice D).
Last year's influenza A vaccine is unlikely to be effective today because influenza A
A. has a heavy polysaccharide coat
B. immunosuppresses the patient
C. kills lymphocytes
D. resists inactivation by complement
E. undergoes genetic reassortment




The correct answer is E. The difficulty with developing a vaccine against influenza A arises because the influenza virus genome is composed of eight strands of single-stranded RNA. Minor shifts (antigenic drift) in surface antigens that occur as point mutations in the genes accumulate. Influenza A can also undergo larger, abrupt changes in antigen expression (antigenic shift), however, as a consequence of reassortment of some of the RNA fragments between human and nonhuman hosts. Thus, last year's vaccine does not necessarily work against this year's virus.

Polysaccharide coats (choice A) are a virulence factor of some bacteria, not of viruses.

Influenza A can compromise the lungs sufficiently to predispose to secondary infections, producing a functional immunosupression (choice B), but this attribute does not make it difficult to produce vaccines against the virus.

Unlike AIDS, influenza virus does not selectively target lymphocytes (choice C).

Influenza A, bound to antibody and complement, can be effectively phagocytized (compare with choice D).
Infections in hyaline cartilage typically destroy the cartilage because
A. cartilage contains chondroitin sulfate
B. cartilage contains collagen
C. cartilage is not innervated
D. cartilage is relatively avascular
E. cartilage tends to become calcified




The correct answer is D. Hyaline cartilage contains very few, sparsely distributed, small blood vessels. The tissue consequently is very vulnerable to infection because the immune system (white cells and serum with clotting factors and antibodies) cannot access the cartilage in an efficient fashion.

Chondroitin sulfate (choice A) is an important constituent of cartilage that can be attacked by bacteria, but this is not the cause of uncontrollable infections in the cartilage, because if the infection is controlled, chondrocytes will continue to synthesize chondroitin.

Collagen (choice B) is present in cartilage, but does not predispose the cartilage for uncontrollable infection.

Collagen is not innervated (choice C), but this does not contribute to the development of uncontrollable infection.

The tendency of cartilage to undergo calcification (choice E), even when not acting as the site of ossification, does not predispose for uncontrollable infection.
The highest level of chronic disease state is associated with which of the following diseases?
A. Hepatitis A
B. Hepatitis B
C. Hepatitis C
D. Hepatitis D
E. None of these diseases have chronic carrier stages




The correct answer is choice C. Hepatitis C is a disease about which much has been learned recently, but much more is unknown. It seems to be transferred through both bloodborne and sexual routes (not fecal/oral). It is harder to contract than HBV, but easier than HIV. It is distinguished by having the highest percentage of cases going into a chronic destructive stage, of all hepatitis types. It is a single stranded RNA virus. It was formerly known as NANB (Non-A Non-B) or "transfusion hepatitis" because tests for hepatitis A or B did not detect it. Hepatitis B also has a destructive carrier state, but it is less common. Hepatitis A has NO carrier state, and exposure to Hepatitis A gives lifelong immunity.
Which of the following organisms would most likely be isolated from the vagina of a normal 5-year-old girl?
A. Candida
B. Lactobacillus
C. Neisseria
D. Staphylococcus
E. Streptococcus




The correct answer is D. The vagina of prepubertal girls and postmenopausal women is colonized by colonic and skin bacteria, including Staphylococcus epidermidis, which is normally found on the skin.

The vagina of women of childbearing age tends to be colonized by Lactobacillus (choice B) species, yeasts such as Candida (choice A), and Streptococcus species (choice E).

The presence of Neisseria (choice C), such as N. gonorrhoeae (the cause of gonorrhea), in the vagina of a 5-year-old girl strongly suggests sexual abuse.
In cases of strepococcus pneumonia, why is the patient's sputum filled with pus?
A. Teichoic acids and peptidoglycan are chemotactic for neutrophils
B. The capsule of the causative agent is chemotactic for neutrophils
C. The causative agent is an intracellular organism
D. The causative agent is beta hemolytic
E. The organism produces an IgA protease




The correct answer is A. The answer to this question requires that the student realize that pus consists of bacteria and dead and dying neutrophils. This fact reflects that the patient has typical (rather than atypical) pneumonia. In the United States, the most common agent of this would be Streptococcus pneumoniae, a gram-positive extracellular pathogen rich in teichoic acids and peptidoglycan that elicit the neutrophilic exudate.

The capsule of this organism is a polysaccharide and primarily elicits an antibody response rather than attracting neutrophils (choice B).

Streptococcus pneumoniae is an extracellular, not an intracellular (choice C) organism.

Streptococcus pneumoniae is alpha hemolytic, not beta hemolytic (choice D). This organism can be distinguished from other bacteria by the inhibition of growth in the presence of optochin or bile. Another common alpha hemolytic bacteria is the Viridans streptococci group.

Streptococcus pneumoniae does produce an IgA protease (choice E) that enhances the ability of the organism to infect the respiratory mucosa, but this does not contribute to pus formation.
Ethylene oxide sterilization procedures usually require exposure times of
A. 20-30 minutes
B. 1-2 hours
C. 2-3 hours
D. 8-10 hours
E. 24 hours




The correct answer is D. Ethylene oxide is used for sterilization of heat sensitive instruments and materials. Its advantage is that it does not melt, damage, or corrode instruments, as heat may. Its disadvantages include long cycle time (8-10 hours) and additional time to air out the materials so that they do not contain ethylene oxide residue. A period of 20-30 minutes is a common steam autoclave time. A period of 1-2 hours is a common dry heat autoclave time. A period of 24 hours was a time recommended for "cold sterilization" by chemicals. Cold sterilization is a misnomer, as it is actually high level disinfecton.
Approximately 7-8 days after starting an antibiotic therapy for an intraoral abscess, the patient begins to experience severe diarrhea. If a diagnosis of antibiotic-induced colitis caused by clostridium difficile is made, the colitis is most likely to be treated with which of the following?
A. Attapulgite
B. Clindamycin
C. Loperamide
D. Metronidazole




The correct answer is D. Pseudomembranous colitis is caused by a necrolytic toxin produced by Clostridium difficile, which can proliferate in the bowel secondary to the use of antibiotics that disrupt the normal bowel flora. Almost all antibiotics (but most commonly clindamycin and ampicillin) can cause this disorder, which results in profuse watery or bloody diarrhea. The most effective treatment for pseudomembranous colitis is vancomycin or metronidazole. Metronidazole is a bactericidal agent that is indicated for the treatment of serious infections caused by anaerobic bacteria and is active against most protozoa. It is primarily used for the treatment of amebiasis and trichomoniasis in outpatient settings and for prophylactic use in colorectal, gynecologic, and abdominal surgery. The use of antidiarrheal agents, such as attapulgite (choice A) and loperamide (choice C), is contraindicated. Clindamycin (choice B) is a bacteriostatic antibiotic used primarily in the treatment of many different anaerobic infections. This is one of the primary antibiotics implicated in the development of pseudomembranous colitis, and so will not be used to treat it.
What is the treatment of choice for an infection with Cryptococcus neoformans?
A. Amphotericin B
B. Isoniazid
C. Clotrimazole
D. Metronidazole
E. Nystatin




The correct answer is A. Amphotericin B is the most appropriate drug listed for the treatment of cryptococcal meningitis. It is a polyene antibiotic that binds to ergosterol in the fungal cell membrane, creating an artificial pore. Flucytosine is often prescribed as an adjunct medication. Fluconazole is used long term to prevent recurrence in AIDS patients. These are all systemic antifungal.

Isoniazid (choice B) inhibits the biosynthesis of mycolic acids in the mycobacterial cell wall. It is the primary drug used against tuberculosis. It is used alone for TB prophylaxis and in combination with other antituberculars to treat patients with active disease.

Clotrimazole (choice C) is a typical antifungal that, like nystatin, is used primarily in the treatment of oral candidiasis. It is from the imidazole class of antifungals.

Metronidazole (choice D) is an antiprotozoal drug useful in treating a variety of parasitic infections. It is the drug of choice for trichomoniasis and giardiasis and provides general anaerobic coverage.

Nystatin (choice E) is an antifungal polyene that is usually used topically but can be taken orally for oral and esophageal candidiasis. Candidal infections of the skin, mucous membranes, and vagina usually respond well to this drug. It may also be used to prevent intestinal fungal overgrowth in patients on chemotherapy.
Which of the following terms describes a different condition from all of the others?
A. ARC
B. AIDS prodrome
C. Pre-AIDS
D. AIDS
E. AIDS-related complex




The correct answer is D. The disease caused by HIV virus progresses in stages that vary greatly in length from individual to individual. HIV+ individuals early in the disease are often asymptomatic and otherwise healthy. The HIV test shows only exposure to the virus and formation of antibody (the test tests for antibody, not virus). This asymptomatic stage may continue for several years. Much later, AIDS defining conditions (e.g., Kaposi sarcoma, cryptococcal meningitis, PCP pneumonia) combined with HIV+ blood test, combine to give the diagnosis of AIDS. In between however, an early symptomatic stage is known as ARC (AIDS-related complex), AIDS prodrome or pre-AIDS. Oral candidiasis, night sweats, and weight loss are common at this stage.
Which of the following is the first antibody that an infant makes and is characteristically produced during the primary immune response?
A. IgG
B. IgM
C. IgA
D. IgE
E. IgD




The correct answer is B. IgM, produced during the immune response and is the first antibody that an infant makes, is the most efficient immunoglobulin at activating complement.

IgG (choice A) is produced during the secondary immune response and is the only maternal immunoglobulin to cross the placenta.

IgA (choice C) is the main immunoglobulin in external secretions, such as saliva and tears. It also protects mucosal surfaces.

IgE (choice D) mediates immune hypersensitivity reactions and is important in providing immunity against some parasites.

IgD (choice E) is present in the membrane of mature B cells and functions in antigen recognition by B cells.
An antibiotic, such as penicillin, which modifies cell wall synthesis, tends to be most effective during which phase of bacterial growth in a closed system?
A. Lag phase
B. Log phase
C. Phase of decline
D. Stationary phase




The correct answer is B. Penicillin is a bactericidal agent with broad gram-negative and gram-positive coverage. This agent inhibits the biosynthesis of cell wall mucopeptides. Bacterial growth in a closed system is characterized by four phases:

(1) In the initial lag phase, no growth occurs as the organisms adapt to the new environment.

(2) In the exponential, or log phase, the organisms grow at the fastest rate and antibiotics that interfere with cell growth or division are most likely to be effective.

(3) In the stationary phase, when nutrients have been largely exhausted, organisms tend to stop growing but may remain viable for long periods of time.

(4) In the phase of decline, cell deaths increase due to cell starvation or exposure to toxins.
A child with sickle cell anemia states that she has been feeling very tired lately, and then states that she may have "come down with a virus." The girl is very pale except for her red cheeks, and a complete blood count shows severe anemia. A bone marrow aspirate contains no erythroid precursor cells. The girl was probably infected with which of the following viruses?
A. Coxsackie virus
B. Echovirus
C. Hepadnavirus
D. Herpes virus
E. Parvovirus




The correct answer is E. Patients with sickle cell anemia typically present with a fiery red "slapped cheek" appearance. Parvoviruses are small single-stranded DNA viruses, of which only serotype B19 is pathogenic for humans. This virus causes three distinct syndromes: a childhood febrile rash known as erythema infectiosum ("Fifth disease"); aplastic crisis in individuals with chronic hemolytic diseases (e.g., sickle cell anemia, thalassemia); and congenital infections that can present as stillbirth, hydrops fetalis (analogous to severe Rh incompatibility), or severe anemia. In immunosuppressed patients and those with sickle cell disease, anemia caused by red cell hypoplasia occurs as a consequence of binding to erythrocyte P antigen.

Coxsackie viruses (choice A) usually cause cold-like illness, but can cause herpangina, myocarditis, and meningitis.

Echoviruses (choice B) can infect a variety of organ systems (gastrointestinal, CNS, eyes, heart, respiratory, skin), but are not a cause of aplastic crises.

Hepadnavirus (choice C) is the causative agent of hepatitis B.

Herpes viruses (choice D) cause a variety of acute to chronic infections, including herpes simplex types I and II, chicken pox, chronic herpes zoster, cytomegalovirus infection, and Epstein Barr virus infections.
A baby born at 32 weeks gestation developed a fever and leukocytosis; lumbar puncture revealed pleocytosis with increased protein, decreased glucose, and gram-positive rods. Which one of the following organisms was most likely isolated from the CSF?
A. Escherichia coli
B. Listeria monocytogenes
C. Neisseria meningitidis
D. Streptococcus agalactiae
E. Streptococcus pneumoniae




The correct answer is B. The three major causes of neonatal meningitis are group B streptococci (Streptococcus agalactiae; choice D), Escherichia coli(choice A), and Listeria monocytogenes. All can be found in the vaginal tract of normal women and may contaminate the infant during passage through the birth canal. They colonize the upper respiratory tract and can cause pneumonia, septicemia, and/or meningitis in the neonate. They are readily distinguished on morphologic grounds; the streptococci are gram-positive cocci in chains, E. coli is a gram-negative rod, and L. monocytogenes is a gram-positive pleomorphic rod. There are other gram-positive rods that resemble Listeria (e.g., the diphtheroid bacilli found in the upper respiratory tract and on the skin), hence a motility test is done to confirm the identification. L. monocytogenes is also associated with drinking unpasteurized milk.

Neisseria meningitidis(choice C) is the most common cause of meningitis in school-age children and young adults. It is a fastidious, non-motile, gram-negative diplococcus that would be a very rare cause of meningeal disease in very young patients, such as this one.

Streptococcus pneumoniae(choice E) is a gram-positive coccus that grows in pairs and short chains. It is the number one cause of pneumonia, septicemia, and meningitis in the elderly. There is a vaccine composed of the capsular carbohydrate of 23 serotypes of this organism that is routinely given to individuals over the age of 60, as well as to individuals with splenic abnormalities (e.g., sickle cell disease) who are at increased risk for the development of pneumococcal sepsis.
A hospitalized burn patient has developed a severe pneumonia caused by a gram-negative, nonfermenting rod. The organism produces a blue-green pigment on growth media and has a grape-like fruity odor. The organism isolated is most likely
A. Escherichia coli
B. Klebsiella pneumoniae
C. Legionella pneumophila
D. Pseudomonas aeruginosa
E. Moratella catarrhalis




The correct answer is D.Pseudomonas aeruginosa is a common opportunist in burn patients, in whom it classically causes secondary wound infections and septicemia. Pseudomonal infections are also commonly seen in patients on ventilators. It may also cause cystitis in patients with urinary catheters and pneumonia in patients with cystic fibrosis. The organism is found in water and usually gains access to the body as a contaminant in the water used in respirators or in water baths used to cleanse wounds. This organism is a nonfermenter, that is, it does not metabolize sugars by classic pathways. It produces a blue-green, water-soluble pigment (pyocyanin), and has a fruity odor when grown on laboratory media.

Escherichia coli(choice A) is a lactose-fermenting, gram-negative rod commonly seen as normal flora of the intestine. It is the most common cause of urinary bladder infections, pyelonephritis, and sepsis in patients with indwelling urinary catheters. It is also the major cause of traveler's diarrhea and is a very important pathogen in neonates who become infected during passage through the birth canal.

Klebsiella pneumoniae(choice B) is a gram-negative, highly encapsulated rod that is a significant pulmonary pathogen in individuals with a respiratory compromise. It is a common cause of aspiration pneumonia and pulmonary abscesses in alcoholics and patients with chronic obstructive pulmonary disease.

Legionella pneumophila(choice C) is a fastidious, gram-negative respiratory pathogen that may cause either a fulminating disease or a mild "walking pneumonia-like" condition (i.e., an atypical pneumonia). It usually affects older or debilitated patients.

Moratella catarrhalis (choice E) is generally seen in patients with preexisting lung disease, elderly patients, and immunosupressed patients. It is a gram-negative diplococcus that produces a patchy infiltrate with occasional lobar consolidation.
A debilitated 72-year-old woman develops dry cough, fever, headache, and muscular pains. She treats herself with ampicillin without any improvement. Chest x-ray films reveal scattered opacities, suggestive of interstitial infiltration. Laboratory investigations demonstrate the presence of cold agglutinins. She is treated with erythromycin, and her symptoms rapidly improve. Which of the following is the most likely etiologic agent of this condition?
A. Influenza virus
B. Mycoplasma pneumoniae
C. Pneumocystis carinii
D. Respiratory syncytial virus
E. Streptococcus pneumoniae




The correct answer is B. The patient's clinical presentation is typical of primary atypical pneumonia. Gram stains will often show PMN's and monocytes. No bacteria is seen. In contrast to bacterial pneumonia, primary atypical pneumonia presents with the following features:

- Caused by M. pneumoniae; less frequently by viruses (influenza, respiratory syncytial virus, adenovirus, rhinoviruses, rubeola and varicella virus), Chlamydia, or Coxiella burnetii

- Characterized pathologically by interstitial, rather than intra-alveolar, inflammation

- Characterized clinically by nonspecific symptomatology and few "localizing" symptoms

Why is M. pneumoniae, and not influenza virus (choice A) or respiratory syncytial virus (choice D), the cause of this patient's pneumonia? First, M. pneumoniae infections are often associated with the appearance of cold agglutinins in the serum, detection of which is diagnostically important. Second, the patient responded quickly to treatment with erythromycin, an antibiotic effective against M. pneumoniae, but obviously not effective in treating viral infections. As a side note: the macrolides, such as erythromycin are the drugs of choice for atypical pneumonia.

Pneumocystis carinii(choice C) is a fungal organism causing pneumonia in severely immunocompromised hosts, especially AIDS patients. P. carinii pneumonia (PCP) is characterized by accumulation of a frothy exudate containing numerous organisms within alveolar spaces. Also, P. carinii is not sensitive to erythromycin. The drug of choice for treatment of PCP is trimethoprim-sulfamethoxazole.

Streptococcus pneumoniae(choice E) is the usual causative agent of lobar pneumonia, characterized by consolidation of a single lobe due to intra-alveolar acute inflammatory exudation. Lobar pneumonia is more prevalent in young, healthy individuals, whereas primary atypical pneumonia favors old, debilitated patients. S. pneumoniae is highly sensitive to penicillin.
Microscopic examination of a urine sample demonstrates many gram-negative rods, many neutrophils, and a few white blood cell (WBC) casts. Which of the following is the specific significance of the presence of WBC casts?
A. One or both kidneys are involved in the infection
B. One or both ureters are involved in the infection
C. The bladder is involved in the infection
D. The urethra is involved in the infection
E. The uterus is involved in the infection




The correct answer is A. The patient described has a urinary tract infection. The presence of white blood cell casts specifically implies renal involvement, because these form when the leukocytes are compressed together in the renal tubules. Similarly, the presence of red blood cell casts in a bloody urine would indicate that at least some of the bleeding was occurring in the kidney. White cell casts are seen in pyelonephritis and interstitial nephritis and are indicative of infection or inflammation. Note also that the presence of bacteria merely indicates infection somewhere in the excretory system.

Infection of other sites in the urinary tract, including ureters (choice B), bladder (choice C), and urethra (choice D), does not cause cast formation.

Although spread of a urinary tract infection to the uterus (choice E) is always of concern in a pregnant woman, the presence of casts does not specifically suggest that this has occurred.
A 3-month-old infant presents with a 3-day history of fever, cough, and poor feeding. The infant seems ill and has a temperature of 102 F and a respiratory rate of 32. A chest x-ray film shows bilateral patchy infiltrates in the lungs. Which of the following is the most likely etiologic agent?
A. Coronavirus
B. Influenza type A
C. Parainfluenza type 1
D. Respiratory syncytial virus
E. Rhinovirus




The correct answer is D. Respiratory syncytial virus is the most common cause of bronchiolitis, tracheobronchitis, and pneumonia in children younger than 1 year of age. The average incubation period is 5 days. Outbreaks occur seasonally in winter and early spring. Infection does not result in lasting immunity, and reinfection can occur. In bronchiolitis, proliferation and necrosis of bronchiolar epithelium develop, producing obstruction from sloughed epithelium and increased mucus secretion. Signs often include fever, tachypnea, and wheezes. Hyperinflated lungs and otitis media are frequent conditions.

Coronavirus (choice A) causes approximately 20% of common colds (nasal obstruction and discharge, sneezing, no fever or mild fever, occasional sore throat, and cough) and acute pharyngitis (sore throat, with or without cervical adenopathy, ulceration, and conjunctivitis).

Influenza type A (choice B) is the leading cause of influenza. Influenza is a systemic illness characterized by the sudden onset of fever, headache, myalgias, malaise, and prostration, followed by cough, nasal obstruction, and sore throat. The lower respiratory tract may also be involved.

Parainfluenza viruses (choice C) are the leading cause of croup, or acute laryngotracheobronchitis, in children. This infection involves the upper and lower respiratory tracts. Inflammation in the subglottal area leads to hoarseness, dyspnea, a barking cough, and inspiratory stridor.

Rhinovirus (choice E) is the most common cause of the common cold.
Which of the following substances takes part in the nonoxidative killing pathway by which pathogenic bacteria can be killed?
A. Hydrogen peroxide
B. Hypochlorous acid
C. Lysozyme
D. Myeloperoxidase
E. Superoxide ions




The correct answer is C. This is a microbiology question that requires knowledge about the physiology of bacteria. Lysozyme is present in tears, saliva, mucus, vaginal secretions, and several other body fluids. This material lyses the peptidoglycan layer of the cell wall of bacteria without participation of any of the elements of the oxidative killing pathway.

Hydrogen peroxide (choice A) is formed in the oxidative killing pathway from the superoxide ions. This material is also very toxic to bacterial pathogens.

Hypochlorous acid (choice B) is formed from hydrogen peroxide and halide ions in the presence of myeloperoxidase.

Myeloperoxidase (choice D) is an enzyme present in the oxidative killing pathway that breaks down hydrogen peroxide.

Superoxide ions (choice E) are formed by NADPH oxidase in the first reaction of the oxidative killing pathway. These superoxide ions are extremely toxic to bacterial organisms.
A 65-year-old man undergoes coronary artery bypass surgery that entails grafting the left internal mammary artery to the left main coronary artery to form an alternative conduit for blood flow. This patient received a(n)
A. allograft
B. autograft
C. homograft
D. isograft
E. xenograft




The correct answer is B. Tissue grafts in which the same individual acts as both the donor and the recipient are termed autografts.

Allografts (choice A) refer to tissue transplants from one person to another.

Homograft (choice C) is a synonym for allograft.

Isografts (choice D) refer to tissue transplants between genetically identical individuals (e.g., an identical twin donates her kidney to her twin sister).

Xenografts (choice E) refer to tissue transplants from another species (e.g., baboon heart transplanted into a human).
A 10-year-old boy is attending summer camp, complains of a sore throat, headache, cough, malaise, a low-grade fever, patchy exudate, and keratoconjunctivitis. Within hours, several other campers show similar symptoms. The most likely pathogen involved is:
A. Adenovirus
B. Chlamydia spp
C. Gram-negative diplococcus
D. Gram-positive enterococcus
E. Herpesvirus




The correct answer is A. There are more than 40 antigenic types of adenoviruses that produce a variety of symptoms. The incubation period is 4-9 days. Adenoviruses are nonenveloped (naked), icosahedral DNA viruses causing a variety of clinical syndromes. Adenoviruses cause a pharyngoconjunctivitis that affects children and sometimes adults who are living in the same household. Contaminated swimming pools have been implicated as sources for the virus. The virus is latent in the lymphoepithelial tissue of the nasopharynx and other sites. Adenoviruses also cause watery, nonbloody diarrhea. Fever often lasts 2-12 days and is accompanied by malaise and myalgia. Sore throat is often manifested by a patchy exudate and cervical lymphadenopathy. Conjunctivitis is often present.

Chlamydia spp. (choice B) produce a variety of clinical syndromes, including a sexually transmitted urethritis, pelvic inflammatory disease, neonatal pneumonia and inclusion conjunctivitis, lymphogranuloma venereum, adult interstitial pneumonia, and a zoonotic pneumonitis.

A gram-negative diplococcus (choice C) would be a Neisseria spp. or Moraxella. Neisseria meningitidis would be the logical choice, because it is associated with outbreaks among children. The clinical syndrome caused by infection with the meningococcus, however, is one of a fulminating, progressive septicemia or meningitis with fever, vascular collapse, and disseminated intravascular coagulation. N. gonorrhoeae does not cause a clinical syndrome as described in the question. Moraxella catarrhalis causes pneumonia in patients with chronic obstructive pulmonary disease.

Gram-positive enterococcus (choice D) would refer to Group D streptococcus. These organisms are associated with endocarditis and genitourinary infections.

Herpesviruses (choice E) are large, enveloped DNA viruses with an icosahedral shape. Possible diagnoses for this patient include Epstein-Barr virus, producing infectious mononucleosis, or cytomegalovirus, producing a mononucleosis-like syndrome. The patients in the question did not have hepatosplenomegaly (characteristic of mononucleosis), but did have keratoconjunctivitis. The organism causing the outbreak in the question also has a higher level of infectivity and a short incubation time.
A 70-year-old nursing home resident develops pneumonia. Examination of sputum demonstrates many neutrophils and many lancet-shaped gram-positive cocci in pairs. Which of the following organisms is the most likely cause of the patient's pneumonia?
A. Haemophilus influenzae
B. Mycobacterium tuberculosis
C. Mycoplasma pneumoniae
D. Staphylococcus aureus
E. Streptococcus pneumoniae




The correct answer is E. Pneumonia can be caused by a large variety of agents including bacteria, Mycoplasma, fungi, and viruses. Streptococcus pneumoniae, or Pneumococci are classically described as lancet-shaped gram-positive cocci in pairs, or diplococci. Historically, this organism was specifically associated with lobar pneumonia, but with modern rapid antibiotic therapy, a bronchopneumonia pattern may also be seen. Pneumococcal pneumonia remains common in community and hospital-acquired settings. Complications include bacteremia, meningitis, endocarditis, pericarditis, and empyema.

Haemophilus influenzae(choice A) is a gram-negative rod, more specifically a pleomorphic gram-negative coccobacilli.

Mycobacterium tuberculosis(choice B) is an acid-fast rod.

Mycoplasma pneumoniae(choice C) is an intracellular organism and an atypical bacteria. Gram stains typically show PMNs and monocytes but no bacteria.

Staphylococcus aureus(choice D) is a gram-positive coccus that grows in grape-like clusters. Complications include empyema and endocarditis.
Bilateral tonsillectomy is performed on an 11-year-old female with recurrent upper respiratory tract infections and trismos. On sectioning the tonsils, numerous small, yellow granules are noted which have dense, gram-positive centers and numerous branching filaments at the periphery. The granules are most likely composed of which of the following organisms?
A. Actinomyces israelii
B. Aspergillus fumigatus
C. Blastomyces dermatitidis
D. Candida albicans
E. Corynebacteria diphtheriae




The correct answer is A. The patient is presenting with cervicofacial actinomycosis. This type of infection commonly follows a tooth extraction or other trauma. Actinomyces are normal inhabitants of the gastrointestinal tract that grow under anaerobic and microaerophilic conditions. Although they are gram-positive rods, they grow as branching filaments and have been confused with fungi. The yellow colonies ("sulfur granules") are found in low-oxygen niches like the tonsils and in actinomycotic abscesses. Cervicofacial actinomycosis often develops slowly. The affected area becomes markedly indurated and the overlying skin becomes reddish or cyanotic. Abscesses draining to the surface may also present. There is little pain unless a secondary infection sets in. Trismus indicates that the muscles of mastication are involved.

Aspergillus fumigatus (choice B) may be present in the respiratory tract as an opportunistic pathogen; however, fungus balls are generally seen only in pre-existing cavities (e.g., bronchiectasis, TB), not in the tonsils.

Blastomyces dermatitidis (choice C) is a respiratory pathogen that is seen as thick-walled yeasts within granulomas.

Candida albicans (choice D), also a normal inhabitant of the oral cavity, would present as whitish plaques and would appear microscopically as budding yeasts. When these white plaques are removed, they reveal a painful bleeding lesion.

Diphtheria, caused by Corynebacteria diphtheriae(choice E), is a gram-positive rod. The disease presents with a gray-white membrane in the oropharynx, and large colonies would not be noted.
A 4-year-old child has repeated infections with staphylococci and streptococci, but has normal phagocytic function and delayed hypersensitivity responses. Lymph node biopsy would most likely reveal
A. absence of postcapillary venules
B. absence of germinal centers
C. defective chemotactic response of neutrophils
D. depletion of paracortical areas
E. depletion of thymus-dependent areas




The correct answer is B. The combination of repeated staphylococcal infections, normal phagocytic function, and normal delayed hypersensitivity responses is suggestive of Bruton hypogammaglobulinemia. The disease is characterized by an IgG level less than 100 mg/dL, other immunoglobulins deficient or absent, B cells deficient or absent, intact cellular (T-cell) immunity, and onset of infections after the sixth month of life, when maternal antibodies are no longer present. B-cell-dependent areas, therefore, such as germinal centers, would be absent or greatly diminished in size and number.

Neither postcapillary venules (choice A) nor thymus-dependent areas (choice E) are depleted in this disease.

The chemotactic response of neutrophils (choice C) is part of the phagocytic function assay, which was normal in the case presented.

Because T-cell functions are intact, the architecture of the portions of lymphoid tissues where T cells reside (e.g., paracortical areas, choice D) should be normal.
A patient with tertiary syphillis has aortic regurgitation. Which of the following organisms is the most likely culprit?
A. Haemophilus influenzae
B. Herpes simplex I
C. Neisseria gonorrhoeae
D. Neisseria meningitidis
E. Treponema pallidum




The correct answer is E. The findings described are those of tabes dorsalis, a form of tertiary syphilis caused by Treponema pallidum. Tabes dorsalis and other forms of tertiary syphilis are now uncommon in this country, possibly because the common use of antibiotics may "treat" many unsuspected cases of syphilis. Late (tertiary) syphilis is associated with infiltrative tumors of skin, bones, and liver (gummas). Aortic regurgitation and CNS disorders (including meningovascular and degenerative changes), shooting pains, and abnormal reflexes are seen.

Haemophilus influenzae (choice A) and Neisseria meningitidis (choice D) can cause meningitis.

Neisseria gonorrhoeae (choice C) causes gonorrhea, which usually does not involve the CNS.

Herpes simplex I (choice B) can cause an encephalitis that typically involves the frontal and temporal lobes. Cardiac and liver involvement, however, are not seen.
A 19-year-old man presents to the emergency department with pneumonia. Since the age of 6 months, he has had recurrent pneumonia and sinusitis caused by Streptococcus pneumoniae and Haemophilus influenzae. Careful assessment of his immune function would likely reveal abnormal function of the
A. B-lymphocytes
B. macrophages
C. natural killer cells
D. platelets
E. T-lymphocytes




The correct answer is A. The symptoms in the question suggest a deficiency known as common variable hypogammaglobulinemia, characterized by very low serum levels of IgG. In this congenital disease, the number of B cells is normal, but their ability to synthesize IgG and the other immunoglobulins is severely compromised, leading to recurrent streptococcal and Haemophilus infections. The onset of the recurrent infections usually begins between 6 and 12 months of age, concurrent with the decreasing levels of maternal IgG in the newborn. Treatment often involves giving intravenous gamma-globulin to reduce the number of infections.

There are no known diseases or infections associated with deficiencies in macrophages (choice B) or natural killer (NK) cells (choice C). Furthermore, one would not expect a deficiency in macrophage or NK cell function to present as recurrent bacterial infections, because these cells are primarily involved in cellular, rather than humoral, immunity.

Platelets (choice D) are not directly involved in the immune response. Deficiencies in platelets lead to problems in clotting and produce bleeding disorders.

T-cell deficiencies (choice E) result in severe viral, fungal, and protozoal infections, rather than recurrent bacterial infections.
Which of the following viruses produces disease or sequelae that is/are more severe if the infection occurs at a very young age?
A. Epstein-Barr virus
B. Hepatitis B virus
C. Measles virus
D. Poliovirus
E. Varicella zoster virus




The correct answer is B. Infection with hepatitis B virus (HBV) at birth or a very young age is associated with chronic HBV infection and the development of hepatocellular carcinoma later in life. In fact, infants born to hepatitis B surface antigen (HBsAg)-positive mothers are commonly infected, and approximately 90% become chronic carriers of the virus. In chronic carriers, hepatocellular carcinoma develops at an incidence more than 200 times higher than in noncarriers. The current recommendation for infants born of HBsAg-positive mothers is administration of hepatitis B immunoglobulin (HBIg) in the delivery room, with the first dose of the hepatitis B vaccine given at the same time or within 1 week. The second and third doses of the vaccine are then given at 1 and 6 months. With this protocol, 94% protection is achieved.

The Epstein-Barr virus (EBV; choice A) is the agent of heterophile-positive infectious mononucleosis. In children, primary EBV infection is often asymptomatic.

The measles virus (choice C) often causes a more severe disease in adults. The incidence of complications, including pneumonia, bacterial superinfection of the respiratory tract, bronchopspasm, and hepatitis, is much higher in adults older than 20 than in children.

Poliovirus (choice D) causes asymptomatic or inapparent infections 95% of the time. Frank paralysis occurs in approximately 0.1% of all poliovirus infections. However, the probability of paralysis increases with increasing age.

Varicella zoster virus (choice E) is the agent of chickenpox and shingles. In immunocompetent children, it is a benign illness with a mortality rate of less than 2 per 100,000 cases. This risk is increased more than 15-fold in adults. Much of the increase is due to varicella pneumonitis, a complication that occurs more frequently in adults.
A 22-year-old woman comes to the sexually transmitted disease (STD) clinic for her first visit. She tells the nurse practitioner that she has had four different sexual partners in the last 6 months and only one of them used a condom. She also admits that she used IV drugs on several occasions 2 years ago. She notes fever, weight loss, lack of appetite, and periodic difficulty breathing over the past few months. She has an HIV test performed, which is positive. The physician decides to do a
A. ELISA (enzyme-linked immunosorbent assay)
B. FACS (fluorescence activated cell sorting)
C. RAST (radioallergosorbent test)
D. RID (radial immunodiffusion)
E. Western blot




The correct answer is E.Because of the prevalence of AIDS, it is essential that the dental student be familiar with these concepts. The Western blot is the most appropriate test for confirmation of HIV infection. It identifies several different antibodies against HIV (anti-gp120, -gp41, -p24, and -p17). The Western blot is generally performed when an ELISA test is positive.

The initial HIV test this patient had was most likely an ELISA. The ELISA (choice A) can be used to detect p24 antibody in the patient, but is not as specific as the Western blot. The ELISA test is the standard test used in the screening of HIV infections.

Fluorescence activated cell sorting (choice B) is a technique used to separate and count specific numbers and types of cells in a sample. An example of this would be to count the number of B cells and T cells in a specific blood sample.

RAST testing (choice C) is used to determine the level of specific IgE present in a patient that reacts with a specific allergen that has been applied to a disk or glass bead.

Radial immunodiffusion (choice D) is an excellent test used for quantitation of immunoglobulin levels in patients. This is used to determine the IgG, IgM, and IgA levels in patient's serum.
A one-week-old baby develops nuchal rigidity and fever. A lumbar puncture is performed and the cerebrospinal fluid demonstrates large numbers of neutrophils. The glucose level of the CSF is lower than normal. If a diagnosis of meningitis is made, which of the following is the most likely causative agent, taking into account the patient's age?
A. Coxsackievirus
B. Escherichia coli
C. Herpes virus
D. Mycobacterium tuberculosis
E. Neisseria meningitidis




The correct answer is B. The nuchal rigidity (stiff neck) suggests meningitis. Numerous neutrophils in the cerebrospinal fluid suggest a bacterial pathogen. Furthermore, low CSF glucose is common in bacterial meningitis. The best answer of those listed is Escherichia coli, which is normally a gut organism, but can infect neonates who acquire the organism during passage through the birth canal. The common characteristics of bacterial meningitis include: elevated polymorphonuclear neutrophils, low CSF glucose, elevated CSF protein and markedly elevated opening pressure.

Coxsackievirus (choice A) is a cause of acute lymphocytic meningitis.

Herpes virus (choice C) is a cause of lymphocytic meningitis.

Mycobacterium tuberculosis(choice D) is a cause of chronic meningitis and is the causitive pathogen in tuberculosis.

Neisseria meningitidis(choice E) causes bacterial meningitis, usually in the second or third decade of life. It is rare to see this bacteria in infants.
Ciprofloxacin is believed to exert its antibacterial effect by which of the following mechanisms?
A. Competitive inhibition of para-amino-benzoic acid
B. Inhibition of bacterial cell wall synthesis
C. Inhibition of DNA-gyrase
D. Irreversible binding to the 30S subunit of bacterial ribosomes
E. Irreversible binding to the 50S subunit of bacterial ribosomes




The correct answer is C. NBDE Part 1 questions often involve antibiotic mechanisms. Fluoroquinolone antibiotics, such as ciprofloxacin and lomefloxacin, inhibit DNA-gyrase, which is an enzyme necessary for bacterial DNA replication and repair. The fluoroquinolones are generally not used for dental procedures because they have very poor anaerobic coverage. They have very good gram-negative coverage, however. These agents are commonly used in the treatment of upper and lower respiratory infections and in genitourinary tract infections.

Sulfonamide antibiotics exert their antibacterial effect through the competitive inhibition of para-amino-benzoic acid (PABA) (choice A), thereby inhibiting folic acid biosynthesis required for bacterial growth.

Cephalosporins (cephalexin, cefuroxime, and cefixime) are believed to exert their antibacterial effect through the inhibition of bacterial cell wall synthesis (choice B). These agents bind to one or more of the penicillin-binding proteins located on the cell walls of susceptible organisms which results in the inhibition of the third and final stage of bacterial cell wall synthesis. These effects account for the bactericidal effect of cephalosporins and their cousins the penicillins.

Aminoglycosides, such as gentamicin and streptomycin, irreversibly bind the 30S subunit of bacterial ribosomes (choice D), which inhibits bacterial protein synthesis.

Lincosamides, such as clindamycin, irreversibly bind to the 50S subunit of bacterial ribosomes (choice E), which suppresses bacterial protein synthesis.

Note: macrolides, such as erythromycin, also bind to the 50S subunit of bacterial ribosomes, which suppresses bacterial protein synthesis.
If a dentist were to administer a bacteriostatic antibiotic, such as a macrolide, and a bactericidal antibiotic, such as a cephalosporin, concomitantly to a patient with a severe intraoral infection, the most likely pharmacological result for the patient would be which of the following?
A. A small extension of bacterial coverage between the two agents
B. Antagonism of the antibacterial effects of both agents
C. Profound toxicity for the patients taking both agents
D. Synergism of the antibacterial effects of both agents
E. The appearance of mild antifungal activity




The correct answer is B. The concomitant administration of bacteriostatic and bactericidal antibiotics would result in antagonism of the antibacterial effects of both agents. The reason for the antagonism between the two is that each agent interferes with the mechanism of action of the other agent. By definition, a bacteriostatic agent is one that prohibits the growth and development of bacteria and a bactericidal agent is one that directly causes the death of a microbe during the growth and development stages.

An extension of bacterial coverage between two agents (choice A) is typically seen when two bacteriostatic or two bactericidal agents are given together.

The administration of a macrolide and a cephalosporin may cause some discomfort for the patient, such as nausea or stomach cramps; however, profound toxicity for the patients taking both agents (choice C) is very unlikely.

Synergism (choice D) is the correlated action of two agents that results in a final effect that is greater than that of each agent acting separately. A classical example of synergism is seen when an aminoglycoside, such as gentamicin, is administered concomitantly with extended spectrum penicillin, such as piperacillin, for the treatment of a pseudomonas infection.

The appearance of mild antifungal activity (choice E) would not be seen when bacteriostatic and bactericidal antibiotics are given together, if neither have any antifungal activity.
If a dentist were to administer a bacteriostatic antibiotic, such as a macrolide, and a bactericidal antibiotic, such as a cephalosporin, concomitantly to a patient with a severe intraoral infection, the most likely pharmacological result for the patient would be which of the following?
A. A small extension of bacterial coverage between the two agents
B. Antagonism of the antibacterial effects of both agents
C. Profound toxicity for the patients taking both agents
D. Synergism of the antibacterial effects of both agents
E. The appearance of mild antifungal activity




The correct answer is B. The concomitant administration of bacteriostatic and bactericidal antibiotics would result in antagonism of the antibacterial effects of both agents. The reason for the antagonism between the two is that each agent interferes with the mechanism of action of the other agent. By definition, a bacteriostatic agent is one that prohibits the growth and development of bacteria and a bactericidal agent is one that directly causes the death of a microbe during the growth and development stages.

An extension of bacterial coverage between two agents (choice A) is typically seen when two bacteriostatic or two bactericidal agents are given together.

The administration of a macrolide and a cephalosporin may cause some discomfort for the patient, such as nausea or stomach cramps; however, profound toxicity for the patients taking both agents (choice C) is very unlikely.

Synergism (choice D) is the correlated action of two agents that results in a final effect that is greater than that of each agent acting separately. A classical example of synergism is seen when an aminoglycoside, such as gentamicin, is administered concomitantly with extended spectrum penicillin, such as piperacillin, for the treatment of a pseudomonas infection.

The appearance of mild antifungal activity (choice E) would not be seen when bacteriostatic and bactericidal antibiotics are given together, if neither have any antifungal activity.
The term "biofilm" as it applies in dentistry is most often used to describe:
A. A safer type of x-ray film
B. a detector that protects tissue from x-ray damage
C. the surface of dentin inside a cavity preparation
D. the surface of dental waterlines
E. none of the above




The correct answer is choice D. The problem of bacteria in dental waterlines is probably going to be the next intensive infection control issue faced by dentistry. The narrow bore of most dental waterlines encourages a thin film colony of bacterial slime (biofilm) to form on the inner circumference of the waterline. Water entering the dental unit, although chlorinated at the source, is clean, but not sterile. The microorganisms it contains can form extensive colonies inside the tubes. While no specific disease cases have been linked to these microorganisms, it is clear that they should be minimized. Several systems of filtration or chemical disinfection are now available. Other alternatives include bottled irrigation water and flushing the lines for several minutes several times each day.
The term "biofilm" as it applies in dentistry is most often used to describe:
A. A safer type of x-ray film
B. a detector that protects tissue from x-ray damage
C. the surface of dentin inside a cavity preparation
D. the surface of dental waterlines
E. none of the above




The correct answer is choice D. The problem of bacteria in dental waterlines is probably going to be the next intensive infection control issue faced by dentistry. The narrow bore of most dental waterlines encourages a thin film colony of bacterial slime (biofilm) to form on the inner circumference of the waterline. Water entering the dental unit, although chlorinated at the source, is clean, but not sterile. The microorganisms it contains can form extensive colonies inside the tubes. While no specific disease cases have been linked to these microorganisms, it is clear that they should be minimized. Several systems of filtration or chemical disinfection are now available. Other alternatives include bottled irrigation water and flushing the lines for several minutes several times each day.
After eating a dinner of leftovers that included rewarmed vegetable fried rice, a 17-year-old boy develops diarrhea and stomach pain. Which of the following is the most likely pathogen?
A. Bacillus cereus
B. Campylobacter jejuni
C. Clostridium botulinum
D. Clostridium difficile
E. Escherichia coli




The correct answer is A. Bacillus cereus contaminates grains, such as rice, and produces spores resistant to quick frying and steaming. If you see the words fried rice on the NBDE, the odds are that the correct answer is B. cereus.

Campylobacter jejuni (choice B) causes enterocolitis with bloody diarrhea, crampy abdominal pain, malaise, and fever.

Clostridium botulinum (choice C) produces a constellation of signs and symptoms, including bulbar palsy, descending weakness or paralysis, progressive respiratory weakness, absence of fever, dry mucous membranes, and autonomic dysfunction.

Clostridium difficile (choice D) causes pseudomembranous colitis, classically resulting from clindamycin use.

Escherichia coli (choice E) comes in a variety of forms. The enterotoxigenic type is the most common cause of traveler's diarrhea.
A 7-year-old boy is brought to a physician because of a nearly confluent, fine, erythematous, macular rash that is most pronounced on his trunk. He has had a mild fever for 36 hours, but does not seem very ill. Physical examination demonstrates a reddened throat with tonsillar exudates, enlarged cervical nodes including the occipital node, and questionable splenomegaly. The most likely cause of his condition is
A. Bullous pemphigoid
B. Dermatitis herpetiformis
C. Herpes simplex
D. Measles
E. Rubella




The correct answer is E. This presentation (fine, nonblotchy, truncal rash in a not-very-ill child) is characteristic of rubella, or German measles, that is caused by a togavirus. The rash typically spreads from trunk to extremities. IgM specific for rubella can often be detected in serum within 1-2 days of developing the rash. The principal significance of this disease is that it can cause a devastating congenital infection characterized by ocular problems (cataracts, retinopathy, microphthalmos, glaucoma), cardiovascular problems (patent ductus arteriosus, ventricular septal defect, pulmonary stenosis), deafness, thrombocytopenic purpura, leukopenia, hepatosplenomegaly, CNS problems, and bony lesions. Arthralgia is seen, particularly in women. Exposure occurs 14-21 days before onset.

Bullous pemphigoid (choice A) produces large, tense blisters in flexural areas that appear with exacerbations and remissions.

Dermatitis herpetiformis (choice B) causes recurrent crops of small vesicles or papules that appear mainly on the elbows, knees, buttocks, posterior neck, and scalp.

Herpes simplex (choice C) is characterized by crops of vesicles on oral or genital sites, where herpes simplex I causes oral and perioral manifestations and herpes simplex II causes genital lesions.

Measles (choice D) causes a blotchy, maculopapular erythematous rash that begins on the face and spreads downward. Patients with measles are usually much sicker than those with German measles, and Koplik spots may be seen on the buccal mucosa.
A patient with diabetes has chronic sinusitis that has not responded to a 6-week course of antibiotics. The physician should suspect infection with which of the following organisms?
A. Actinomyces
B. Aspergillus
C. Cryptococcus
D. Mucor
E. Pneumocystis




The correct answer is D. There is a specific association between diabetes (particularly in brittle diabetics who may have episodes of ketoacidosis) and chronic sinusitis caused by saprophytic Zygomycetes, including Mucor and Rhizopus. These fungi can spread rapidly from the sinuses to the nearby skull bones and brain, potentially causing massive tissue destruction and death. The term "rhinocerebral mucormycosis" is used in these cases. Less commonly, other sites may be involved (lung, gastrointestinal tract), depending on the port of entry. The physician should suspect mucormycosis in any patient with chronic sinusitis who appears unusually ill and does not respond to antibiotic therapy. Unfortunately, most cases are diagnosed at autopsy.

Actinomyces (choice A) are part of the normal flora of the mouth. Actinomycosis also occurs in humans and may affect the cervicofacial region (typically following dental procedures or maxillofacial injuries), lungs, abdomen (typically following surgery, trauma, or intestinal penetration), or pelvis (related to IUD use). There is no specific association with diabetes mellitus.

Aspergillus (choice B) can be present in the sinuses, and does have somewhat increased incidence in patients with diabetes, but it is not the organism about which the physician should be most concerned.

Cryptococcus (choice C) is found in pigeon feces and is usually introduced into the body by way of the respiratory tract. It can disseminiate to the meninges and other sites in immunocompromised patients (often AIDS patients). It would not be of particular concern in this patient with sinusitis.

Pneumocystis carinii (choice E) causes pneumonia in severely immunosupressed patient ( e.g., patients with AIDS).
A patient with diabetes has chronic sinusitis that has not responded to a 6-week course of antibiotics. The physician should suspect infection with which of the following organisms?
A. Actinomyces
B. Aspergillus
C. Cryptococcus
D. Mucor
E. Pneumocystis




The correct answer is D. There is a specific association between diabetes (particularly in brittle diabetics who may have episodes of ketoacidosis) and chronic sinusitis caused by saprophytic Zygomycetes, including Mucor and Rhizopus. These fungi can spread rapidly from the sinuses to the nearby skull bones and brain, potentially causing massive tissue destruction and death. The term "rhinocerebral mucormycosis" is used in these cases. Less commonly, other sites may be involved (lung, gastrointestinal tract), depending on the port of entry. The physician should suspect mucormycosis in any patient with chronic sinusitis who appears unusually ill and does not respond to antibiotic therapy. Unfortunately, most cases are diagnosed at autopsy.

Actinomyces (choice A) are part of the normal flora of the mouth. Actinomycosis also occurs in humans and may affect the cervicofacial region (typically following dental procedures or maxillofacial injuries), lungs, abdomen (typically following surgery, trauma, or intestinal penetration), or pelvis (related to IUD use). There is no specific association with diabetes mellitus.

Aspergillus (choice B) can be present in the sinuses, and does have somewhat increased incidence in patients with diabetes, but it is not the organism about which the physician should be most concerned.

Cryptococcus (choice C) is found in pigeon feces and is usually introduced into the body by way of the respiratory tract. It can disseminiate to the meninges and other sites in immunocompromised patients (often AIDS patients). It would not be of particular concern in this patient with sinusitis.

Pneumocystis carinii (choice E) causes pneumonia in severely immunosupressed patient ( e.g., patients with AIDS).
A 39-year-old HIV-positive man has a seizure accompanied by loss of consciousness and leg and arm jerking. The patient is lethargic, unable to answer simple questions, and has an obvious left-sided hemiparesis. The causative organism is most likely:
A. Cryptococcus neoformans
B. Herpes simplex
C. Mycobacteria tuberculosis
D. Isospora belli
E. Toxoplasma gondii




The correct answer is E. Although all five pathogens can cause central nervous system (CNS) manifestations, toxoplasmosis (caused by Toxoplasma gondii) presents with seizures in 15-25% of cases. It is the most common cause of secondary CNS infections in AIDS patients. The disease is spread by ingestion of cysts from undercooked meat or from cat feces. Tachyzoites develop from cysts phagocytized by macrophages, then spread to the brain, muscle, and other tissues, where they encyst and multiply.

Cryptococcal meningitis (choice A) usually presents as a subacute meningitis with headache, nausea, vomiting, and confusion. Cranial nerve abnormalities are common with cryptococcal infections.

Herpes simplex encephalitis (choice B) typically has a subacute onset with headache, meningismus, and personality changes.

Mycobacterium tuberculosis(choice C) is a cause of basilar meningitis, which can present insidiously with headache and mental changes over a week or two, or can present acutely as confusion, lethargy, altered sensorium, and a stiff neck. Cranial nerve palsies, focal cerebral ischemia, and hydrocephalus are characteristic.

Isospora belli(choice D) is a leading cause of diarrhea in AIDS patients. It typically occurs when the CD4 count is less than 300/uL.
An otherwise healthy patient who wears contact lenses develops a small ulceration of the eye. Which of the following organisms is most likely involved?
A. Acanthamoeba
B. Cytomegalovirus
C. Herpes simplex
D. Toxocara
E. Toxoplasma




The correct answer is A. All the agents listed can infect the eyeball. The agent specifically associated with contact lens use is Acanthamoeba, which can infect lens solution. This amoeba is dangerous because it causes an intractable ulcerative keratitis that may progress to uveitis. If the lesion is suspected, the clinical laboratory should be notified and specific directions for collecting samples for culture obtained. The parasites may be difficult to see in histologic sections or corneal scrapings.

Cytomegalovirus (choice B) and herpes (choice C) infections are most often seen in immunocompromised patients, particularly AIDS patients.

Circulating larvae of the helminth Toxocara(choice D) can lodge in the eye (particularly in the vitreous or retina); Toxocara infections are seen more commonly in children.

Toxoplasmosis (choice E) of the eye is most often congenital, but it can be acquired.
An otherwise healthy patient who wears contact lenses develops a small ulceration of the eye. Which of the following organisms is most likely involved?
A. Acanthamoeba
B. Cytomegalovirus
C. Herpes simplex
D. Toxocara
E. Toxoplasma




The correct answer is A. All the agents listed can infect the eyeball. The agent specifically associated with contact lens use is Acanthamoeba, which can infect lens solution. This amoeba is dangerous because it causes an intractable ulcerative keratitis that may progress to uveitis. If the lesion is suspected, the clinical laboratory should be notified and specific directions for collecting samples for culture obtained. The parasites may be difficult to see in histologic sections or corneal scrapings.

Cytomegalovirus (choice B) and herpes (choice C) infections are most often seen in immunocompromised patients, particularly AIDS patients.

Circulating larvae of the helminth Toxocara(choice D) can lodge in the eye (particularly in the vitreous or retina); Toxocara infections are seen more commonly in children.

Toxoplasmosis (choice E) of the eye is most often congenital, but it can be acquired.
An otherwise healthy patient who wears contact lenses develops a small ulceration of the eye. Which of the following organisms is most likely involved?
A. Acanthamoeba
B. Cytomegalovirus
C. Herpes simplex
D. Toxocara
E. Toxoplasma




The correct answer is A. All the agents listed can infect the eyeball. The agent specifically associated with contact lens use is Acanthamoeba, which can infect lens solution. This amoeba is dangerous because it causes an intractable ulcerative keratitis that may progress to uveitis. If the lesion is suspected, the clinical laboratory should be notified and specific directions for collecting samples for culture obtained. The parasites may be difficult to see in histologic sections or corneal scrapings.

Cytomegalovirus (choice B) and herpes (choice C) infections are most often seen in immunocompromised patients, particularly AIDS patients.

Circulating larvae of the helminth Toxocara(choice D) can lodge in the eye (particularly in the vitreous or retina); Toxocara infections are seen more commonly in children.

Toxoplasmosis (choice E) of the eye is most often congenital, but it can be acquired.
A 4-year-old boy is brought to the emergency department in extreme respiratory distress, with a temperature of 103.8 F. He is drooling, has difficulty swallowing, and exhibits inspiratory stridor and swelling of the epiglottis. He has had no previous vaccinations. Which of the following agents is the most likely cause of these symptoms?
A. Haemophilus influenzae
B. Klebsiella pneumoniae
C. Legionella pneumophila
D. Mycoplasma pneumoniae
E. Streptococcus pyogenes




The correct answer is A. Epiglottitis is the most common disease of the upper respiratory tract produced by Haemophilus influenzae type b, a gram-negative encapsulated rod. Epiglottitis is characterized by an abrupt onset of high fever, drooling, and inability to handle secretions. Stridor and respiratory distress result from laryngeal obstruction. The epiglottis is described as being cherry red and swollen. H. influenzae is also a common cause of otitis media in children and may cause bronchitis, bronchiolitis, and pneumonia in adults. The incidence of serious disease caused by Haemophilus influenzae type b decreased greatly with the introduction of an effective vaccine, which is composed of the H. influenzae type b capsular polysaccharides coupled to a carrier molecule and given to children from 2-15 months of age. The patient had not received the Hib conjugate vaccine and therefore was susceptible to this organism.

Klebsiella pneumoniae(choice B) causes pneumonia and pulmonary abscesses, but is not considered to be a pathogen in the upper respiratory tract. Klebsiella is a gram-negative encapsulated rod.

Legionella pneumophila(choice C) causes pneumonia in humans. The disease may be mild (an atypical pneumonia) or a fulminating disease with high mortality (30%). On a gram stain, one would see no bacteria because it is an "atypical" organism.

Mycoplasma pneumoniae(choice D) causes community-acquired atypical pneumonia. It is the most common cause of pneumonia in young adults. On a gram stain, one would see no bacteria because it is an "atypical" organism.

Streptococcus pyogenes(choice E) is the most common cause of pharyngitis; however, this patient's presentation strongly suggests epiglottitis. It is a gram-positive organism.
Which of the following is an example of type II hypersensitivity?
A. Allergic rhinitis
B. Erythroblastosis fetalis
C. Food allergy
D. Serum sickness
E. Tuberculosis




The correct answer is B. Type II hypersensitivity occurs when antibodies react with antigens present on the surface of cells or other tissue components. It is conveniently subclassified into diseases produced by three distinct mechanisms. One mechanism involves complement fixation by the antibody-antigen complex, which facilitates phagocytosis; this mechanism occurs in transfusion reactions, erythroblastosis fetalis, autoimmune hemolytic anemia, agranulocytosis, or thrombocytopenia. In a second form of type II hypersensitivity, target cells coated with low levels of IgG are lysed (without phagocytosis) by monocytes, neutrophils, or natural killer cells; this mechanism is thought to operate in the destruction of large parasites, possibly some tumor cells, and in graft rejection. The third mechanism involves antibody-mediated cellular dysfunction, such as occurs in Graves disease or myasthenia gravis.

Allergic rhinitis (choice A) is an example of type I (allergic or anaphylactic) hypersensitivity.

Food allergy (choice C) is an example of type I (anaphylactic) hypersensitivity.

Serum sickness (choice D) is an example of type III (immune complex diseases) hypersensitivity.

Tuberculosis (choice E) is an example of type IV (cell-mediated or delayed) hypersensitivity.
A gram stain of the sputum from a patient with lobar pneumonia involving the left lower lobe demonstrates gram-positive, encapsulated, lancet-shaped diplococci. Which of the following is the most probable causative organism?
A. Haemophilus influenzae
B. Neisseria gonorrhoeae
C. Pneumocystic carinii
D. Staphylococcus aureus
E. Streptococcus pneumoniae




The correct answer is E. This is the classic microscopic description of the pneumococcus Streptococcus pneumoniae, which is a common cause of lobar pneumonia. Most strains are still sensitive to penicillins, although some drug-resistant strains have been isolated.

Haemophilus influenzae (choice A) is a gram-negative bacillus. Atypical strains may cause pneumonia in elderly patients with chronic respiratory disease.

Neisseria gonorrhoeae (choice B) is a gram-negative diplococcus that is not typically associated with pneumonia.

Pneumocystis carinii (choice C) is a small, hat-shaped fungus that is a common cause of pneumonia in HIV-positive patients.

Staphylococcus aureus (choice D) occurs as grapelike clusters of large, gram-positive cocci. It may cause pneumonia after surgery or after a viral respiratory infection, such as influenza, and is associated with empyema formation.
Global eradication of Lyme disease is unlikely in the foreseeable future because Borrelia burgdorferi
A. can be maintained in nature indefinitely by a tick vector
B. has a tough outer coat that is resistant to environmental stresses
C. has humans as its primary reservoir
D. is resistant to antibiotics and disinfectants
E. may reactivate and cause Brill-Zinsser disease




The correct answer is A.Borrelia burgdorferi, the tick-transmitted spirochete that causes Lyme disease, can be maintained in nature indefinitely by a tick vector. The tick, Ixodes dammini, can infect white-footed mice and large mammals such as deer during its life cycle, making these animals into reservoirs. The tick itself, however, is a reservoir because it acquires the disease through transovarial passage of the organism. Together these factors make Lyme disease an endemic infection with little hope for eradication.

B. burgdorferi is a delicate spirochete that is vulnerable to several chemical and physical agents. It does not have a tough outer coat (unlike choice B).

Humans are incidental hosts, not primary reservoirs (choice C), for B. burgdorferi. The primary reservoirs are ticks, mice, and large mammals.

B. burgdorferi can be successfully treated with penicillins, tetracycline, and ceftriaxone (unlike choice D).

Brill-Zinsser disease (choice E) is actually the reactivation of epidemic typhus infection caused by Rickettsia prowazekii. It can occur many years after an infection that was not treated with antibiotics.
A patient experiences a severe bite wound from a pit bull. He receives a booster injection of tetanus toxoid and an injection of penicillin G. Several days later, the wound is inflamed and purulent. The exudate is cultured on blood agar and yields gram-negative rods. Antibiotic sensitivity tests are pending. Which of the following is the most likely pathogen?
A. Bartonella henselae
B. Brucella canis
C. Clostridium tetani
D. Pasteurella multocida
E. Toxocara canis




The correct answer is D. Pasteurella multocida is a gram-negative rod that is normal flora of the oral cavity of dogs and cats. It often causes a local abscess following introduction under the skin by an animal bite. Most cases occur in children who are injured while playing with a pet. Note that penicillin G is not highly effective against gram negative rods.

Bartonella henselae (choice A)is a very small gram-negative bacterium that is closely related to the rickettsia. It is the cause of cat-scratch disease (a local, chronic lymphadenitis most commonly seen in children) and bacillary angiomatosis (seen particularly in AIDS patients). In this latter patient population, the organism causes proliferation of blood and lymphatic vessels, causing a characteristic "mulberry" lesion in the skin and subcutaneous tissues of the afflicted individual.

Brucella canis(choice B) is a gram-negative rod that is a zoonotic agent. Its normal host is the dog, but when it gains access to humans, it causes an undulating febrile disease with malaise, lymphadenopathy, and hepatosplenomegaly. The normal route of exposure is by way of ingestion of the organism.

Clostridium tetani (choice C) is a gram-positive spore-forming anaerobic rod. It causes tetanus (a spastic paralysis caused by tetanospasmin, which blocks the release of the inhibitory neurotransmitters glycine and gamma-aminobutyric acid [GABA]). There may be no lesion at the site of innoculation and exudation would be extremely rare.

Toxocara canis (choice E), a common intestinal parasite of dogs, is a metazoan parasite that causes visceral larva migrans. Young children are most likely to be affected, as they are most likely to ingest soil contaminated with eggs of the parasite.
A patient with nuchal rigidity and headache undergoes lumbar puncture. The CSF contains markedly increased numbers of lymphocytes, leading to a presumptive diagnosis of viral meningitis. Which of the following groups of viruses is most likely to be involved?
A. Adenoviruses
B. Enteroviruses
C. Human papillomaviruses
D. Poxviruses
E. Reoviruses




The correct answer is B. Viral meningitis is relatively common, accounting for 10,000 cases of meningitis per year in the United States. The vast majority of cases occur in individuals younger than 30 years of age. Usually the symptoms are relatively mild and death is uncommon. Enteroviruses, arboviruses, and type 2 herpes simplex virus are the most common causes of viral meningitis. Also, up to 10% of HIV patients develop an acute meningitis, typically at the time of seroconversion.

Adenovirus (choice A) infection is associated with upper respiratory tract infections (URIs), sinusitis, ocular disease, enteric infections, and bladder infections. It does not typically cause aseptic meningitis.

Human papillomaviruses (choice C) are associated with warts on the skin and genital areas.

Poxviruses (choice D) include the causative agents of smallpox, cowpox, and molluscum contagiosum. Reoviruses include the most common family member, rotavirus, responsible for gastroenteritis in young children. These agents do not typically cause meningitis.
A patient who is unable to tolerate clindamycin therapy for the treatment of an intraoral anaerobic infection would be most likely prescribed which of the following agents?
A. Ciprofloxacin
B. Metronidazole
C. Tetracycline
D. Tobramycin
E. Vancomycin




The correct answer is B. Anaerobic bacteria exist as part of the normal flora on the mucosal surfaces (the mouth, gastrointestinal tract, skin, and female genital tract) of humans and animals. Anaerobic infections occur when the normal balance between aerobic and anaerobic bacteria is disrupted.The two medications primarily used for the treatment of anaerobic infections are clindamycin and metronidazole. Clindamycin is typically used in the treatment of serious anaerobic infections when other less toxic medications cannot be used. A potential serious side effect is the onset of pseudomembranous colitis. Metronidazole is a bacterial agent that is indicated for the treatment of serious infections caused by anaerobic bacteria and is active against most protozoa. It is primarily used for the treatment of gynecologic and abdominal infections and can be used for the treatment of antibiotic-induced pseudomembranous colitis.

The fluoroquinolones, such as ciprofloxacin (choice A), have very poor anaerobic coverage. Ciprofloxacin therefore would be relatively ineffective for the treatment of an oral infection caused by an anaerobic infection. These agents are commonly used in the treatment of upper and lower respiratory infections and in genitourinary tract infections.

Tetracycline (choice C) is primarily used in the treatment of chlamydial and mycoplasmal infections, gonorrhea, and acute exacerbations of chronic bronchitis.

Tobramycin (choice D) is an intravenous antibiotic typically reserved for the treatment of serious and life-threatening gram-negative bacterial infections and some gram-positive infections.

Vancomycin (choice E) is also typically reserved for the treatment of serious and life-threatening bacterial infections; however, it is used primarily in the treatment of gram-positive infections.
Serum analysis of a symptomatic patient yields elevated ALT, HBsAg, Anti-HBc, HBeAg, and bilirubin. All other values are normal. What is the hepatitus B status of this recruit?
A. Asymptomatic carrier
B. Chronic active carrier
C. Fulminant hepatitis B
D. Recovered from acute self-limited HBV
E. Vaccinated against HBV




The correct choice is B. The presence of elevated ALT, HBsAg, anti-HBc, HBeAg, and bilirubin all point to active hepatitis B.

An asymptomatic carrier (choice A) would not have elevated ALT and bilirubin.

The absence of other symptoms rules out fulminant hepatitis B (choice C).

Recovery from acute self-limited HBV (choice D) is associated with the presence of anti-HBs and a decrease in HBsAg and HBeAg.

Someone who is vaccinated with HBV (choice E) has only anti-HBs in serum.
A 23-year-old woman has Toxic Shock Syndrome. A diffuse erythematous rash with areas of desquamation over the hands and feet is noted. Infection with which of the following agents is the most likely cause of these signs and symptoms?
A. Clostridium perfringens
B. HIV-1
C. Shigella dysenteriae
D. Staphylococcus aureus
E. Staphylococcus epidermidis




The correct answer is D. Toxic Shock Syndrome (TSS), a multisystem syndrome caused by a toxin (TSST-1) formed by certain strains of S. aureus. TSS usually affects several organ systems (gastrointestinal, renal, hepatic, hematopoietic, musculoskeletal, pulmonary) and can result in death. TSS historically has been associated with the use of tampons (as well as cervical cups and diaphragms) in young women, but also can occur in other patient populations. Symptoms often have a very abrupt onset, including fever, hypotension, diarrhea, and diffuse rash with desquamation of the hands and feet. Blood cultures are negative, because sypmtoms are caused by the toxin not the invasive organism. Management of shock, renal failure, and adult respiratory distress syndrome (ARDS) are a priority if these conditions are present, in addition to appropriate antibacterial treatment.

C. perfringens (choice A) causes gas gangrene, with necrosis of soft tissues, usually after a traumatic wound. It is also a cause of food poisoning.

Although HIV-1 (choice B) can cause many diverse findings and should never be immediately ruled out, the findings in this patient are most specific for TSS.

Shigella dysenteriae(choice C) is a cause of dysentery characterized by fever, abdominal cramps, and bloody diarrhea.

S. epidermidis (choice E) is part of the normal skin flora, but is notorious for causing infections of intravenous lines and prosthetic heart valves.
A patient was recently prescribed fludrocortisone inhalation therapy to improve his current asthma treatment regimen. Ten days after starting therapy he presents with white patches on the inside of the cheeks that can be easily wiped off leaving a red, bleeding, sore surface. This patient would be best treated with which of the following?
A. Acyclovir Capsules
B. Ampicillin Capsules
C. Cephalexin Suspension
D. Clarithromycin Tablets
E. Clotrimazole Troches




The correct answer is E. Candidiasis can appear in any area of the oral mucosa. One of the most common causes of candidiasis in the adult population is the use of oral inhalation corticosteroids, such as fludrocortisone. The pseudomembranous form typically appears with white patches that can be easily wiped off leaving red, bleeding, sore surfaces. There is also an erythematous form that presents with flat red and white lesions that cannot be rubbed off. Based on the description of the signs and symptoms in this question, the patient has the pseudomembranous form of oral candidiasis or oral thrush. Oral candidiasis responds very well to antifungal therapy. Clotrimazole and nystatin are antifungal agents used locally for treatment of infections caused by many different Candida species. Clotrimazole Troches are the generic name for the brand name product Mycelex Troches. A troche is essentially an orally disintegrating tablet that dissolves in the mouth and is generally used in adults with candidiasis.

Acyclovir (choice A) is an antiviral agent used in the treatment of infections caused by herpes simplex virus types 1 and 2 and varicella-zoster virus. This agent would be indicated in individuals with herpes zoster infections that typically appear as vesicular eruptions and/or ulcers on the cheek, tongue, gingival or palate.

Both ampicillin (choice B) and cephalexin (choice C) are beta-lactam antibiotics used to treat a number of bacterial infections. Both agents would be ineffective in fungi.

Clarithromycin (choice D) is a macrolide antibiotic that may be safely and effectively used for the treatment of bacterial pharyngitis, as well as various other infections caused by susceptible organisms. It is also used in SBE prophylaxis for dental procedures.
A patient is suffering from pneumonia, and culture shows numerous gram-positive cocci that are identified as Streptococcus pneumoniae. Which of the following immune effector mechanisms is most important in completely clearing this infection?
A. ADCC (antibody-dependent cell cytotoxicity)
B. Complement-mediated opsonization
C. Cytotoxic T cell lymphocytes
D. LAK cells
E. Natural killer cells




The correct answer is B. One of the most efficient mechanisms for eliminating extracellular pathogenic bacteria is by opsonization and phagocytosis by macrophages. The IgG and IgM antibody produced in response to the organism reacts with the capsular structure, stimulating the activation of the classic pathway of the complement system. This pathway produces large amounts of C3b that coat the organism, preparing it for phagocytosis.

ADCC cells (choice A) are actually natural killer (NK) cells that find virally infected cells and tumor cells that have been coated with antibody and react with and destroy them. These cells do not destroy antibody-coated bacteria, only body cells that are coated with antibody.

Cytotoxic T lymphocytes (choice C) react only with cells that have antigen epitopes presented in association with class I MHC molecules. An example would be a virus epitope from a virally infected cell presented by class I molecules on the surface of the cell.

LAK cells (choice D) are NK cells that have been activated by IL-2. They are considered to be superactivated NK cells.

NK cells (choice E) are cells of the innate immune system that destroy virally infected cells or tumor cells. This does not involve antibody and it does not involve extracellular pathogens.
A 33-year-old man with AIDS and a history of shingles develops a severe, multifocal encephalitis. Which of the following viruses is the most likely cause of his encephalitis?
A. Cytomegalovirus
B. Herpes simplex type I
C. Herpes simplex type II
D. Herpes zoster-varicella
E. Measles virus




The correct answer is D. The specific clue to the cause of the severe encephalitis in this patient is the history of shingles, caused by reactivation of the herpes zoster-varicella virus. In otherwise healthy adults, the virus (which is usually introduced to the body as a childhood case of chickenpox) remains dormant in a dorsal root ganglion, only to reactivate in later life as a painful dermatomal vesicular eruption. In patients with AIDS, the virus can cause a severe, multifocal encephalitis that may be resistant to acyclovir therapy.

Cytomegalovirus (choice A), Herpes simplex type I (choice B) and herpes simplex type II (choice C), can cause disseminated disease (including brain infection) in AIDS patients, but is less likely in this patient, given the past history of shingles and the lack of response to acyclovir.

Measles virus (choice E) appears to be related to subacute sclerosing panencephalitis, but this condition is not specifically increased in AIDS patients.
A patient with a lung lesion coughs up sputum that contains thin, acid-fast positive rods. Which of the following features would most likely be associated with these bacteria?
A. Nutritional requirement for factors V and X
B. Streptokinase
C. Toxic shock syndrome toxin
D. Visible under dark field illumination
E. Waxy envelope




The correct answer is E. M. tuberculosis is a non-spore-forming thin aerobic bacteria. Once stained the bacilli are not discolorized by acid alcohol; hence the term acid-fast. Mycobacteria are also considered "acid fast" because they have an envelope that contains large amounts of lipids and even true waxes (unlike envelopes of other types of bacteria) that prevents the acid-fast stain (carbolfuchsin) from leaking out.

The other characteristics listed in the answers are commonly tested features of specific bacteria:

Nutritional requirement for factors V and X (choice A) is a feature of Haemophilus influenzae.

Streptokinase (choice B) is produced by Streptococci.

Toxic shock syndrome toxin (choice C) is a feature of Staphylococcus aureus.

Visibility under dark field illumination (choice D) is a feature of the syphilis organism Treponema pallidum. Furthermore, T. pallidum does not grow on artificial media and therefore cannot be cultured in a laboratory.
A 58-year-old alcoholic with multiple dental caries develops a pulmonary abscess and is treated with antibiotics. Several days later he develops nausea, vomiting, abdominal pain, and voluminous green diarrhea. Which of the following antibiotics is most likely responsible for this patient's symptoms?
A. Chloramphenicol
B. Clindamycin
C. Gentamicin
D. Metronidazole
E. Vancomycin




The correct answer is B. Any time you see the development of diarrhea in the same question stem with "treated with antibiotics," you should immediately think of pseudomembranous colitis. This condition is caused by Clostridium difficile and typically occurs as a result of treatment with clindamycin or ampicillin. You would confirm your suspicion by sending a stool culture to be tested for the presence of the C. difficile toxin.

The most test-worthy side effect of chloramphenicol (choice A) is aplastic anemia, not diarrhea. In addition, you might have been able to eliminate this choice simply because of the extremely low probability that this patient would receive this antibiotic in the United States.

The key side effects of gentamicin (choice C) include ototoxicity and nephrotoxicity.

Metronidazole (choice D) and vancomycin (choice E) do not cause pseudomembranous colitis; they are used to treat it.
A 37-year-old man is admitted to the hospital with shortness of breath, cyanosis, and fever. Chest x-rays reveal consolidation of the right lower lobe with relative sparing of the remaining lobes. A clinical diagnosis of lobar pneumonia is made and supported by the results of sputum cultures. Which of the following is the genus of the bacterium most likely to be isolated from this patient's sputum?
A. Haemophilus
B. Klebsiella
C. Streptococcus, alpha-hemolytic
D. Streptococcus, beta-hemolytic
E. Streptococcus, gamma-hemolytic




The correct answer is C. The clinical and radiologic characteristics of this patient's condition are consistent with lobar pneumonia, a respiratory infection that, in its classic presentation, involves a single pulmonary lobe. In 90-95% of cases, the etiologic agent is Streptococcus pneumoniae (AKA pneumococcus), an alpha-hemolytic streptococcus present in the throat of 40-70% of healthy individuals. This organism is best described as a gram-positive diplococcus. Penicillin is the drug of choice for pneumococcal pneumonia, but sputum cultures are necessary to identify the infectious agent and determine its antibiotic sensitivity.S. pneumoniae is a major cause of purulent meningitis in the elderly. Other alpha-hemolytic streptococci (viridans streptococci) cause subacute endocarditis in patients with previously altered cardiac valves.

Bacteria belonging to the genera Haemophilus(choice A) and Klebsiella (choice B) cause respiratory infections; however, in immunocompetent individuals, these bacterial infections usually result in bronchopneumonia, which leads to multilobar, and often bilateral, pulmonary involvement.

Beta-hemolytic streptococci (choice D) cause countless infections in humans. Recall that classification of beta-hemolytic streptococci is based on their surface antigens known as Lancefield antigens. Human diseases caused by this group of bacteria include:

- Streptococcus pyogenes (group A): pharyngitis, scarlet fever, erysipelas, impetigo, rheumatic fever, and glomerulonephritis.

- Streptococcus agalactiae (group B): neonatal sepsis and urinary infections.

- Enterococcus faecalis (group D): endocarditis and urinary infections. Respiratory involvement is rarely seen.

Gamma-hemolytic streptococci (choice E) are streptococci that do not produce hemolysins (nonhemolytic streptococci) and are not a significant cause of human disease.
Which of the following organisms would most likely be isolated from shunt tubing or indwelling catheters in hospitalized children who develop meningitis?
A. Bacteroides fragilis
B. Corynebacterium diphtheriae
C. Escherichia coli
D. Staphylococcus epidermidis
E. Streptococcus pneumoniae




The correct answer is D. Staphylococcal meningitis is fairly rare, occurring mostly in patients with indwelling ventricular-peritoneal shunts. Staphylococcus epidermidis, a coagulase-negative organism that normally colonizes the skin, is the most common organism causing this disorder; Staphylococcus aureus meningitis occasionally occurs.

Bacteroides fragilis(choice A) is a common cause of anaerobic infections, including sepsis and peritonitis, but it does not commonly cause meningitis.

Diphtheroids (including Corynebacterium diphtheriae, choice B) are sometimes isolated from indwelling shunts, but in many cases they are simply contaminants, because some diphtheroids are normal skin flora.

Escherichia coli(choice C) is isolated from 30-50% of neonates with bacterial meningitis, but is not typically associated with shunt infections.

Streptococcus pneumoniae(choice E) is the most common cause of bacterial meningitis in people older than 30 years of age; it is much less common in children.
A 25-year-old woman presents with an erythematous rash that began on her face, then spread downward over her trunk. She has a fever and headache, bilateral pain of the front and back of her neck, and arthralgia. Which of the following diseases does she most likely have?
A. Infectious mononucleosis
B. Lyme disease
C. Roseola
D. Rubella
E. Rubeola




The correct answer is D. Rubella, or German measles, is a disease caused by a Togavirus, which is a small, enveloped, single-stranded, (+) linear RNA virus. Approximately 40% of patients are asymptomatic or have mild symptoms. In symptomatic patients, the clinical presentation typically consists of an erythematous rash beginning on the head that spreads downward to involve the trunk, lasting for approximately 3 days. In addition to the rash, symptoms include fever, malaise, and arthralgias coinciding with the eruption. Leukopenia also may be seen. Enteroviral rashes may mimic rubella and rubeola.

Infectious mononucleosis (choice A) is caused by the Epstein-Barr virus, a herpesvirus. Classic findings include fever, exudative pharyngitis, generalized lymphadenopathy, severe malaise (most common complaint), and hepatosplenomegaly. A rash is not a characteristic feature unless the patient has been treated with ampicillin. This is a self-limited disease that is rarely fatal unless splenic rupture occurs.

Lyme disease (choice B) is caused by the spirochete Borrelia burgdorferi. The disease is transmitted by the bite of the tick, Ixodes dammini. Reservoirs in nature include deer and mice. The initial lesion is an annular rash with central clearing and a raised red border (erythema chronicum migrans) at the bite site. The rash is warm, but not painful or itchy. Patients also have fever, malaise, myalgias, arthralgias, headache, generalized lymphadenopathy, and, occasionally, neurologic findings.

Roseola (choice C) is caused by human herpesvirus 6. Other names include exanthem subitum or sixth disease. Children have a febrile period of 3-5 days with rapid defervescence followed by an erythematous maculopapular rash lasting 1-3 days.

Rubeola (choice E), or measles, is caused by a paramyxovirus. Patients present with an upper respiratory prodrome and characteristic oral lesions (Koplik spots) that precede the rash. The nonpruritic maculopapular rash begins on the face and spreads to the trunk and extremities, including palms and soles. The incubation period is 10-14 days. Patients also have a posterior cervical lymphadenopathy. The virus is not associated with risk to a fetus.
A 60-year-old alcoholic smoker abruptly develops high fever, shakes, a severe headache, muscle pain, and a dry, insignificant cough. Later he develops marked shortness of breath requiring assisted ventilation. Chest x-ray reveals left lower lobe shadowing that spreads until both lungs are extensively involved. Culture of bronchoalveolar lavage fluid demonstrates a coccobacillary pathogen. Which is the most likely causative organism?
A. Legionella pneumophila
B. Listeria monocytogenes
C. Pseudomonas aeruginosa
D. Staphylococcus aureus
E. Streptococcus pneumoniae




The correct answer is A. The patient has a severe, potentially fatal, pneumonia with prominent systemic symptoms. Culture revealing a coccobacillus is the specific clue that the organism is Legionella pneumophila. The disease is respiratory Legionellosis, also known as Legionnaire disease. Patients tend to be older (40-70 years of age) and may have risk factors including cigarette use, alcoholism, diabetes, chronic illness, or immunosuppressive therapy. The bacteria can also be seen in the sputum with direct fluorescent antibody staining.

Listeria monocytogenes(choice B) causes listeriosis and is not a notable cause of pneumonia.

Pseudomonas aeruginosa (choice C) is a gram-negative rod that is a cause of pneumonia in patients on ventilators and those with cystic fibrosis.

Staphylococcus aureus(choice D) can cause pneumonia, but is a plump gram-positive cocci in clumps.

Streptococcus pneumoniae(choice E) can cause pneumonia, but is a gram-positive diplococci.
The electron transport system of Neisseria is located on which of the following structures?
A. Cytoplasmic membrane
B. Mesosome
C. Mitochondria
D. Nuclear membrane
E. Polyribosome DNA aggregates




The correct answer is A. This is a good microbiology question that uses the principles of bacterial physiology. Unlike eukaryotic cells in which the electron transport system is located on mitochondria, the electron transport system of bacteria is located on the cytoplasmic (plasma) membrane.

Mesosomes (choice B) and polyribosome DNA aggregates (choice E) are also found in bacterial cells and function in cell division and protein synthesis, respectively.

Bacteria do not have mitochondria (choice C) or membrane-bounded nuclei (choice D).
A 32-year-old man presents to the emergency room with a severe headache. Nuchal rigidity is found on physical examination. Lumbar puncture demonstrates cerebrospinal fluid with markedly increased lymphocytes. Other cell populations are not increased. If the fluid is clear in appearance, which of the following agents is the most likely cause of his symptoms?
A. Escherichia coli
B. Haemophilus influenzae
C. Herpes virus
D. Mycobacterium tuberculosis
E. Treponema pallidum




The correct answer is C. The clinically suspected diagnosis is meningitis, which is confirmed by the abnormal cerebrospinal fluid. The markedly increased lymphocytes suggests acute lymphocytic meningitis, which is distinguished from acute pyogenic meningitis (increased neutrophils as well as lymphocytes). Acute lymphocytic meningitis is usually viral in origin. Among the many viruses that have been implicated, mumps, herpes, Epstein-Barr, echovirus, and Coxsackie virus are the most common. One of the easiest ways to differentiate between a viral and bacterial meningitis is the appearance of the cerebral spinal fluid. A viral meningitis will not generally change the color of the CSF. However, a bacterial meningitis often produces a cloudy CFS.

Escherichia coli(choice A) and Haemophilus influenzae(choice B) cause acute pyogenic meningitis and would be expected to produce a cloudy cerebral spinal fluid.

Mycobacterium tuberculosis(choice D) and Treponema pallidum(choice E) cause chronic meningitis and would be expected to produce a cloudy fluid.
Which of the following diseases is associated with the "Dane Particle"?
A. Hepatitis A
B. Hepatitis B
C. Hepatitis C
D. Hepatitis D
E. Hepatitis E




The correct answer is choice B. A Dane particle is the name for the entire Hepatitis B virion (viral particle). It is one of three entities that will test positive as Hepatitis B surface antigens. The other two are pieces of viral coats known as spheres or filaments, which, although they are not complete viral particles, they do contain the surface antigen (HBsAg). Other hepatitis B antigens include the core antigen (HBcAg) and an "e-antigen" (HBeAg). The "e-antigen" is associated with increased viral replication and infectivity of the patient.
Which of the following is an example of a type II hypersensitivity?
A. Cutaneous reactions to drugs
B. Heat intolerance, sinus tachycardia, and proptosis caused by endocrine disease
C. Eczematous reaction on the dorsum of the foot after new detergent use
D. Glomerulonephritis with systemic lupus erythematosus
E. Wheal and flare reactions and vesicles due to fire ant bites




The correct answer is B. Type II hypersensitivity is mediated by antibodies directed toward antigens that are present on the surface of cells or other tissue components. The antigen may be intrinsic to the cell membrane or may take the form of an exogenous antigen that is adsorbed to the cell surface. The situation described is Grave's disease, which is an autoimmune form of hyperthyroidism produced by autoantibodies directed against the TSH (thyroid stimulating hormone) receptor. These antibodies are called LATS (long-acting thyroid stimulator) and stimulate thyroid function, resulting in the release of thyroid hormones.

A drug reaction (choice A) and wheal and flare reactions (choice E) are cutaneous manifestations of type I hypersensitivity. Certain allergens, especially drugs, insect venoms, latex, and foods may induce a type I IgE antibody response.

An eczematous reaction (choice C) associated with washing clothes in a new detergent may either represent type IV hypersensitivity or a nonimmune reaction associated with direct toxicity from some component of the soap penetrating the skin.

Glomerulonephritis in systemic lupus erythematosus (choice D) is caused by the deposition of antigens in the glomerular basement membrane with the resultant formation of antigen-antibody complexes. These complexes activate the complement cascade, which causes neutrophils to enter the area and produce tissue damage.
Examination of a peripheral blood smear demonstrates a leukemia composed of small mature lymphocytes without blast forms. If a diagnosis of chronic lymphocytic leukemia is made, which of the following is the most likely age of this patient?
A. 1 year
B. 5 years
C. 20 years
D. 45 years
E. 65 years




The correct answer is E. Different leukemias tend to affect populations of different ages. The disease described is chronic lymphocytic leukemia (CLL), a disease of older adults.

Both the 1-year-old (choice A) and the 5-year-old (choice B) would be most likely to have acute lymphocytic leukemia (ALL).

The 20-year-old (choice C) would be most likely to have acute myelocytic leukemia (AML).

The 45-year-old (choice D) would be likely to have either AML or chronic myelogenous leukemia (CML).
Which of the following serum chemistry studies will MOST likely be abnormal in a preteen with polydipsia that develops weight loss despite eating large amounts of food over a three month period?
A. Blood urea nitrogen
B. Serum bicarbonate
C. Serum calcium
D. Serum glucose
E. Serum sodium




The correct answer is D. Although it is commonly known that diabetes mellitus is associated with polyuria and polydipsia, many do not realize it is also associated with weight loss despite increased eating, also known as polyphagia. In our extremely weight- and food-conscious society, this phenomenon may be much more striking to the patient and his family than a change in the amount of fluid intake. The lesson learned is that a child who develops weight loss despite increased food intake should have a blood glucose test.

All of the other answer choices would be expected to be normal in this patient.
Which of the following is involved in hypoxia in a smoker with noticeable increase in anteroposterior chest width if oxygen therapy is effective?
A. Anemia caused by blood loss
B. Edematous tissues
C. Emphysema
D. Localized circulatory deficiencies
E. Right-to-left cardiac shunts




The correct answer is C. Chronic pulmonary emphysema is characterized by distention of small air spaces distal to the respiratory bronchioles and destruction of alveolar septa. Long-term cigarette smoking is the usual cause. The marked loss of lung parenchyma associated with emphysema leads to a decrease in the diffusion capacity of the lungs that reduces their ability to oxygenate blood and remove carbon dioxide, contributing to the "barrel chest" appearance of these patients. Cigarettes also inhibit Alpha-1 antitrypsin allowing trypsin to break down proteins in the alveoli. When arterial hypoxemia is persistent and severe, oxygen therapy should therefore be considered.

Oxygen therapy is less effective for treating the hypoxia associated with anemia (choice A), edematous tissues (choice B), localized circulatory deficiencies (choice D), and right-to-left cardiac shunts (choice E), because in each case, there is already adequate oxygen available in the alveoli. The problem in each of these situations is inadequate transport of oxygen to the tissues, blunting the effects of increasing the oxygen tension of the inspired air. Oxygen therapy nonetheless increases the amount of dissolved oxygen carried in the blood, which may be life-saving in some instances.
Which of the following characterizes an eczematous reaction that occurs 72 hours after wearing latex gloves?
A. Irritant dermatitis
B. Type I reaction
C. Type II reaction
D. Type III reaction
E. Type IV reaction




The correct answer is E. Sensitization to latex has become a major healthcare problem. Local skin irritations are common but more severe allergic reactions occur, up to and including rare anaphylactic reactions that are occasionally fatal. The immune responses to latex are immediate-type hypersensitivity (type I) reactions, expressing themselves in minutes, or delayed-type hypersensitivity (type IV) reactions that express themselves in 48-72 hours. The type I reactions are caused by the IgE-mediated sensitivity to latex proteins, whereas the type IV reactions are caused by a cell-mediated response to the chemicals that are added in the processing of latex. The type IV response in this circumstance would be referred to as contact dermatitis.

Irritant dermatitis (choice A) can be observed in the early stages of sensitization and can be caused by sweating, rubbing, and residual soap. The timing of the reaction after 72 hours points to the type IV reaction rather than an irritant dermatitis.

A type I reaction (choice B) would have been apparent within minutes with characteristic rhinitis, conjunctivitis, urticaria, asthma, angioedema, or anaphylaxis immediately after wearing latex gloves.

A type II reaction (choice C) is a cytolytic response mediated by an antibody. The type II reaction is observed in the context of hemolytic anemias, thrombocytopenia, and neutropenia.

A type III reaction (choice D) is an immune-complex response that sets into motion an inflammatory response.
What is the explanation for an older person having a fainting spell with a resting blood pressure of 130/60 mm Hg and a pulse reading of 40 beats per minute?
A. Aortic valve obstruction
B. Cardiac tamponade
C. Complete heart block
D. Heart failure
E. Hypertension




The correct answer is C. In complete (third degree) heart block, the ventricles beat independently of SA node activity and P waves become completely dissociated from QRS-T complexes. The rate of the ventricular beat is usually 30-45 per minute. Because resting cardiac output (CO) is normal and because CO = stroke volume x heart rate, the stroke volume is increased in complete heart block. When the stroke volume increases, a greater amount of blood must be accommodated in the arterial tree with each heartbeat, which causes a greater increase and decrease in pressure during systole and diastole. Note that the pulse pressure is 70 mm Hg (normal pulse pressure is 30-50 mm Hg).

The pulse pressure is decreased in aortic valve obstruction (choice A), cardiac tamponade (choice B), heart failure (choice D). This is clearly not hypertension (choice E).
A well-behaved toddler, accompanied by his parents, presents with multiple fractures, humpback and blue sclera. What is the probable diagnosis?
A. Child abuse
B. Ehlers- Danlos syndrome
C. Osteogenesis imperfecta
D. Scurvy




The correct answer is choice C. Osteogenesis imperfecta, also known as Brittle bone disease, is characterized by retarded wound healing. . The most common form is autosomal dominant with abnormal collagen type 1 synthesis due to a mutation in the collagen gene.

Choice A- Child abuse is unlikely in this well-behaved toddler with no other signs and symptoms of abuse. Blue sclera are also typical of osteogenesis imperfecta, and not related to child abuse.

Choice B- Mutation in the collagen gene and lysine hydroxylase gene results in hyperextensible, fragile skin, hypermobile joints, dislocations, varicose veins, ecchymoses, and arterial intestinal ruptures. Inheritance varies. There are 10 types with any one of several enzyme deficiencies.

Choice D- Scurvy is characterized by anemia, petechiae, ecchymoses, bleeding gums, loose teeth, poor wound healing and poor bone development. Deficient hydroxylation of collagen due to vitamin C deficiency results in defective connective tissue.
When a histologic section is taken of an intraoral abscess, many of the observed neutrophils show a degenerative change in which the nucleus has undergone fragmentation. This process is known as
A. caseous necrosis
B. coagulative necrosis
C. karyolysis
D. karyorrhexis
E. pyknosis




The correct answer is D. Karyorrhexis refers to a pattern of nuclear degradation in which a pyknotic or partially pyknotic nucleus undergoes fragmentation followed by complete lysis. This pattern is common in the neutrophils present in acute inflammation.

The type of necrosis seen in an abscess is liquefactive necrosis. Caseous necrosis (choice A) is seen in tuberculosis and some other granulomatous diseases; coagulative necrosis (choice B) is seen following infarctions of many organs (other than the brain).

Karyolysis (choice C) is also a degenerative change affecting nuclei. In this case, however, it is seen as a decrease in nuclear basophilia, which is presumably the result of DNAse activity.

Pyknosis (choice E) is characterized by nuclear shrinking and basophilia, apparently as a result of DNA condensation.
If hypocalcemia is noted, excessive secretion of which of the following hormones MOST likely contributed to hand and forearm bone erosion and resorption during chronic renal failure?
A. Aldosterone
B. Calcitonin
C. Parathyroid hormone
D. Renin
E. Vitamin D




The correct answer is C. In renal failure, the ability of the kidney to secrete phosphate is impaired. The resultant hyperphosphatemia causes hypocalcemia and triggers excretion of large amounts of parathyroid hormone. The released parathyroid hormone is a major contributor to bony changes (e.g., osteitis fibrosa, a form of localized bone resorption) seen with chronic renal failure. Note that excess parathyroid hormone normally causes hypercalcemia, but calcium resorption still cannot bring calcium levels back to normal.

Aldosterone (choice A) is a regulator of serum sodium.

Calcitonin (choice B) levels are usually low in chronic renal failure unless the parathyroids have so hypertrophied as to cause "tertiary hyperparathyroidism" with hypercalcemia.

Renin (choice D) is normally secreted by the kidney and may be decreased or increased in varying stages and forms of kidney disease. Renin regulates blood pressure and aldosterone secretion, rather than bone metabolism.

The active form of vitamin D (choice E), cholecalciferol, is formed in the kidney from vitamin D absorbed from the gut and then processed by the liver. Uremia interrupts this pathway and consequently causes a functional vitamin D deficiency.
Which of the following would be MOST likely present on a biopsy of a thyroid gland and associated tissues that has a woody hardness when palpated but the needle biopsy shows no evidence of malignancy?
A. Marked fibrous reaction with gland destruction
B. Masses of hyperplastic follicles
C. Multinucleated giant cells
D. Small foci of lymphocytic infiltration
E. Prominent lymphocytic infiltrate with gland destruction




The correct answer is A. With Riedel's thyroiditis, an uncommon form of chronic (possibly autoimmune) thyroiditis that is characterized by dense fibrosis that destroys the thyroid gland and also extends into the adjacent muscle and connective tissue of the neck. The condition is clinically important because it may mimic malignancy. A key to the diagnosis is the woody hardness of fibrosis. All other choices are likely to be swollen and soft by palpation.

Masses of hyperplastic follicles (choice B) are a feature of multinodular goiter.

Multinucleated giant cells (choice C) are a feature of de Quervain thyroiditis.

Lymphocytic infiltration (choice D) is a part of many thyroid diseases, but is seen in isolation in subacute lymphocytic thyroiditis.

Sheets of lymphocytes with gland destruction (choice E) are seen in Hashimoto's thyroiditis.
Which of the following would be MOST likely present on a biopsy of a thyroid gland and associated tissues that has a woody hardness when palpated but the needle biopsy shows no evidence of malignancy?
A. Marked fibrous reaction with gland destruction
B. Masses of hyperplastic follicles
C. Multinucleated giant cells
D. Small foci of lymphocytic infiltration
E. Prominent lymphocytic infiltrate with gland destruction




The correct answer is A. With Riedel's thyroiditis, an uncommon form of chronic (possibly autoimmune) thyroiditis that is characterized by dense fibrosis that destroys the thyroid gland and also extends into the adjacent muscle and connective tissue of the neck. The condition is clinically important because it may mimic malignancy. A key to the diagnosis is the woody hardness of fibrosis. All other choices are likely to be swollen and soft by palpation.

Masses of hyperplastic follicles (choice B) are a feature of multinodular goiter.

Multinucleated giant cells (choice C) are a feature of de Quervain thyroiditis.

Lymphocytic infiltration (choice D) is a part of many thyroid diseases, but is seen in isolation in subacute lymphocytic thyroiditis.

Sheets of lymphocytes with gland destruction (choice E) are seen in Hashimoto's thyroiditis.
Which of the following gastrointestinal diseases is most likely to be associated with megaloblastic anemic who has been found to be deficient in vitamin B12, while the levels of all other essential vitamins are within normal limits?
A. Atrophic gastritis
B. Celiac sprue
C. Duodenal ulcer
D. Ulcerative colitis
E. Zollinger-Ellison syndrome




The correct answer is A. An important cause of vitamin B12 deficiency is pernicious anemia (megaloblastic), an autoimmune disease associated with atrophic gastritis. In atrophic gastritis, the gastric epithelium undergoes intestinal metaplasia, replacing gastric chief and parietal cells with goblet cells. The metaplastic epithelium produces insufficient intrinsic factor to bind the dietary vitamin B12deficiency.

Celiac sprue (choice B) is a condition associated with weight loss, flatulence, greasy stools, and increased fecal fat. Clinical improvement is seen with a gluten-free diet (a diet free of wheat, rye, barley, and oats).

Duodenal ulcers (choice C) may also produce iron deficiency anemia because of chronic blood loss. They are caused by damage to the duodenal epithelium by gastric acids and are associated with cirrhosis, COPD, chronic renal failure, and hyperparathyroidism.

Ulcerative colitis (choice D) is an inflammatory bowel disease almost entirely restricted to the large intestine. It usually causes intestinal distress and diarrhea, and it may produce a malabsorption syndrome that includes vitamin B12deficiency.

Zollinger-Ellison syndrome (choice E) is caused by gastric acid hypersecretion leading to peptic ulcer disease. Diarrhea is common.
If left untreated, severe squamous dysplasia on a vocal cord of a hoarse smoker may progress to which of the following?
A. Adenocarcinoma
B. Lymphoepithelioma
C. Mucoepidermoid carcinoma
D. Squamous cell carcinoma
E. Squamous papilloma




The correct answer is D. Squamous cell carcinoma is the most frequent type of cancer of the larynx. As with squamous cell carcinoma of the uterine cervix, the development of laryngeal carcinoma is related to an orderly sequence of morphologic changes. These begin with epithelial hyperplasia, proceed through increasingly severe degrees of dysplasia up to in situ carcinoma, and culminate with invasive carcinoma. Cigarette smoking is the most important risk factor for the development of laryngeal carcinoma. Any patient over the age of 50 years with hoarseness that has persisted beyond 2-3 weeks should be evaluated by indirect laryngoscopy. Odynophagia, hemoptysis, weight loss, referred otalgia, vocal cord immobility and cervical adenopathy suggest more advanced disease.

Adenocarcinoma (choice A) and mucoepidermoid carcinoma (choice C) are rare forms of laryngeal cancer. Squamous dysplasia is not a precursor of either type of tumor.

Lymphoepithelioma (choice B) is a form of squamous cell carcinoma that most frequently occurs in the nasopharynx, although it has been reported in the larynx also. Its name is derived from the fact the tumor is rich in lymphocytes. This tumor occurs frequently in southern China and certain regions in Africa. Epstein-Barr virus is implicated in its pathogenesis.

Squamous papilloma (choice E) is a benign laryngeal neoplasm caused by human papillomavirus types 6 and 11. It is not associated with squamous dysplasia.
What is the BEST explanation for the significant elevation of liver enzymes, ammonia, and hypoglycemia experienced five days after the onset of chicken pox in a child that was given aspirin for fever-control?
A. Crigler-Najjar syndrome
B. Dubin-Johnson syndrome
C. Gilbert's syndrome
D. Reye's syndrome
E. Rotor's syndrome




The correct answer is D. The use of aspirin in a child with chicken pox can cause Reye's syndrome. Reye's syndrome (fatty liver with encephalopathy) is an acute (and potentially fatal) postviral injury that is characterized by severe mitochondrial damage affecting the liver, brain, skeletal muscle, heart, and kidneys. The rapidly progressive hepatic failure and encephalopathy is associated with a 30% fatality rate. Most are children, although adult cases have been described. Varicella and influenza A and B are the most common precipitating illnesses. Aspirin use has been linked to the development of this disorder, but cases occur in the absence of salicylate ingestion. Hypoglycemia, elevated serum aminotransferases and blood ammonia, prolonged prothrombin time, and change in mental status all occur within 2-3 weeks after onset.

Crigler-Najjar (choice A) syndrome is a rare, mild to severe form of inherited unconjugated hyperbilirubinemia.

Dubin-Johnson syndrome (choice B) is an inherited conjugated hyperbilirubinemia associated with a darkly pigmented liver.

Gilbert's syndrome (choice C) is a common, benign form of inherited unconjugated hyperbilirubinemia.

Rotor's syndrome (choice E) resembles Dubin-Johnson syndrome, but is associated with a normal-colored liver.

Although most of these other symptoms are rare, it is important to recognize the signs, symptoms, and causes of Reye's syndrome.
Which of the following is the MOST likely the reason for persistent and marked anasarca noted with jaundice?
A. Lymphatic obstruction
B. Reduced central venous pressure
C. Reduced plasma oncotic pressure
D. Sodium retention
E. Venous thrombosis




The correct answer is C. Hepatic failure occurring in cirrhosis reduces the capacity of the liver to synthesize sufficient quantities of plasma proteins (mostly albumin) necessary to maintain plasma oncotic pressure. Low plasma oncotic pressure allows fluid from the intravascular fluid component to move into the interstitial space, producing plasma volume contraction and edema.

Lymphatic obstruction (choice A) occurs as a result of mechanical blockage of lymphatics by tumor, inflammatory processes, or certain parasitic infections. Cirrhosis does not lead to lymphatic obstruction.

Reduced central venous pressure (choice B) does not cause edema. Conversely, increased central venous pressure, which may arise with congestive heart failure, thrombosis, or cirrhosis can lead to increased hydrostatic pressure and edema.

Sodium retention (choice D) is an important cause of edema in patients with poor renal perfusion. The kidneys respond by retaining sodium and increasing plasma volume in an effort to increase renal blood flow. Any sodium retention in cirrhosis is secondary to the decrease in plasma oncotic pressure and consequent decrease in plasma volume.

Venous thrombosis (choice E) can lead to edema; however, the diminished synthesis of coagulation proteins in cirrhosis predisposes to bleeding, not thrombosis.
What of the following is characterized by left lower quadrant periumbilical pain in an elderly person, with the presence of fever, tender abdomen, leukocytosis, nausea, and vomiting?
A. Acute appendicitis
B. Diverticulitis
C. Gallstones
D. Pancreatitis
E. Pyelonephritis




The correct answer is B. Diverticulitis is a disease of the elderly and usually involves the distal colon. In severe cases, however, the diverticula may extend throughout the colon and up to the cecum. Inflammation of a cecal diverticulum can closely mimic acute appendicitis. The essentials of diagnosis for diverticulitis are acute abdominal pain and fever, left lower abdominal tenderness, and mass. Leukocytosis is commonly present together with nausea and vomiting.

Acute appendicitis (choice A) is usually a disease of young adults (and sometimes children). It is rarely seen in the elderly.

Pancreatitis (choice D), pyelonephritis (choice E), and gall bladder disease (choice C), refer pain to the mid back, lateral back, and right upper quadrant, respectively.
Which of the following would most likely be associated with Type A chronic gastritis resulting from autoimmune destruction of parietal cells?
A. Decreased growth of luminal bacteria
B. Decreased likelihood of developing gastric carcinoma
C. Decreased plasma concentration of gastrin
D. Increased production of macrocytic red blood cells
E. Increased secretion of pancreatic bicarbonate




The correct answer is D. Autoimmune destruction of parietal cells would lead to decreased secretion of gastric acid and intrinsic factor. The diminished availability of intrinsic factor would result in poor absorption of dietary vitamin B12. Over time, the vitamin B12 deficiency could lead to pernicious anemia, which is characterized by increased production of macrocytes (megaloblasts) by the bone marrow.

Because of the decrease in gastric acid secretion, luminal bacteria (choice A) would most likely exhibit increased (not decreased) growth. One of the functions of HCl secreted by the parietal cells is to sterilize the gastric lumen.

Patients with Type A gastritis have an increased likelihood of developing gastric carcinoma (not decreased, choice B).

A decrease in acid secretion leads to increased secretion of gastrin (not decreased, choice C) by antral G cells. This is because low gastric pH (less than 3.0) inhibits gastrin secretion by way of paracrine release of somatostatin from cells in the gastric mucosa that can sense the acidity. With decreased parietal cells, the pH of the gastric lumen would rise and remove this inhibitory component.

Because less acid would be delivered to the duodenum with parietal cell destruction, less secretin would be released into the blood. This would result in decreased pancreatic bicarbonate secretion (not increased, choice E).
What is present in an older woman when there is increased serum levels of calcium and urinary cAMP levels, below normal levels of serum phosphate, and no other complaints but weakness?
A. A calcitonin-secreting tumor
B. Primary hyperparathyroidism
C. Primary hypoparathyroidism
D. Thyrotoxicosis
E. Vitamin D deficiency




The correct answer is B. Primary hyperparathyroidism is often asymptomatic and only incidentally discovered during routine blood work, however, there may be vague complaints of fatigue or weakness and constipation. These neuromuscular manifestations are caused by the hypercalcemia that can "hyperstabilize" excitable tissue membranes and reduce normal responsiveness. Primary hyperparathyroidism incidence increases greatly after age 50 years, and is more common in women than men. The hypercalcemia is caused by the excess plasma concentration of parathyroid hormone (PTH). Approximately 80% of cases are caused by a single adenoma in a parathyroid gland. In the other 20% of cases, the hypersecretion of PTH is caused by hyperplasia in multiple parathyroid glands. The increased PTH also causes renal excretion of phosphate, producing hypophosphatemia. PTH acts by increasing cAMP formation in target tissues. The cAMP formed in renal tubules can diffuse into the lumen and be measured in the urine.

Tumors that secrete calcitonin (choice A) include medullary carcinoma of the thyroid and occasionally small and large cell carcinomas of the lung. Despite calcitonin's high blood concentration, serum phosphate is rarely abnormal and calcium levels would decrease rather than increase.

Primary hypoparathyroidism (choice C), which is caused by decreased secretion of PTH, is associated with hypocalcemia and hyperphosphatemia. Furthermore, urinary cAMP concentration would be decreased. The low calcium in extracellular fluid "destabilizes" excitable tissue membranes and can lead to spontaneous action potentials that produce tetany.

Thyrotoxicosis (choice D) or hyperthyroidism is associated with sweating, anxiety, heat intolerance, irritability, fatigue, muscle weakness, tachycardia, and warm moist skin.

With vitamin D deficiency (choice E), serum calcium is decreased because of diminished absorption from the diet. PTH secretion is increased to compensate, resulting in bone demineralization (osteomalacia).
What is present in an older woman when there is increased serum levels of calcium and urinary cAMP levels, below normal levels of serum phosphate, and no other complaints but weakness?
A. A calcitonin-secreting tumor
B. Primary hyperparathyroidism
C. Primary hypoparathyroidism
D. Thyrotoxicosis
E. Vitamin D deficiency




The correct answer is B. Primary hyperparathyroidism is often asymptomatic and only incidentally discovered during routine blood work, however, there may be vague complaints of fatigue or weakness and constipation. These neuromuscular manifestations are caused by the hypercalcemia that can "hyperstabilize" excitable tissue membranes and reduce normal responsiveness. Primary hyperparathyroidism incidence increases greatly after age 50 years, and is more common in women than men. The hypercalcemia is caused by the excess plasma concentration of parathyroid hormone (PTH). Approximately 80% of cases are caused by a single adenoma in a parathyroid gland. In the other 20% of cases, the hypersecretion of PTH is caused by hyperplasia in multiple parathyroid glands. The increased PTH also causes renal excretion of phosphate, producing hypophosphatemia. PTH acts by increasing cAMP formation in target tissues. The cAMP formed in renal tubules can diffuse into the lumen and be measured in the urine.

Tumors that secrete calcitonin (choice A) include medullary carcinoma of the thyroid and occasionally small and large cell carcinomas of the lung. Despite calcitonin's high blood concentration, serum phosphate is rarely abnormal and calcium levels would decrease rather than increase.

Primary hypoparathyroidism (choice C), which is caused by decreased secretion of PTH, is associated with hypocalcemia and hyperphosphatemia. Furthermore, urinary cAMP concentration would be decreased. The low calcium in extracellular fluid "destabilizes" excitable tissue membranes and can lead to spontaneous action potentials that produce tetany.

Thyrotoxicosis (choice D) or hyperthyroidism is associated with sweating, anxiety, heat intolerance, irritability, fatigue, muscle weakness, tachycardia, and warm moist skin.

With vitamin D deficiency (choice E), serum calcium is decreased because of diminished absorption from the diet. PTH secretion is increased to compensate, resulting in bone demineralization (osteomalacia).
If there is a large retinal detachment in the right eye and nothing noted in the left, how do the eyes react to a pupillary light reflex examination?
A. Constriction of the right pupil and constriction of the left
B. Constriction of the right pupil and dilatation of the left
C. Dilatation of the right pupil and constriction of the left
D. Dilatation of the right pupil and dilatation of the left
E. No reaction of the right pupil and constriction of the left




The correct answer is D. The eyes are exhibiting the Marcus-Gunn phenomenon. When light strikes the retina, the pupillary light reflex is automatically triggered, leading to simultaneous constriction of both pupils. In the absence of adequate light entering the eye, for example, following retinal detachment or optic neuritis, paradoxic dilatation of the pupils occurs. The retina receives far less light than it normally would, and the pupils dilate to absorb as much light as possible.
Which of the following findings is particularly indicated with a marked elevation in serum C-reactive protein?
A. Developing autoimmune reaction
B. Ineffective immune response
C. Respiratory compromise
D. Nonspecific inflammation




The correct answer is D. C-reactive protein is one of the most commonly measured acute-phase reactants, which are a group of serum proteins showing a rapid increase in concentration in response to any inflammatory process. This finding is entirely nonspecific. It only indicates a recent inflammatory process. C-reactive protein would be expected to be elevated following a dental procedure and in cases of pharyngitis (viral and bacterial).

An autoimmune reaction (choice A), which is certainly a concern with streptococcal pharyngitis, is suggested by the development of a rising ASO (antistreptolysin O) titer weeks after the illness. Acute phase reactants are not specific to autoimmune processes.

Increases in C-reactive protein indicate a healthy immune response to an infective pathogen. An ineffective immune response (choice B) would not elicit acute-phase reactions.

Respiratory compromise (choice C) produces changes in arterial blood gases and blood pH. Acute-phase reactants do not reflect respiratory status.
Which of the following is characteristic when there is a blood pressure of 165/95 mm Hg, tiredness, muscle weakness, polydipsia, while plasma sodium and serum aldosterone is slightly increased and plasma potassium and plasma renin activity is significantly decreased?
A. Addison's disease
B. Conn's syndrome
C. Cushing's syndrome
D. Type 1 diabetes mellitus
E. Pheochromocytoma




The correct answer is B. Conn's syndrome, or primary hyperaldosteronism, results from an adrenal tumor that secretes excessive aldosterone. The increased mineralocorticoid effects of aldosterone lead to renal sodium and water retention (which explains the hypertension) and increased renal potassium excretion (hypokalemia). The volume expansion also explains the decrease in hematocrit. The increased blood volume, increased blood pressure, and hypernatremia all tend to suppress renin secretion in an attempt to compensate for the increased aldosterone.

Addison's disease (choice A), or primary adrenal insufficiency, is characterized by low plasma concentration of aldosterone, hyponatremia, hypotension, and hyperkalemia.

In Cushing's syndrome (choice C), blood pressure may be increased because of crossover mineralocorticoid activity of the increased plasma cortisol. Furthermore, cortisol makes blood vessels more responsive to catecholamines, which could increase peripheral resistance. The combination of increased blood pressure and hypokalemia would, if anything, tend to suppress secretion of aldosterone.

Type 1 diabetes mellitus (choice D) is associated with polyuria, polydipsia, muscle weakness, and chronic tiredness. Renin and aldosterone levels are normal.

Pheochromocytoma (choice E) is another endocrine cause of hypertension. The increased plasma concentration of catecholamines can cause increased cardiac output and increased peripheral resistance. Plasma renin activity may be increased because of increased beta receptor activation on juxtaglomerular cells. This could produce increased aldosterone secretion and subsequent salt retention.
Which of the following BEST describes the inflammatory response that occurs upon contact with certain plants and presents as a weeping, vesicular, erythematous, and itchy rash on exposed arms, legs, neck, and face?
A. Erythema nodosum
B. Pemphigus
C. Psoriasis
D. Spongiotic dermatitis
E. Urticaria




The correct answer is D. Spongiotic dermatitis (intracellular edema of the epidermis) is seen in with contact dermatitis, such as poison ivy exposure. The accumulation of inflammatory cells in the superficial dermis causes marked edema, which splays epidermal keratinocytes apart and gives a spongy appearance to intercellular bridges. Grossly, the skin has a weepy appearance with frequent blistering.

Erythema nodosum (choice A) is a form of panniculitis, which is chronic inflammation in the subcutaneous fat lobules. Erythema nodosum presents as painful erythematous nodules, often with fever and malaise. It is associated with infections and drug reactions and is not a contact dermatitis.

Pemphigus (choice B) is a genetic blistering disorder caused by the production of antibodies to the intercellular cement substances in skin and mucous membranes.

Psoriasis (choice C) is a common chronic inflammatory disease causing plaques and scales, typically on elbows, knees, and scalp. The pathogenesis of psoriasis is still unclear; it may be a complement-mediated autoimmune process.

Urticaria (choice E) is an IgE-driven hypersensitivity process. Urticaria is characterized by wheals (edematous pruritic plaques) and typically affects the trunk and distal extremities.
Which of the following genetic conditions is MOST likely to be the cause of a child born with with a small head, small eyes, six fingers on each hand, and congenital heart defects?
A. Trisomy 13
B. Trisomy 18
C. Trisomy 21
D. XXY
E. XYY




The correct answer is A. This is a description of Patau's syndrome or trisomy 13. This disorder is also associated with severe mental retardation, abnormal forebrain structures, and death within 1 year of birth. Patau's syndrome has an incidence of 1:6,000 births, making it the second most common form of autosomal trisomy.

Trisomy 18 (choice B) is Edwards syndrome, characterized by severe mental retardation, rocker bottom feet (also sometimes seen in Patau's syndrome), low-set ears, micrognathia, clenched hands, prominent occiput, and death within 1 year. Edwards syndrome has an incidence of 1:8,000, making it the third most common autosomal trisomy.

Trisomy 21 (choice C) is Down syndrome, and is characterized by mental retardation, flat facial profile, prominent epicanthal folds, simian crease, duodenal atresia, and congenital heart disease. Down syndrome is the most common autosomal trisomy, with an incidence of 1:700.

XXY (choice D) is Klinefelter's syndrome and is associated with male hypogonadism and infertility, eunuchoid body habitus, gynecomastia, and lack of male secondary sexual characteristics.

XYY (choice E) is double Y syndrome. Affected individuals often go undetected, but may be taller than average and may be more likely to exhibit aggressive, antisocial behavior.
Which of the following is the MOST likely explanation for when two days after removal of a cancerous neck lesion, there is thigh and calf cramps, tingling around the lips, low serum calcium, and high serum phosphate?
A. Hyperparathyroidism
B. Primary hypoparathyroidism
C. Pseudohypoparathyroidism
D. Renal failure
E. Vitamin D deficiency




The correct answer is B. Surgically related hypoparathyroidism is the most common cause of primary hypoparathyroidism. Exploration of the anterior neck during thyroidectomy, parathyroidectomy, or removal of neck lesions can all compromise parathyroid gland function. Often the problem occurs because the blood supply to the parathyroid glands is interrupted during the surgery. The decrease in plasma parathyroid hormone leads to hypocalcemia and hyperphosphatemia. The decreased serum calcium is caused by decreased absorption of dietary calcium (because vitamin D activation is decreased) and decreased movement of calcium from bone to extracellular fluid. Low serum calcium can lead to tetany and paresthesias because of destabilization of excitable tissue membranes. Tetany is most often observed when there is a rapid decrease in serum calcium, such as that occurring with surgical hypoparathyroidism. The increased serum phosphate is caused by decreased renal excretion.

Hyperparathyroidism (choice A) is frequently asymptomatic; however, renal stones, polyurine, hypertension, constipation, fatigue, and mental status changes may be seen. Serum and urine calcium is elevated. Urine phosphate is high and a low to normal serum phosphate is generally seen.

Pseudohypoparathyroidism (choice C) is a rare genetic defect in which the target tissues are insensitive to parathyroid hormone. Because parathyroid hormone is less effective, serum calcium decreases and serum phosphate increases. Pseudohypoparathyroidism is also accompanied by developmental defects, including mental retardation, short stature, and missing metatarsal or metacarpal bones.

In renal failure (choice D), hyperphosphatemia occurs because of decreased renal excretion. This can lead to hypocalcemia as the equilibrium between serum phosphate and serum calcium is pushed toward hydroxyapatite. Furthermore, dietary absorption of calcium is decreased because vitamin D activation is decreased by hyperphosphatemia, even before there is significant decrease in renal 1-alpha-hydroxylase activity. The low serum calcium produces a compensatory increase in parathyroid hormone with subsequent bone demineralization (renal osteodystrophy).

With vitamin D deficiency (choice E), serum calcium and phosphate are typically decreased. The calcium is low because of decreased dietary absorption. The phosphate is low (not high) because of decreased dietary absorption, and because the secondary increase in parathyroid hormone secretion (caused by the hypocalcemia) increases renal excretion of phosphate.
At what level in the skin do bullae MOST likely develop in a localized cutaneous infection around the mouth with phage group II Staphylococcus aureus?
A. Across the basal cells
B. Below the basement membrane
C. Between the basal cells and the basement membrane
D. High in the epidermis
E. Just above the basal cells




The correct answer is D. Bullous diseases of the skin are subdivided on the basis of the level at which the cleavage for blister formation occurs. In general, the lower in the epidermis/dermis that the cleavage plane occurs, the more dangerous and widespread the blistering. This is because blistering at lower levels, particularly those involving the basal cell layer, permits loss of substantial amounts of fluid and heals slowly (often with significant scarring). Scalded-skin syndrome (toxic epidermal necrolysis) may follow staphylococcal (often phage group II) skin infection. This disorder fortunately involves the very superficial squamous cells just beneath the granular layer. Consequently, the disease (which typically produces bright red skin sloughing) usually resolves without sequelae after antibiotic therapy. "Scalded-skin" may also be observed in association with drug-induced erythema multiforme. Major inciting agents include phenylbutazones, sulfonamides, barbiturates, aminopenicillins, oxicam, nonsteroidal antiinflammatory agents, and allopurinol. This form usually affects the mucosa (eyes, mouth) first, and is much more dangerous because the blistering is subepidermal, and the entire overlying epidermis becomes necrotic.
Which of the following cell types show abnormal function when there is a history in a young adult of multiple arm and leg fractures following minor falls, with a slight weakness of facial muscles on the left, mild anemia, and with generalized bony widening with partial obliteration of marrow spaces?
A. Granulocytic stem cells
B. Megakaryocytes
C. Plasma cells
D. Osteoblasts
E. Osteoclasts




The correct answer is E. The disease described is osteopetrosis (Albers-Schonberg disease), which is a group of hereditary diseases in which impaired osteoclast function leads to reduced bone resorption. The abnormal osteoclasts frequently are enlarged, with bizarre shapes. The bones become thick and brittle; other features include anemia secondary to marrow loss and cranial nerve deficits secondary to narrowing of bony ostea. An autosomal recessive, severe form of the disease produces death in childhood. A relatively benign, autosomal dominant form presents in adulthood.

Abnormal proliferation of granulocytic stem cells (choice A) can produce myelocytic leukemias.

Megakaryocyte abnormalities (choice B) can produce platelet disorders, but not deficient bone resorption.

In multiple myeloma, neoplastic plasma cells (choice C) can cause lytic bone lesions characterized by excessive resorption of bone.

Abnormally low osteoclast, rather than osteoblast (choice D), function is the problem in osteopetrosis.
What of the following is characterized by left lower quadrant periumbilical pain in an elderly person, with the presence of fever, tender abdomen, leukocytosis, nausea, and vomiting?
A. Acute appendicitis
B. Diverticulitis
C. Gallstones
D. Pancreatitis
E. Pyelonephritis




The correct answer is B. Diverticulitis is a disease of the elderly and usually involves the distal colon. In severe cases, however, the diverticula may extend throughout the colon and up to the cecum. Inflammation of a cecal diverticulum can closely mimic acute appendicitis. The essentials of diagnosis for diverticulitis are acute abdominal pain and fever, left lower abdominal tenderness, and mass. Leukocytosis is commonly present together with nausea and vomiting.

Acute appendicitis (choice A) is usually a disease of young adults (and sometimes children). It is rarely seen in the elderly.

Pancreatitis (choice D), pyelonephritis (choice E), and gall bladder disease (choice C), refer pain to the mid back, lateral back, and right upper quadrant, respectively.
Which of the following genetic conditions is MOST likely to be the cause of a child born with with a small head, small eyes, six fingers on each hand, and congenital heart defects?
A. Trisomy 13
B. Trisomy 18
C. Trisomy 21
D. XXY
E. XYY




The correct answer is A. This is a description of Patau's syndrome or trisomy 13. This disorder is also associated with severe mental retardation, abnormal forebrain structures, and death within 1 year of birth. Patau's syndrome has an incidence of 1:6,000 births, making it the second most common form of autosomal trisomy.

Trisomy 18 (choice B) is Edwards syndrome, characterized by severe mental retardation, rocker bottom feet (also sometimes seen in Patau's syndrome), low-set ears, micrognathia, clenched hands, prominent occiput, and death within 1 year. Edwards syndrome has an incidence of 1:8,000, making it the third most common autosomal trisomy.

Trisomy 21 (choice C) is Down syndrome, and is characterized by mental retardation, flat facial profile, prominent epicanthal folds, simian crease, duodenal atresia, and congenital heart disease. Down syndrome is the most common autosomal trisomy, with an incidence of 1:700.

XXY (choice D) is Klinefelter's syndrome and is associated with male hypogonadism and infertility, eunuchoid body habitus, gynecomastia, and lack of male secondary sexual characteristics.

XYY (choice E) is double Y syndrome. Affected individuals often go undetected, but may be taller than average and may be more likely to exhibit aggressive, antisocial behavior.
If a woman 22 weeks pregnant has ankle edema and proteinuria, the presence of which of the following would determine if she has preeclampsia?
A. Diabetes mellitus
B. Hyperuricemia
C. Hypertension
D. Systemic lupus erythematosus
E. Thrombocytopenia




The correct answer is C. A pregnant patient is considered to be in preeclampsia if she develops hypertension, proteinuria, and edema. The hypertension is defined as a sustained elevation of blood pressure of 140 mm Hg systolic or 90 mm Hg diastolic or more in the absence of chronic hypertension after 20 weeks' gestation. Eclampsia includes the addition of seizures to the triad. Approximately 7% of pregnant women develop preeclampsia, typically between 20 weeks' gestation to 6 weeks postpartum. Predisposing conditions include preexisting hypertension, diabetes (choice A), and autoimmune diseases such as lupus (choice D). Laboratory features can include hyperuricemia (choice B) and thrombocytopenia (choice E), but these are not used to define the presence of preeclampsia.
Which of the following blood components can be expected to increase with disseminated intravascular coagulation?
A. Factor V
B. Fibrin degradation products
C. Fibrinogen
D. Plasminogen
E. Platelets




The correct answer is B. Disseminated intravascular coagulation (DIC or consumptive coagulopathy) represents pathologic activation of the coagulation system by another underlying disease, with consequent consumption and depletion of the cellular and humoral components of the coagulation cascade. The fibrinolytic mechanisms are also activated, and an uncontrolled cycle of bleeding and clotting develops. The essentials of diagnosis include underlying serious illness, hypofibrinogenemia, thrombocytopenia, fibrin degradation products, and prolonged prothrombin time. As a consequence, levels of all clotting proteins (choices A and C) become depleted, platelet counts drop (choice E), and the fibrinolytic proteins are depleted also (choice D). Fibrin degradation products (choice B), which are normally low in the serum, increase markedly because of increased fibrinolysis seen in this disease; identification of these proteins can be an important indicator of DIC.
Which of the following additional clinical findings would MOST be associated with a woman who has oral mucosal swelling, xerostomia, and intense salivary gland destructive inflammation, as well as antibodies against ribonucleoprotein?
A. Conjunctivitis
B. Goiter
C. Hemolytic anemia
D. Proximal muscle weakness




The correct answer is A. This describes Sjogren's syndrome, an autoimmune disease characterized by dry eyes (keratoconjunctivitis) and a dry mouth (xerostomia) caused by destruction of the lacrimal and salivary glands. Sjogren's syndrome is also characterized by autoantibody production. The most diagnostic autoantibodies are those against ribonucleoproteins.

Goiters (choice B) are not typical of Sjogren's syndrome. Although autoimmune thyroiditis is associated with Sjogren's syndrome, ocular involvement is much more characteristic than thyroid involvement.

Hemolytic anemia (choice C) is not characteristic of Sjogren's syndrome. Primary autoantibodies, drugs, and systemic lupus erythematosus may be associated with hemolytic anemia.

Proximal muscle weakness (choice D), in association with autoantibodies, is expected in polymyositis or dermatomyositis. Although polymyositis may occur in association with Sjogren's syndrome, keratoconjunctivitis would be much more common than muscle weakness.
What phenomenon is responsible when a person sets off the metal detector at the airport, despite removing watch, belt buckle, and every other obvious source of metal?
A. Argyria
B. Gall stones
C. Hemochromatosis
D. Kidney stones
E. Wilson's disease




The correct answer is C. Hemochromatosis is an iron storage disorder that can cause cirrhosis (with increased risk for hepatocellular carcinoma), skin pigmentation, pancreatic damage leading to diabetes mellitus, and congestive heart failure. These complications are attributable to damage caused by deposition of iron in tissues; the total body iron in some of these individuals may reach 50 g, sufficient to set off some airport metal detectors.

Argyria (choice A) is a blue-gray skin discoloration related to silver poisoning.

Neither gallstones (choice B) nor kidney stones (choice D) contain metal.

In Wilson's disease (choice E), copper is deposited in the liver and brain, but not enough to be detected by metal detectors.
Which of the following is the explanation for the laboratory findings in an alcoholic with chronic obstructive lung disease, secondary to cigarette smoking, of persistently lower serum levels than expected of theophylline which is being used as a bronchodilator, even though the drug is taken according to schedule?
A. Cirrhosis of the liver
B. Decreased absorption
C. Enhanced liver metabolism
D. Increased urinary clearance
E. Noncompliance




The correct answer is C. Alcohol and smoking normally enhance the cytochrome P450 system in the smooth endoplasmic reticulum (SER) of the liver. This system is responsible for the metabolism of drugs, hence, the low theophylline levels are most likely caused by enhanced liver metabolism. The hepatocyte SER undergoes hyperplasia as a response to alcohol ingestion and synthesizes the enzyme gamma-glutamyl transferase (GGT). An elevation of GGT would help confirm the likelihood of increased hepatic drug metabolism as the cause of low drug levels.

Cirrhosis of the liver (choice A) would likely increase the serum levels of theophylline because of poor metabolism of the drug.

Decreased absorption of the drug (choice B) in the gastrointestinal tract is a possible choice. The history of excess alcohol intake, chronic smoking, and lack of a history of malabsorption, however, suggest increased hepatic metabolism.

Increased clearance of theophylline in the urine (choice D) implies an increase in the glomerular filtration rate, which would not be expected in this patient.

In most circumstances, the lack of an expected response to a medication is because of patient noncompliance until proven otherwise; however, the question stem rules this out.
Which of the following could develop with a mucosal neuroma on the lower lip, family history of medullary thyroid carcinoma, and recent introduction of severe headaches, perspiration, palpitations, and hypertension?
A. Gastrinoma
B. Insulinoma
C. Parathyroid adenoma
D. Pheochromocytoma
E. Pituitary adenoma




The correct answer is D. This describes multiple endocrine neoplasia, specifically, MEN III (formerly MEN II b). Features of this autosomal dominant condition include medullary carcinoma of the thyroid, pheochromocytoma, and oral and intestinal ganglioneuromatosis (including mucosal neuromas). Pheochromocytomas typically cause attacks of severe headache, perspiration, palpitations, hypertension, anxiety, and tremor.

Gastrinomas (choice A) and insulinomas (choice B) are found in MEN I.

Parathyroid adenomas (choice C) are found in MEN I and II.

Pituitary adenomas (choice E) are found in MEN I.
Which of the following substances is significantly elevated in the serum of a young adult male with bilateral parotid gland swelling, orchitis, and fever and malaise?
A. Gastrinoma
B. Insulinoma
C. Parathyroid adenoma
D. Pheochromocytoma
E. Pituitary adenoma




The correct answer is D. This describes multiple endocrine neoplasia, specifically, MEN III (formerly MEN II b). Features of this autosomal dominant condition include medullary carcinoma of the thyroid, pheochromocytoma, and oral and intestinal ganglioneuromatosis (including mucosal neuromas). Pheochromocytomas typically cause attacks of severe headache, perspiration, palpitations, hypertension, anxiety, and tremor.

Gastrinomas (choice A) and insulinomas (choice B) are found in MEN I.

Parathyroid adenomas (choice C) are found in MEN I and II.

Pituitary adenomas (choice E) are found in MEN I.
Which of the following substances is significantly elevated in the serum of a young adult male with bilateral parotid gland swelling, orchitis, and fever and malaise?
A. Alanine aminotransferase
B. Amylase
C. Aspartate aminotransferase
D. Cortisol
E. Creatine phosphokinase, MB isoenzyme




The correct answer is B. The disease described is mumps, caused by a paramyxovirus. In children, mumps causes a transient inflammation of the parotid glands, and less commonly the testes, pancreas, or central nervous system. Mumps tends to be a more severe disease in adults than in children as it involves the testes (causing orchitis) and pancreas with some frequency. Pancreatic involvement can cause elevation of serum amylase.

Alanine aminotransferase (ALT) (choice A) and Aspartate aminotransferase (AST) (choice C) are markers for hepatocellular damage.

Cortisol (choice D) levels are increased in patients with Cushing's syndrome. This condition is associated with central obesity, muscle wasting, thin skin, easy bruisability, psychologic changes, hirsutism, and purple stria. Osteoporosis, hypertension, hyperglycemia, and glycosuria are also noted.

Creatine phosphokinase, MB isoenzyme (CPK-MB) (choice E) is the isoenzyme of CPK that is relatively specific for the myocardium. This enzyme is increased in the early stages of a myocardial infarction.
Which of the following forms of gastritis would MOST likely be found with a history of long-term use of nonsteroidal antiinflammatory drugs and complaints of heartburn, nausea, and vomiting for several days?
A. Acute gastritis
B. Chronic antral gastritis
C. Chronic fundal gastritis
D. Hypertrophic gastritis
E. Lymphocytic gastritis




The correct answer is A. Acute gastritis, characterized by patches of erythematous mucosa, sometimes with petechiae and ulceration, can be seen as a complication of a variety of other conditions (alcohol use, aspirin and other NSAIDs use, smoking, shock, steroid use, and uremia), which usually have in common disruption of the mucosal barrier of the stomach.

Chronic antral (type B) gastritis (choice B) is associated with Helicobacter pylori.

Chronic fundal (type A) gastritis (choice C) is associated with pernicious anemia.

Hypertrophic gastritis (Menetrier's disease; choice D) is an idiopathic condition characterized by markedly enlarged mucosal folds.

Lymphocytic gastritis (choice E) is believed to be a gastric manifestation of celiac sprue.
What is responsible for mild temporary hyperthyroidism after a person complains of a severe sore throat?
A. Diffuse nontoxic goiter
B. Grave's disease
C. Hashimoto's thyroiditis
D. Subacute granulomatous thyroiditis




The correct answer is D. It is due to subacute granulomatous (de Quervain) thyroiditis, which frequently develops after a viral infection. Microscopically, it is characterized by microabscess formation within the thyroid, eventually progressing to granulomatous inflammation with multinucleated giant cells. Clinically, patients may experience fever, sudden painful enlargement of the thyroid, or symptoms of transient hyperthyroidism. The disease usually abates within 6-8 weeks.

Diffuse nontoxic goiter (choice A) by definition does not produce hyperthyroidism. Goiter is usually associated with low thyroid function.

The hyperthyroidism of Grave's disease (choice B) does not spontaneously remit.

Hashimoto's thyroiditis (choice C) can cause transient hyperthyroidism, but then goes on to cause hypothyroidism.
What reaction has occured if after receiving a tuberculosis vaccine, the area becomes indurated and erythematous, having a 12 mm diameter?
A. Antibody-dependent cell-mediated cytotoxicity
B. Local anaphylaxis
C. T-cell mediated cytotoxicity
D. Type III hypersensitivity
E. Type IV hypersensitivity




The correct answer is E.The tuberculin reaction is an example of delayed-type hypersensitivity (a form of Type IV hypersensitivity) in which the bulk of the tissue damage is done by macrophages that are stimulated by a few previously sensitized CD4+ memory T-cells recognizing antigens presented by the macrophages. In contrast, in T-cell mediated cytotoxicity (choice C, another form of Type IV sensitivity) the damage is done by CD8+ cytotoxic T-cells that recognize "foreign" cell surface antigens and directly lyse targeted cells.

Antibody-dependent cell-mediated cytotoxicity reactions (choice A, a form of Type II hypersensitivity) involves cells coated with a thin layer of antibody that triggers attack by cells (monocytes, neutrophils, eosinophils, and natural killer cells) that can bind to Fc receptors.

Local anaphylaxis (choice B, a form of Type I hypersensitivity) is caused by the release of vasoactive substances by mast cells and basophils stimulated by memory (CD4+) T-cells reacting to antigen.

Type III (choice D) hypersensitivity is caused by deposition of circulating antigen-antibody complexes, often in small blood vessels.
Which of the following is characterized by extreme weakness, fatigue, nausea, stomach cramps, hypotensive while sitting and even more on standing, hyperkalemia, increased freckling around the eyes, and darkening of the palmar creases?
A. Addison's disease
B. Conn's syndrome
C. Cushing's syndrome
D. Secondary adrenal insufficiency
E. Tertiary adrenal insufficiency




The correct answer is A. Addison's disease usually occurs because of autoimmune destruction of the adrenal cortex (all three zones are typically involved), resulting in decreased secretion of cortisol, aldosterone, and adrenal androgens. Hyperpigmentation is the classic physical finding, resulting from increased serum ACTH caused by loss of negative feedback inhibition by cortisol at the pituitary or hypothalamus. The increase in pigmentation may occur because the first 13 amino acids of ACTH are identical to alpha-melanocyte stimulating hormone. Low serum levels of cortisol produce gastrointestinal symptoms such as nausea, vomiting, and anorexia. Fatigue and weakness are almost always reported. Blood pressure is usually low and orthostatic hypotension may be present, because arterioles are less responsive to the constrictor effects of catecholamines in the absence of cortisol. The cardiovascular symptoms are worsened by the loss of blood volume caused by aldosterone deficiency. Hyperkalemia is a manifestation of the low serum aldosterone; hyponatremia may also be present.

Conn's syndrome (choice B) results from hypersecretion of aldosterone. It is characterized by hypertension, hypernatremia, and hypokalemia.

Cushing's syndrome (choice C) occurs because of excessive secretion of cortisol. It is characterized by central obesity, buffalo hump, moon facies, hypertension, and hypokalemia.

Secondary (choice D) and tertiary (choice E) adrenal insufficiency result in low serum levels of ACTH. The subsequent hypocortisolism can produce the gastrointestinal complaints and fatigability but not hyperkalemia. With deficiency of CRH or ACTH, serum aldosterone usually remains in the normal range, and signs of mineralocorticoid deficiency are not present. Furthermore, low serum levels of ACTH would not produce hyperpigmentation.
A 45-year-old man presents to a physician with back pain, facial pain, coarse facial features, and kyphosis. His laboratory studies show elevated alkaline phosphatase. X-ray studies demonstrate skull thickening with narrowing of foramina and bowing of the femur and tibia. Bone biopsy reveals a mosaic pattern of bone spicules with prominent osteoid seams. Which of the following neoplasms occurs at an increased frequency in patients with this disorder?
A. Astrocytoma
B. Hodgkin lymphoma
C. Meningioma
D. Non-Hodgkin lymphoma
E. Osteosarcoma




The correct answer is E. The phrase "mosaic pattern" of newly formed woven bone is specific for Paget disease of bone and is not seen in other bone conditions. The clinical and radiologic presentation are typical; an increased hat size may also be a clue. In early stages, Paget disease is characterized by osteolysis, producing patchwork areas of bone resorption with bizarre, large osteoclasts. In the middle stage of the disease, secondary osteoblastic activity compensates with new bone formation, producing the mosaic pattern. In late Paget disease, the bones are dense and osteosclerotic. Paget disease is suspected to be related to prior viral infection, but the cause remains unknown. Complications include myelophthisic anemia, high output cardiac failure, pain secondary to nerve compression, deformities secondary to skeletal changes, and in approximately 1% of patients, osteosarcoma or other sarcoma, typically involving the jaw, pelvis, or femur.

An increased incidence of astrocytomas (choice A) is associated with tuberous sclerosis.

Hodgkin lymphoma (choice B) is usually a disease of young adults, although older patients may have the lymphocyte-depleted form.

Meningiomas (choice C) are mostly benign tumors that affect adults, especially women. There may be an association with breast cancer, possibly related to high estrogen states.

Non-Hodgkin lymphoma (choice D) is more common in AIDS and other immunodeficiency states, although the incidence in the immunocompetent is increasing.
Which of the following can be present with swollen and painful toes and knees, morning low back stiffness, conjunctivitis, as well as mouth and skin ulcerations in a young adult?
A. Gout
B. Lyme disease
C. Reiter's syndrome
D. Rheumatoid arthritis
E. Septic arthritis




The correct answer is C. This is a description of Reiter's syndrome. Patients typically present with the acute onset of arthritis (usually asymmetric and additive), with involvement of new joints occurring over a period of a few days to 2 weeks. Joints of the lower extremities are the most commonly involved, but wrists and fingers can also be affected. Dactylitis (sausage digit), a diffuse swelling of a solitary finger or toe, is a distinctive feature of Reiter's arthritis and psoriatic arthritis. Tendonitis and fasciitis are common, as are spinal pain and low back pain. Oligoarthritis, conjunctivitis, urethritis, and mouth ulcers are the most common features. The mucocutaneous lesions may include balanitis, stomatitis, and keratoderma blennorrhagicum. Microorganisms that can trigger Reiter's syndrome include Shigella spp., Salmonella spp., Yersinia spp., Campylobacter jejuni, and Chlamydia trachomatis. Most patients are younger males.

Gout (choice A) usually presents as an explosive attack of acute, very painful, monarticular inflammatory arthritis. Hyperuricemia is the cardinal feature and prerequisite for gout. The first metatarsophalangeal joint is involved in more than 50% of first attacks.

Lyme disease (choice B), caused by Borrelia burgdorferi, presents with a red macule or papule at the site of the tick bite. This lesion slowly expands to form a large annular lesion with a red border and central clearing. The lesion is warm, but usually not painful. The patient also has severe headache, stiff neck, chills, arthralgias, and profound malaise and fatigue. Untreated infection is associated with development of arthritis in the large joints (e.g., knees) lasting for weeks to months.

Rheumatoid arthritis (choice D) begins insidiously with fatigue, anorexia, generalized weakness, and vague musculoskeletal symptoms leading up to the appearance of synovitis. Pain in the affected joints, aggravated by movement, is the most common manifestation of established rheumatoid arthritis. Generalized stiffness is frequent, especially in the morning, and is usually greatest after periods of inactivity. Rheumatoid arthritis is more common in females. The metacarpophalangeal and proximal interphalangeal joints of the hands are characteristically involved.

Septic arthritis (choice E) is caused by a variety of microorganisms, including Neisseria gonorrhoeae and Staphylococcus aureus. Hematogenous spread is the most common route in all age groups. Approximately 90% of patients present with involvement of a single joint, usually the knee. The usual presentation is moderate to severe pain, effusion, muscle spasm, and decreased range of motion.
Which of the following antigens would autoantibodies be directed against when there is symmetrical swelling of the proximal phalangeal joints and large subcutaneous nodules over the extensor surfaces of both arms of a older woman?
A. Acetylcholine receptor
B. Double stranded DNA
C. Histones
D. IgG
E. Ribonucleoprotein




The correct answer is D. The disease described is rheumatoid arthritis, and the autoantibody is rheumatoid factor, which is usually an IgM or IgG (or less commonly IgA) directed against the constant region of autologous IgG.

Autoantibody directed against acetylcholine receptors (choice A) is a feature of myasthenia gravis.

Autoantibody directed against double stranded DNA (choice B) is a feature of systemic lupus erythematosus.

Autoantibody directed against histones (choice C) is a feature of drug-induced lupus.

Autoantibody directed against ribonucleoprotein (choice E) is a feature of mixed connective tissue disease.
What is the BEST explanation for rapid weight gain, buffalo hump formation, prominent vertical purple abdominal striae, and the increase of fasting blood glucose, plasma levels of ACTH and cortisol, and also the presence of osteoporosis, hypertension, poor wound healing, and hypokalemia in an older male?
A. Addison's disease
B. An ectopic ACTH-secreting tumor
C. Conn's syndrome
D. Cushing's disease
E. Primary hypercortisolism




The correct answer is D. This is a description of "Cushingoid" signs and symptoms caused by hypercortisolism. Although the acute effect of cortisol is to produce lipolysis, patients with chronically increased cortisol levels develop a characteristic central obesity and buffalo hump. The extremities are often thinned. The mechanism for the redistribution of body fat is not known but may involve an interaction between cortisol and insulin. The weight gain with hypercortisolism usually results from increased appetite. Cortisol excess causes protein catabolism, which leads to poor wound healing, decreased connective tissue, and fragile blood vessels. The combination of thin skin and fragile blood vessels leads to abdominal stretch marks (striae) that are characteristically purple in color. Because of increased gluconeogenesis and decreased peripheral insulin sensitivity, blood glucose may be increased. Osteoporosis, hypertension, poor wound healing, and hyperkalemia are also commonly seen. If the hypercortisolism is caused by a functional tumor in the adrenal cortex (primary hypercortisolism, choice E), plasma concentration of ACTH should be low because of negative feedback suppression. Increased cortisol and increased ACTH could result from a functional ACTH-secreting tumor in the pituitary (Cushing's disease) or an ectopic tumor (such as a small cell carcinoma of the lung, choice B).

Addison's disease (choice A) is primary adrenal insufficiency, and whereas plasma ACTH is increased (producing hyperpigmentation), plasma cortisol and aldosterone are decreased (not increased) compared with normal.

Conn's syndrome (choice C) results from hypersecretion of aldosterone by the adrenal cortex. Some of the clinical manifestations overlap with Cushing's disease: for example, both may exhibit hypertension. In the case of Conn's syndrome, this is caused by excessive renal sodium and water reabsorption because of increased aldosterone levels. In Cushing's disease, it is due in part to the mineralocorticoid-like effects of high plasma cortisol.
What condition is marked by dilated cardiomyopathy, slow speech and intellectual function, fatigue, lethargy, cold intolerance, listlessness, thickened facial features, periorbital edema, dry, coarse skin, and peripheral edema, and also with low serum levels of T4 and high ones of TSH?
A. Cretinism
B. Grave's disease
C. Hyperthyroidism
D. Myxedema
E. Thyroid cancer




The correct answer is D. Myxedema is caused by long-standing hypothyroidism in adults. Myxedema can result from the many causes of hypothyroidism: Hashimoto's thyroiditis, idiopathic primary hypothyroidism, iodine deficiency, drugs, pituitary lesions, hypothalamic lesions, and damage to the thyroid by surgery or radiation.

Cretinism (choice A) is caused by hypothyroidism in infancy.

Grave's disease (choice B) usually produces hyperthyroidism.

Hyperthyroidism (choice C) describes the signs and symptoms associated with an overproduction of thyroid hormone. Because the body's metabolism is increased,patients often feel hotter than those around them and can slowly lose weight even though they may be eating more. The weight issue is confusing sometimes since some patients actually gain weight because of an increase in their appetite. There is usually fatigue at the end of the day, but have trouble sleeping. Trembling of the hands and a hard or irregular heartbeat (called palpitations) may develop. These individuals may become irritable and easily upset. When severe, there can be shortness of breath, chest pain, and muscle weakness.

Thyroid cancer (choice E) often causes a painless swelling in the region of the thyroid. Thyroid function tests are usually normal.
Which of the following occurs after an upper respiratory infection and involves severe lower back pain, generalized muscle weakness, and distal paresthesia and demyelination, but there is no appreciable sensory loss and there is minimal residual sequelae?
A. Creutzfeldt-Jakob disease
B. Friedreich ataxia
C. Huntington's disease
D. Multiple sclerosis




The correct answer is D. This describes Guillain-Barr syndrome, also known as acute idiopathic inflammatory polyneuropathy. This condition, which typically follows an upper respiratory or other infection by several days to a month, is caused by an autoimmune attack on the myelin of peripheral nerves. In this respect, it is most similar to multiple sclerosis, which is an autoimmune attack on the myelin in the brain and spinal cord. Most cases of Guillain-Barr syndrome resolve spontaneously. A few patients have recurrences, and rare patients die during the acute episode of respiratory muscle failure (artificial ventilation may be required).

Creutzfeldt-Jakob disease (choice A) is an Alzheimer-like condition caused by a prion (protein infectious agent).

Friedreich ataxia (choice B) is an autosomal recessive disorder associated with spinocerebellar degeneration. Ataxia and paralysis are seen beginning in adolescence.

Huntington's disease (choice C) is an autosomal dominant degeneration of the caudate and frontal lobes characterized by movement disorder and dementia.
Which of the following can involve lassitude, myalgia, mylar rash, joint pain, elevated urinary protein, with the blood showing leukopenia and a high titer of antinuclear antibodies?
A. Generalized fatigue
B. Goodpasture's syndrome
C. Systemic lupus erythematosus
D. Scleroderma




The correct answer is C. Systemic lupus erythematosus (SLE) is a prototype connective tissue disease. The diagnosis requires four criteria to be met from a list of 11 possible criteria: malar rash, discoid rash, photosensitivity, oral ulcers, arthritis, serositis, renal disorder, neurologic disorder, hematologic disorder, immunologic disorder, and antinuclear antibody. The appearance of a malar or "butterfly" rash seen in approximately half of all patients is generally the easiest way to diagnose this condition on the dental boards. Antinuclear antibodies (ANA) are present in 95-100% of cases of SLE; anti-double-stranded DNA is found in 70% of the cases.

Generalized fatigue (choice A) can occur in anyone, but the presence of the other criteria make SLE more likely.

Goodpasture's syndrome (choice B) is characterized by linear disposition of immunoglobulin and often C3 along the glomerular basement membrane (GBM). Glomerulonephritis, pulmonary hemorrhage, and occasionally idiopathic pulmonary hemosiderosis occur.

Scleroderma (choice D) is characterized by thickening of the skin caused by swelling and thickening of fibrous tissue, with eventual atrophy of the epidermis. ANA are often associated with the disease, but the staining pattern is generally nucleolar.
Which of the following cancers may develop when there are islands of red tissue noted above the gastroesophageal junction in situations involving chronic reflux of gastric contents into the esophagus?
A. Adenocarcinoma of the esophagus
B. Adenocarcinoma of the stomach
C. Sarcoma of the esophagus
D. Sarcoma of the stomach
E. Squamous cell carcinoma of the esophagus




The correct answer is A. The lesion is Barrett's esophagus, which is related to chronic reflux of gastric contents into the esophagus. This lesion predisposes for the development of adenocarcinoma of the distal esophagus, which is the most serious complication of Barrett's esophagus. The development of adenocarcinoma of the esophagus is approximately 40 times higher in patients with Barrett's esophagus compared with those without the lesion.

Conditions predisposing for adenocarcinoma of the stomach (choice B) include chronic atrophic gastritis, pernicious anemia, and postsurgical gastric remnants.

Sarcoma of the esophagus (choice C) or stomach (choice D) is rare.

Plummer-Vinson syndrome predisposes for squamous cell carcinoma of the esophagus (choice E).
Which of the following when removed from the diet will help improve a history of weight loss, diarrhea, flatulence, greasy stools, and increased fecal fat and marked atrophy of villi?
A. Beef
B. Eggs
C. Potatoes
D. Tomatoes
E. Wheat




The correct answer is E. This describes celiac sprue which is caused by an allergic, immunologic, or toxic reaction to the gliadin component of gluten from wheat, as well as rye, barley, and oats, but not rice or corn. The symptoms and pathologic changes usually reverse with complete removal of gliadin from the diet. Therapeutic failures are frequently caused by hidden wheat in the diet. Patients with celiac disease have an increased risk for developing gastrointestinal lymphoma. The classic signs and symptoms of celiac sprue include weight loss, flatuence, greasy stools, and increased fecal fat. A diagnosis can be confirmed when clinical improvement is made on a gluten-free diet.
Which of the following is the MOST likely explanation for a rash in a leukemic patient that consists of multiple erythematous patches on the arms, legs, palms, and soles, some of which show central clearing and surrounding ring formation?
A. Urticaria
B. Erythema multiforme
C. Kaposi's sarcoma
D. Psoriasis




The correct answer is B. The presence of erythematous patches with central clearing, known clinically as target lesions, is associated with erythema multiforme. Both erythema multiforme and its severe, life-threatening version, known as Stevens-Johnson syndrome, are produced by immune complex deposition in dermal blood vessels. In approximately 50% of patients, no specific precipitating cause is identified. In the remainder of patients, however, a variety of causes have been implicated, including certain infections (herpes simplex, enteroviruses, Mycoplasma pneumoniae, Chlamydia, histoplasmosis), drugs, neoplasia, sarcoidosis, and foods. Some penicillins and corticosteroids can also cause this condition.

Urticaria (choice A) causes wheals that are intensely pruritic, but does not produce target lesions.

Kaposi's sarcoma (choice C) causes purple lesions with no target lesions.

Psoriasis (choice D) causes erythematous plaques with silvery scales but does not produce target lesions.
A 72-year-old man with a significant smoking history presents with dyspnea, facial erythema, and facial, truncal, and arm edema with prominence of thoracic and neck veins. Chest x-ray reveals a mass in the right mediastinum with adenopathy. Which of the following is the most likely diagnosis?
A. Adenocarcinoma
B. Hodgkin lymphoma
C. Large cell carcinoma
D. Non-small cell carcinoma
E. Small cell carcinoma




The correct answer is E. Superior vena cava (SVC) syndrome is characterized by obstruction of venous return from the head, neck, and upper extremities. More than 85% of cases of SVC syndrome are related to malignancy. Bronchogenic carcinomas (most commonly small cell cancer and squamous cell cancer) account for more than 80% of these cases. Among bronchogenic carcinomas, the most common causes of SVC syndrome (in order of frequency) are small cell carcinoma, epidermoid carcinoma, adenocarcinoma (choice A), and large cell carcinoma (choice C). Lymphomas such as Hodgkin disease (choice B) and non-Hodgkin lymphoma are uncommon causes of SVC syndrome. Rare tumors associated with SVC syndrome include primary leiomyosarcomas and plasmacytomas. Infectious etiologies include tuberculosis, syphilis, and histoplasmosis. SVC syndrome can also occur as a result of an enlarged goiter, and from thrombus formation caused by indwelling intravenous lines or pacemaker wires.

Non-small cell carcinoma (choice D) is not commonly associated with SVC syndrome.
What can be present when there is digital clubbing and tenderness over the distal ends of the radius, ulna, and fibula?
A. Gastrointestinal cancer
B. Liver cancer
C. Lung cancer
D. Renal cancer
E. Testicular cancer




The correct answer is C. This patient has the finger clubbing and hypertrophic pulmonary osteoarthropathy that can be associated with bronchogenic carcinoma (other than squamous cell carcinoma), benign mesothelioma, and diaphragmatic neurilemmoma. X-ray of the bones generally shows formation of new periosteal bone; arthritis may be present. The etiology of these changes remains a mystery. An alert clinician may identify a cancer at an earlier, potentially curable stage by investigating a possible paraneoplastic syndrome. Digital clubbing is caused by chronically low oxygen levels. This condition affects the fingers and toes in which proliferation of the distal tissues, especially the nail beds, results in broadening of the extremities of these digits. The affected nails are abnormally curved and shiny.
Which of the following lesions are present in an adult with newly diagnosed tuberculosis?
A. A single lesion in a lung apex
B. A single lesion in the gastrointestinal tract
C. A single lesion subjacent to the pleura
D. A lesion subjacent to the pleura in the lower part of an upper lobe and active disease in the mediastinal lymph nodes
E. Multiple tiny masses throughout the body




The correct answer is A. Because tuberculosis is transmitted primarily by "droplet" transmission by way of the nasopharynx, it is essential that the dental student understand the basic principles of tuberculosis. Primary tuberculosis infection characteristically involves the lung subjacent to the pleura in either the lower part of the upper lobe or the upper part of a lower lobe of one lung. The mediastinal nodes are also usually involved (choice D), rather than having a single lesion subjacent to the pleura (choice C) without lymph node involvement. Roughly 80% of newly diagnosed pulmonary tuberculosis cases in adults are actually caused by reactivation of an often clinically unsuspected infection acquired years to decades previously. The reinfection site usually is in the apex of the lung. The source of reinfection is usually a Gohn complex, or calcified granuloma of giant cells, mycobacteria leukocytes, and fibrous cells.

The lungs are not the only site where tuberculosis can occur (it can occur throughout the body), and isolated gastrointestinal involvement (choice B) is (uncommonly) also seen.

Miliary tuberculosis (rare) is a widely disseminated and dangerous form of tuberculosis characterized by small lesions throughout the body (choice E). It is more common in immunocompromised patients.
Which of the following is involved when there is excessive bleeding after labor, and the laboratory studies demonstrate decreased platelets, prolonged prothrombin time and partial thromboplastin time, and increased fibrin split products?
A. Disseminated intravascular coagulation
B. Hemophilia A
C. Severe liver disease
D. Vitamin K deficiency
E. Von Willebrand's disease




The correct answer is A. Thsi is a description of disseminated intravascular coagulation (DIC), a feared and often life-threatening complication of many other disorders, including amniotic fluid embolism, infections (particularly gram-negative sepsis), malignancy, and major trauma. This is suspected when a decrease in platelets and a prolongation of PT and PTT times are observed. The observed hematologic abnormalities are caused by consumption of platelets and clotting factors, caused by extensive microclot formation with accompanying fibrinolysis (reflected by the increased fibrin split products).

Hemophilia (choice B) alters the PTT without affecting the other indices.

Severe liver disease (choice C) produces alterations comparable to those in vitamin K deficiency; platelets can also be decreased secondary to a generalized metabolic marrow dysfunction, but fibrin split products would not be increased.

Vitamin K deficiency (choice D) is associated with alterations in PT and PTT, but platelets will not be decreased, nor will fibrin split products be increased.

Von Willebrand's disease (choice E) produces impaired platelet adhesion and increases the bleeding time and the PTT, but will not produce the other features described.
Which of the following can account for a child developing fever, conjunctivitis, photophobia, cough, white spots on a bright red background on the buccal mucosa, and a rash that begins around the hairline, then spreads to the trunk and extremities?
A. Aphthous ulcers
B. Herpetic stomatitis
C. Laryngeal papillomas
D. Measles
E. Oral thrush




The correct answer is D. This description is of measles, with the appearance of Koplik's spots (white spots on the buccal mucosa) followed by a rash beginning along the neck and hairline and spreading to the trunk and extremities. The sequela of this condition may be postinfectious encephalomyelitis that can follow infection with measles, varicella, rubella, mumps, influenza, or vaccination with vaccinia vaccine or rabies vaccine derived from nervous tissue.

Aphthous ulcers (choice A) are easy to recognize because they are round ulcerations with yellow-gray fibrinoid centers surrounded by red halos.

Herpetic stomatitis (choice B) is common, mild, short-lived, and requires no intervention. There is an initial burning followed by small vessicles that rupture to form scabs.

Laryngeal papillomas (choice C) are common lesions of the larynx and other sites where ciliated and squamous epithelia meet. Hoarseness progresses to stridor over the course of weeks to months.

Oral thrush (choice E) is associated with the development of painful, creamy white, curd- like patches overlying erythematous mucosa.
What is present when an elderly person can no longer recognize people and common objects, plan activities, and wanders with an absent look on their face, but who is still happy, yet frustrated by the memory loss, and speech is limited to simple two- or three-word sentences?
A. Alzheimer's disease
B. Amnestic disorder
C. Clinical depression
D. Substance-induced persisting dementia
E. Parkinson's disease




The correct answer is A. This is a description of dementia of the Alzheimer's type. A gradual onset of symptoms, general pervasive memory deficit, difficulties with language, and inability to plan, leading to severe impairment of daily functioning are all characteristic of dementia of the Alzheimer's type.

Amnestic disorder (choice B) is limited to memory problems; and not cognitive dysfunction, such as alterations in language and the loss of the ability to plan.

Clinical depression (choice C) is associated with feelings of helplessness and hopelessness. Cognitive function is not impaired.

The diagnosis of substance-induced persisting dementia (choice D) requires evidence of a history of substance abuse. It is the second most likely diagnosis, however, and should be carefully explored.

Parkinson's disease (choice E) is associated with develpment of dyskinesia, postural instability, moonlike facies, and difficulty performing daily activities.
Which of the following substances would be MOST likely be elevated due to the development of large osteoblastic bone lesions in a patient with prostate cancer?
A. Prostatic acid phosphatase
B. Prostate specific antigen
C. Serum alkaline phosphatase
D. Serum uric acid
E. Leukocytes




The correct answer is C. Osteoblastic cells respond to metastatic prostate carcinoma by forming bone (osteoid), and secreting alkaline phosphatase, which is believed to initiate or facilitate mineralization.

Prostatic acid phosphatase (choice A) and prostatic-specific antigen (choice B) are synthesized by the tumor and would most likely be elevated; however, they are elevated because of the prostatic cancer independent of the bony metastasis.

Serum uric acid (choice D) would be expected to be seen in patients with gout.

White blood cells (choice E) can be elevated by a bacterial infection, rheumatoid arthritis, and several medications, including glucocorticoids and lithium. In other words, elevated white blood cells is often a nonspecific indicator.
Which of the following pair of hormones regulates the hormone responsible for a middle-aged man experiencing gradual coarsening of facial features, progressive protrusion of the lower jaw, and having to wear larger shoes and gloves?
A. Dopamine and norepinephrine
B. LH and hCG
C. Prolactin and FSH
D. Somatostatin and GHRH
E. TSH and ACTH




The correct answer is D. The disease described is acromegaly, which is typically produced by a growth hormone-secreting pituitary adenoma. Growth hormone synthesis is predominantly regulated by hypothalamic GHRH (growth hormone-releasing hormone), and its pulsatile secretion is predominantly regulated by hypothalamic somatostatin. Clinical features of acromegaly include excessive growth of hands and feet in adults, protusion of the lower jaw, coarsening of facial features, and a deeper voice.

Dopamine and norepinephrine (choice A) are catecholamines that regulate smooth muscle tone and cardiac function.

Choice B is incorrect because luteinizing hormone (LH) regulates sex steroid hormone production by testes and ovaries; human chorionic gonadotropin (hCG) is produced by the placenta and has actions similar to LH.

Choice C is incorrect because prolactin regulates menstruation and lactation, whereas follicle stimulating hormone (FSH) regulates ovarian and testicular function.

Choice E is incorrect because thyroid stimulating hormone (TSH) regulates secretion of thyroid hormones and adrenocorticotropin (ACTH) regulates glucocorticoid secretion.
Which of the following tissues or organs may be associated with hyperplasia and neoplastic proliferation if there is fatigue, feeling of constant coldness, diffusely enlarged and rubbery thyroid gland, while laboratory tests show low T3 and T4, high TSH, large numbers of lymphocytes of all degrees of maturation, a few abnormal follicular cells with eosinophilic granular cytoplasm, and only rare normal follicular cells?
A. Colon
B. Esophagus
C. Peripheral nerve
D. Skin
E. Thymus




The correct answer is E. The thyroid disease is Hashimoto's thyroiditis, an autoimmune disease in which the thyroid parenchyma is destroyed by a lymphocytic infiltrate. The infiltrate typically contains mature follicles; the remaining scanty follicular cells often have eosinophilic granular cytoplasm and are called Hurthle cells or oncocytes. Clinically, patients usually have hypothyroidism, although brief periods of hyperthyroidism ("Hashitoxicosis") may also be seen. Like myasthenia gravis, Hashimoto's disease may be associated with thymic disorders, including thymic hyperplasia, benign thymomas, and malignant thymomas.

Colon (choice A) cancer is associated with ulcerative colitis and adenomatous polyps.

The risk for esophageal cancer (choice B) is increased with Barrett's esophagus and in Plummer-Vinson syndrome.

You should associate neurofibromas of peripheral nerve (choice C) with cafe au lait spots on the skin.

Skin cancer (choice D) occurs with greater frequency in association with xeroderma pigmentosa and actinic keratosis.
What is occuring when an esophagus is dilated, kinked, tortuous, partly filled with undigested foods, and is associated with chronic dysphagia?
A. Achalasia
B. Barrett's esophagus
C. Hiatal hernia
D. Plummer-Vinson syndrome
E. Zenker's diverticulum




The correct answer is A. Achalasia (from the Greek "unrelaxed") is a disease of ganglion cells in the esophageal myenteric plexus causing a failure of relaxation in the lower esophageal (cardiac) sphincter. The cause of achalasia is usually not determined. The peristaltic waves in the esophagus stop before the sphincter, and the food collects in the esophagus, which becomes dilated and elongated.

Barrett's esophagus (choice B) is metaplastic replacement of the squamous esophageal epithelium with columnar epithelium. Barrett's esophagus is an important risk factor for esophageal adenocarcinoma.

Hiatal hernia (choice C) is a protrusion of the stomach into the thorax by way of the diaphragmatic hiatus, at the lower esophageal sphincter. Although hiatal hernia can produce gastroesophageal reflux, the esophagus does not become distended and food passes normally into the stomach.

Plummer-Vinson syndrome (choice D) is a constellation of physical findings associated with severe iron-deficiency anemia including koilonychia, atrophic glossitis, and dysphagia caused by atrophy of the pharyngeal mucosa and mucosal webs in the upper esophagus.

Zenker's diverticulum (choice E) is an oropharyngeal diverticulum occurring at the junction of the pharynx and esophagus which occurs because of wall weakness in the esophagus at this location, and may produce dysphagia.
What is involved in a smoker with scant clear mucoid sputum, quiet sounding chest, hemoptysis, weight loss, detruction of alveolar septae around the respiratory bronchioles, with marked enlargement of the airspaces, and heavy deposits of anthracotic pigment?
A. Asthma
B. Chronic bronchitis
C. Emphysema
D. Pulmonary hypertension
E. Silicosis




The correct answer is C. Emphysema is a pulmonary disease characterized by enlargement of the alveolar airspaces caused by destruction of the septae without consequent fibrosis. The gross appearance of emphysematous lungs is characteristic: alveoli are sufficiently dilated to allow visualization with the naked eye and destruction of structural support to lymphatic vessels produces heavy pigment deposition in the tissue. Microscopic findings that confirm the diagnosis include enlarged, round airspaces with club-like ends of broken septae sticking into the alveoli. There is scant clear mucoid sputum. The chest is very quiet without adventitious sounds.

Asthma (choice A) is a disease of airway hyperreactivity and is characterized by hypertrophy of the bronchial basement membranes and smooth muscle, with glandular hyperplasia and thick mucus plugs in the bronchi. Wheezing will be heard on expiration.

Chronic bronchitis (choice B) produces marked hypersecretion of mucopurulent mucus in the large airways and can be identified by hypertrophy of mucous glands in the bronchi and goblet cell hyperplasia in the smaller airways. The chest is noisy with rhonci invariably present. Wheezing is common.

Pulmonary hypertension (choice D) affects neither the airways nor the alveoli. It is characterized by thickening of the arterial smooth muscle with intimal hyperplasia and fibrosis. Atherosclerotic changes in the normally plaque-free larger pulmonary arteries may be seen.

Silicosis (choice E), one of the forms of pneumoconiosis, is an interstitial fibrosing disease that produces thick pleural scars and dense nodules of collagen that may calcify. The silica particles may be visualized within the nodules using polarized light.
What presents with painful widespread erosions on the mucous membranes with friability, no well-defined borders, acantholysis, and with direct immunofluorescence demonstrate an intraepidermal band of IgG and C3?
A. Bullous pemphigoid
B. Dermatitis herpetiformis
C. Herpes simplex I
D. Herpes simplex II
E. Pemphigus vulgaris




The correct answer is E. This is a description of pemphigus vulgaris, in which autoantibody directed against transmembrane cadherin adhesion molecules induces acantholysis (breakdown of epithelial cell-cell connections) with resulting intraepidermal blister formation. It has an insidious onset of flaccid bullae in crops or waves. It may develop spontaneously or following triggers such as drugs (thiols, penicillamine), physical injury (burns), cancer, pregnancy, other skin diseases, and emotional stress. Pemphigus vulgaris is a relatively rare blistering disease; it is seen more commonly in patients with Jewish or Mediterranean heritage. The epidermis at the edge of these erosions is often easily disrupted by sliding pressure (Nikolsky sign). Pemphigus vulgaris begins in the mouth in 50% of cases. Acantholysis is seen on biopsy.

Bullous pemphigoid (choice A) is characterized by deeper blisters occurring at the dermal-epidermal junction.

Dermatitis herpetiformis (choice B) is characterized by severe, intense pruritus and groups of papules and vesicles.

Herpes simplex I (choice C) or II (choice D) can show multinucleated giant cells on scrapings of the ulcer base.
Which of the following thyroid diseases is most likely related to restlessness, fever, profuse sweating, marked tachycardia and tremor during labor, later developing delirium, nausea, vomiting, and abdominal pain between contractions?
A. Follicular carcinoma
B. Grave's disease
C. Hashimoto's thyroiditis
D. Hypertensive urgency
E. Papillary carcinoma




The correct answer is B. This is a description of a thyrotoxic crisis that occurs most commonly in untreated or inadequately treated Grave's disease. The onset is typically abrupt and may be precipitated by stressors that can include infection, trauma, radioiodine treatment, and childbirth. The condition, if unrecognized, may progress to congestive cardiac failure, pulmonary edema, and death.

Both follicular (choice A) and papillary (choice E) carcinomas of the thyroid gland are usually nonsecretory and consequently do not produce hyperthyroidism.

Hashimoto's thyroiditis (choice C) is an autoimmune thyroiditis that may transiently produce hyperthyroidism before producing hypothyroidism, but thyrotoxic crisis is not usually a feature.

Hypertensive urgency (choice D) is a situation in which blood pressure must be reduced in a few hours. Most patients are asymptomatic with severe hypertension (systolic > 220 mmHg or diastolic > 125 mmHg).
Which of the following is the most probable etiology for a young person with acute bacterial endocarditis limited to the tricuspid valve?
A. Congenital heart disease
B. Illicit drug use
C. Rheumatic fever
D. Rheumatoid arthritis
E. Rheumatic heart disease




The correct answer is B. The most probable etiology of bacterial endocarditis involving the tricuspid valve is illicit intravenous drug use, which can introduce skin organisms into the venous system that then attack the tricuspid valve. Staphylococcus aureus accounts for 60-90% of cases of endocarditis in intravenous drug users. This is an important concept for dentists because some patients with drug seeking behaviors, such as the intravenous drug abuser, may present to a dental office in search of narcotic medications.

The endocarditis associated with congenital heart disease (choice A) typically involves either damaged valves or atrial or ventricular septal defects. The tricuspid valve is not particularly vulnerable.

Rheumatic fever (choice C) most commonly damages the mitral and aortic valves, and tricuspid damage is usually less severe and seen only when the mitral and aortic valves are heavily involved. Consequently, secondary bacterial endocarditis involving only the tricuspid valve in a patient with a history of rheumatic fever would be unusual.

Rheumatoid arthritis (choice D) is not associated with bacterial endocarditis, unless there is an association with SLE.

Rheumatic heart disease (choice E) which occurs after rheumatic fever and again involves mainly the mitral and aortic valves.
Which of the following is the most probable etiology for a young person with acute bacterial endocarditis limited to the tricuspid valve?
A . Congenital heart disease
B. Illicit drug use
C. Rheumatic fever
D. Rheumatoid arthritis
E. Rheumatic heart disease




The correct answer is B. The most probable etiology of bacterial endocarditis involving the tricuspid valve is illicit intravenous drug use, which can introduce skin organisms into the venous system that then attack the tricuspid valve. Staphylococcus aureus accounts for 60-90% of cases of endocarditis in intravenous drug users. This is an important concept for dentists because some patients with drug seeking behaviors, such as the intravenous drug abuser, may present to a dental office in search of narcotic medications.

The endocarditis associated with congenital heart disease (choice A) typically involves either damaged valves or atrial or ventricular septal defects. The tricuspid valve is not particularly vulnerable.

Rheumatic fever (choice C) most commonly damages the mitral and aortic valves, and tricuspid damage is usually less severe and seen only when the mitral and aortic valves are heavily involved. Consequently, secondary bacterial endocarditis involving only the tricuspid valve in a patient with a history of rheumatic fever would be unusual.

Rheumatoid arthritis (choice D) is not associated with bacterial endocarditis, unless there is an association with SLE.

Rheumatic heart disease (choice E) which occurs after rheumatic fever and again involves mainly the mitral and aortic valves.
What is the specific mechanism by which death is produced from breathing carbon monoxide in a closed space?
A. Damage to the plasmalemma
B. Decreased oxygen-carrying capacity of blood
C. Increased calcium transport into mitochondria
D. Poisoning of oxidative phosphorylation
E. Rupture of lysosomes




The correct answer is B. Carbon monoxide has a very high affinity for hemoglobin, and binds, nearly irreversibly, in such a manner that oxygen cannot bind, drastically decreasing the oxygen-carrying capacity of the blood. Carbon monoxide also causes the oxygen-hemoglobin dissociation curve to shift to the left, making oxygen more difficult to unload. Traditionally, carbon monoxide poisoning have been described as having "cherry red" blood and skin, but this change is somewhat unreliable in real life.

Choices A, C, and E list secondary changes that are commonly observed in injured cells, no matter what the cause of the injury.

Cyanide acts by poisoning oxidative phosphorylation (choice D).
Which of the following clinical features would be most likely to be present when thyroid function tests reveal increased TSH, decreased total T4, decreased free T4, and decreased T3 uptake?
A. Diarrhea
B. Heat intolerance
C. Hyperactivity
D. Palpitations
E. Weight gain




The correct answer is E. The laboratory studies are typical for primary hypothyroidism, in which the thyroid fails to produce adequate T4 despite appropriate TSH signals from the pituitary gland. Clinical features of hypothyroidism include weight gain, cold intolerance, hypoactivity, fatigue, lethargy, decreased appetite, constipation, weakness, decreased reflexes, facial and periorbital myxedema, dry and cool skin, and brittle hair.

In contrast, diarrhea (choice A), heat intolerance (choice B), hyperactivity (choice C), and palpitations (choice D) are features of hyperthyroidism.
Which of the following findings would be MOST consistent for a young person in a coma having an acetone-like odor?
A. Alcohol intoxication
B. Diabetic hyperosmolar coma
C. Diabetic ketoacidosis
D. Heroin overdose
E. Profound hypoglycemia




The correct answer is C. The smell of acetone on the breath of a comatose person is an important, rapid diagnostic clue that strongly suggests ketoacidosis and is usually seen in patients with poorly controlled type 1 diabetes mellitus. Other features of diabetic ketoacidosis include high blood glucose, increased serum osmolality, hypovolemia, acidosis, and electrolyte imbalance. It is important to note that diabetic ketoacidotic coma is usually preceded by a day or more of polyuria and polydipsia associated with marked fatigue, confusion, and nausea.

In alcohol intoxication (choice A), the breath will smell like alcohol. Some may confuse a patient with diabetes in ketoacidosis as being "drunk," however, because of the similar smell.

Diabetic hyperosmolar coma (choice B) usually is seen in older patients with type 2 diabetes mellitus and is not characterized by ketoacidosis. Because there is no acetone production, there is no specific scent to the breath.

In heroin overdose (choice D), no acetone production occurs and there is no specific scent to the breath.

In hypoglycemic coma (choice E), which can occur in diabetics with insulin overdose, no acetone production occurs and there is no specific scent to the breath.
A 24-year-old man presents with complaints of itching on his arms and face. He states that he took four 500-mg capsules of amoxicillin 1 hour before a dental procedure. Physical examination reveals well circumscribed wheals with raised, erythematous borders and blanched centers. Which form of hypersensitivity is this patient probably exhibiting?
A. Acute serum sickness (Type III)
B. Antibody-dependent cell-mediated cytotoxicity (Type II)
C. Antireceptor antibodies (Type II)
D. Delayed type hypersensitivity (Type IV)
E. Immediate type hypersensitivity (Type I)




The correct answer is E. Urticaria (hives) is a good example of a local anaphylaxis reaction that is classified as a Type I hypersensitivity reaction. Type I hypersensitivity reactions involve preformed IgE antibody bound to mast cells or basophils that release vasoactive and spasmogenic substances when they react with antigens. Certain allergens, especially drugs, insect venoms, latex, and foods may induce an IgE antibody response, causing a generalized release of mediators from mast cells leading to a systemic reaction.

Acute serum sickness (choice A) is now uncommon but was formerly seen when animal sera were used for passive immunization. Serum-like sickness has also been seen in patients receiving cephalosporins, such as cefaclor.

The eosinophil-mediated cytotoxicity against parasites is an example of antibody-dependent cell-mediated cytotoxicity (choice B).

Myasthenia gravis is an example of a disease caused by antireceptor antibodies (choice C).

The tuberculin (PPD) reaction used to test for tuberculosis exposure is an example of delayed-type hypersensitivity (choice D).
Which of the following notations in a chart indicates the greatest likelihood that a 1-cm, flat,white patch on the buccal mucosa will progress to a malignancy?
A. Hairy leukoplakia
B. Leukoplakia
C. Linea alba
D. Oral thrush
E. Squamous papilloma




The correct answer is B. Leukoplakia is a white plaque on the oral mucosa for which a more specific diagnosis cannot be rendered. For example, unlike oral candidiasis the white lesion cannot be removed by rubbing the oral mucosal surface. Leukoplakia is often associated with hyperkeratosis and may or may not show dysplastic squamous epithelium. On average, 2-6% represent either dysplasia or early invasive squamous cell carcinoma.

Hairy leukoplakia (choice A) is an oral infective lesion seen almost exclusively in patients with HIV infection. It is a fluffy, white, hyperkeratotic lesion in which a destructive piling up of keratotic squames is seen. Hairy leukoplakia is associated with viral infection, mostly EBV, HPV, or HIV. It does not progress to cancer.

Linea alba (choice C) is a linear white area on the buccal mucosa formed from hyperkeratosis due to occlusion, and may be excessive in those that clench or grind.

Oral thrush (choice D) is a superficial candidal infection, typically occurring in immunosuppressed or very young patients. Thrush is an infectious, non-neoplastic disease.

Squamous papilloma (choice E) is a benign human papillomavirus (HPV) infection of the oral mucosa. Typically associated with HPV genotypes 6 and 11, squamous papilloma only rarely progresses to squamous carcinoma.
Which of the following is present in a elderly person that has thin arms and legs, a swollen abdomen, red tongue, dry, thin, and slightly yellow skin, gynecomastia, testicular atrophy, multiple spider angiomas, tremor, yellow discoloration of sclera, and short-term memory loss?
A. Bronchogenic carcinoma
B. Colon carcinoma
C. Congestive heart failure
D. Type 2 diabetes mellitus
E. Hepatic cirrhosis




The correct answer is E. The physical examination is typical for an alcoholic with advanced hepatic cirrhosis. It is important to recognize these symptoms, as these patients are notorious for "underestimating" and even denying their alcohol use.

Bronchogenic carcinoma (choice A) typically presents with cough or respiratory changes, but can present with mass effects in the chest or involvement of mediastinal nerves or vessels.

Colon cancer (choice B) typically presents with changes in the stool or bowel habits.

Congestive heart failure (choice C) is typically heralded by shortness of breath or peripheral edema, or both.

Type 2 diabetic patients (choice D) are typically obese and present with usual signs and symptoms of diabetes, such as polyuria, polydipsia, and polyphagia (but not ketoacidosis).
Which of the following laboratory tests is performed following a head injury so as to differentiate between nephrogenic and neurogenic diabetes insipidus?
A. Creatinine in a 24-hour urine collection
B. Urine osmolality after vasopressin administration
C. Serum renin after infusion of hypotonic saline
D. Serum sodium after infusion of hypertonic saline




The correct answer is B. Diabetes insipidus is characterized by the excretion of abnormally large volumes of dilute urine (polyuria) with a commensurate increase in fluid intake (polydipsia). There are two types, neurogenic and nephrogenic. The most common type is neurogenic diabetes insipidus, which is caused by inadequate secretion of antidiuretic hormone (ADH) or vasopressin. In the absence of ADH, a brisk diuresis results; the osmolality of the urine may be extremely low. Many with neurogenic (central) diabetes insipidus have a history of head trauma, brain tumors, or brain infections that damage the hypothalamus or neurohypophysis. Nephrogenic diabetes insipidus, which presents in much the same way, can be seen in association with certain renal diseases caused by a defect in the renal V2 vasopressin receptor, Gs protein, or other steps in the formation of cyclic AMP. Plasma levels of vasopressin are usually increased, because of the hyperosmolarity of the serum. Water deprivation will fail to increase urine osmolarity in neurogenic and nephrogenic diabetes insipidus. Because V2 receptors are functional in neurogenic diabetes insipidus, however, administration of exogenous ADH will still concentrate the urine, whereas this would be ineffective in nephrogenic diabetes insipidus.

Measurement of creatinine (choice A) in a 24-hour urine collection, coupled with plasma creatinine values and the urine flow rate can be used to approximate glomerular filtration rate.

Renin secretion would be inhibited by infusion of hypotonic saline (choice C) in neurogenic and nephrogenic diabetes.

Serum sodium (choice D) would rise with infusion of hypertonic saline in either type of diabetes insipidus.
Which of the following laboratory results would be expected in a child with 2 standard deviations below the expected mean height, delayed bone maturation, goiter, and a point mutation in the thyroid hormone receptor?
A. Decreased radioactive iodine uptake test
B. Increased plasma T4 concentration
C. Decreased plasma TSH concentration
D. Increased basal metabolic rate




The correct answer is B. Generalized resistance to thyroid hormone is a rare genetic abnormality (Refetoff syndrome). It results from mutations of the thyroid hormone receptor gene. Depending on the severity of the disorder, patients may be only mildly affected or may exhibit striking hypothyroid-like symptoms including decreased basal metabolic rate (BMR) (not increased, choice D). Growth can be stunted, there may be deaf mutism, and attention span may be short. Because the thyroid hormone resistance is generalized, the normal negative feedback effects of T4 and T3 at the hypothalamus and pituitary are also deficient. This would lead to an increased plasma TSH concentration (not decreased, choice C). Because of the increase in plasma TSH, iodine trapping by the thyroid follicular cells will be increased, leading to an increase in radioactive iodine uptake test (RAIU) (not decreased, choice A) and an increase in serum T4.
What form of arthritis is present in an elderly woman that complains of ill-fitting dentures, dry eyes and mouth, and severe arthritic pain?
A. Gonococcal arthritis
B. Gouty arthritis
C. Osteoarthritis
D. Pseudogout
E. Rheumatoid arthritis




The correct answer is E. Dry eyes and mouth in an elderly woman are probably caused by Sjogren's syndrome, an autoimmune attack on the salivary and tear glands. Sjogren's syndrome is associated, in some cases, with rheumatoid arthritis.

Gonococcal arthritis (choice A) is a systemic complication of gonorrhea, typically observed in young women who have had a recent sexual encounter.

Gout (choice B) and pseudogout (choice D) can cause arthritis secondary to crystal deposition in joints.

Osteoarthritis (choice C) seems to be caused by repetitive low-level trauma to joints.
A 7-year-old boy is not having any behavioral problems in school, but he is having a great deal of trouble paying attention in class. He has normal IQ and cognitive function, but occasionally asks that questions be repeated after staring blankly into space for a few seconds. Which of the following disorders most likely accounts for these symptoms?
A. Absence seizures
B. Attention deficit hyperactivity disorder
C. Schizophrenia with childhood onset
D. Parkinson disease




The correct answer is A. The patient described in this question is suffering from absence seizures that typically appear during childhood between the ages of 5 and 7 years. In absence seizures, the patient has many episodes of brief disruption of consciousness throughout the day. These seizures are not accompanied by the convulsions and complete loss of consciousness often associated with epilepsy, but rather by the absence of motor or sensory symptoms (hence the blank look on the patient's face).

Children with attention deficit hyperactivity disorder (choice B) also have a limited attention span and normal intelligence. They also exhibit hyperactivity, impulsiveness, emotional lability, and irritability, however, which lead to behavioral problems in school.

Schizophrenia with childhood onset (choice C) is rare. Children with this disorder demonstrate normal intelligence and may show a limited attention span. These children also manifest the same psychiatric symptoms seen in adult-onset schizophrenics, however, including hallucinations, delusions, abnormal affect, and limited social skills that lead to behavioral problems in school.

Parkinson disease (choice D) most commonly onsets between ages 45-65 years. It is associated with any combination of tremor, rigidity, bradykinesia, and progressive postural instability.
Which of the following hormones is most likely secreted by mass in the head of the pancreas in a person with multiple intractable gastric and duodenal peptic ulcers and marked elevation of gastric acid secretion?
A. Calcitonin
B. Epinephrine
C. Gastrin
D. Parathormone
E. Vasoactive intestinal peptide




The correct answer is C.This is a description of Zollinger-Ellison syndrome in which a duodenal or pancreatic gastrin-secreting endocrine tumor causes hypersecretion of gastric acid. Two thirds of these tumors are malignant. One third are related to MEN I that also causes parathyroid hyperplasia/adenomas and pituitary adenomas.

Calcitonin (choice A) is usually secreted by medullary carcinoma of the thyroid.

Epinephrine (choice B) is secreted in pheochromocytoma.

Pancreatic endocrine tumors may also secrete vasoactive intestinal peptide (VIP, choice E), parathyroid hormone (choice D), parathyroid hormone-related peptide, insulin, glucagon, somatostatin, ACTH, or growth hormone-releasing hormone (GHRH). Secretion of these hormones is less common than secretion of gastrin and is not specifically suggested by the peptic ulcer history.
Which of the following findings would MOST likely also be present with a history of scleroderma that has a near absence of smooth muscle peristalsis and lower esophageal sphincter tone?
A. Anemia
B. Atrophic glossitis
C. Hourglass-shaped stomach on barium swallow
D. Massively dilated colon
E. Muscle replaced with connective tissue




The correct answer is E. Near complete absence of muscle tone and peristalsis is characteristic of involvement of the esophagus with scleroderma, which causes replacement of muscle by dense connective tissue. Similar changes in the dermis cause the skin to be thickened. The thickened, shiny skin of the hands may cause them to resemble claws with telangiectasia and areas of increased pigmentation and depigmentation. Systemic features include dysphagia and hypomotility of the gastrointestinal tract.

Anemia (choice A) and atrophic glossitis (choice B) are associated with esophageal webs in Plummer-Vinson syndrome.

An hourglass-shaped stomach within the thoracic cavity (choice C) is a feature of a sliding hiatal hernia.

A massively dilated esophagus (megaesophagus) can be caused by Chagas' disease, a trypanosomal disease that can also cause massive dilation of the colon (choice D).
Which of the following is the MOST significant risk factor for breast cancer in a 40-year-old woman?
A. Breast-feeding after pregnancy
B. Caffeine consumption
C. Cigarette smoking
D. Family history
E. Nulliparity




The correct answer is D. Positive family history is definitively the most significant risk factor for developing breast cancer among the ones mentioned. Approximately 5-10% of cases are attributable to inheritance of autosomal dominant genes. Most hereditary cases of breast cancer are caused by two genes, BRCA1 and BRCA2, which are tumor suppressor genes probably involved in DNA repair. Note, however, that 80-90% of women with breast cancer do not have a positive family history.

Breast-feeding after pregnancy (choice A) is not considered a risk factor for breast cancer. Although research is ongoing, it may be that the breast cancer risk in women who breast-feed is decreased.

There is no convincing evidence linking caffeine consumption or cigarette smoking (choices B and C) to breast cancer.

Breast cancer is more frequent in nulliparous women than in multiparous women. Nulliparity (choice E) is therefore a risk factor, but its role is considered less important than family history. Similar to a long duration of reproductive life and late age at first intercourse, nulliparity (never given birth to a child) seems to increase breast cancer risk by increasing exposure to endogenous estrogen during the menstrual cycle.
A child with cystic fibrosis may have a history of meconium ileus, chronic cough, frequent respiratory tract infections, and bulky, foul-smelling stools and _______ ?
A. Cystinuria
B. Hypoglycemia
C. Iron deficiency anemia
D. Laryngeal papillomas
E. Pancreatic insufficiency




The correct answer is E. This is a descriptionof cystic fibrosis. In this disorder, an abnormality of chloride channels causes all exocrine secretions to be more viscous than normal. Because of pancreatic insufficiency, the pancreatic secretion of digestive enzymes is often severely impaired, with consequent steatorrhea and deficiency of fat-soluble vitamins, including vitamin A.

Cystinuria (choice A) is a common disorder in which a defective transporter for dibasic amino acids (cystine, ornithine, lysine, arginine) leads to saturation of the urine with cystine, which is not very soluble in urine, and precipitates out to form stones.

Hypoglycemia (choice B) is not a prominent feature of children with cystic fibrosis who are on a normal diet. Hyperglycemia may occur late in the course of the disease.

Iron deficiency anemia (choice C) is not found with any regularity in children with cystic fibrosis.

Laryngeal papillomas (choice D) are common lesions of the larynx. Unlike in the oral cavity, when found in the larynx they are likely to be symptomatic, with hoarseness that progresses to stridor over weeks to months.
What are multiple long-standing tan to brownish lesions on the face and back of older individuals that are slightly raised with a rough surface,typically 0.5-1.5 cm in diameter, and that show an ability to be peeled off?
A. Eczema
B. Melanoma
C. Psoriasis
D. Seborrheic keratoses
E. Verruca vulgaris




The correct answer is D.Chronic eczema (choice A) produces dry, thick, and sometimes discolored skin.

Melanomas (choice B) characteristically look like dark moles with irregular margins and variations in the degree of pigmentation.

Psoriasis (choice C) produces erythematous plaques with a silvery scale.

Verruca vulgaris (choice E), the common wart, produces verrucous papules that are most commonly found on the hands. The face and back would be unusual sites.
Which of the following laboratory findings would be MOST likely in a teenage girl with delayed puberty, short stature, webbed neck, low-set ears, fish-like mouth, and ptosis?
A. Decreased plasma growth hormone
B. Decreased plasma growth hormone
C. Increased plasma follicle stimulating hormone
D. Increased plasma inhibin
E. Increased plasma estrogen




The correct answer is C. This is a description of Turner's syndrome which is a result of a 45, XO karyotype and which is characterized by ovarian dysgenesis and a variety of somatic abnormalities including micrognathia, a fish-like mouth, a shield chest, low-set ears, ptosis, and a webbed neck. Other findings can include coarctation of the aorta, hypertension, and renal abnormalities. Short stature is invariably present; the cause is not known because plasma levels of growth hormone (choice A) and thyroid hormone (choice B) are typically not decreased. Clinical studies have shown, however, that injections of human growth hormone can increase the final height. The ovaries are usually streak-like and exhibit only fibrous stroma which leads to decreased secretion of estrogen (not increased, choice E) and inhibin (not increased, choice D) and persistent infantilism. Plasma levels of FSH are markedly increased because of the lack of feedback inhibition by ovarian secretions.
Which of the following presents with a painful, swollen knee, geographic tongue, and resolving psoriasiform rash on the palms and soles, as well as urethritis and conjunctivitis?
A. Gout
B. Bacillary arthritis
C. Osteoarthritis
D. Osteoporosis
E. Rheumatoid arthritis




The correct answer is E. This is an example of Reiter's syndrome (urethritis, cervicitis in females, conjunctivitis, arthritis, and mucocutaneous lesions). The term keratoderma blennorrhagicum is used for the lesions of the palms and soles that resemble pustular psoriasis. Geographic tongue is caused by erosions histologically characterized by prominent spongiform pustules. Lesions of the glans penis (balanitis circinata) may also be present. Reiter's syndrome is associated with chlamydial infection in 70% (or more) of cases. The arthritis that can be seen in the disease is an inflammatory arthritis similar to that of rheumatoid arthritis.

Gout (choice A) is a crystal arthritis; other examples of crystal arthritis include apatite-associated arthritis and pseudogout caused by calcium pyrophosphate dihydrate crystals.

If you believed the patient has gonococcal arthritis (a septic arthritis), then gram-negative bacillary arthritis (choice B) would be a good choice. Neisseria gonorrhoeae is not noted for causing conjunctivitis, however, which should have steered you toward Reiter's syndrome.

Osteoarthritis (choice C) is a degenerative disorder without systemic manifestations. X-ray findings typically show narrowed joint space, osteophytes, and an increased density of subchondral bone.

Osteoporosis (choice D) can be asymptomatic to severe backache from vertebral fractures. Demineralization of the spine, hip, and pelvis is common.
Which of the following can be noted in a rheumatoid arthritic that has acute caries and chronic parotid gland enlargement?
A. Oral squamous cell carcinoma
B. Polyarteritis nodosa
C. Sjogren's syndrome
D. Systemic lupus erythematosus
E. Thyrotoxicosis




The correct answer is C. Rheumatoid arthritis can coexist with a variety of autoimmune diseases (including those listed in the answers), but is most frequently associated with Sjogren's syndrome. Sjogren's syndrome classically presents with keratoconjunctivitis (dry eyes) and xerostomia (dry mouth, often resulting in dental caries and fissures in the oral mucosa). These symptoms are caused by autoimmune involvement with subsequent scarring of the salivary and lacrimal glands. Parotid gland enlargement is common and may be chronic or relapsing and develops in approximately one third of patients with Sjogren's syndrome. Vasculitis, Raynaud's phenomenon, hyperviscosity syndrome, and peripheral neuropathy may also be seen.

The development of oral squamous cell carcinoma (choice A) is not related to the presence of dental caries.

Polyarteritis nodosa (choice B) is a systemic necrotizing vasculitis. There is typically present with low-grade fever, weakness, and weight loss. Abdominal pain, hematuria, renal failure, hypertension, and leukocytosis may occur.

Systemic lupus erythematosus (choice D) is an autoimmune disease characterized by vasculitis, rash, renal disease, hemolytic anemia, and neurologic disturbances.

Thyrotoxicosis (choice E) is not related to the development of dental caries.
Which of the following is the cause of joint pain involving the large leg joints in a young adult accompanied by diarrhea that is often bloody?
A. Amebic colitis
B. Chronic appendicitis
C. Diverticulosis
D. Pseudomembranous colitis
E. Ulcerative colitis




The correct answer is E. Several gastrointestinal diseases are associated with rheumatologic complaints. The most frequent of these are the chronic inflammatory bowel diseases, ulcerative colitis, and Crohn's disease, which can be associated with sacroiliitis or lower limb arthritis. Other gastrointestinal diseases associated with arthropathy include bypass surgery, Whipple's disease, Behcet's syndrome, and celiac disease. Ulcerative colitis is accompanied by bloody diarrhea, lower abdominal cramps and fecal urgency, anemia, low serum albumin, and negative stool culture. Sigmoidoscopy is the key to diagnosis.

Amebic colitis (choice A) is caused by ingestion of infectious cysts (typically from Entamoeba histolytica). Symptoms include abdominal pain and diarrhea; malaise and weight loss may occur. Cecal amebiasis can resemble acute appendicitis.

Chronic appendicitis (choice B) may be asymptomatic or cause poorly defined abdominal pain.

Diverticulosis (choice C) is usually a disease of older adults. It is often asymptomatic unless inflammation supervenes.

Pseudomembranous colitis (choice D) is a severe form of diarrhea usually seen in the setting of prior antibiotic use. The causative organism is almost always Clostridium difficile.
Two diabetic patients are seen by a clinician. The first patient Comparing an older person just diagnosed with diabetes that was identified by the presence of hyperglycemia and diabetes in a young person who had previously presented with polyuria and polydipsia, the younger person is more likely to:
A. be obese
B. become euglycemic with oral hypoglycemic agents
C. develop ketoacidosis
D. have relatively high endogenous insulin levels
E. have tissue insulin insensitivity




The correct answer is C. The younger person would have type 1 (used to be called juvenile onset) diabetes mellitus, whereas the older person probably has type 2 (used to be called maturity onset) diabetes mellitus. These two types of diabetes differ in many respects. Ketoacidosis is more apt to develop in type 1, although it can occur in patients with type 2 after several years. As type 2 progresses, the amount of insulin secretion decreases more like type 1; therefore, the incidence of ketoacidosis increases.

Type 2 tend to be obese (choice A), whereas type 1 are often thin.

Type 1 is usually apparently caused by viral or immune destruction of beta cells, whereas type 2 is apparently usually caused by increased resistance to insulin; consequently the older diabetic rather than the younger diabetic is more likely to have relatively high endogenous levels of insulin (choice B).

Type 2 can often be controlled with oral hypoglycemic agents (choice D), whereas IDDM generally require insulin. Note that some NIDDM also may require insulin as the disease evolves.

Tissue insulin insensitivity (choice E) or insulin resistance are the primary etiologic factors causing NIDDM.
Which of the following processes is related to a asymptomatic perimenopausal woman demonstrating bony swellings in the distal interpharyngeal joints; however no inflammation is apparent and the proximal interphalangeal joints, hands, and wrists are not involved?
A. Autoantibody formation
B. Bacterial infection
C. Crystal deposition
D. Joint trauma
E. Viral infection




The correct answer is D. The disease is osteoarthritis and the bony swellings are called Heberden's nodes, which may or may not be symptomatic. Osteoarthritis is believed to be related to repetitive joint trauma. Osteoarthritis is a degenerative disorder with systemic manifestations. Pain is relieved by rest. Radiographic findings include narrowed joint space, osteophyte, increased density of subchondral bone, and bony cysts.

Autoantibodies (choice A) are important in rheumatoid arthritis.

Bacterial infections (choice B) cause septic arthritis.

Crystal deposition (choice C) is important in gout and pseudogout.

Viral infections (choice E) can cause transient arthralgias and arthritis.
Which of the following is present with a complaint of tiredness and muscle weakness, as well as increased blood pressure, plasma sodium and plasma aldosterone, and decreased plasma potassium, hematocrit, and plasma renin activity?
A. Addison's disease
B. Conn's syndrome
C. Cushing's syndrome
D. Hypothyroidism
E. Pheochromocytoma




The correct answer is B. Conn's syndrome is hyperaldosteronism caused by a hypersecreting adrenal adenoma. Conn's syndrome is one of several endocrine causes of hypertension. The hypertension is caused by volume expansion secondary to increased renal sodium and water retention. The excessive aldosterone also causes increased renal excretion of potassium leading to hypokalemia, which can explain the tiredness and muscle weakness. The decreased hematocrit is also consistent with blood volume expansion. The increase in blood volume, blood pressure, and plasma sodium all contribute to the suppression of renin secretion.

Addison's disease (choice A) is primary adrenal insufficiency and is characterized by decreased secretion of cortisol and aldosterone. It is accompanied by hyponatremia and hyperkalemia.

Patients with Cushing's syndrome (choice C) can also be hypertensive. This may be, in part, to an increased permissive action of cortisol on catecholamine-mediated vascular tone. In addition, cortisol in high levels can have significant mineralocorticoid activity and can produce sodium retention and potassium loss. The hypertension, sodium retention, and hypervolemia tend to suppress renin secretion and may decrease aldosterone secretion. Cushing's syndrome is also characterized by a redistribution of body fat, producing central obesity and a buffalo hump with the extremities being thinned.

Hypothyroidism (choice D) is associated with weakness, fatigue, cold intolerance, constipation, weight changes, and hoarseness. Dry skin, bradycardia, and a delayed return of deep tendon reflexes are also present. This condition is caused by low T4 levels. TSH levels are generally elevated in patients with primary hypothyroidism.

Pheochromocytoma (choice E) is another endocrine cause of hypertension. It is caused by a catecholamine-secreting tumor. The hypertension is caused by excessive vasoconstriction and increased cardiac output.
Which of the following additional findings would MOST likely be present in a child with freckles all over his body, including the buccal mucosa, lips, palms, soles, and skin not exposed to sun?
A. Colonic polyps
B. Desmoid tumors
C. Epidermoid cysts
D. Osteomas of the jaw
E. Pigmented ocular fundus




The correct answer is A. The widespread freckles (spots of melanin pigmentation) described are associated with hamartomatous colonic polyps in the autosomal dominant condition Peutz-Jeghers syndrome. The polyps in Peutz-Jeghers syndrome do not progress to colon cancer. Peutz-Jeghers syndrome is associated with an increased potential to develop carcinomas of the pancreas, breast, ovary, uterus, and lung. The hamartomas can become large enough to cause obstruction, bleeding, or intussusception.

All of the other features listed are components of Gardner's syndrome, a variant of familial adenomatous polyposis syndrome that carries a greatly increased risk for colon cancer.
In a PPD skin test that shows >10 mm of induration, which of the following type of reactive cells would be found?
A. B lymphocyte
B. Eosinophil
C. Mast cell
D. Neutrophil
E. T lymphocyte




The correct answer is E. The CD4+ population of T lymphocytes, specifically TH1 cells, are responsible for the delayed hypersensitivity reaction seen with a skin test in a previously sensitized patient. This response to a PPD test is typically seen in reactivation or adult-type tuberculosis. A 10-mm reaction on a skin test is considered positive if the person is in a high-incidence group (foreign-born, medically underserved, low-income population, or resident of a long-term care facility).

B lymphocytes (choice A) are involved in humoral immune reactions. Antibody production is not a feature of tuberculosis hypersensitivity.

Eosinophils (choice B) are important in type I hypersensitivity reactions and in immune-mediated responses to parasitic infections (ADCC). They are not associated with tuberculosis hypersensitivity.

Mast cells (choice C) are tissue cells that are involved in type I hypersensitivity reactions. They have surface receptors for the Fc fragment of the IgE molecule.

Neutrophils (choice D) are associated with acute inflammatory reactions. They are considered a "nonspecific" cell in that they do not interact with an antigen. The neutrophil is not a classic cell type seen in tuberculosis hypersensitivity.
Which of the following is the MAJOR chemotactic factor responsible for attracting neutrophils?
A. C3b
B. C5a
C. IgM
D. IL-2
E. Lysozyme




The correct answer is B. In active inflammation, the complement system has been activated and C5a is being produced, which is a strong chemoattractant to neutrophils and other phagocytic cells.

C3b (choice A) is an excellent opsonin of pathogenic organisms; when an organism is coated with C3b, it is more easily phagocytized. C3b is formed by way of the classic and alternative complement pathways.

IgM (choice C) is the first immunoglobulin produced in the primary immune response. IgM cannot cross the placenta, but it is a powerful activator of complement; elevated levels in the newborn are associated with an acute infection with a pathogen.

IL-2 (choice D) is a powerful interleukin that stimulates T helper 1 cells. It also stimulates natural killer cells and T cytotoxic CD8 lymphocytes, but is not chemotactic for neutrophils.

Lysozyme (choice E) is a material present in tears, mucus, vaginal secretions, and other body fluids. It is active against the peptidoglycan of bacterial cell walls, splitting the backbone structure of the peptidoglycan (N-acetylglucosamine and N-acetyl muramic acid polymers).
Which of the following is characterized by a young boy with failure to thrive, recurrent seizures, hepatomegaly, renomegaly, severe hypoglycemia, hyperlipidemia, lactic acidosis, and ketosis?
A. Gaucher's disease
B. McArdle's disease
C. Niemann-Pick disease
D. Pompe's disease
E. von Gierke's disease




The correct answer is E. von Gierke's disease is a glycogen storage disease caused by a deficiency of glucose-6-phosphatase. It typically presents with neonatal hypoglycemia, hyperlipidemia, lactic acidosis, and ketosis. Failure to thrive is common in early life; convulsions may occur because of profound hypoglycemia. The glycogen accumulation in von Gierke's disease occurs primarily in the liver and kidneys, accounting for the enlargement of these organs. Gout may develop later because of the derangement of glucose metabolism.

Even if you do not remember all of the details of the presentation of these genetic diseases, you should be able to narrow the choices:

Gaucher's disease (choice A) and Niemann-Pick disease (choice C) are lipid storage diseases and would not be expected to produce hypoglycemia.

The other diseases are glycogen storage diseases, but McArdle's disease (choice B) and Pompe's (choice D) disease affect muscle rather than liver and would not be expected to produce profound hypoglycemia, because the liver is the major source for blood glucose.
Which of the following disorders is associated with the same oncogenic virus that is the likely cause of nonkeratinizing squamous cell carcinoma of the nose, pharynx, and sinuses?
A. Adult T-cell leukemia
B. Burkitt's lymphoma
C. Cervical carcinoma
D. Hepatocellular carcinoma
E. Kaposi's sarcoma




The correct answer is B. The cancer described is nasopharyngeal carcinoma, which is associated with Epstein Barr virus (EBV). This virus is also associated with the African form of Burkitt's lymphoma that characteristically involves the jaw.

HTLV-1, or human T-lymphocyte virus, is associated with adult T-cell leukemia (choice A).

HPV, or human papillomavirus, is associated with cervical carcinoma (choice C), penile carcinoma, and anal carcinoma.

Hepatitis B virus (HBV) is associated with hepatocellular carcinoma (choice D).

HHV 8, a member of the herpes family, is associated with Kaposi's sarcoma (choice E).
What is at risk to develop when there is substernal burning pain exacerbated by large meals, cigarettes, and caffeine, and laying on the back, especially when sleeping at night, but improved with antacids?
A. Cardiac ischemia
B. Columnar metaplasia of the distal esophagus
C. Mallory-Weiss lesion in the esophagus
D. Squamous cell carcinoma
E. Zenker's diverticulum




The correct answer is B. This is a description of reflux esophagitis, a condition in which the lower esophageal sphincter (LES) does not adequately prevent acidic gastric contents from refluxing back into the distal esophagus. Most commonly, there is a defect in the LES mechanism itself, in addition to secondary causes such as pregnancy (from increased abdominal pressure) and some medications. Symptoms can mimic cardiac chest pain and must be carefully evaluated for complications including esophageal strictures, ulcerations, laryngitis, pulmonary aspiration, and Barrett's esophagus (columnar metaplasia of the distal esophagus). Barrett's esophagus is considered a premalignant state, with roughly a 40-fold increase in the incidence of esophageal adenocarcinoma. The normal squamous epithelium of the esophagus transforms into columnar epithelium similar to gastric epithelium as a result of recurrent reflux of acidic gastric contents.

Anginal pain, signaling cardiac ischemia (choice A), is generally not burning in nature and is not relieved by antacids.

Mallory-Weiss lesions (choice C) are actual tears of the epithelia of the proximal stomach or distal esophagus as a result of retching, nausea, and vomiting (seen in patients with anorexia and alcoholism).

The incidence of pure squamous cell carcinoma (choice D) is not increased by acid reflux disease.

Zenker's diverticula (choice E), the most common of esophageal diverticuli, are not true diverticuli, but only mucosal herniations that can cause obstructive symptomatology.
Which of the following in a middle-aged person can cause a persistent, long term productive cough with a past history of smoking and with the recent presence of a green sputum and fever?
A. Bronchogenic carcinoma
B. Chronic bronchitis with superimposed infection
C. Cystic fibrosis
D. Emphysema
E. Pulmonary tuberculosis




The correct answer is B. Chronic bronchitis requires the presence of chronic productive cough over at least 3 months of the year for 2 successive years. The green productive sputum and fever suggest that there is an acute infection superimposed on chronic bronchitis (history of cigarette smoking, history of excessive mucus production over many years). Histologically, the mucus-producing glands in the bronchi would show hyperplasia and hypertrophy and extend to a greater depth in the bronchial wall, resulting in a higher Reid index (ratio of thickness of mucus gland to thickness of bronchial wall).

Although smoking is involved and there is an increased risk for bronchogenic carcinoma (choice A), this is unlikely. Hemoptysis and weight loss would also be present if cancer was involved.

Cystic fibrosis (choice C) presents earlier in life and may be associated with severe production of mucus, especially if bronchiectasis supervenes. Age and the relative late onset of disease preclude this.

Emphysema may be involved (choice D), as chronic bronchitis and emphysema are often coexistent. Although pure emphysema might cause dyspnea, it would not, however, be associated with a fever or a productive cough.

Pulmonary tuberculosis (choice E) would typically present with hemoptysis rather than abundant green sputum. Weight loss, night anorexia, malaise, and weakness may also be present.
What is noted histologically with extensive pruritic wheals following ingestion of certain seafoods?
A. Dilated superficial lymphatic channels
B. Granular complement and IgG at the dermal/epidermal junction
C. Microscopic blisters
D. Munro microabscesses
E. Solar elastosis




The correct answer is A. Urticaria (hives) are pruritic wheals that form after mast cells degranulate and trigger localized dermal edema with dilated superficial lymphatic channels with an allergic response (commonly to certain types of shellfish). The mast cell degranulation is sometimes but not always triggered by IgE-antigen interactions.

Granular complement and IgG deposition at the dermal/epidermal junction (choice B) is a characteristic of systemic lupus erythematosus.

Microscopic blisters (choice C) are a characteristic of dermatitis herpetiformis.

Munro abscesses (choice D) are a characteristic of psoriasis.

Solar elastosis (choice E) is found in actinic keratoses.
Which will increase in the plasma and can cause an acid-base disorder in an uncontrolled type 1 diabetic having breath that smells like nail polish remover?
A. Formic acid
B. Glycolic acid
C. Ketoacids
D. Lactic acid
E. Oxalic acid




The correct answer is C. Ketoacidosis can easily occur in patients with type 1 diabetes mellitus secondary to the body's inability to produce insulin. This is metabolic acidosis (low pH, low HCO3-). The decrease in arterial CO2 is a compensatory response to the acidosis. The smell of acetone (which is the primary solvent of nail polish) on the breath of a diabetic suggests ketoacidosis and uncontrolled type 1 diabetes mellitus. Because acetone is highly volatile, it is excreted mainly by the lungs. Other common causes of ketoacidosis include starvation and the acute drinking-vomiting binge of the alcoholic.

Plasma levels of formic acid (choice A) increase when methanol is ingested.

Plasma levels of glycolic acid (choice B) and oxalic acid (choice E) increase when ethylene glycol is ingested.

Because lactic acid (choice D) is a product of anaerobic metabolism, its rate of production is increased by decreases in tissue blood flow and oxygen delivery and when oxygen use is impaired. Lactic acidosis commonly occurs in circulatory failure. Although diabetes mellitus can cause lactic acidosis, it is far less common than ketoacidosis.
A 25-year-old man experiences the gradual onset of intermittent diarrhea that over years progresses to severe diarrhea, alternating with constipation, rectal bleeding, and passage of mucus. On examination, the abdomen is tender over the colon, and stool examination fails to reveal parasites. Colonoscopy demonstrates inflammation limited to the rectum, with no higher lesions. Which of the following diseases is most consistent with this history?
A. Celiac disease
B. Pseudomembranous colitis
C. Traveler's diarrhea
D. Ulcerative colitis
E. Whipple disease




The correct answer is B. The presentation is classic for Crohn disease and ulcerative colitis. Family members of those afflicted show increased incidence of ulcerative colitis and Crohn disease, evidence that these two diseases are actually different ends of the same spectrum. In contrast to Crohn disease in which the lesions may be patchy and involve the distal ileum and even the esophagus, ulcerative colitis lesions involve the rectum and may extend continuously proximally for varying distances up to the cecum and very distal end of the ileum.

Celiac disease (choice A) is a small intestinal disease related to gluten intolerance. Flattening of villi, elongated crypts, and marked inflammation in the lamina propria are noted histologically.

Pseudomembranous colitis (choice B), or antibiotic induced colitis, is commonly seen following antibiotic therapy, especially with clindamicin. This condition is treated with metronidazole or vancomycin and is caused by the toxin produced by Clostridium difficile.

Traveler's diarrhea (choice C) is generally seen in patients traveling from one country to another. It has an acute onset and is generally associated with severe diarrhea.

Whipple disease (choice E) is an intestinal diarrheal disease that has been shown to be caused by a bacterial infection.
What disease state can be present when focal calcification is found above the sella turcica, and there is visual field abnormalities and diabetes insipidus, while needle biopsy shows tissue resembling tooth enamel?
A. Craniopharyngioma
B. Glioblastoma multiforme
C. Large pituitary adenoma
D. Medulloblastoma
E. Pituitary microadenoma




The correct answer is A. The tumor described is a craniopharyngioma, alternatively known as an adamantinoma or ameloblastoma. Craniopharyngiomas may arise in, or more commonly above, the sella turcica. The histologic pattern recapitulates the enamel organ of the tooth, with nests or cords of stratified squamous or columnar epithelium embedded in a loose fibrous stroma. Calcification (and even metaplastic bone formation) is common in these benign tumors that are believed to arise from vestigial remnants of Rathke's pouch.

Glioblastoma multiforme (choice B) characteristically shows at least some enlarged cells with bizarre nuclei.

Large pituitary adenomas (choice C) contain nests of uniform glandular cells.

Medulloblastoma (choice D) is made of small basophilic cells with relatively large nuclei for their size.

Pituitary microadenomas (choice E) contain nests of uniform glandular cells.
Which of the following locations listed is the more common primary site for oral squamous cell carcinoma?
A. Ventral tongue surface
B. Buccal mucosa
C. Floor of mouth
D. Palate
E. Tip of tongue




The correct answer is C. Oral cancers are highly associated with alcohol and tobacco use, excessive sun exposure, and HPV-16 (human papilloma virus type 16, which is also found in cervical carcinomas) is found in nearly half of all oral cancers. The pattern of tumor development possibly reflects exposure-related or tissue classification factors. In decreasing order of frequency in regards to risk of oral cancer, the following can be stated: (1) floor of the mouth (choice C); (2) tip of the tongue (choice E); (3) hard palate (choice D); (4) ventral tongue surface (choice A); and (5) buccal mucosa (choice B).

It is important to note that a systematic intraoral examination, including the entire oral cavity and lips, with particular emphasis on the lateral tongue, floor of the mouth, and soft palate complex, as well as palpation of the neck for enlarged hard lymph nodes should be part of any general head and neck examination. This is especially true for patients over the age of 45 years who smoke tobacco or drink alcohol excessively or are exposed to excessive amounts of sun.
Which of the following conditions explains the pain in the upper portion of the neck on swallowing, with regurgitation of undigested food shortly after eating?
A. Mallory-Weiss tears
B. Plummer-Vinson syndrome
C. Schatzki rings
D. Traction diverticula
E. Zenker's diverticulum




The correct answer is E. This is a description of Zenker's diverticulum, which is a false diverticulum formed by herniation of the mucosa at a point of weakness at the junction of the pharynx and esophagus in the posterior hypopharyngeal wall. The cause is believed to be loss of elasticity of the upper esophageal sphincter. Zenker's diverticulum is also associated with halitosis, and if the diverticulum fills completely with food, it can cause dysphagia or obstruction of the esophagus.

Mallory-Weiss tears (choice A) are mucosal tears at the gastroesophageal junction secondary to repeated, forceful vomiting. They are often seen in alcoholics and result in the appearance of blood in the emesis.

Plummer-Vinson syndrome (choice B) is the triad of dysphagia (caused by esophageal webs in the upper esophagus), atrophic glossitis, and iron-deficiency anemia.

Schatzki rings (choice C) are mucosal rings found in the distal esophagus at the squamocolumnar junction.

In contrast to a Zenker's diverticulum, the usually asymptomatic traction diverticula (choice D) are true diverticula involving all of the layers of the esophagus. They are typically caused by adherence of the esophagus to a scarred mediastinal structure.
Which of the following findings would suggest pemphigus vulgaris as opposed to bullous pemphigoid?
A. Eosinophils within bullae
B. Negative Nikolsky's sign
C. IgG autoantibody activity
D. Oral mucosal lesions




The correct answer is D. Pemphigus vulgaris is associated with chronic, severe bullae formation on the skin and oral mucosa. Mucosal lesions are extremely rare in bullous pemphigoid. This can be used clinically to guide therapy, although a skin biopsy should be taken to confirm the diagnosis. Both diseases are characterized by formation of tender bullae that can rupture, leaving red, raw areas. Pemphigus vulgaris patients eventually become febrile and lose weight, and if untreated, most will die within 1 year. Bullous pemphigoid lesions tend to heal and the patients do very well. This prognostic difference is an important distinction in dermatologic medicine.

Eosinophils within blisters (choice A) provide an important clue supporting bullous pemphigoid as the diagnosis that must be ascertained with histologic examination. The vesicles in pemphigus vulgaris mostly contain rounded acantholytic keratinocytes "floating" within.

Nikolsky's sign consists of separation of the epidermis on manual stroking of the skin. Bullous pemphigoid is characterized by a negative Nikolsky's sign (choice B), but Nikolsky's sign is positive in pemphigus vulgaris because of the IgG-mediated destruction of intercellular bridges between keratinocytes.

IgG autoantibody activity (choice C) is common to pemphigus vulgaris and bullous pemphigoid.
A 30-year smoker walks into clinic, breathing heavily and complaining of dyspnea and constant fatigue. On physical examination, the physician observes a "barrel chest" (expanded, with increased anteroposterior diameter) and hypertrophy of the accessory respiratory muscles. No cyanosis is evident. Occasionally he develops episodes of nonproductive cough, but he denies asthma attacks. Blood gas analysis shows minimal hypoxemia and normal CO2. Which of the following underlying pathogenetic mechanisms is most likely responsible for this patient's condition?
A. Airway obstruction
B. Bronchospasm
C. Chest wall deformity
D. Interstitial infiltration
E. Loss of elastic recoil




The correct answer is E. This patient is the classic pink puffer with chronic obstructive pulmonary disease (COPD). COPD is an umbrella term that refers to overlapping clinical conditions resulting from a combination of emphysema, asthma, bronchiectasis, and chronic bronchitis. If emphysema is predominant, patients with COPD have severe dyspnea (puffers), scanty sputum production, and nearly normal O2 arterial pressure, and thus no cyanosis (pink). Loss of elastic recoil is characteristic of emphysema, which is caused by destruction of alveolar walls and enlargement of airspaces distal to terminal bronchioles. Cigarette smoking is clearly the most important cause of COPD. Destruction of the pulmonary elastic fibers brings about increased resistance to airflow. The lungs become overexpanded; although total pulmonary capacity increases, the functioning lung parenchyma decreases. In emphysema the major complaint is dyspnea, often severe. Cough is rare with scant clear mucoid sputum. No peripheral edema is noted.

Airway obstruction (choice A) is prevalent in patients who have COPD with predominant chronic bronchitis (i.e., blue bloaters. Decreased PaO2 manifests with cyanosis (blue), and bronchitis causes abundant sputum production. Pulmonary hypertension and right ventricular overload produce peripheral edema (bloaters). The patient in this case does not fit this description.

Bronchospasm (choice B) is associated with asthma, a frequent component of COPD. Attacks of asthma are caused by spasm of bronchiolar smooth muscles, resulting in increased resistance to expiration. The clinical history clearly rules out bronchospasm as the fundamental mechanism of this patient's condition.

Chest wall deformity (choice C), such as severe kyphoscoliosis and obesity, and interstitial infiltration (choice D), usually caused by interstitial fibrosis, are responsible for restrictive pulmonary disease. Restrictive pulmonary disease leads to decreased lung compliance and reduction in all respiratory volumes. The barrel-chest deformity of this patient is a consequence, not a cause, of the underlying pathologic change (i.e., overexpansion of the lungs).
What is the MOST likely cause for profuse upper gastrointestinal bleeding with hematemesis and unconsciousness in an alcholic?
A. Barrett's esophagus
B. Helicobacter gastritis
C. Mallory-Weiss tear
D. Schatzki ring
E. Zenker's diverticulum




The correct answer is C. This is a description of a Mallory-Weiss tear, which is a mucosal tear at the gastroesophageal junction secondary to recurrent vomiting (the stomach temporarily evulses through the esophagus, tearing the esophagus). The result can be massive hematemesis, but the lesions usually heal without problems if the patient does not die from exsanguination. Mallory-Weiss tears account for 5-10% of cases of upper gastrointestinal bleeding. Most patients report history of heavy alcohol abuse.

Barrett's esophagus (choice A) is characterized by the replacement of normal esophageal epithelium with gastric-type epithelium. Barrett's causes an increased risk for adenocarcinoma, not bleeding.

Helicobacter gastritis (choice B) does not usually cause profuse bleeding.

Schatzki rings (choice D) are benign mucosal rings at the squamocolumnar junction below the aortic arch.

Zenker's diverticula (choice E) are esophageal evaginations at the junction of the pharynx and esophagus. They are not typically associated with bleeding.
Which of the following clinical conditions is most likely to result in hoarseness during speech?
A. Right lung apical segment bronchogenic carcinoma
B. Congestive heart failure
C. Emphysema
D. Pulmonary thromboembolism
E. Thymoma




The correct answer is A. A tumor of the apical segment of the upper lobe may impinge on the recurrent laryngeal nerve because it ascends from the superior mediastinum to the root of the neck in a groove between the trachea and esophagus. The recurrent laryngeal nerve supplies all intrinsic muscles of the larynx except the cricothyroid.

Congestive heart failure (choice B) is associated with exertional dyspnea, fatigue, orthopnea, paroxysmal nocturnal dyspnea, and rales. Speech patterns are often normal.

Emphysema (choice C) is a condition associated with dyspnea, often severe, in which the presence of a cough is rare. Speech patterns are generally normal.

Pulmonary embolus (choice D) causes an obstruction to arterial blood flow of the lung, resulting in infarction of the affected segment. It may also cause pleurisy, resulting in pain conveyed by intercostal nerves.

Thymoma (choice E), or tumor of the thymus gland, may cause dyspnea (difficulty breathing) caused by pressure on the trachea. It may also cause engorgement of deep and superficial veins of the neck caused by pressure on the superior vena cava.
Which of the following features characterizes apoptosis but not necrosis?
A. Disaggregation of polyribosomes
B. Eosinophilia
C. Inflammation
D. Karyolysis
E. Peripheral aggregation of chromatin




The correct answer is E. Apoptosis is a form of cell death that serves to eliminate unwanted cells during development, maintain cell numbers in intact organs or tissues, and eliminate immune cells after an immune response has faded. Apoptosis also occurs in response to noxious agents and in the aging process by way of execution of a genetic program. In the process of apoptosis, cells shrink and cytoplasmic organelles become more densely packed. Cytoplasmic blebs may form, and apoptotic bodies (membrane-bound cellular fragments) can be produced. The most characteristic feature of apoptosis is a distinctive peripheral aggregation of chromatin, sometimes accompanied by breaking up of the nucleus into several fragments. In necrosis, the chromatin may become more pale (karyolysis; choice D), or form irregular clumps, and the nucleus itself may shrink into a dense pyknotic body.

Disaggregation of polyribosomes (choice A) is characteristic of the initial stages of cellular injury and necrosis.

Eosinophilia (choice B) characterizes both apoptosis and necrosis.

Inflammation (choice C) is typically absent in apoptosis, in contrast to necrosis.
Which of the following can occur in a child after an upper respiratory infection who starts to form multiple petechial hemorrhages and blood tests show marked reduction in platelets?
A. Idiopathic thrombocytopenic purpura
B. Iron deficiency anemia
C. Pernicious anemia
D. Von Willebrand's disease




The correct answer is A. The process described is commonly called acute idiopathic thrombocytopenic purpura (ITP), even though the autoimmune basis has been clearly established (some investigators use "immune thrombocytopenic purpura," so that the initials still work). The thrombocytopenia in this disorder seems to be secondary to splenic destruction of opsonized platelets and usually follows a viral upper respiratory tract infection. The acute form of ITP is usually explosive but self-limited; a chronic form in adults may respond to steroid therapy or splenectomy.

Iron deficiency anemia (choice B) is almost always caused by bleeding in adults. Absent bone marrow iron stores are noted or serum ferritin is low.

Pernicious anemia (choice C) is a macrocytic anemia with macro and hyposegmented neutrophils on peripheral blood smear.

In von Willebrand's disease (choice E), deficient von Willebrand's factor produces platelet dysfunction, but thrombocytopenia is not prominent.
Patients with Plummer-Vinson syndrome are at an increased risk for
A. Adenocarcinoma of the esophagus
B. Barrett's esophagus
C. Candida infection
D. Cytomegalovirus esophagitis
E. Squamous cell carcinoma of esophagus




The correct answer is E. Plummer-Vinson syndrome is characterized by atrophic glossitis, esophageal webs, and iron-deficiency anemia. Patients with this syndrome are at increased risk for developing squamous cell carcinoma of the esophagus.

Barrett's esophagus (choice B) and adenocarcinoma of the esophagus (choice A) are associated with reflux esophagitis.

Candida (choice C) and cytomegalovirus (choice D) esophagitis can be seen in immunosuppressed patients, including AIDS patients.
What is a flesh-colored lesion that presents on the nose that is 1 cm in diameter with induration and central ulceration?
A. Basal cell carcinoma
B. Eczema
C. Psoriasis
D. Urticaria
E. Verruca vulgaris




The correct answer is A. The description is typical for basal cell carcinoma. These skin cancers typically occur on sun-exposed areas of the head, neck, and upper trunk. Basal cell carcinoma only rarely metastasizes, but can become locally mutilating if neglected. When located on the face, it may be difficult to adequately excise without damaging facial structures.

Eczema (choice B) typically involves a larger area of skin and may cause dryness, discoloration, and thickening of the involved area. Blistering, erythema, or oozing may also be observed.

Psoriasis (choice C) is characterized by erythematous plaques with a silvery surface.

Urticaria (choice D) causes transient, nonpitting, erythematous wheals.

Verruca vulgaris (choice E), the common wart, causes well-demarcated verrucous papules, often on the hands.
Which of the following is may cause sudden extreme hematemesis and eventual loss of consciousness?
A. Esophageal varices
B. Mallory-Weiss tear
C. Barrett's esophagus
D. Schatzki ring
E. Zenker's diverticulum




The correct answer is A. This is a description of life-threatening bleeding from esophageal varices (tortuous, dilated, submucosal esophageal vessels). Sclerotherapy of the vessels and pressure on the bleeding site with an "esophageal balloon" may temporarily control the problem, but unfortunately bleeding often recurs and exsanguination is a frequent cause of death in these patients.

In contrast, Mallory-Weiss tears (choice B) occur as a complication of repeated vomiting and do not often result in this amount of blood loss.

The other esophageal conditions listed do not usually cause hematemesis.

Barrett's esophagus (choice C) does not produce specific symptoms. Associated symptoms, however, are generally a consequence of gastroesophageal reflux disease.

Schatzki rings (choice D) are benign mucosal rings found at the squamocolumnar junction of the esophagus, below the aortic arch.

Zenker's diverticulum (choice E) is an esophageal evagination at the junction of the pharynx and esophagus.
Which of the following characteristics is typical in the early stages of type 1 diabetes mellitus, but not the early stages of type 2 diabetes mellitus?
A. Adult onset
B. Nearly complete twin concordance
C. Increased serum insulin levels
D. Ketoacidosis
E. Obesity




The correct answer is D. Type 1 diabetes mellitus, previously known as juvenile onset, is caused by low insulin production as a consequence of autoimmune destruction of pancreatic beta cells. Severe insulin deficiency causes marked increases in the use of fats as a source of energy. Ketones, acetoacetate, and beta-hydroxybutyrate are produced in excess, and diabetic ketoacidosis may develop with potentially dire consequences. Type 2 diabetes (or adult onset) is a consequence of insulin resistance by the tissues, despite very high levels of serum insulin, initially (insulin levels typically decrease as the disease progresses). Ketoacidosis is highly unusual in type 2 in the early stages, because insulin is present. As the disease progresses, however, and insulin levels begin to decrease, the incidence of ketoacidosis increases.

In type 1 there is usually complete loss of beta cells by puberty; thus, insulin dependence generally begins in childhood. Type 2 usually has an adult age of onset (choice A).

There is approximately 50% twin concordance in type 1, suggesting that environmental factors must also play a "triggering role" in type 1. The twin concordance rate is much higher in type 2 (~90%) (choice B).

Insulin levels are nearly zero in type 1. Conversely, type 2 is a disease of insulin resistance and is usually associated with increased insulin levels (choice C).

Body weight has no bearing on the pathogenesis of type 1, whereas type 2 occurs predominantly in the obese (choice E).
Which of the following tests should be performed on a young adult woman who presents with with fatigue, malaise, low-grade fever, arthralgias, elevated BUN, proteinuria, and butterfly rash on her face?
A. Anticentromeric antibody
B. Antimitochondrial antibody
C. Antinuclear antibody
D. Anti-TSH receptor antibody
E. Rheumatoid factor




The correct answer is C. Systemic lupus erythematosus (SLE) and should be considered in any 15-45-year-old woman with a malar (butterfly) rash, anti-double-stranded DNA antibodies, and renal involvement. Antinuclear antibody is a good screening test (although it is also positive in some other autoimmune diseases). Renal involvement is unique to this autoimmune disease. SLE mainly occurs in young women and joint symptoms are seen in 90% of all patients. The most common features of SLE include: malar rash, discoid rash, photosensitivity, oral ulcers, arthritis, serositis, renal disease, neurologic disease, and positive ANA.

Anticentromeric antibody (choice A) is a marker for the CREST form of scleroderma.

Antimitochondrial antibody (choice B) is a marker for primary biliary cirrhosis.

Anti-TSH receptor antibody (choice D) is a marker for Grave's disease.

Rheumatoid factor (choice E) is a marker for rheumatoid arthritis.
Which of the following can develop in a diabetic with a long-standing intrauterine contraceptive device that develops chronic pelvic pain and whose latest endometrial biospy shows a prominent infiltrate composed of lymphocytes, plasma cells, and histiocytes?
A. Acute endometritis
B. Candida infection
C. Chronic endometritis
D. Endometriosis
E. Simple hyperplasia of endometrium




The correct answer is C. This is a description of chronic endometritis, evidenced by the chronic inflammatory infiltrate of lymphocytes, plasma cells, and histiocytes. This disorder may be idiopathic but is more often associated with an obvious predisposing factor, such as chronic pelvic inflammatory disease, tuberculosis, retained gestational tissue, or, as in this case, an intrauterine contraceptive device. Chronic endometritis can cause abnormal bleeding, pain, and infertility.

Acute endometritis (choice A) is characterized by a prominent neutrophilic infiltrate and usually occurs after delivery or miscarriage.

Candidiasis (choice B) occurs in an estimated 75% of women during their lifetime. Although diabetes is an important risk factor, this is not a description of vulvovaginal candidiasis. Common symptoms include acute vulvar pruritus, burning vaginal discharge and dyspareunia, and the characteristic "curd-like" discharge.

Endometriosis (choice D) refers to abnormally located patches of endometrium (except in the myometrium, where it would be called adenomyosis).

Simple hyperplasia of endometrium (choice E) causes cystically dilated glands in endometrium.
A patient has intermittent diarrhea and abdominal pain and begins to pass fecal material in his urine. Which of the following diseases is most likely to cause this complication?
A. Celiac disease
B. Crohn disease
C. Diverticulitis
D. Pseudomembranous colitis
E. Ulcerative colitis




The correct answer is B. Passing fecal material in urine strongly suggests the possibility of a fistula between the bowel and bladder. Of the diseases listed, only Crohn disease (a type of inflammatory bowel disease) commonly produces fistulas. Fistulas are produced in Crohn disease because the disease affects the entire thickness of the bowel wall rather than being restricted to the mucosa (e.g., ulcerative colitis - choice E).

Celiac disease (choice A) is a mucosal disorder of the small intestine caused by intolerance to certain components of gluten from wheat and other grains.

Diverticulitis (choice C) can cause bowel perforation with peritonitis but does not usually cause fistula formation.

Pseudomembranous colitis (choice D), or antibiotic induced colitis, is usually caused by the toxin produced by the bacteria Clostridium difficile. The most common complaint is severe diarrhea.
A patient has intermittent diarrhea and abdominal pain and begins to pass fecal material in his urine. Which of the following diseases is most likely to cause this complication?
A. Celiac disease
B. Crohn disease
C. Diverticulitis
D. Pseudomembranous colitis
E. Ulcerative colitis




The correct answer is B. Passing fecal material in urine strongly suggests the possibility of a fistula between the bowel and bladder. Of the diseases listed, only Crohn disease (a type of inflammatory bowel disease) commonly produces fistulas. Fistulas are produced in Crohn disease because the disease affects the entire thickness of the bowel wall rather than being restricted to the mucosa (e.g., ulcerative colitis - choice E).

Celiac disease (choice A) is a mucosal disorder of the small intestine caused by intolerance to certain components of gluten from wheat and other grains.

Diverticulitis (choice C) can cause bowel perforation with peritonitis but does not usually cause fistula formation.

Pseudomembranous colitis (choice D), or antibiotic induced colitis, is usually caused by the toxin produced by the bacteria Clostridium difficile. The most common complaint is severe diarrhea.
A patient has intermittent diarrhea and abdominal pain and begins to pass fecal material in his urine. Which of the following diseases is most likely to cause this complication?
A. Celiac disease
B. Crohn disease
C. Diverticulitis
D. Pseudomembranous colitis
E. Ulcerative colitis




The correct answer is B. Passing fecal material in urine strongly suggests the possibility of a fistula between the bowel and bladder. Of the diseases listed, only Crohn disease (a type of inflammatory bowel disease) commonly produces fistulas. Fistulas are produced in Crohn disease because the disease affects the entire thickness of the bowel wall rather than being restricted to the mucosa (e.g., ulcerative colitis - choice E).

Celiac disease (choice A) is a mucosal disorder of the small intestine caused by intolerance to certain components of gluten from wheat and other grains.

Diverticulitis (choice C) can cause bowel perforation with peritonitis but does not usually cause fistula formation.

Pseudomembranous colitis (choice D), or antibiotic induced colitis, is usually caused by the toxin produced by the bacteria Clostridium difficile. The most common complaint is severe diarrhea.
A patient's complete blood count demonstrates a hematocrit of 62%. The peripheral smear shows normocellular erythrocytes, with increased reticulocytes and nucleated red cells. Bone marrow biopsy demonstrates increased numbers of erythrocytic precursors. Cancer of which of the following organs would be most likely to cause these findings?
A. Colon
B. Kidney
C. Ovary
D. Prostate
E. Thyroid




The correct answer is B. This is a pathophysiology question that can be answered easily by simply understanding that a low hematocrit is probably caused by a low production of erythropoietin that is synthesized and secreted in the kidney. The hematologic finding is erythrocytosis, which can be caused by abnormal erythropoietin secretion by renal cell carcinoma (i.e., a paraneoplastic syndrome). Absolute erythrocytosis also occurs in several other conditions, such as hypoxia, other types of renal disease, some tumors (e.g., hepatocellular carcinoma, meningioma, pheochromocytoma, cerebellar hemangioblastoma, adrenal adenoma), androgen therapy, Bartter syndrome, or in polycythemia vera.

Cancers of the colon (choice A), prostate (choice D), and thyroid (choice E) do not usually produce inappropriate hormones.

Cancers of the ovary (choice C) can produce male or female sex steroids, but do not produce erythropoietin.
A patient's complete blood count demonstrates a hematocrit of 62%. The peripheral smear shows normocellular erythrocytes, with increased reticulocytes and nucleated red cells. Bone marrow biopsy demonstrates increased numbers of erythrocytic precursors. Cancer of which of the following organs would be most likely to cause these findings?
A. Colon
B. Kidney
C. Ovary
D. Prostate
E. Thyroid




The correct answer is B. This is a pathophysiology question that can be answered easily by simply understanding that a low hematocrit is probably caused by a low production of erythropoietin that is synthesized and secreted in the kidney. The hematologic finding is erythrocytosis, which can be caused by abnormal erythropoietin secretion by renal cell carcinoma (i.e., a paraneoplastic syndrome). Absolute erythrocytosis also occurs in several other conditions, such as hypoxia, other types of renal disease, some tumors (e.g., hepatocellular carcinoma, meningioma, pheochromocytoma, cerebellar hemangioblastoma, adrenal adenoma), androgen therapy, Bartter syndrome, or in polycythemia vera.

Cancers of the colon (choice A), prostate (choice D), and thyroid (choice E) do not usually produce inappropriate hormones.

Cancers of the ovary (choice C) can produce male or female sex steroids, but do not produce erythropoietin.
Arthritis with a history significant for morning stiffness and symmetric deformities of the affected joints is most likely associated with which condition
A. Ankylosing spondylitis
B. Gouty arthritis
C. Osteoarthritis
D. Rheumatoid arthritis
E. Septic arthritis




The correct answer is D. Involvement of the cervical spine occurs in almost 80% of patients with rheumatoid arthritis and involves the atlanto-axial joint in up to 25% of those hospitalized for rheumatoid arthritis. Fortunately, large degrees of subluxation of the joint are rare, but if present can be associated with quadriplegia and even sudden death if the odontoid peg separates from the arch of the atlas and compresses the spinal cord. The onset of rheumatoid arthritis is usually insidious and in the small joints. Progression is centripetal and symmetric. Deformities are common. Extraarticular manifestations include subcutaneous nodules, pleural effusion, pericarditis, lymphadenopathy, and splenomegaly.

Serious complications of ankylosing spondylitis (choice A) include atrioventricular block, bladder and bowel dysfunction, uveitis, pulmonary fibrosis, psoriasis, and inflammatory bowel disease.

Serious complications of gout (choice B) include renal impairment.

Serious complications of osteoarthritis (choice C) are uncommon, but the local manifestations can be crippling. They do not cause quadriplegia. Pain and stiffness are most extreme at the end of the day, which differs from rheumatoid arthritis.

Serious complications of septic arthritis (choice E) include joint destruction, osteomyelitis, and systemic infection.
Which of the following can predispose a women to have vaginal itchiness and white curdy discharge that upon microscopic examination demonstrates fungal hyphae and yeast forms? W
A. Crohn's disease
B. Diabetes mellitus
C. Disseminated gonococcal infection
D. Rheumatoid arthritis
E. Systemic lupus erythematosus




The correct answer is B. The patient has vulvovaginitis secondary to Candida infection. Predisposing factors include a high vaginal pH, diabetes, and use of antibiotics. The increased vulnerability in patients with diabetes may reflect increased glucose concentrations in vaginal secretions and relative immunosuppression, both of which provide an excellent environment for fungus to grow.

Crohn's disease (choice A) can predispose for fistulas involving the vagina, rather than vulvovaginitis.

Dissemination of Neisseria gonorrhoeae (choice C) can cause septic arthritis.

Neither rheumatoid arthritis (choice D) nor systemic lupus erythematosus (choice E) are specifically associated with Candida vulvovaginitis.
Which of the following most likely precedes the development of non-Hodgkin's lymphoma in the gland?
A. Follicular thyroid carcinoma
B. Grave's disease
C. Hashimoto's thyroiditis
D. Nodular goiter
E. Papillary thyroid carcinoma




The correct answer is C. Hashimoto's disease (autoimmune thyroiditis) confers a 60-80-fold increased risk for developing thyroid lymphoma. This increased risk is not surprising considering the histology exhibited in Hashimoto's disease, in which sheets of benign but activated lymphocytes infiltrate and eventually destroy the thyroid gland, producing eventual thyroid failure.

Neither follicular (choice A) nor papillary (choice E) thyroid carcinomas predispose for lymphoma.

Lymphoma is not related to Grave's disease (choice B) or benign nodular goiter (choice D).
Which of the following most likely describes a person who is pleasant, emotionally warm and happy, lives at home, works in a sheltered workshop, did not complete high school, and talks in three-word sentences?
A. Autistic disorder
B. Mental depression
C. Mental retardation
D. Mixed receptive-expressive language disorder
E. Parkinson's disease




The correct answer is C. This describes simple mental retardation.

Autistic disorder (choice A) is not correct because persons with this are rarely able to interact with others to the point of holding a job, and they characteristically do not use language for purposes of communication.

Mental depression (choice B) is associated with feelings of worthlessness and helplessness. It generally does not affect speech patterns.

Mixed receptive-expressive language disorder (choice D) is a neurologic condition and affects language skills. This description does not discuss the use of both receptive and expressive language skills used to the point of being productively employed.

Parkinson's disease (choice E) is associated with postural instability, bradykinesia, and mask-like facies. It does not affect cognitive function.
If polydipsia and polyuria are present, but without glycosuria, which of the following should be considered?
A. Addison's disease
B. Diabetes insipidus
C. Diabetes mellitus
D. Grave's disease




The correct answer is B. Diabetes insipidus is characterized by the excretion of abnormally large volumes of dilute urine (polyuria) with a commensurate increase in fluid intake (polydipsia). The most common type is caused by inadequate secretion of antidiuretic hormone (also called vasopressin) and is usually referred to as "neurogenic" diabetes insipidus. This condition rarely causes severe problems as long as the person has plenty of water to drink. Placing a patient on overnight water restriction can cause severe dehydration and a greatly elevated plasma sodium concentration. The possibility of diabetes mellitus (choice C), which can also be associated with polyuria and polydipsia, but it does not have glucosuria.

Addison's disease (choice A) results from failure of the adrenal cortices to produce adrenocortical hormones. The lack of aldosterone leads to decreases in sodium reabsorption allowing large amounts of sodium to be lost into the urine. Polyuria and polydipsia are not characteristic of Addison's disease.

Grave's disease (choice D) or thyrotoxicosis is associated with marked tachycardia, profuse sweating, delirium, nausea, vomiting, and fever.
Microscopically, which of following do the proliferating cells in Kaposi's sarcoma associated with HIV infection MOST closely resemble? M
A. Angiosarcoma
B. Carcinosarcoma
C. Lymphoma
D. Malignant fibrous histiocytoma
E. Melanoma




The correct answer is A. Kaposi's sarcoma is a spindle cell neoplasm that is highly associated with AIDS and with the Herpes simplex virus type 8. The tumor has an appearance similar to that of angiosarcoma-proliferating stromal cells and endothelium, creating vascular channels that contain blood cells.

Carcinosarcoma (choice B) is a tumor that contains malignant epithelial cells and malignant stromal cells. There is no epithelial element in Kaposi's sarcoma.

Although lymphoma (choice C) occurs with increased frequency in AIDS, it does not resemble Kaposi's sarcoma. Lymphoma involves neoplastic lymphocytes, whereas Kaposi's sarcoma involves neoplastic vascular structures.

Malignant fibrous histiocytoma (MFH); (choice D) is an extremely poorly differentiated (anaplastic) stromal malignancy. MFH does not produce any recognizable mesenchymal structures; thus, the production of vascular structures by Kaposi's sarcoma differentiates the two tumors.

Melanoma (choice E) does produce colored skin lesions; however, the histologic appearance of the malignant melanocytes is unlike Kaposi's sarcoma. Melanoma in situ appears as small nests of cells with large, red nucleoli in the dermis and epidermis; this lesion can progress to a variety of forms, but none resemble Kaposi's sarcoma.
What is the cause for the anemia that develops in a person with systemic lupus erythematosus, chronic renal failure, and with a slightly yellow sclerae, enlarged spleen and positive Coombs test?
A. Bone marrow aplasia
B. Antibody directed against red blood cells
C. Spleen sequestration
D. Renal failure




The correct answer is B. The anemia must be a hemolytic form, because it is associated with unconjugated hyperbilirubinemia (hence the yellow sclerae), resulting from increased destruction of red blood cells. Increased erythrocyte destruction is the cause (not the effect) of splenomegaly. A positive Coombs test implies that hemolysis is mediated by antibodies attached to red blood cells. Antibody-coated red cells are then sequestered by the spleen, where hemolysis occurs, thus explaining splenomegaly.

Bone marrow aplasia (choice A) is caused by failure or suppression of myeloid stem cells, with decreased production of red blood cells, platelets, and leukocytes (pancytopenia).

Spleen sequestration (choice C) may cause anemia in case of massive splenomegaly because of an exaggeration of the normal role of the spleen as repository of blood cells.

Renal failure (choice D) causes normochromic, normocytic anemia because of decreased synthesis of erythropoietin. Erythropoietin administration is currently the standard treatment for this form of anemia.
Autoantibodies to which of the following components would most likely be found in a person who is not taking any medications but has multiple oral ulcers and flaccid skin bullae that were biopsied and showed a separation of epithelial cells above the basal layer?
A. Epidermal basement membrane proteins
B. Glycoprotein IIb/IIIa
C. Intercellular junctions of epidermal cells
D. Intrinsic factor
E. Type IV collagen




The correct answer is C. Bullae with the cleavage plane above the basal layer of the epidermis suggests pemphigus vulgaris, which is caused by autoantibodies to intercellular junctions of epidermal cells. The autoantibodies decrease the ability of the keratinocytes to adhere to one another, permitting formation of vesicles and bullae. Oral involvement is common, and often precedes the characteristic skin lesions. There will also be separation of the epidermis on manual stroking of the skin which is known as Nikolsky's sign. This sign is present in other disorders such as Stevens-Johnson syndrome, but we are told this person is healthy, and thus is not taking any medications, a typical cause of Stevens-Johnson syndrome in the adult population.

Antibodies to epidermal basement membrane proteins (choice A) are seen in bullous pemphigoid, which is a bullous disease characterized by blisters with a cleavage line between the epidermis and dermis.

Antibodies to glycoprotein IIb/IIIa (choice B) are seen in autoimmune thrombocytopenic purpura.

Antibodies to intrinsic factor (choice D) are seen in pernicious anemia.

Antibodies to Type IV collagen (choice E) are seen in Goodpasture syndrome.
Which of the following is the reason for a CBC with reticulocyte count that shows very low white blood cells and platelet counts, as well as having a bone marrow biopsy that demonstrates cellularity less than 25%?
A. Anemia of chronic disease
B. Aplastic anemia
C. Myelodysplasia with myelofibrosis
D. Pure red cell aplasia




The correct answer is B. Aplastic anemia is characterized by pancytopenia (low levels of all blood cells and platelets). Bone marrow biopsy typically reveals a normal architecture with a decrease in cellularity to levels less than 25% of normal. Absolute neutrophil counts are also often extremely low. Aplastic anemia may be either hereditary or acquired. Chloramphenicol, some anticonvulsant drugs, phenylbutazone, and a variety of other agents may produce aplastic anemia in an idiosyncratic manner. Aplastic anemias have also been linked to exposure to environmental toxins, such as benzene and insecticides, and are often found to be preceded by viral infections from cytomegalovirus, parvovirus, and hepatitis.

Anemia of chronic disease (choice A) produces an isolated deficit of red cell production and may resemble iron deficiency.

The myelodysplastic syndromes (e.g., myelodysplasia with myelofibrosis, choice C) are characterized by replacement of the bone marrow with abnormal (dysplastic) stem cells and ineffective hematopoiesis.

Pure red cell aplasia (choice D) would produce a selective deficit of the erythrocytic lineage, not all three lineages.
Which of the following should a person be evaluated for if they have rheumatoid arthritis and suddenly develop moderate caries?
A. Diabetes insipidus
B. Polyarteritis nodosa
C. Sjogren's syndrome
D. Systemic lupus erythematosus




The correct answer is C. Rheumatoid arthritis can coexist with a variety of autoimmune diseases (including those listed in the answers), but is most frequently associated with Sjogren's syndrome. Sjogren's syndrome is caused by autoimmune involvement with subsequent scarring of the salivary and lacrimal glands, leading to dry eyes and dry mouth. Secondary effects include parotid gland enlargement, dental caries (from dry mouth or xerostomia), and recurrent tracheobronchitis.

Diabetes insipidus (choice A) is associated with polyuria and polydipsia but not dental caries.

Polyarteritis nodosa (choice B) is a systemic necrotizing vasculitis. Patients present with low-grade fever, weakness, and weight loss. They may also have abdominal pain, hematuria, renal failure, hypertension, and leukocytosis.

Systemic lupus erythematosus (choice D) is an autoimmune disease characterized by vasculitis (which may produce a variety of symptoms depending on the site of the lesion), rash, renal disease, hemolytic anemia, and neurologic disturbances.
Which of the following should a person be evaluated for if they have rheumatoid arthritis and suddenly develop moderate caries?
A. Diabetes insipidus
B. Polyarteritis nodosa
C. Sjogren's syndrome
D. Systemic lupus erythematosus




The correct answer is C. Rheumatoid arthritis can coexist with a variety of autoimmune diseases (including those listed in the answers), but is most frequently associated with Sjogren's syndrome. Sjogren's syndrome is caused by autoimmune involvement with subsequent scarring of the salivary and lacrimal glands, leading to dry eyes and dry mouth. Secondary effects include parotid gland enlargement, dental caries (from dry mouth or xerostomia), and recurrent tracheobronchitis.

Diabetes insipidus (choice A) is associated with polyuria and polydipsia but not dental caries.

Polyarteritis nodosa (choice B) is a systemic necrotizing vasculitis. Patients present with low-grade fever, weakness, and weight loss. They may also have abdominal pain, hematuria, renal failure, hypertension, and leukocytosis.

Systemic lupus erythematosus (choice D) is an autoimmune disease characterized by vasculitis (which may produce a variety of symptoms depending on the site of the lesion), rash, renal disease, hemolytic anemia, and neurologic disturbances.
Which of the following physical findings would be the MOST reliable indicator that an older woman with shortness of breath is experiencing heart failure?
A. Diabetes insipidus
B. Polyarteritis nodosa
C. Sjogren's syndrome
D. Systemic lupus erythematosus




The correct answer is C. Rheumatoid arthritis can coexist with a variety of autoimmune diseases (including those listed in the answers), but is most frequently associated with Sjogren's syndrome. Sjogren's syndrome is caused by autoimmune involvement with subsequent scarring of the salivary and lacrimal glands, leading to dry eyes and dry mouth. Secondary effects include parotid gland enlargement, dental caries (from dry mouth or xerostomia), and recurrent tracheobronchitis.

Diabetes insipidus (choice A) is associated with polyuria and polydipsia but not dental caries.

Polyarteritis nodosa (choice B) is a systemic necrotizing vasculitis. Patients present with low-grade fever, weakness, and weight loss. They may also have abdominal pain, hematuria, renal failure, hypertension, and leukocytosis.

Systemic lupus erythematosus (choice D) is an autoimmune disease characterized by vasculitis (which may produce a variety of symptoms depending on the site of the lesion), rash, renal disease, hemolytic anemia, and neurologic disturbances.
Which of the following physical findings would be the MOST reliable indicator that an older woman with shortness of breath is experiencing heart failure?
A. A third heart sound (S3)
B. A fourth heart sound (S4)
C. Ascites
D. Orthopnea
E. Pulmonary rales




The correct answer is A. A third heart sound (S3) is a low-pitched sound occurring at the termination of rapid filling. In patients over 40 years of age, the appearance of a third heart sound strongly suggests congestive heart failure. It also occurs in patients with atrioventricular valve incompetence and can be a normal finding in some young athletes. Other common signs and symptoms of heart failure include the following:

Left ventricular failure: exertional dyspnea, fatigue, orthopnea, cough, cardiac enlargement, rales, gallop rhythm, and pulmonary venous congestion.

Right ventricular failure: elevated venous pressure, hepatomegaly, and dependent edema.

A fourth heart sound (S4; choice B) can be a normal finding in some older patients who do not have congestive heart failure.

Ascites (choice C) can also occur in patients with renal, hepatic, or local conditions not associated with cardiac factors.

Both orthopnea (choice D) and pulmonary rales (choice E) often occur secondary to heart failure; however, they both are associated with noncardiac disorders also.
What are multiple long-standing tan to brownish lesions on the face and back of older individuals that are slightly raised with a rough surface,typically 0.5-1.5 cm in diameter, and that show an ability to be peeled off?
A. Eczema
B. Melanoma
C. Psoriasis
D. Seborrheic keratoses
E. Verruca vulgaris




The correct answer is D.Chronic eczema (choice A) produces dry, thick, and sometimes discolored skin.

Melanomas (choice B) characteristically look like dark moles with irregular margins and variations in the degree of pigmentation.

Psoriasis (choice C) produces erythematous plaques with a silvery scale.

Verruca vulgaris (choice E), the common wart, produces verrucous papules that are most commonly found on the hands. The face and back would be unusual sites.
Serum chemistry studies of a patient reveal that her aspartate aminotransferase (AST) is markedly elevated, whereas her alanine aminotransferase (ALT), gamma-glutamyl transpeptidase (GGT), and alkaline phosphatase are all within normal limits. Disease of which of the following organs would be most likely to cause this serum enzyme pattern?
A. Colon
B. Duodenum
C. Heart
D. Pancreas
E. Stomach




The correct answer is C. Myocardial infarction (MI) can cause AST elevation without accompanying elevation of ALT or other liver enzymes. This is an important fact to remember because it may be the first clue for heart disease in a patient who has an atypical presentation of MI (as is common in women with MI). MI can be confirmed with measurement of the MB fraction of creatine phosphokinase (CPK-MB). In addition, Troponin T and Troponin I can be diagnostic.

Unfortunately, diseases of the tubular organs of the gastrointestinal tract, including colon (choice A), duodenum (choice B), and stomach (choice E), do not produce distinctive serum enzyme patterns.

Damage to the pancreas (choice D) is associated with elevated amylase levels.
A newborn infant has multiple hemorrhages. Clotting studies demonstrate an elevated prothrombin time and elevated INR. An abnormality of which of the following biochemical processes is likely present in this patient?
A. Conversion of homocysteine to methionine
B. Conversion of methylmalonyl CoA to succinyl CoA
C. Degradation of cystathionine
D. Formation of gamma-carboxyglutamate residues
E. Hydroxylation of proline




The correct answer is D. Deficiency of vitamin K produces a clotting disorder characterized by an elevated prothrombin time and easy bleeding, particularly in neonates (hemorrhagic disease of the newborn). The biochemical basis for this hemorrhagic tendency is that glutamate residues on Factors II (Thrombin), VII, IX, and X must be converted to gamma-carboxyglutamate residues (in a vitamin K-requiring reaction) for optimal activity.

The conversion of homocysteine to methionine (choice A) requires vitamin B12. Vitamin B12 deficiency can result in the development of pernicious anemia.

Conversion of methylmalonyl CoA to succinyl CoA (choice B) requires vitamin B12.

Degradation of cystathionine (choice C) requires vitamin B6 and can result in the development of mouth soreness, glossitis, cheilosis, and weakness.

Hydroxylation of proline (choice E) requires vitamin C. Vitamin C deficiency can cause easy bruising, but will not prolong the prothrombin time. Vitamin C deficiency is commonly known as scurvy.
Where are the body's temperature regulation centers located?
A. Midbrain
B. Pons
C. Medulla
D. Hypothalamus




The correct answer is D. Temperature regulation centers and thirst and food intake regulatory centers are located in the hypothalamus.

Choice A - Midbrain contains the micturation center.

Choice B - Pneumotaxic centers are located in the pons.

Choice C - The medulla contains the vasomotor center, respiratory center, vomiting, swallowing, and coughing centers.
An animal is made diabetic by injection of a drug that destroys pancreatic β cells. Removal of which of the following organs would most likely produce a decrease in blood glucose concentration in this animal?
A. Anterior pituitary
B. Colon
C. Gonads
D. Kidney
E. Pancreas




The correct answer is A. The anterior pituitary produces the following hormones: thyroid-stimulating hormone (TSH), adrenocorticotropic hormone (ACTH), follicle stimulating hormone (FSH), leutinizing hormone (LH), prolactin, and growth hormone (GH). Two of these secretions (GH, ACTH) affect the sensitivity of peripheral tissues to the action of insulin. Growth hormone has a direct effect on liver and muscle to decrease insulin sensitivity. This may be partly through a growth hormone-induced decline in insulin receptors or to unknown postreceptor defects. In excess, growth hormone is "diabetogenic," and approximately 25% of patients with acromegaly have diabetes. ACTH indirectly has antiinsulin effects by virtue of the cortisol secretion it evokes. Like growth hormone, cortisol also decreases insulin sensitivity in peripheral tissues. A third anterior pituitary hormone, TSH, also tends to increase blood glucose levels. In this case, the effect is probably mediated mostly through increased glucose absorption by the gut. Patients with hyperthyroidism can sometimes exhibit a postprandial glucosuria because of excessive intestinal glucose absorption. In diabetic animals, the removal of the anterior pituitary may lower blood glucose by increasing tissue sensitivity to whatever insulin remains.

Removal of the colon (choice B) should have little effect on blood glucose because dietary glucose is absorbed in the small intestine.

Sex steroids secreted by the gonads (choice C) have little effect on blood glucose concentration.

The kidney (choice D) plays an important role in reabsorbing filtered glucose. In diabetes, the tubular reabsorption maximum is exceeded and glucose spills over into the urine. The loss of glucose in the urine helps to reduce the severity of the plasma hyperglycemia. Removal of the kidneys would, if anything, make the hyperglycemia worse.

Pancreatectomy (choice E) would make the hyperglycemia worse by removing the source of any remaining insulin.
Which of the following is most likely to decrease in the skeletal muscles of a healthy 22-year-old woman during exercise?
A. Arteriolar resistance
B. Carbon dioxide concentration
C. Lactic acid concentration
D. Sympathetic nervous activity
E. Vascular conductance




The correct answer is A. The increase in muscle blood flow that occurs during exercise is caused by dilation of the arterioles (i.e., decreased arteriolar resistance) attributed to the dilatory actions of metabolic factors (e.g., adenosine, lactic acid, carbon dioxide) produced by the exercising muscles. In normal skeletal muscles, the blood flow can increase as much as 20-fold during strenuous exercise.

Exercise causes the concentration of carbon dioxide (choice B) and lactic acid (choice C) to increase in the muscles.

Mass discharge of the sympathetic nervous system (choice D) occurs throughout the body during exercise, causing arterioles to constrict in most tissues. The arterioles in the exercising muscles, however, are strongly dilated by vasodilator substances released from the muscles.

A decrease in vascular conductance (choice E) occurs when the vasculature is constricted. Resistance and conductance are inversely related, so that a decrease in arteriolar resistance is associated with an increase in arteriolar conductance.
Which of the following parameters is expected to increase in response to a 50% reduction in sodium intake for a 2-month period?
A. Arterial pressure
B. Atrial natriuretic peptide release
C. Extracellular fluid volume
D. Renin release
E. Sodium excretion




The correct answer is D. Renin is an enzyme released by the juxtaglomerular cells when renal blood pressure or pO2 declines. Renin converts angiotensinogen to angiotensin I. A reduction in sodium intake leads to a decrease in extracellular fluid volume (choice C) and therefore a decrease in arterial pressure (choice A). The decrease in arterial pressure stimulates renin release that in turn leads to an increase in the formation of angiotensin II. The angiotensin II increases the renal retention of salt and water (i.e., decreases sodium excretion, choice E), which returns the extracellular fluid volume nearly back to normal.

Atrial natriuretic peptide (choice B) is released from the two atria of the heart as a result of an increase in the extracellular fluid volume. A decrease in sodium intake therefore would tend to decrease the release of atrial natriuretic peptide.
Which of the following hormones is most important in the initiation of gallbladder contraction following a fatty meal?
A. CCK
B. Gastrin
C. GIP
D. Secretin
E. VIP




The correct answer is A. Cholecystokinin, or CCK, is synthesized in the duodenal and jejunal mucosa and stimulates gallbladder contraction and pancreatic enzyme secretion. Other functions include slowing of gastric emptying, an atrophic effect on the pancreas, and secretion of antral somatostatin, which in turn decreases gastric acid secretion.

Gastrin (choice B) prepares the stomach and small intestine for food processing, including stimulating secretion of HCl, histamine, and pepsinogen, increasing gastric blood flow, lower esophageal sphincter tone, and gastric contractions.

Gastric inhibitory peptide, or GIP (choice C), stimulates pancreatic insulin secretion at physiologic doses and inhibits gastric acid secretion and gastric motility at pharmacologic doses.

Secretin (choice D) stimulates secretion of bicarbonate-containing fluid from the pancreas and biliary ducts.

Vasoactive intestinal polypeptide, or VIP (choice E), relaxes intestinal smooth muscle and stimulates gut secretion of water and electrolytes.
During spermatogenesis, crossing over occurs during the meiotic division of which of the following cells?
A. Primary spermatocytes
B. Secondary spermatocytes
C. Spermatids
D. Spermatogonia
E. Spermatozoa




The correct answer is A. Crossing over, a transposition of genetic information, occurs during the first meiotic (reduction) division, when the primary spermatocyte divides to form two secondary spermatocytes. This division does not consist of separation of sister chromatids after DNA replication, but rather involves the separation of previously paired, homologous chromosomes. Crossing over occurs during prophase of meiosis I.

The secondary spermatocyte (choice B) undergoes the second meiotic division, which results in four spermatids (choice C), each with the haploid number of chromosomes. The spermatids are located adjacent to the lumen of the seminiferous tubules and are distinguished by their small size. These cells undergo no further division, but become transformed into mature spermatozoa (choice E) through the prcess of spermiogenesis.

The spermatogonia (choice D) are the primitive germ cells. These cells give rise to the primary spermatocytes through repeated mitotic divisions.

Remember the sequence of spermatogenesis:

Spermatogonia (2n) → REPEATED MITOSES → Primary spermatocytes (2n) → FIRST MEIOTIC DIVISION → Secondary spermatocytes (n) → SECOND MEIOTIC DIVISION → Spermatids (n) → SPERMIOGENESIS → Spermatozoa (n)
A decrease in which of the following is the most likely cause of peripheral edema in a patient with long-term alcoholism and liver disease?
A. Capillary hydrostatic pressure
B. Interstitial colloid osmotic pressure
C. Interstitial hydrostatic pressure
D. Plasma colloid osmotic pressure
E. Precapillary arteriolar resistance




The correct answer is D. Osmotic pressure of a solution is an indicator of the force of water movement resulting from its solute concentration. The higher the solute concentration of a solution, the greater its osmotic pressure. The plasma colloid osmotic pressure is often low in alcoholics with chronic liver disease (cirrhosis). The diseased liver cannot produce adequate amounts of albumin, which leads to a decrease in the concentration of albumin in the plasma (i.e., hypoalbuminemia). Because approximately 75% of the plasma colloid osmotic pressure can be attributed to the presence of albumin in the plasma, the decrease in plasma albumin concentration that occurs in the latter stages of cirrhosis often leads to peripheral edema. Cirrhosis also causes excess fluid to accumulate in the peritoneal cavity as ascites. In the case of ascites, the edema results not only from hypoalbuminemia, but also from portal vein obstruction (which increases capillary hydrostatic pressure) and the obstruction of lymphatic drainage of the liver. In fact, ascites is observed more often than peripheral edema in liver disease.

A decrease in capillary hydrostatic pressure (choice A) would tend to decrease fluid loss from the capillaries, and thereby oppose the development of edema.

A decrease in the colloid osmotic pressure of the interstitial fluid (choice B) would decrease fluid loss from the capillaries, thereby opposing the development of edema.

A decrease in interstitial hydrostatic pressure (choice C) would tend to increase fluid loss from the capillaries, but this cannot be considered a primary cause of edema because the interstitial hydrostatic pressure actually increases when a tissue becomes edematous.

A decrease in precapillary arteriolar resistance (choice E), which means arteriolar dilation, would increase capillary hydrostatic pressure and tend to cause edema. Decreased precapillary arteriolar resistance in the peripheral vasculature, however, is not associated with cirrhosis.
The medical record of a patient indicates a systolic murmur due to increase in afterload, producing a pressure gradient between the ventricle and aorta during ejection. Which of the following best describes the condition?
A. Aortic insufficiency
B. Aortic stenosis
C. Mitral insufficiency
D. Mitral stenosis




The correct answer is B.Aortic stenosis increases afterload and produces a pressure gradient between ventricle and aorta during ejection. Aortic insufficiency (choice A) increases preload and produces retrograde flow from the aorta to the ventricle, leading to a diastolic murmur. Mitral insufficiency (choice C) increases volume and pressure in the atrium and ventricle, producing a systolic murmur. Mitral stenosis (choice D) increases left-atrial volume and pressure, producing a diastolic murmur.
At 25 weeks of pregnancy, an unidentified infection greatly compromises the viability of a developing fetus. The level of which of the following hormones in the mother's blood is most likely to be affected?
A. Estriol
B. Free thyroxine
C. Human chorionic gonadotropin
D. Human chorionic somatomammotropin
E. Progesterone




The correct answer is A. Plasma levels of maternal estrogens during pregnancy depend on a functioning fetus. The fetal adrenal cortex and liver produce the weak androgens, DHEA-S and 16-OH DHEA-S, which are carried to the placenta by the fetal circulation. The placenta then desulfates the androgens and aromatizes them to estrogens (16-OH DHEA-S, estriol) before delivery to the maternal circulation. Estradiol and estrone increase approximately 50-fold during pregnancy, but estriol increases approximately 1,000 fold. When estriol is assayed daily, a significant drop may be a sensitive early indicator of fetal jeopardy.

Total serum thyroxine concentration may be increased in pregnancy because of an increase in circulating TBG resulting from increased estrogen. Free thyroxine, however, (choice B) remains within the normal range because of feedback regulation. The decline in estrogen with fetal compromise may gradually decrease serum thyroxine, but the free thyroxine will remain unchanged.

Human chorionic gonadotropin (choice C) and human chorionic somatomammotropin (choice D) are both secreted by syncytiotrophoblasts of the placenta. As long as placental function is intact, blood levels of these two hormones should not change with fetal compromise.

Placental secretion of progesterone (choice E) during pregnancy is also independent of any fetal contribution. The placenta relies on maternal cholesterol for progesterone production. Fetal death has no immediate influence on progesterone production by the placenta.
A healthy 20-year-old man deprived of water for several days has a plasma concentration of antidiuretic hormone (ADH) 5 times greater than normal. Which of the following is the most likely explanation for the increase in ADH concentration?
A. Decreased plasma aldosterone
B. Decreased plasma renin activity
C. Increased extracellular fluid volume
D. Increased left atrial pressure
E. Increased plasma osmolality




The correct answer is E. An obligatory loss of water from the body continues to occur even when a person is deprived of water. This loss of water from the body tends to concentrate the extracellular fluid, causing it to become hypertonic. Both the decrease in extracellular fluid (compare with choice C) and the increase in osmolarity act as stimuli for increased thirst and increased secretion of ADH. The decrease in extracellular fluid volume also tends to decrease arterial pressure that in turn increases plasma renin activity (compare with choice B) and aldosterone levels in the plasma (compare with choice A).

Water deprivation tends to decrease left atrial pressure (compare with choice D).